Preview (15 of 64 pages)

Chapter 10—Product Concepts TRUE/FALSE 1. Sebastian got a haircut this afternoon. His haircut cannot be called a product, however, because it is a service, not a tangible good. Answer: False Rationale: To many people, the term product means a tangible good. But services and ideas are also products. A product is everything, both favorable and unfavorable, that a person receives in an exchange. 2. An elementary school teacher purchased a ream of art paper, a box of crayons, and some modeling clay. The teacher intends to use these products in her classroom, so they would be classified as business products. Answer: True Rationale: The art paper, crayons, and modeling clay purchased by the elementary school teacher are intended for use in the classroom, making them business products as they support educational activities. 3. Convenience products, though inexpensive, still require considerable shopping effort by buyers. Answer: False Rationale: Convenience products are relatively inexpensive items that merit little shopping effort. 4. Because there is not much difference between different brands of milk, milk would be classified as a homogeneous product. Answer: True Rationale: Homogeneous products are goods that are perceived as similar enough by consumers that they can substitute one for another with little consequence. Since milk from different brands generally meets this criterion, it can be classified as a homogeneous product. 5. Inez believes that makeup is the most important part of her wardrobe. She spends considerable time comparing the prices, skin tones, and color options available at various cosmetics counters in department stores. For Inez, makeup is a convenience product. Answer: False Rationale: For Inez, makeup is a shopping product. 6. Rolex watches are very expensive and can only be purchased in high-end jewelry stores located in large metropolitan areas. Most buyers will accept no substitutes. Rolex watches are an example of a shopping product. Answer: False Rationale: This is an example of a specialty product. 7. A product unknown to the potential buyer or a known-product that the buyer does not actively seek is referred to as an undesirable product. Answer: False Rationale: This is an unsought product. 8. Coca-Cola is one of the world’s largest manufacturers of beverages. It makes over 2,800 different beverage products, including sparkling drinks, waters, juices, juice drinks, teas, coffees, sports drinks, and energy drinks. The great variety of beverages made by Coca-Cola represents its product mix. Answer: True Rationale: Coca-Cola produces a wide range of beverages across various categories, demonstrating a diverse product mix that includes sparkling drinks, waters, juices, teas, and more. 9. Product modification occurs when changes are made in how the product functions, its quality, or some aesthetic characteristic of the product. Answer: True Rationale: Product modification involves altering the function, quality, or aesthetic features of a product to meet changing consumer needs or improve its market competitiveness. 10. Procter & Gamble’s Olay line of skin products is very popular and profitable. P&G has launched an Olay body wash that is supposed to fight wrinkles. This is an example of repositioning. Answer: False Rationale: This would be an example of a product line extension. Repositioning involves changing consumers’ perceptions of a brand. 11. The best generator of repeat sales is satisfied customers. Answer: True Rationale: Satisfied customers are more likely to make repeat purchases, thereby generating repeat sales for a business. 12. Isaac Golding, owner of a tutoring service, has developed a design for his firm that includes an owl wearing large spectacles and carrying a satchel crammed full of books and papers. Golding plans to use this design on all communications the firm sends out. This design is called the firm’s brand name. Answer: False Rationale: Such a design would be called the firm’s brand mark, which is the element of a brand that cannot be spoken. 13. Branding has three main purposes: product identification, repeat sales, and new-product sales. Answer: True Rationale: Branding serves several key purposes, including helping consumers identify products, encouraging repeat purchases due to brand loyalty, and facilitating the introduction of new products under a trusted brand name. 14. Devore loves southern-style cooking and always buys Glory Foods brand of collard greens, black eyed peas, and hot sauces. In fact, she refuses to purchase any other brand because Glory Foods products taste just like her grandmother’s cooking. Devore is showing intense brand loyalty. Answer: False Rationale: They are likely to have more brand equity, which is the value of company and brand names, because of their familiarity. 15. Kellogg’s, Hanes, and General Electric are examples of a manufacturer’s brand. Answer: True Rationale: Kellogg’s, Hanes, and General Electric are examples of manufacturer's brands. 16. Retailers love national brands because overhead is low and there are no marketing costs, and they bring higher profit margins than private brands. Answer: False Rationale: Private brands, not national brands, bring these benefits to retailers. 17. When the manufacturer of French’s classic yellow mustard introduced French’s Dijon mustard, French’s Honey mustard, and French’s Sweet Onion mustard, it was an example of co-branding. Answer: False Rationale: This was an example of family branding. Co-branding is placing two or more brand names on a product or its package. 18. Trademark protection and rights only lasts for five years. Answer: False Rationale: Trademark protection typically lasts for 10 years; however, rights to a trademark last as long as the mark is used. 19. The most important functions of packaging are to contain and protect products, promote products, facilitate product storage, and facilitate recycling. Answer: True Rationale: Packaging serves multiple functions including containment, protection, promotion, storage facilitation, and recycling facilitation. 20. The nutrition label on a package is an example of persuasive labeling. Answer: False Rationale: This is better described as informational labeling. 21. Due to the time required to scan them, universal product codes (UPCs) are typically placed on merchandise that would be classified as shopping products. Answer: False Rationale: UPCs appear on most items in supermarkets and other high-volume outlets. Bar codes also appear on shopping products such as clothing, appliances, high-end wines, etc. The UPC is essential in supply chain management for a wide variety of products, not just shopping products. 22. A company that wants to market its product globally has only two branding strategies available: either use the brand name everywhere or develop a new brand name for each different market. Answer: False Rationale: There is a third possible branding strategy––adapt and modify. 23. Cultural traits can make a package design successful in one country and a failure in another. Answer: True Rationale: Cultural traits significantly influence the success or failure of package designs across different countries. 24. A warranty does not confirm the quality or performance of a good or service. Answer: False Rationale: A warranty is a confirmation of the quality or performance of a good or service. 25. All sales have an implied warranty under the Uniform Commercial Code. Answer: True Rationale: According to the Uniform Commercial Code (UCC), all sales have an implied warranty, which ensures that the goods sold meet certain standards of quality and performance expected by the buyer. MULTIPLE CHOICE 1. All of the following are examples of products EXCEPT: A. carton of milk B. contact lenses C. haircut D. pair of shoes E. All of the above are products. Answer: E Rationale: A product may be a tangible good, a service, an idea, or any combination of these three. 2. When a greenhouse plant grower sells plants to nursery retailers, the grower is selling _____ products. A. specialty B. consumer C. convenience D. business E. unsought Answer: D Rationale: Retailers resell the plants, so they are considered a business product. 3. RockTenn produces cardboard cartons to be used by companies in the food industry. RockTenn is selling _____ products. A. specialty B. consumer C. business D. convenience E. unsought Answer: C Rationale: Business products are purchased for (1) use in the production of other goods or services, (2) use in an organization’s operations, or (3) resale to other customers. 4. The consumer product classification system is based on: A. how the market is segmented B. the way the products are manufactured C. the way products are used D. the physical attributes of the product E. the amount of effort consumers spend to acquire the product Answer: E Rationale: Products are classified by the amount of effort that is normally expended in the acquisition process. 5. There are many cell phones on the market, and they vary so much on price, quality, and features that consumers often have trouble comparing them. A cell phone would probably be considered a _____ product. A. convenience B. specialty C. heterogeneous shopping good D. homogeneous shopping good E. generic Answer: C Rationale: Heterogeneous shopping products are essentially different, and consumers often have trouble comparing them. 6. _____ products are purchased with little shopping effort. These products typically are purchased regularly, usually with little planning, and require wide distribution. A. Convenience B. Specialty C. Branded shopping D. Unbranded shopping E. Generic Answer: A Rationale: This is the definition of convenience products. 7. The convenience product marketing strategy includes: A. wide distribution of the product. B. higher than ordinary prices. C. few retail outlets other than convenience stores. D. significantly lower promotion budgets. E. products that are not easily substitutable. Answer: A Rationale: To sell large amounts of products that consumers are unwilling to search for, there must be a large number of retail outlets. 8. Amy stopped by the grocery store to pick up a gallon of milk. As she was waiting to check out, she noticed that People Magazine had an interesting cover story about Scarlett Johansson. As Amy is a huge fan of Scarlett, she had to buy a copy. In this example, the People Magazine is an example of a(n) _____ product. A. specialty B. convenience C. business D. shopping E. unsought Answer: B Rationale: Convenience products are usually relatively inexpensive items that receive little shopping effort, such as an item bought on an impulse at a grocery-store checkout lane. 9. Compared to the other classifications of consumer products, shopping products are: A. widely available, so they need little or no promotion. B. usually less expensive than convenience products. C. purchased without significant planning. D. usually more expensive than convenience products and are found in fewer stores. E. purchased immediately after the consumer realizes he or she needs them. Answer: D Rationale: Shopping products typically are items such as clothing, automobiles, and major appliances. Consumers usually compare items across brands or stores. 10. Kathy is driving Chaz, her young son, home from soccer practice when Chaz says, “Mom, I’m thirsty.” Kathy immediately pulls the car into the nearest gas station and buys Chaz a Coke. For Kathy, the Coke is best described as an example of buying a _____ product. A. homogeneous shopping B. heterogeneous shopping C. convenience D. specialty E. component Answer: C Rationale: She spent very little shopping effort in buying the Coke. 11. Which of the following is the best example of a convenience product? A. Concert tickets B. Chewing gum C. Jeans D. Chemical for the swimming pool E. A birthday present Answer: B Rationale: Chewing gum is a relatively inexpensive purchase and requires very little, if any, comparison shopping. 12. Which type of consumer product requires comparison shopping because it is usually more expensive than a convenience product and is found in fewer stores? A. An unsought product B. A secondary product C. A unique product D. A shopping product E. A selective product Answer: D Rationale: This is the definition of a shopping product. 13. Harrison wanted a gas water heater. He watched newspaper ads until he found one on sale. For Harrison, the water heater is a(n) _____ product. A. shopping B. convenience C. component D. unsought E. specialty Answer: A Rationale: Harrison is comparing ads looking for a low price. This is shopping product behavior. 14. The two types of shopping products are: A. unsought and convenience. B. generic and family. C. exclusive and intensive. D. heterogeneous and homogeneous. E. consumer and business. Answer: D Rationale: Consumers perceive homogeneous shopping products as basically similar, but they perceive heterogeneous shopping products as essentially different. 15. Which of the following is the best example of a shopping product for most consumers? A. Washing detergent B. Crackers C. A soft drink D. A digital camera E. A magazine Answer: D Rationale: Shopping products are typically more expensive than convenience products and are found in fewer shops. 16. _____ are products consumers see as being basically the same, so consumers shop for the lowest price. A. Low-prestige specialty products B. Product equivalents C. Heterogeneous shopping products D. Generic convenience products E. Homogeneous shopping products Answer: E Rationale: With homogeneous shopping products, consumers typically look for the lowest-priced brand that has the desired features. 17. Andrea needs to find a kennel for boarding her son’s dogs. There are several different kennels in her community, and Andrea believes all of them to be basically alike. The only feature she is genuinely concerned about is low price. For Andrea, the kennel for boarding dogs would be classified as a(n) _____ product. A. specialty B. unsought C. heterogeneous shopping D. basic convenience E. homogeneous shopping Answer: E Rationale: Homogeneous products are products that consumers see as being basically the same, so consumers shop for the lowest price. 18. _____ products are seen by consumers to differ in quality, style, suitability, and lifestyle compatibility. Comparisons between these products are often quite difficult because they may have unique features and different levels of quality and price. A. Product equivalents B. Comparative convenience products C. Homogeneous shopping products D. Product counterparts E. Heterogeneous shopping products Answer: E Rationale: This describes heterogeneous shopping products. 19. When Kyla wanted patio furniture, she visited stores that had outdoor furniture, including Home Depot, Lowe’s, Target, and Howard Lumber Company to find the set that would look just right on her patio. If Kyla represents a typical patio furniture shopper, you know this product is a(n): A. homogeneous luxury product. B. shopping specialty good. C. operating supply good. D. convenience item. E. heterogeneous shopping good. Answer: E Rationale: Patio furniture would be considered heterogeneous shopping products because consumers perceive patio furniture to have distinctive qualities such as sizes, shapes, and levels of functionality that match the style of the customer. 20. Which of the following products is most likely to be considered an example of a heterogeneous shopping product? A. An apartment B. Laundry detergent C. A blender D. A package of pens E. Dog food Answer: A Rationale: Heterogeneous shopping products are essentially different. Housing is one of the examples given in the text. 21. _____ products are searched for extensively, and substitutes are not acceptable. These products may be quite expensive, and often distribution is limited. A. Exclusive shopping B. Homogeneous convenience C. Branded shopping D. Specialty E. Heterogeneous convenience Answer: D Rationale: This describes specialty products. 22. Bang & Olufsen make some of the best sound systems in the world. After years of development, Bang & Olufsen recently released its new BeoLab 5 speakers. For audiophiles who demand the best in audio equipment, the $19,950 Bang & Olufsen BeoLab 5 speakers represent an example of a(n) _____ product. A. heterogeneous shopping B. specialty C. homogeneous shopping D. convenience E. exclusive Answer: B Rationale: Bang & Olufsen speakers are expensive, and no substitutes are acceptable to this consumer group. 23. Janet will only purchase Crate and Barrel products for her home. The products are expensive, but Janet feels they have the highest quality and will last a very long time. These products represent _____ products. A. heterogeneous shopping B. specialty C. homogeneous shopping D. convenience E. business Answer: B Rationale: Crate and Barrel home furnishings are expensive, and no substitutes are acceptable to this consumer. 24. Evan Zimmermann loves watches. He collects vintage Rolex sport models made between 1950 and 1960. He paid $1,500 for his first watch in 2000. Now the watch is worth $15,000. For Evan, the vintage watches are an example of a(n) _____ product. A. specialty B. consumer C. convenience D. business E. unsought Answer: A Rationale: The fact that Evan will only purchase the vintage Rolex sports watches made between 1950 and 1960 shows that the watches are a specialty product. 25. When deciding on distribution plans for specialty products, companies generally ensure that the items are: A. made available only as private brands. B. made available in a large number of stores in a geographic area. C. made available only through the mail. D. distributed to a considerable number of stores in a geographic area. E. distributed to only a few stores in the geographic area. Answer: E Rationale: Specialty products are searched for extensively, and substitutes are not acceptable. These products may be quite expensive, and often distribution is limited. 26. _____ products are products that are not known about or not actively searched for by consumers. These products require aggressive personal selling and highly persuasive advertising. A. Hidden B. Specialty C. Equity D. Unsought E. Shopping Answer: D Rationale: This describes unsought products. 27. Unsought products typically are products that: A. consumers purchase when they see them in a convenient location. B. consumers definitely don’t want. C. only require reminder advertising to be successful. D. require little or no decision making by the buyer. E. consumers don’t know about or don’t actively look for. Answer: E Rationale: This is the definition of unsought products. 28. Every three months or so, Stanley cleans the lint out of the long vent to his clothes dryer. It is a messy job that Stanley hates. It is too bad that Stanley has never heard of the Casabella Bendable Dryer Vent Brush, a specially designed brush that would make his life a little easier. For Stanley, the dryer vent brush is a(n) _____ product because he doesn’t know the product exists. A. heterogeneous shopping B. specialty C. unsought D. convenience E. exclusive Answer: C Rationale: New products are often examples of unsought products until promotion reveals their existence. 29. Freda was proud of her shiny new iPhone. But she was disappointed when she began to notice scratches on its display surface. If Freda had known about the clear protective films that were available to protect the iPhone display from scratches, she could have avoided this annoying problem. For Freda, the protective films are a(n) _____ product. A. heterogeneous shopping B. specialty C. unsought D. convenience E. consumer Answer: C Rationale: New products are often examples of unsought products until promotion reveals their existence. 30. Which of the following is an example of a product item? A. Tarter Protection Crest toothpaste B. Campbell’s soup C. Chevrolet automobiles D. Ore-Ida frozen foods E. All of the above are product items. Answer: A Rationale: A product item is a specific version of a product that can be designated as a distinct offering among an organization’s products. 31. Campbell’s can talk about its soups being “Mm! Mm! Good!” and promote its entire line. This is an example of: A. macroeconomics. B. package uniformity. C. advertising economies. D. guerilla marketing. E. economies of scale. Answer: C Rationale: Equivalent quality is considered an advantage because purchasers usually expect and believe that all products in a line are about equal in quality. 32. Unsought products are often sold through: A. aggressive personal selling. B. highly persuasive advertising. C. direct mail. D. direct-response advertising. E. all of the above. Answer: E Rationale: Because the buyers do not actively seek to buy unsought products, companies that sell these products must go directly to them through all of the methods listed here. 33. A product line is a group of products that are closely related because the: A. products share the same product managers. B. products all function in a similar manner and provide similar benefits. C. same company has developed the idea for each product. D. products are all sold under the same brand name. E. products are all priced about the same. Answer: B Rationale: A product line is a group of closely related products offered by the organization that basically perform the same tasks and provide the same benefits. 34. A number of yogurts are marketed under the Dannon brand, including Dannon All Natural, Dannon Fruit on the Bottom, Dannon Light & Fit, DanActive, and Dan-o-nino. The large variety of yogurts under the Dannon brand is an example of a: A. marketing mix. B. product line. C. product mix. D. product equity. E. product modification. Answer: B Rationale: A product line is a group of closely related products. 35. Along with its Macintosh computer line, Apple markets its iPod, iPhone, and AppleTV products. This is an abbreviated listing of the company’s: A. customer mix. B. product line. C. product mix. D. line depth. E. product modification. Answer: C Rationale: An organization’s product mix includes all of the products that it sells. 36. Product mix width may be defined as: A. the number of different product categories found within an industry B. the average number of products within each product line in the company C. the average number of products offered by the company D. the number of different product lines an organization offers for sale E. the extensiveness of the coverage of the line from high end (expensive) to low end (inexpensive) Answer: D Rationale: Product mix width represents the number of product lines an organization offers. 37. Most people recognize Crayola as a brand of crayon, but Crayola also markets paints, chalk, pencils, markers, toys, coloring books, clothing, and even software. All these products are part of Crayola’s: A. marketing equity. B. product line. C. product mix. D. line depth. E. product modification. Answer: C Rationale: An organization’s product mix includes all of the products that it sells. 38. ForeverLawn has recently added two new synthetic lawn products. One of the products includes fake brown grass that makes the synthetic lawn look more realistic, as it appears to have some patches of brown thatch. The other product is K9Grass for consumers who have pet dogs. The addition of these two new synthetic grass products is a way for ForeverLawn to increase its: A. line width. B. breadth mix. C. mix width. D. line depth. E. mix depth. Answer: D Rationale: Product line depth is the number of product items in a product line. 39. Procter & Gamble manufactures Tide laundry detergent, the best-selling brand in the United States. But is also offers Tide to Go, which is a stick product consumers can use to clean a spot on their clothing between washings, and small travel packets of Tide detergent. These other products are part of Tide’s product line: A. width. B. breadth. C. mix. D. depth. E. synergy. Answer: D Rationale: Product line depth is the number of product items in a product line. 40. There are a number of toothpastes marketed under the Crest brand, including Cavity Protection Crest, Pro-Health Sensitive Shield Crest, Baking Soda Crest, Tartar Protection Crest, and Crest for Kids. This is an example of Crest’s: A. product line width. B. product mix. C. product line depth. D. product mix inconsistency. E. marketing mix. Answer: C Rationale: Product line depth is the number of product items in a product line. 41. Proctor & Gamble is a well-known producer of home and beauty products. Along with these products, P&G also markets pet products, prescription drugs, and small appliances. This range of products provides the company with product: A. line height. B. line consistency. C. mix width. D. line depth. E. mix depth. Answer: C Rationale: Product mix width refers to the number of different product lines that an organization offers for sale. 42. All of the following are benefits from organizing related items into product lines EXCEPT: A. package uniformity. B. varying quality. C. advertising economies. D. efficient sales and distribution. E. standardized components. Answer: B Rationale: Equivalent quality—not varying quality—is considered an advantage because purchasers usually expect and believe that all products in a line are about equal in quality. 43. Firms increase the depth of their product lines for all of the following reasons EXCEPT: A. to attract buyers with different preferences. B. to diversify risk. C. to further segment the market. D. to capitalize on economies of scale. E. to even out seasonal sales patterns. Answer: B Rationale: Firms increase the width of their product mix to diversify risk. 44. Changing one or more of a product’s characteristics is called: A. product modification. B. product repositioning. C. product adjustment. D. planned obsolescence. E. product extension. Answer: A Rationale: This is the definition of product modification. 45. Which type of product modification changes a product’s dependability or durability? A. Functional B. Style C. Aesthetic D. Quality E. Primary Answer: D Rationale: This is a description of a quality modification. 46. A go-cart manufacturer recently added shock absorbers to make the ride in its go-carts smoother. It has not changed its prices. This is a(n) _____ modification. A. upsale B. style C. dysfunctional D. repositioning E. quality Answer: E Rationale: Quality modifications entail changing a product’s dependability or durability. 47. When Honda automobiles first entered the U.S. market, they were small and not very durable. However, over the years, Honda has modified its automobiles to be one of the most dependable cars on the market. This change in dependability and durability is representative of which type of product modification? A. Style B. Planned obsolescence C. Functional D. Quality extension E. Quality Answer: E Rationale: Quality modification involves a change in a product’s dependability or durability. 48. Which type of product modification changes a product’s versatility, effectiveness, convenience, or safety? A. Quality B. Planned C. Functional D. Style E. Use Answer: C Rationale: This is a description of functional product modification. 49. When Proctor & Gamble removed potentially dangerous dyes and chemicals from its Tide detergent product and introduced Tide Free, the company was making a(n): A. product line contraction B. quality modification C. aesthetics modification D. obsolescence factor E. functional modification Answer: E Rationale: Functional modification is a change in a product’s versatility, safety, convenience. or effectiveness. 50. Which of the following is an example of a product modification? A. Lowering the price of textbooks B. Dropping a product from the product line C. Changing the color of a laundry detergent D. Changing the amount of management time spent overseeing production E. Increasing the number of distribution outlets Answer: C Rationale: Product modification entails changing one or more of a product’s characteristics. 51. Apple recently updated its iPod Touch product, expanding its storage from 8 GB to 16 GB. Enlarging the iPod Touch’s storage capacity is an example of a: A. product modification. B. brand mix extension. C. product diversification. D. brand repositioning. E. demographic modification. Answer: A Rationale: A product modification changes one of more of a product’s characteristics––in this example, its storage capacity. 52. Which type of product modification is an aesthetic product change? A. Functional B. Quality C. Repositioning D. Style E. Planned Answer: D Rationale: Style modifications are aesthetic product changes rather than quality or functional changes. 53. A style modification is a(n): A. change in the product’s durability or dependability B. safety improvement C. aesthetic product change D. way to add convenience E. improvement in product versatility and effectiveness Answer: C Rationale: Style modifications change the aesthetics of a product rather than its quality or functionality. 54. Which of the following is a symptom of product line overextension? A. Overuse of standardized components B. Manufacturing or marketing resources are disproportionately allocated to slow-moving products C. Strong economy of scale D. Planned obsolescence of certain products E. All of the above. Answer: B Rationale: Other symptoms include products not contributing to profits, products cannibalizing sales of other items in the line, and products becoming obsolete. 55. One of the leading manufacturers of choir robes has added different types of trim and fabric in the making of the robes. The company has not changed its prices. This is a(n) _____ modification. A. style B. obsolescence C. quality D. repositioning E. upward extension Answer: A Rationale: A style modification is an aesthetic change rather than a quality or functional change. 56. Which of the following statements about planned obsolescence is true? A. Environmentalists support planned obsolescence. B. A company that is interested in producing a quality product would never engage in planned obsolescence. C. Style modification creates planned obsolescence. D. Planned obsolescence is not an ethical issue. E. All of the statements about planned obsolescence are true. Answer: C Rationale: Planned obsolescence is the practice of modifying products so that those that have already been sold become obsolete before they actually need replacement, and style modifications are commonly used to motivate customers to replace products before they are worn out. 57. A regional utility company needs to change consumers’ perceptions of its current service as being harmful to the environment. Which of the following strategies would best allow the company to accomplish this goal? A. Contraction of the number of services offered by the utility company B. Adding new services to its product line C. Repositioning D. Disintermediation E. Use of product cannibalization Answer: C Rationale: Repositioning is a firm’s attempt to change customer perceptions of a product. 58. Apple Computer, Inc. recently dropped “Computer” from the company’s name to become Apple, Inc. This was a part of its _____ away from simply being perceived as a computer company. A. quality modification B. aesthetic modification C. demarketing D. repositioning E. functional extension Answer: D Rationale: Repositioning involves changing consumers’ perceptions of a product. 59. Over the years Cadillac has become to be thought of as “your grandparent’s car.” The new Cadillac ads showing a beautiful young woman driving a Cadillac along with the statement, “When you turn your car on, does it turn you on?” This is an example of marketing designed to help _____ Cadillac as a car for today’s younger driver. A. diversify B. reposition C. develop D. expand E. display Answer: B Rationale: Repositioning is changing consumers’ perceptions about a brand. 60. Adding additional products to an existing product line in order to compete more broadly in the industry is referred to as: A. product line extension. B. product modification. C. planned obsolescence. D. repositioning. E. cannibalization. Answer: A Rationale: This is the definition of product line extension. 61. Over the years, Crest has added a variety of different toothpastes to its brand, such as Cavity-Protection Crest, Sensitive-Teeth Protection Crest, Baking Soda Crest, Tartar Protection Crest, and Crest for Kids. These additions are examples of: A. product line extensions. B. product portfolio width adjustments. C. product item contractions. D. SBU expansions. E. product mix widths. Answer: A Rationale: Product line extension occurs when a firm adds a product to an existing product line to compete more broadly in the industry. 62. When Coca Cola introduced Coke Zero, this was an example of a _____ strategy. A. repositioning B. product line extension C. cannibalization D. disintermediation E. demarketing Answer: B Rationale: Product line extension is the adding of additional products to an existing product line in order to compete more broadly in the industry. 63. Cover Girl and Revlon are both recognized as leading manufacturers of cosmetics. Both companies have recently introduced cosmetics for the over-50 woman who does not have the same concerns as a 20-year-old. The introductions of these cosmetics are examples of the implementation of a _____ strategy. A. portfolio breadth expansion B. product portfolio width adjustment C. product item contraction D. product line extension E. repositioned marketing mix Answer: D Rationale: Product line extension occurs when a firm adds a product to an existing product line to compete more broadly in the industry. 64. At one point, Heinz made ketchup in multiple colors, including red, green, purple, pink, orange, and teal. While they were popular for a few years, consumers stopped buying the unusual colors and Heinz has dropped back to just making red ketchup. This is an example of the implementation of a _____ strategy. A. repositioning B. cannibalization C. product line extension D. product line contraction E. divestment Answer: D Rationale: Product line contraction occurs when a company cuts its product offerings due to overextension of the product. 65. A _____ is a name, term, symbol, design, or combination thereof that identifies a seller’s products and differentiates them from competitors’ products. A. brand mark B. trademark C. brand name D. UPC E. brand Answer: E Rationale: This is the definition of a brand. 66. The well-known Nike “swoosh” is an example of a: A. quality mark. B. product line. C. brand name. D. brand mark. E. manufacturer’s brand. Answer: D Rationale: Well-known symbols for brands are called their brand marks, but these elements cannot be spoken. 67. A _____ is everything, both favorable and unfavorable, that a person receives in an exchange. It can be tangible, intangible, a service, an idea, or a combination of these things. A. brand equity B. quality C. warranty D. transaction E. product Answer: E Rationale: This is the definition of a product. 68. The _____ is the starting point in creating a marketing mix. A. price B. product C. distribution channel D. promotional media E. production line Answer: B Rationale: The product must be specified before other marketing mix variables can be planned. 69. Why is creation of a product the starting point for the marketing mix? A. The production department must know what to produce first. B. Production strategy is the first of the four Ps listed in the marketing mix. C. Determination of the price, promotional campaign, and distribution network cannot begin until the product has been specified. D. Product development takes the longest amount of time to complete. E. Actually, the product does not have to be the starting point––promotional strategies are often the starting point. Answer: C Rationale: The product is the first decision around which the others are based. 70. Which of the following is NOT an example of a product’s tangible feature? A. Brand equity B. Packaging C. Color D. Options E. Size Answer: A Rationale: Brand equity is the value of company and brand names. 71. Which of the following may qualify for trademark protection? A. Shapes, such as the Coca-Cola bottle B. Catchy phrases, such as “Every kiss begins with Kay” C. Abbreviations, such as Bud D. Sounds, such as Intel’s famous five-note jingle E. All of the above. Answer: E Rationale: Parts of a brand or other product identification may qualify for trademark protection, including all of the examples listed. 72. A _____ is the part of the brand that can be spoken. A. brand equity B. service mark C. trademark D. brand name E. certification mark Answer: D Rationale: Brand names are the part of the brand that can be spoken, including letters, words, and numbers. 73. The _____ is the element of a brand that cannot be spoken. A. brand mark B. trademark C. brand equity D. quality mark E. intangible product Answer: A Rationale: Well-known symbols for brands are called their brand marks, but these elements cannot be spoken. 74. A Boston diamond distributor has developed the first branded diamond called Hearts on Fire to help smaller retailers counter the volume jewelers’ aggressive price discounting. This move is meant to: A. take away trademark rights. B. build product identity and customer loyalty. C. create a catchy phrase to promote diamonds. D. combat the quality appeal of generic products. E. combat low-cost production of generic brands. Answer: B Rationale: This represents one benefit of branding. 75. The value of company and brand names is referred to as: A. brand equity. B. brand loyalty. C. brand power. D. brand equivalency. E. brand strength. Answer: A Rationale: A brand that has high awareness, perceived quality, and brand loyalty among customers has high brand equity. 76. According to recent market research, Google is one of the most valuable brands in the world. According to this research, the company and its well-known name are worth about $100 billion. This research indicates that Google has a high: A. brand loyalty hierarchy. B. evoked set. C. brand quality standard. D. perceptual expectation. E. brand equity. Answer: E Rationale: The term brand equity refers to the value of company and brand names. 77. A(n) _____ refers to a brand that obtains at least a third of its earnings from outside its home country. A. evoked set B. global brand C. equity brand name D. master brand E. ethnocentric trademark Answer: B Rationale: This is the definition of a global brand. 78. Rexona, marketed by Unilever (a Dutch company), is the world’s number one deodorant brand. The brand is a leader in Europe, South America, Asia, Africa, and the Middle East. Rexona is an example of a: A. master brand. B. global brand. C. cannibalized brand. D. standard brand. E. family brand. Answer: B Rationale: At least one third of a global brand is sold outside its home country. 79. What is the best generator of repeat sales? A. Advertising B. Sales promotion C. Brand equity D. Satisfied customers E. Global brand Answer: D Rationale: Satisfied customers are the best generator of repeat sales. 80. A consumer who consistently and repeatedly purchases the same brand is said to have: A. brand loyalty B. product equity C. product loyalty D. product repetition E. store loyalty Answer: A Rationale: Brand loyalty is a consistent preference for one brand over all others. Product loyalty could easily involve a variety of brands. 81. Brand loyalty can: A. increase product diffusion B. encourage competition C. shorten the amount of time a product spends in the maturity stage D. help ensure repeat sales E. sell a bad product Answer: D Rationale: Brand loyalty ensures repeat and regular sales. 82. The brand name of a manufacturer is known as a(n) _____ brand. A. private B. manufacturer’s C. individual D. captive E. family Answer: B Rationale: This is the definition of a manufacturer’s brand, which is also referred to as a national brand. 83. Vick’s, Nestlé, Pepsi, and Toyota products are examples of _____ brands because of who owns them. A. manufacturers’ B. private C. family D. individual E. master Answer: A Rationale: Manufacturers’ brands are the brand name of a manufacturer. 84. A(n) _____ brand is one owned by the wholesaler or retailer. A. intermediate B. private C. generic D. corporate E. primary Answer: B Rationale: A private brand, also known as a private label or store brand, is a brand name owned by a wholesaler or a retailer. 85. Target stores sell many premium foods under the name Archer Farms. This brand can only be purchased in Target stores and is an example of a(n) _____ brand. A. manufacturers’ B. international C. family D. private E. corporate Answer: D Rationale: A private brand is one owned by the wholesaler or retailer. 86. Which type of private label brand carries no evidence of a retailer’s affiliation, is manufactured by a third party, and is sold exclusively at the retailer? A. A captive brand B. A complementary brand C. A cooperative brand D. An exclusive brand E. A generic brand Answer: A Rationale: This describes a captive brand. This strategy allows the retailer to ask a price similar to manufacturer’s brands. 87. A small chain of supermarkets in the western United States sells only manufacturers’ brands. Which of the following is one of the arguments you could use to dissuade the supermarket chain owner from selling only manufacturers’ brands? A. A well-known manufacturers’ brand will not enhance the chain’s image. B. Manufacturers rarely spend money advertising the brand name to consumers. C. Manufacturers can decide to drop a brand or reseller at any time. D. Manufacturers force the chain to carry a large in-store inventory. E. Relying on the manufacturer or wholesaler to deliver a national brand quickly is optimistic at best. Answer: C Rationale: Manufacturers can decide to drop a brand or reseller at any time See Exhibit 10.2. 88. Which of the following statements describes an advantage to retailers associated with developing their own brands? A. Dealers must market the brand, thus cutting into the gross margin. B. Higher profit margins are available on private brands. C. Private brands, particularly those owned by discounters, are perceived to be of a higher quality. D. Dealers often buy in large quantities and thus always have a ready supply. E. Private brands are normally delivered more promptly by the manufacturer. Answer: B Rationale: Private brands carry higher margins because manufacturers do not need to promote them. See Exhibit 10.2. 89. When a company uses different brand names for different products, it is using _____ branding. A. private B. generic C. compound D. family E. individual Answer: E Rationale: This is the definition of individual branding. 90. Heinz is a leading global food manufacturer. It manufactures and markets Farleys (baby food), Jack Daniel’s sauces, and Weight Watcher’s diet/slimming meals and supplements. The use of these brand names instead of the Heinz name is an example of a(n) _____ branding strategy. A. individual B. synergistic C. umbrella D. family E. piggyback Answer: A Rationale: Individual branding is using different brand names for different products. 91. Procter & Gamble makes Gain laundry detergent for people who like a strong scent and Dreft for those who want a gentle detergent for their baby’s clothes. It is using a(n) _____ strategy. A. individual branding B. family branding C. combination branding D. trademarked branding E. private branding Answer: A Rationale: An individual brand strategy allows for development of a unique brand image to distinguish the different products. 92. When a company markets several different products under the same brand name, it is referred to as a(n) _____ brand. A. family B. generic C. bargain D. dealer E. umbrella Answer: A Rationale: This is the definition of family branding. 93. Most people recognize Butterball as a brand of turkey, but Butterball brand is also found on fresh turkey breast cuts, turkey sausages, and ground turkey; lunchmeat cold cuts; fresh marinated bone-in, boneless, and whole chicken; frozen chicken products; and Butterball stuffing and gravy mixes. Butterball uses: A. dealer branding B. brand grouping C. family branding D. generic branding E. co-branding Answer: C Rationale: Family branding refers to the strategy of marketing several different products under the same brand name. 94. Which of the following entails placing two or more brand names on a product or its package? A. Family branding B. Private branding C. Dual-code branding D. Co-branding E. Complementary branding Answer: D Rationale: This is the definition of co-branding. 95. Maureen purchased a package of Bounce fabric sheets with Febreeze citrus scent. Since both brands were prominently listed on the package, this would be an example of: A. equity branding B. co-branding C. conjunctive branding D. private branding E. complementary branding Answer: B Rationale: Using two or more brand names in conjunction is co-branding. 96. Jennifer purchased Cascade 2in1 ActionPacs for her dishwasher. The packaging said the product combines the scrubbing power of Cascade with the grease-fighting power of Dawn dishwashing detergent. What type of co-branding does this product represent? A. Cooperative B. Complementary C. Piggyback D. Ingredient E. Synergistic Answer: D Rationale: Using two or more brand names in conjunction is co-branding. Ingredient branding identifies the additional brand that makes up part of the product. 97. Rosa’s husband Phil has a cold. Rosa went to the store to buy something that would help Phil sleep. Rosa purchased Breathe Right nasal strips embedded with Vicks mentholated vapor rub. The package showed both the Breathe Right and the Vicks brand names. This is an example of _____ branding. A. equity B. ingredient C. family D. private E. functional Answer: B Rationale: Using two or more brand names in conjunction is co-branding. Ingredient branding identifies the additional brand that makes up part of the product. 98. Which of the following describes cooperative branding? A. When advertising identifies a brand as a part that makes up another product B. When two brands receive equal treatment, borrowing from each other’s brand equity C. When products are advertised together to suggest usage together D. When advertising identifies a brand that should not be used with another product E. When advertising identifies a family of brands Answer: B Rationale: Cooperative branding occurs when two brands receive equal treatment and borrow from each other’s brand equity. 99. Betty Crocker brand sells a Hershey’s Ultimate Fudge cake mix that prominently features both the Betty Crocker and Hershey’s names and logos on the box. This is an example of _____ cobranding. A. cooperative B. ingredient C. umbrella D. complementary E. family Answer: A Rationale: Cooperative branding occurs when two brands receive equal treatment and borrow from each other’s brand equity. 100. Butterball, a well-known brand of turkey, advertised Ocean Spray cranberry sauce in its magazine ads and depicted suggested usage with its turkey. This is an example of which type of co-branding? A. Ingredient B. Complementary C. Family D. Captive E. Product use Answer: B Rationale: With complementary branding, products are advertised or marketed together to suggest usage. 101. A _____ is a legal term indicating the owner’s exclusive right to use a brand name or part of a brand. A. trademark B. patent C. brand D. private brand E. right of warranty Answer: A Rationale: This is the definition of a trademark. 102. When a product name becomes generic: A. the firm must use black-and-white packaging B. competitors are prohibited by law from using the trademark C. the price will have to drop to appeal to lower-income consumers D. the product name is no longer recognized as the exclusive property of one firm E. a firm can reapply for exclusive trademark protection at the U.S. Patent Office Answer: D Rationale: A generic product name can be used by any firm and is public property, therefore if a product name becomes generic, it is no longer the property of one firm. 103. Kleenex is a well-known brand name of facial tissue. People often refer to Kleenex as if it were the product name. The company that makes Kleenex may someday find its brand name becoming a(n): A. equity brand B. certified name C. trademark D. faux brand E. generic product name Answer: E Rationale: If a brand name becomes synonymous with the product category, it can become generic. 104. All of the following are functions of packaging EXCEPT: A. containing and protecting the product. B. guaranteeing product quality. C. facilitating recycling and reducing environmental damage. D. promoting the product. E. facilitating product storage, use, and convenience. Answer: B Rationale: More than packaging is necessary to guarantee product quality. 105. Sherwin-Williams’ Dutch Boy paint has packaging that is getting rave reviews. The new Twist & Pour paint is packaged in an all-plastic gallon container with an easy twist-off lid, comfortable side handle, and even a pouring spout. There’s no other paint product like it on the market. This innovative packaging was most likely designed for which of the following functions? A. To facilitate product use and convenience B. To contain and protect the product C. To reduce environmental damage D. To facilitate recycling E. To promote the product Answer: A Rationale: The other functions of packaging are containment and protection, promotion, the facilitation of recycling, and the reduction of environmental damage. 106. The two types of package labeling in common usage today are: A. informational and persuasive. B. promotional and nonpromotional. C. functional and persuasive. D. government mandated and seller controlled. E. motivational and required. Answer: A Rationale: Persuasive labeling focuses on a promotional theme or logo, whereas informational labeling is designed to help consumers make proper product selections and lower their cognitive dissonance after the purchase. 107. Labels that contain standard promotional claims like “new,” “improved,” and “super” are examples of _____ labels. A. descriptive B. functional C. repositioning D. informational E. persuasive Answer: E Rationale: Persuasive labels primarily focus on a promotional theme or logo and try to sell the customer on a new or changed feature. 108. Labeling that is designed to help consumers make proper product selections and lower their cognitive dissonance after the purchase is referred to as: A. informational labeling. B. persuasive labeling. C. cognitive labeling. D. rational labeling. E. restrictive labeling. Answer: A Rationale: This is the definition of informational labeling. 109. Statements on Kashi cereals and breakfast bars stating that they have no artificial colors, flavors, or preservatives would most likely be an example of _____ labeling. A. primary B. required C. informational D. functional E. environmental Answer: C Rationale: Informational labeling is designed to help consumers make proper product selections. 110. Mia’s new raincoat contains a label that reads “Professional Dry Clean Only.” This is an example of a(n) _____ label. A. data B. union mandated C. persuasive D. informational E. functional Answer: D Rationale: Informational labeling is designed to help consumers in making proper product selections and to lower cognitive dissonance after the purchase. These labels often include care and use information and an explanation of construction standards. 111. Which of the following statements about bar codes is true? A. Bar codes are also called universal pricing codes (UPCs). B. Bar codes were first used in 2000. C. Bar codes can be read by optical scanners. D. Bar codes cannot be used as a marketing research tool. E. All of these statements about bar codes are true. Answer: C Rationale: Bar codes are also called universal product codes. They were first introduced in 1974. Also, bar codes can be used in single-source research. 112. Universal product codes (UPCs) can perform all the following functions EXCEPT: A. allow scanners to match codes with brand names, package sizes, and prices. B. print product and price information on cash register tapes. C. assist retailers in preparing records of customer purchases. D. allow retailers to accurately track sales and control inventories. E. provide the detailed nutritional information required by the FDA. Answer: E Rationale: Universal product codes do not contain nutritional information. 113. Advantages of this global branding strategy include greater identification of the product from market to market and ease of coordinating promotion from market to market. A. adaptation B. one brand name everywhere C. piggybacking D. different brand names in different markets E. product modification Answer: B Rationale: This strategy is useful when the company markets mainly one product and the brand name does not have any negative connotations in the local market. 114. Green is a sign of danger in some countries, so a U.S. firm that sells its goods in those countries would probably want to avoid using green on its packaging. Which aspect of packaging is addressed by this issue? A. convenience B. safety C. descriptive labeling D. aesthetics E. informational labeling Answer: D Rationale: Cultural traits in certain countries require attention to package aesthetics. 115. Three aspects of packaging that are especially important in international marketing are labeling, aesthetics, and: A. price. B. length of distribution channel. C. counterfeiting potential. D. warranties. E. climate considerations. Answer: E Rationale: Extreme climates and long-distance shipping necessitate sturdier and more durable packages for goods sold overseas. 116. Which type of warranty is a written guarantee? A. Complete B. Express C. Implied D. Explicit E. Primary Answer: B Rationale: This is the definition of an express warranty. 117. Well-known retailer Lands’ End, offers a written satisfaction guarantee on all merchandise it sells. In other words, Lands’ End gives its customers a(n) _____ warranty. A. implied B. descriptive C. limited D. full E. express Answer: E Rationale: An express warranty is a written guarantee. 118. The label on Darwin’s new blazer that reads “Will Not Shrink” is an example of a(n) _____ warranty. A. descriptive B. limited C. implied D. express E. superfluous Answer: D Rationale: An express warranty is any written guarantee. 119. The statement on a toothpaste box that brushing twice a day with the paste will remove 90 percent of all tartar from the user’s teeth is an example of a(n): A. implied warranty. B. functional label. C. UPC disclaimer. D. express warranty. E. universal warranty. Answer: D Rationale: An express warranty is any written guarantee. 120. Shane had every reason to believe that the antacid he purchased would relieve his stomach problems if he followed the directions on the box. This is an example of a(n): A. implied warranty. B. functional label. C. persuasive label. D. express warranty. E. universal warranty. Answer: A Rationale: An implied warranty is an unwritten guarantee that the good or service is fit for the purpose for which it was sold. All sales have an implied warranty under the Uniform Commercial Code. 121. Under the _____, all sales have an implied warranty. A. Label Law B. Lanham Act C. Bill of Rights D. Good Housekeeping Seal E. Uniform Commercial Code Answer: E Rationale: The Uniform Commercial Code regulates all sales. Campbell’s Most people recognize Campbell’s as a brand of soup, but Campbell’s also manufactures Pace Picante Sauce, Pepperidge Farms products, Prego Sauce, Swanson Broth, and V8 juice. Campbell’s International division offers more than 20 brands of soups, sauces, beverages, and baked snacks in the Asia Pacific region, Greater Europe and Latin America. Products such as Arnott’s biscuits and crackers and Bià Band soups are well known overseas. Campbell’s products are sold in more than 120 countries. 122. Refer to Campbell’s. Marco likes to pack a V8 juice in his lunch bag every day. Based on its intended use, Marco’s V8 juice could best be classified as a _____ product. A. industrial B. consumer C. business D. convenience E. unsought Answer: B Rationale: A consumer product is purchased to satisfy an individual’s personal wants. 123. Refer to Campbell’s. Campbell’s Creamy Chicken Soup could be called a: A. product mix. B. product item. C. product depth. D. product width. E. product consistency. Answer: B Rationale: A product item is a specific version of a product that can be distinguished as a distinct offering among Campbell’s products. 124. Refer to Campbell’s. All the products that Campbell’s sells—both domestically and worldwide—are the company’s product: A. item. B. depth. C. Width. D. mix. E. marketing consistency. Answer: D Rationale: The organization’s product mix includes all the products it sells. 125. Refer to Campbell’s. The soups, juices, broths, sauces, and baked goods sold by Campbell’s compose the company’s product mix _____. A. depth B. length C. width D. reliability E. consistency Answer: C Rationale: Product mix width refers to the number of product lines that an organization offers. 126. Refer to Campbell’s. If a marketer looks only at the canned soups produced by Campbell’s and notes the brands and items only within that category, he or she is analyzing the _____ of the product line. A. width B. length C. depth D. levels E. consistency Answer: C Rationale: Product line depth is the number of product items in a product line. 127. Refer to Campbell’s. Of the following what is probably Campbell’s biggest labeling concern in its international markets? A. printing accurate warranties on each label B. correctly affixing the UPC code on each label C. eliminating any persuasive labeling as such labels are not effective on the international market D. making sure the labels are aesthetically pleasing E. translating ingredient, promotional, and instructional information Answer: E Rationale: Translating ingredients, promotional and instruction information are major labeling concerns. Choice Homes, Inc. Choice Homes, Inc., based in Texas, built its reputation by building high-quality new homes, often at prices less than “used” ones. Choice Homes used mass purchasing power, innovative high-tech communications among employees to coordinate the building process, and other entrepreneurial strategies to hold costs down. Its homes are primarily targeted at first-time homeowners-to-be in an effort to get people out of apartments so they can experience the “American Dream” of home ownership for the same price as rent. Choice Homes later developed the “Choice Classic” brand name for higher-end homes targeted at wealthier dual-income couples. The Choice Classic name was designed to capitalize on the reputation of Choice Homes, especially for current Choice home owners who are ready to “move up” to a larger home. Recently, Choice Homes started a third line of homes called “Reflections,” which are expensive, custom-designed homes located in exclusive, prestigious neighborhoods. 128. Refer to Choice Homes, Inc. Kurt McKinney has just received a large inheritance and wants to have his “dream” home built. He knows exactly the architectural design he wants. He wants a particular Reflections home. The type of consumer product he wishes to buy is best classified as a(n): A. shopping product. B. convenience product. C. nondurable good. D. specialty product. E. unsought product. Answer: D Rationale: McKinney is searching for an expensive item for which he will not accept a substitute. 129. Refer to Choice Homes, Inc. The three different brands of homes (Choice, Choice Classic, and Reflections) represent the firm’s: A. line portfolio. B. mix depth. C. line breadth. D. product line. E. product itemization. Answer: D Rationale: A group of closely related product items is a product line. 130. Refer to Choice Homes, Inc. Choice Homes is considering acquiring a mortgage and title company to add to its marketing power in selling homes. This acquisition would expand the firm’s: A. product mix width. B. line breadth. C. product item width. D. product line length. E. product breadth. Answer: A Rationale: By adding more product lines, Choice is expanding its product mix width. 131. Refer to Choice Homes, Inc. Choice Homes is considering the development of a new category of small homes designed to replace low-income housing in the inner city. This strategy represents a: A. quality modification. B. product line extension. C. style modification. D. product line contraction. E. functional modification. Answer: B Rationale: Modifications are for existing product lines and items. A new line represents an extension. 132. Refer to Choice Homes, Inc. In a recent survey, Choice Homes’ customers indicated that they thought their homes were of high quality, made lots of referrals to friends, and planned to buy a Choice Classic or Reflections home in the future. The customers’ reactions indicate that Choice Homes has achieved: A. generic status. B. private brand status. C. brand equity. D. superficial skills. E. brand superiority. Answer: C Rationale: Brand equity indicates the brand has value because of its high awareness, perceived quality, and brand loyalty among customers. Hardie Siding Products James Hardie began selling fiber cement siding products in the United States in 1989 to leading builders, lumberyards, and home improvement centers. Even though ads guaranteed the product would not rot or crack for 50 years, many builders hated the product. It was heavy to install, and it showed every flaw in a bad framing job. In 1997, Hardie decided to run ads in traditional lifestyle magazines and emphasize the emotional appeal of houses made with strong, weather-resistant materials. Soon consumers began asking their builders or remodelers to use the product. Trade ads were used to explain how builders could take advantage of the interest created by the ads in lifestyle magazines. By 2000, the HardiePlank was the number one brand of siding in North America. James Hardie is now the third most recognized brand of building material in the world. 133. Refer to Hardie Siding Products. HardiePlank siding is best described as a(n) _____ product. A. business B. specialty C. unsought D. shopping E. convenience Answer: A Rationale: This product is used to manufacture houses. Consumers are not purchasing it directly; contractors (businesses) purchase the siding to use on clients’ projects. 134. Refer to Hardie Siding Products. HardiePlank siding is an example of a product: A. line. B. mix. C. reference point. D. item. E. standardization. Answer: D Rationale: HardiePlank is a specific item made by the company. 135. Refer to Hardie Siding Products. James Hardie now makes multiple types of fiber cement siding products. There are different widths, textures, and profiles. The different types of siding the company makes are called its product: A. line. B. mix. C. reference point. D. item. E. standardization. Answer: A Rationale: The siding is a group of closely related products. 136. Refer to Hardie Siding Products. By changing the focus of its promotion from the building industry to home owners who saw their homes as sources of security, warmth, and stability, Hardie Siding used a _____ strategy. A. market penetration B. repositioning C. product penetration D. harvesting E. divestment Answer: B Rationale: The company repositioned their product in order to change consumers’ perceptions of the brand. 137. Refer to Hardie Siding Products. Since all of the siding products sold by Hardie are sold under the James Hardie brand name, it is clear that the company uses a _____ strategy. A. family branding B. private branding C. business branding D. co-branding E. repositioning Answer: A Rationale: A company using family branding markets several different products under the same brand name. 138. Refer to Hardie Siding Products. What type of warranty is James Hardie offering in its promotions? A. Implied B. Responsive C. Intangible D. Express E. Valid Answer: D Rationale: There is a written guarantee in the ads. Nestlé When consumers think of Nestlé, they probably think of chocolate. Historically, though, Nestlé’s confectionary business is its weakest area. Based in Switzerland, it is the world’s largest food company, with a brand arsenal of Nescafé, Jenny Craig, Perrier, Purina, and PowerBar, just to name a few of its 30 product lines. Nestlé is hoping to become the “world’s leading health, nutrition, and wellness firm” by spending billions of dollars on research and development of functional foods––foods that have pharmaceutical-like capabilities to enhance energy and heart, bone, gut, and other health. Nestlé wants consumers to see chocolate as a pharmaceutical product rather than just a treat. Part of this new focus includes streamlining the product mix by selling underperforming items or lines that do not fit its new direction. Critics claim that this new focus could hurt the company’s existing brands if the new products fail. Additionally, it may be an uphill battle convincing consumers that a company known for indulgence is now a wellness company. 139. Refer to Nestlé. Some of Nestlé’s products include bottled water, candy, and coffee, which would be classified as what type of consumer products? A. Heterogeneous shopping products B. Homogeneous shopping products C. Convenience products D. Specialty products E. Unsought products Answer: C Rationale: Convenience products are relatively inexpensive items that merit little shopping effort. 140. Refer to Nestlé. Nestlé’s quest is to make some of its existing products more nutritious and provide a health benefit. This is an example of which type of product modification? A. Style B. Extensive C. Brand D. Ingredient E. Functional Answer: E Rationale: Functional modifications involve changes in a product’s versatility, effectiveness, convenience, or safety. Nestlé is attempting to make its existing products more versatile and effective. 141. Refer to Nestlé. After developing these new, functional products, Nestlé will have to change consumers’ perceptions of its brands because it is known for indulgence, not wellness. Nestlé will have to _____ itself. A. reinvent B. reposition C. rename D. modify E. standardize Answer: B Rationale: Repositioning involves changing consumers’ perceptions of a brand. 142. Refer to Nestlé. What adjustment to its product mix is Nestlé doing when it removes underperforming items from the market? A. Product line extension B. Product line contraction C. Product pruning D. Product line retraction E. Functional modification Answer: B Rationale: Product line contractions involve removing items from the product mix. 143. Refer to Nestlé. One new product Nestlé intends to introduce includes fiber added to chocolate. The packages will include the fact that each chocolate bar contains 5 grams of fiber and the various health benefits of including more fiber in one’s diet. Which form of labeling will this perform? A. Persuasive B. Informational C. Regulatory D. Competitive E. Perceptive Answer: B Rationale: The two forms of labeling are persuasive and informational. This is an example of informational labeling, which is designed to help consumers make proper product selections and lower their cognitive dissonance after the purchase. ESSAY 1. How would a marketer define the term product? Answer: A product is everything, both favorable and unfavorable, that a consumer receives in an exchange. A product may be a tangible good, a service, an idea, or any combination of these three. 2. Products can be classified as either business/industrial or consumer products. Explain what distinguishes business products from consumer products and why it is important to understand this distinction. Answer: A product’s classification depends on the buyer’s intentions. Business products are purchased (1) for use in the manufacture of other goods or services, (2) to facilitate an organization’s operations, or (3) to resell the products to other customers. Consumer products are purchased to satisfy an individual’s personal wants. The same item can be classified as either a business or a consumer product depending on the intended use. It is important to know about these product classifications because business and consumer products are marketed differently. These two target markets require different distribution, promotion, and pricing strategies. 3. What is the difference between a homogeneous shopping product and a heterogeneous shopping product? Give specific examples of products that fit into each category to help illustrate your answer. Answer: HOMOGENEOUS SHOPPING PRODUCTS are those that consumers see as being basically the same. The main purpose of shopping is to find the lowest-priced product that has the required features. Appliances and televisions are homogeneous shopping products for many people. HETEROGENEOUS SHOPPING PRODUCTS are those that consumers see as different in terms of quality, style, suitability, and lifestyle compatibility. Comparisons among heterogeneous shopping products are difficult because the alternatives possess unique features as well as different prices and quality levels. These products may include furniture, clothing, housing, and the choice of a university. 4. Name and briefly define the four categories of consumer products. For each category, list three specific examples of products that would most likely be classified in that category. Answer: CONVENIENCE PRODUCTS are relatively inexpensive items that require little shopping effort. The products are bought regularly, usually without significant planning. Convenience products may include candy, soft drinks, aspirin, small hardware items, dry cleaning, car wash services, and so on. SHOPPING PRODUCTS are usually more expensive than convenience products and are found in fewer stores. Consumers spend some effort comparing brands and stores. Shopping products may include washers, dryers, refrigerators, televisions, furniture, clothing, housing, choice of university, and so on. SPECIALTY PRODUCTS are those exclusive items for which consumers are willing to search extensively. Consumers are extremely reluctant to accept substitutes for specialty products. Brand names and service quality are important. Fine watches, luxury cars, expensive stereo equipment, gourmet restaurants, and specialized medical services could all be considered specialty products. UNSOUGHT PRODUCTS are those that the buyer does not know about or does not actively seek to buy. These products include insurance, burial plots, and other similar items. 5. The Crayola brand is currently placed on a wide variety of products, including crayons (standard and fluorescent colors packaged in a wide variety of box sizes), markers (regular and washable), paints (water color and acrylic), scissors, glue, and children’s clothing. Using Crayola as an example, describe the concepts of product item, product line, and product mix. Answer: A PRODUCT ITEM is a specific version of a product that can be designated as a distinct offering among an organization’s products. Any single product identified in the Crayola example could be an item. Examples include a crayon package with 64 colors and a sharpener, an eight-pack of washable markers, and so on. A PRODUCT LINE is a group of closely related products offered by the organization. One could identify two Crayola product lines: arts and crafts, and clothing. Alternatively, three product lines could be identified: clothing, drawing/painting items, and supplies. Finally, as many as six product lines could be interpreted: crayons, markers, paints, scissors, glue, and clothing. A PRODUCT MIX includes all of the products and item varieties that the company offers for sale. All Crayola items and package variations identified above make up Crayola’s product mix. 6. Binney & Smith’s Crayola makes various products, all targeted toward elementary schoolchildren. Drawing, painting, and supply items are sold at convenience stores, toy stores, and grocery stores. Clothing is sold in department stores. Crayola might arrange its product items in the following way: What is the product mix width for Crayola? What is its product line depth? Answer: Product mix width refers to the number of product lines that an organization offers. In this case, the width of the product mix is three. Product line depth is the number of product items in a product line. There are seven items in the drawing/painting line, five items in the supplies line, and three items in the clothing line. 7. Discuss the benefits for organizations by organizing related items into product lines. Answer: ADVERTISING ECONOMIES. Product lines provide economies of scale because several products can be advertised under the umbrella of the line. Additionally, advertising a product line can enhance the corporate name. PACKAGE UNIFORMITIES. All packages in the line may have a common look without causing any product item to lose its individual identity. Thus, one item in a line can advertise another. STANDARDIZED COMPONENTS. Product lines provide an opportunity to standardize components, thus reducing manufacturing and inventory costs. EFFICIENT SALES AND DISTRIBUTION. Product lines enable sales personnel to provide a full range of product alternatives to their customers. Distributors and retailers are often more willing to stock the company’s products if a full line is offered. Transportation and warehousing costs are also likely to be less for product lines than for a collection of individual items. EQUIVALENT QUALITY. A brand name symbolizes a certain quality level to buyers. Purchasers expect all products in a line to be of comparable quality, so further evaluation of individual product items need not take place. 8. Compare and contrast the three types of product modifications and give an example of how each could be implemented. Answer: QUALITY MODIFICATIONS entail changing the product’s dependability or durability. A company could reduce the product quality of some of its offerings to enable a lower price, which would appeal to a broader or price-sensitive target market. It could also increase quality, which would give the corporation a competitive advantage, encourage brand loyalty, allow higher prices, or provide new market segmentation opportunities. FUNCTIONAL MODIFICATIONS are changes in a product’s versatility, effectiveness, convenience, or safety. A company could add further capabilities to its patio furniture such as making it mildew resistant, designing it to fold and store easily, and adding wider seats for greater comfort. The furniture could also add a safety feature to prevent unfolded furniture from collapsing when someone sits in it. STYLE MODIFICATIONS are aesthetic product changes. A company could make its product more fashionable. 9. What is planned obsolescence? Give at least two examples. Is planned obsolescence ethical, or does it serve a function? Explain your answer. Answer: Planned obsolescence is the practice of modifying products so those that have already been sold become obsolete before they actually need replacement. Examples include products such as printers, computers, and cell phones, which often become outdated before they stop working and need to be replaced. Students’ arguments about the ethics of planned obsolescence will vary. Arguments could be made that the practice is unethical because it uses up resources to make changes to a product that are not of great value to the customer. Arguments could also be made that planned obsolescence simply gives customers what they want (e.g., changes the appearance of goods such as autos) and that it drives technological advances and speeds innovation. 10. Explain the differences among the terms brand, brand name, brand mark, trademark, and service mark. Answer: A brand is a name, term, symbol, design, or combination of these that identifies a seller’s products and differentiates them from competitors’ products. A brand name is that part of a brand that can be spoken. The brand mark is the part of the brand that cannot be spoken––a picture, logo, special decoration, or design. A trademark is a legal term indicating the owner has the exclusive right to use a brand or part of a brand; others are prohibited from using the brand without permission. A service mark performs the same functions as a trademark but is reserved for services. 11. What purposes does branding serve? Answer: Branding has three main purposes: identification, repeat sales, and new-product sales. In terms of identification, branding allows marketers to distinguish their products from all others. Brands become familiar to consumers as indicators of quality. Brand equity refers to the value of a well-recognized brand name. Repeat sales come from satisfied customers who recognize which brands to purchase again and which brands to avoid. Branding also helps word-of-mouth and mass media promotion. Brand loyalty occurs when a consumer consistently prefers one brand over all others. The third main purpose of branding is to facilitate new-product sales. Brands with high equity are extremely useful when introducing new products. 12. Distinguish between manufacturer’s brands and private brands. From the reseller’s perspective, what are they key advantages of carrying each type of brand? Answer: A manufacturer’s brand (also sometimes called a national brand) is the brand name of a manufacturer. A private brand (also sometimes called a private label or store brand) is a brand name owned by a wholesaler or retailer. The key advantages of carrying manufacturer’s brands include: • Heavy advertising to the consumer by manufacturers helps develop strong consumer loyalties. • Well-known manufacturer’s brands can attract new customers and enhance the dealer’s prestige. • Many manufacturers offer rapid delivery, allowing the dealer to carry less inventory. • If a dealer sells a manufacturer’s brand that is of poor quality, the customer may simply switch brands but remain loyal to the dealer. The key advantages of carrying private brands include: • A dealer can usually earn higher profits on its own brand, and there is less pressure to mark prices down to meet competition. • Manufacturers can decide to drop a brand or reseller at any time or even to become a direct competitor to its dealers. • Private brands tie the customer to the reseller. • Resellers have no control over the intensity of distribution of manufacturer’s brands. 13. Packaging is an important component of the product strategy. List and briefly describe the four major functions of packaging. Answer: CONTAIN AND PROTECT PRODUCTS. Packaging contains products in liquid, granular, or other divisible forms. Packaging allows firms to market products in specific quantities. Physical protection of the product is also necessary because most products are handled often between manufacture and final consumption. Packages protect products from breakage, light, heat, cold, infestation, spoilage, and so on. PROMOTE PRODUCTS. A key role of packaging is promotion. Labels not only provide brand identification, ingredient listings, feature specifications, and directions but also use designs, colors, shapes, and materials to influence consumers’ perceptions and buying behavior. A package differentiates a product from competing products and may associate a new product with a family of other products from the same manufacturer. Packages are the last opportunity marketers have to influence buyers before they make purchase decisions. This promotional role differentiates the product from competitors’ products and establishes the brand’s image. FACILITATE STORAGE, USE, AND CONVENIENCE. Packages can make products easier to ship, store, and stock on shelves. Additionally, package features or designs can make packages convenient. Features such as disposable packages, easy opening devices, and reusability are a plus. Package size can also be used to segment markets. FACILITATE RECYCLING AND REDUCE ENVIRONMENTAL DAMAGE. Environmental compatibility is preferred, if not demanded, by consumers. Packages should be recyclable, biodegradable, and reusable. 14. Name and describe the two labeling forms. Answer: PERSUASIVE LABELING focuses on a promotional theme or logo, with information for the consumer of secondary importance. The promotional theme may contain the words “new,” “improved,” or “super.” INFORMATIONAL LABELING is designed to help consumers make proper product selections and lower postpurchase cognitive dissonance. These labels may explain construction standards, color selection, cleaning instructions, or other use information. 15. What is a universal product code (UPC)? How does the use of UPCs benefit retailers? Answer: A universal product code—often called a bar code—is a series of thick and thin vertical lines, readable by computerized optical scanners that represent numbers used to track products. They print information on cash register tapes and help retailers rapidly and accurately prepare records of customer purchases, control inventories, and track sales. 16. You are the U.S. brand manager of the Fluffit family of products. The products include a wide variety of synthetic and natural stuffing materials for furniture cushions, sleeping bags, quilts, and winter coats. The brand is a favorite among home repair enthusiasts and hobbyists of various kinds. Your firm would like to enter several foreign markets. Name and describe the three major alternative brand name choices for this global strategy. Discuss the viability of each alternative for Fluffit. Answer: ONE BRAND NAME EVERYWHERE. This strategy is useful when the company markets mainly one product and the brand name does not have negative connotations in any local market. Advantages of a one-brand strategy are greater identification of the product from market to market and ease of coordinating promotion from market to market. This strategy may be difficult for Fluffit because the variety of products are sold. Additionally, it is not likely that “fluff” carries the same meaning in all languages or communicates the benefits of the product. ADAPTATIONS AND MODIFICATIONS. If the Fluffit name is not pronounceable in the local language, the brand name is owned by someone else, or the brand has a negative connotation in the local language, minor modifications can make the brand name more suitable. This could be a viable alternative for Fluffit. DIFFERENT BRAND NAMES IN DIFFERENT MARKETS. Local brand names are often used when translation or pronunciation problems occur, when the marketer wants the brand to appear to be a local brand, or when regulations require localization. This could also be a viable alternative for Fluffit. 17. Compare and contrast an express warranty and an implied warranty. Answer: An express warranty is made in writing and may range from a simple statement to an extensive document. An implied warranty is an unwritten guarantee that the good or service is fit for the purpose for which it was sold. All sales have an implied warranty under the Uniform Commercial Code. Chapter 11—Developing and Managing Products TRUE/FALSE 1. A product must be a discontinuous innovation to be considered a “new” product. Answer: False Rationale: There are six categories of new products, only one of which is new-to-the-world discontinuous innovative products. 2. Central Bark is a dog day care center. When you check your dog into a Central Bark you know your pooch will be pampered. Since kennels have been boarding dogs for years, the dog day care is not really a new product. Answer: False Rationale: There are many types of new product ranging from new-to-the-world products to low-price products. 3. The development of Coke Zero by Coca Cola would not be considered a new product since the company was already making soft drinks. It would simply be called a line extension. Answer: False Rationale: Line extensions are considered to be a type of new product. Coke Zero is an example of an addition to an existing product line. 4. Very few firms follow a formal new-product development process. Answer: False Rationale: Most companies follow a formal new-product development process. 5. A firm’s new-product strategy links the new-product development process with the objectives of the marketing departments, the business unit, and the corporation. Answer: True Rationale: A firm’s new-product strategy aligns the new-product development process with marketing department, business unit, and corporate objectives. 6. For a firm that adheres to the product concept, the logical place to start generating new-product ideas is with the customers of the firm. Answer: False Rationale: Firms adhering the marketing concept—not the product content—logically start generating new-product ideas from customers. 7. The two techniques considered most useful for generating new-product ideas are customer-generation and competitive imitation. Answer: False Rationale: The two techniques considered most useful for generating new-product ideas are brainstorming and focus group exercises. 8. Seven employees from a company are trying to come up with ideas for new-product development. The employees are all in a room together, shouting out potential ideas. The ideas are written down and evaluated as they are generated. This is an example of brainstorming. Answer: False Rationale: Brainstorming does not involve evaluation of the ideas as they are generated; criticism of any kind is avoided. 9. The first filter in the product development process is called screening. Answer: True Rationale: The initial stage of filtering ideas in the product development process is called screening. 10. It is considered inefficient to use concept tests during the idea screening stage of the new-product development process. Answer: False Rationale: During the idea screening stage of the new-product development process, it is appropriate to use concept tests. 11. Concept tests are considered fairly good predictors of success for line extensions and for products that do not require major changes in consumer behavior. Answer: True Rationale: Concept tests are indeed useful for predicting success, especially for line extensions and products with minimal changes in consumer behavior because they gauge initial consumer reactions. 12. Andersen Electronics is currently engrossed in its first estimations of costs and revenues for a new amusement park video skill game. This firm is engaged in a new-product business analysis. Answer: True Rationale: Andersen Electronics' activities align with the initial stages of new-product business analysis, focusing on estimating costs and revenues for their new amusement park video skill game. 13. Without the Internet, it would be impossible to conduct simultaneous product development from different parts of the world. Answer: True Rationale: The Internet facilitates simultaneous product development across different locations, enabling real-time collaboration and communication that would be otherwise impossible without it. 14. A test market is one example of a focus group test where consumers are encouraged to try a new product and provide feedback to the manufacturer. Answer: False Rationale: A test market is not a focus group. 15. One benefit of test marketing is that the marketing activity is shielded from competitors. Answer: False Rationale: Test marketing may result in a serious problem if a firm’s competitors find out about the new product before it has been fully introduced. 16. The Internet offers a fast, cost-effective way to conduct test marketing. Answer: True Rationale: The Internet provides a rapid and cost-effective platform for conducting test marketing, allowing businesses to reach diverse audiences quickly and gather feedback efficiently. 17. The decision to market a product is called commercialization. Answer: True Rationale: Commercialization refers to the process of launching and promoting a product in the market, encompassing decisions and actions taken to bring a product to market successfully. 18. Increasing globalization of markets and competition provides a reason for multinational firms to consider new-product development from a worldwide perspective. Answer: True Rationale: Globalization increases market access and competition, compelling multinational firms to consider global perspectives in new-product development to meet diverse market needs efficiently. 19. Adaptation is the process by which the adoption of an innovation spreads. Answer: False Rationale: This process is referred to as diffusion. 20. Martina loves her new Kindle and has tweeted to her followers several times about how great the product is. This is an example of communication directly from the marketer to potential adopters. Answer: False Rationale: This is an example of word-of-mouth communication among consumers. 21. Consumers described as laggards tend to rely on group norms when deciding to adopt an innovation. Answer: False Rationale: Laggards do not rely on group norms, and their independence is rooted in their ties to tradition. 22. The Sumatra Coffee Company has two new products. One is a simple hand-cranked coffee grinder. The other is a combination grinder and brewer that also can be used to dispense hot milk. Of these two, the hand-cranked grinder will probably be diffused more quickly because it is less complex. Answer: True Rationale: Simple products like the hand-cranked coffee grinder tend to diffuse more quickly because they are easier to understand and use compared to complex products like the combination grinder and brewer. 23. Procter & Gamble introduced a new fabric softener dispenser that consumers stick to the inside of the dryer drum and sent consumers free samples in the mail. This strategy likely increased the rate of adoption of the new product. Answer: True Rationale: This will encourage trial, which can increase the rate of adoption. 24. The product life cycle is a biological metaphor that traces the stages of a product’s acceptance, from its introduction (birth) to its decline (death). Answer: True Rationale: The product life cycle metaphorically represents the stages of a product's market acceptance, from introduction through growth, maturity, and decline. 25. Messages directed directly from marketers toward early adopters should normally use different appeals than messages directed toward the early majority, the late majority, or the laggards. Answer: True Rationale: Marketers should tailor messages differently for early adopters, early majority, late majority, and laggards based on their differing motivations and readiness to adopt new products. 26. The growth stage is the first stage of the product life cycle. Answer: False Rationale: The introductory stage is the first stage. 27. Increasing competition, aggressive brand advertising, and healthy profits are all characteristics found in the introductory stage of the product life cycle. Answer: False Rationale: This describes the growth stage. 28. Profits typically reach their peak during the growth stage of the product life cycle. Answer: True Rationale: Profits typically peak during the growth stage of the product life cycle as sales increase rapidly while marketing and production costs remain relatively stable. 29. A VCR player is a good example of a product in the maturity stage of the product life cycle. Answer: False Rationale: VCR players are in the decline stage, not the maturity stage. 30. The concept of “organized abandonment” is based upon a periodic audit of all goods and services that a firm markets. Answer: True Rationale: Organized abandonment involves periodically evaluating all goods and services a firm offers to discontinue those that are no longer profitable or relevant. 31. The product life cycle concept encourages marketing managers to act reactively. Answer: False Rationale: The product life cycle concept encourages marketing managers to plan so that they can take the initiative instead of reacting to past events. MULTIPLE CHOICE 1. New products are important to a company for all the following reasons EXCEPT: A. increased revenues B. immediate profits C. sustained growth D. replacement of obsolete items E. increased profits Answer: B Rationale: While new products bring the potential advantage of increased margins, they do not bring immediate profit. 2. New-to-the-world products, where the product category itself is new, are also called: A. discontinuous innovations B. moderate innovations C. slow-diffusing products D. venture products E. creative offerings Answer: A Rationale: These products create an entirely new market. 3. Sony has developed a wireless speaker that sits unobtrusively on a desktop. The tiny yet strong twin speakers use Bluetooth to stream music directly from a PC or MP3 player. This small Sony wireless speaker, which is radically different from anything currently on the market, would be classified as a(n): A. discontinuous innovation B. slow-diffusing product C. episodic innovation D. venture product E. sporadic innovation Answer: A Rationale: New-to-the-world products, where the product category itself is new, are also called discontinuous innovations. 4. Which of the following would be considered a new product? A. A new product line B. A new-to-the-world product C. An addition to an existing product line D. An improvement or revision of an existing product E. All of these choices Answer: E Rationale: A new product is a product new to the world, the market, the producer, the seller, or some combination of these, and there are six categories of new products. 5. Marketers consider all of the following examples of new products EXCEPT: A. a 2010 model year automobile with the same style, parts, and color as the 2009 model. B. the addition of a management consultant service to a company that once specialized in investment management. C. a new design for Reach brand toothbrushes. D. a new flavor of Ben & Jerry’s brand ice cream. E. the development of a less-painful procedure for the treatment of varicose veins. Answer: A Rationale: All these are new products except the new model year with no other changes than the date. 6. When P&G began marketing Tide Free & Gentle, which is dye- and perfume-free, this represented which new product category? A. Repositioning strategy B. New product line C. Addition to existing product line D. Discontinuous innovation E. Revision of existing products Answer: C Rationale: This was a new product that supplemented, but did not replace, a firm’s established line. 7. Heinz is expected to introduce more than 400 new products in the next two years. One of the products is a spicier ketchup aimed at the baby boomer market. The spicier ketchup would be considered what type of new product? A. Discontinuous innovation B. New to market C. New product line D. Lower priced E. Addition to existing product line Answer: E Rationale: Heinz is well established in the ketchup market. The spicier ketchup will be an addition to its existing product line. 8. For years, Diet Dr. Pepper has been considered a diet drink. After declining sales, the company is attempting to present Diet Dr. Pepper as an alternative to having a dessert. This is an example of a _____ strategy. A. discontinuous innovation B. niche C. new-product-line D. retargeting E. repositioning Answer: E Rationale: One way to create innovative products is to target new markets through repositioning. Repositioning is an attempt to change the market’s perception of a product. 9. Apple recently introduced a newly designed iPod nano with Multi-Touch capability, a built-in FM radio, and mix-making Genius. Since it was not trying to reach a new target market, Apple was using which new product strategy? A. Improvement or revision of existing product B. Addition to existing product line C. Discontinuous innovation D. Repositioned product E. New product line Answer: A Rationale: Apple was improving its iPod nano product. 10. When Procter & Gamble wanted to enter the hair-care market, it purchased Wella, a leading German company specializing in hair-care products. What new product category does this acquisition represent for P&G? A. Repositioning strategy B. New product line C. Addition to existing product line D. Discontinuous innovation E. Revision of existing products Answer: B Rationale: New product lines are defined as products that a firm has not previously offered. 11. Oakley, a manufacturer of protective eyewear for various kinds of sports, has introduced sunglasses that play music. The glasses weigh just a few ounces and can hold up to 120 songs. These sunglasses would be an example of a(n): A. repositioning strategy B. new product line C. addition to existing product line D. discontinuous innovation E. revision of existing products Answer: C Rationale: The new sunglasses supplement the firm’s established product line. 12. Dial Corporation has introduced Dial Basics, a premium soap product with an emphasis on value. Dial Basics would be an example of which strategy to create a new product? A. New-to-the-world product B. New product line C. Repositioned product D. Addition to a current product line E. Lower-priced products Answer: B Rationale: Dial Basics is marketed as a value brand which had not previously been offered. 13. _____ occurs when an existing product is targeted toward new market segments. It is another type of new-product development because the product is new to that segment. A. Brainstorming B. Diffusing C. Repositioning D. Screening E. Reciprocity Answer: C Rationale: Repositioned products are existing products targeted at new markets or market segments, or ones repositioned to change the current market’s perception of the product. 14. Companies that are most likely to succeed in the development and introduction of new products typically are characterized by all of the following EXCEPT: A. establishing an environment conducive to achieving new-product objectives. B. making the long-term commitment needed to support innovation and new-product development. C. capitalizing on experience to achieve and maintain competitive advantage. D. using a company-specific approach, driven by corporate objectives and strategies, with a well-defined new-product strategy at its core. E. introducing ten new products every year. Answer: E Rationale: Introducing a specific number of new products each year is not one of the factors identified for success. 15. A(n) _____ links the new-product development process with the objectives of the marketing department, the business unit, and the corporation. A. product-review committee B. concept test C. episodic innovation D. new-product strategy E. product strategy bridge Answer: D Rationale: This is the definition of new-product strategy. 16. The first stage of the new-product development process is: A. screening and concept testing B. establishing the new-product strategy C. exploring opportunities D. developing a business analysis E. the building of a prototype Answer: B Rationale: See Exhibit 11.1. New-product strategy is the first subset of the organization’s new-product development plan. It specifies what roles new products play in the organization’s overall plan and describes goals. 17. A new-product strategy: A. links the new-product development process with the objectives of the marketing department, the business unit, and the corporation B. is part of an organization’s overall marketing strategy C. specifies the roles new products play in the organization’s overall plans D. describes the characteristics of products the organization wants to offer and the markets it wants to serve E. is accurately described by all of these Answer: E Rationale: All of these statements accurately describe a new-product strategy. 18. Which of the following is the LEAST likely source for new-product ideas? A. A company’s distributors B. Its customers C. Its employees D. Its financial lenders E. Its competitors Answer: D Rationale: Customers, employees, distributors, and competitors are interacting with the marketplace and may have ideas for goods to serve customer needs. The bank from which the company gets its debt financing is the least likely alternative here. 19. A manufacturer of office furnishings is finding it difficult to compete with cheaper imported merchandise. Which of the following is a potential source of new-product ideas that would allow it to compete more effectively? A. Current retailers who carry the manufacturer’s equipment B. Its foreign competitors C. The company’s employees D. Customers who have requested its catalogs E. All of these Answer: E Rationale: Customers, employees, distributors, and competitors are interacting with the marketplace and may have ideas for goods to serve customer needs. 20. The process of converting applications for new technologies into marketable products is called: A. basic research B. product modification C. marketing development D. product development E. correlation analysis Answer: D Rationale: Product development is a marketing strategy that entails the creation of marketable new products. 21. _____ is a process where a group thinks of as many ways to vary a product or solve a problem as possible without considering the practicality of the ideas. A. New-product brain dumping B. Screening C. A focus group interview D. Brainstorming E. Diffusion Answer: D Rationale: The goal of brainstorming is to get a group to think of unlimited ways to vary a product or solve a problem, no matter how ridiculous it may seem. 22. Mosaic Labs has developed a chemical compound that prevents mildew in even the most humid climates. The week after the compound was invented, a group of the firm’s employees got together and listed ways the product might be used. This is an example of: A. quality control B. brainstorming C. concept testing D. venture group activities E. screening Answer: B Rationale: Brainstorming is a process where a group thinks of as many ways to vary a product or solve a problem. 23. The same manufacturer produces Hanes, Playtex, and Bali clothing for women. This manufacturer assembled a group of nine regular customers and asked them to discuss what they like and do not like about its current product line for full-figured women and to suggest new-product ideas, such as jewelry, wide shoes, and belts. The manufacturer used a research technique called a: A. focus group B. brainstorming session C. buying center D. product-review committee E. venture team Answer: A Rationale: An objective of focus group interviews is to stimulate insightful comments through group interaction. 24. Which of the following stages of the new-product development process is the first filter, which serves to eliminate new-product ideas that are inconsistent with the organization’s new-product strategy or are obviously inappropriate for some other reason? A. Applied diffusion B. Introductory diffusion C. Business analysis D. Test marketing E. Idea screening Answer: E Rationale: Most new-product ideas are rejected at this stage, called the idea screening stage. 25. At what stage of the new-product development process are most new-product ideas rejected? A. Test marketing B. Diffusion C. Business analysis D. Idea screening E. Idea generation Answer: D Rationale: Screening is the stage when the bulk of the ideas are rejected for being incompatible or impractical. 26. After the research team at Nabisco had generated more than two dozen new-product ideas for a substitute for partially hydrogenated oils that tastes the same but has zero trans fat, the company formed a committee to analyze whether the product ideas were consistent with the organization’s new-product strategy. This is called: A. business analysis B. diffusion C. idea screening D. test marketing E. idea generation Answer: C Rationale: Idea screening is the first filter in the product development process. It serves to eliminate new product ideas that are inconsistent with the organization’s new-product strategy. 27. Top managers at a commercial real estate firm that specializes in site location and installation of cell phone towers considered providing the same service for windmill location and installation but decided this was not within the company’s corporate mission. Which stage of the new-product development process was reached? A. Development B. Idea generation C. Idea screening D. Business analysis E. Customer testing Answer: C Rationale: Screening involves rejecting products that don’t fit with company plans. 28. A European dairy that is famous for its rich ice creams has introduced basil-, tomato-, fennel-, and oregano-flavored ice creams. In the process of developing these new flavors, the dairy surveyed world-famous chefs to determine which nontraditional flavors had the greatest level of marketability. This survey would have happened during which stage of the product development process? A. Concept testing B. Brainstorming C. Basic research D. Idea screening E. Applied research Answer: D Rationale: The idea screening stage eliminates ideas that are inconsistent with the organization’s new-product strategy or are obviously inappropriate for some other reason. 29. The screening and concept testing stage of the new-product development process is used to: A. eliminate undesirable ideas and predict consumer acceptance B. refine the promotion campaign to be used with the new product C. determine the potential profits to be gained from the new product D. determine how long the test market should be run E. generate new-product ideas Answer: A Rationale: Concept tests involve describing the product idea to consumers, usually before any prototype has been created, and asking for their evaluations. 30. _____ evaluate new-product ideas usually before any prototype has been created. A. Concept tests B. Simulated market tests C. Market tests D. User tests E. Venture analyses Answer: A Rationale: Concepts test are often used at the screening stage to rate concept (or product) alternatives. 31. After Mattel evaluated many new products to add to its Barbie product line and before any prototype was created, the toy manufacturer instructed a committee to select three of the ideas to present to a group of consumers. The consumer group was asked to evaluate the ideas in terms of their marketability. This stage of new-product development is called: A. prototype screening B. concept testing C. market testing D. idea manipulation E. idea diffusing Answer: B Rationale: Concept tests evaluate new-product ideas usually before any prototype has been created. 32. In the _____ stage of new-product development, preliminary demand, cost, sales, and profitability estimates are made. A. prototype screening B. idea generation C. concept testing D. post-brainstorming E. business analysis Answer: E Rationale: New-product ideas that survive the initial screening process move to the business analysis stage, where preliminary figures for demand, cost, sales, and profitability are calculated. 33. Which of the following questions is NOT likely to be asked during the business analysis stage? A. What is the likely demand for the product? B. Which celebrity spokesperson can we hire to promote the product? C. Would current customers benefit from the product? D. What is the risk of failure? E. What new facilities would be needed? Answer: B Rationale: In the business analysis stage, preliminary figures for demand, cost, sales, and profitability are calculated. Finding an appropriate spokesperson is not a concern at this stage. 34. The accuracy of revenue projections for products are affected by: A. the newness of the product. B. the size of the market. C. the nature of the competition. D. all of the above. E. none of the above. Answer: D Rationale: The newness of the product, the size of the market, and the nature of the competition all affect the accuracy of revenue projections. 35. In the _____ stage of new-product development, the technical feasibility of manufacturing the new product is examined as a prototype is produced. A. screening B. development C. market testing D. concept testing E. business analysis Answer: B Rationale: During the development stage, the technical feasibility of manufacturing the product at a reasonable cost is thoroughly examined. This stage can be long and expensive and can involve a great deal of testing. Products are often modified after the testing. 36. A team-oriented approach to new-product development is referred to as: A. simultaneous product development B. synergistic product development C. commercialized product development D. synchronized product development E. parallel product development Answer: A Rationale: The development process works best when all the involved areas (research and development, marketing, engineering, production, and even suppliers) work together rather than sequentially, a process call simultaneous product development. 37. Which of the following statements about simultaneous product development is FALSE? A. Simultaneous product development refers to the practice of developing multiple new products at the same time. B. With simultaneous product development, all relevant functional areas and outside suppliers participate in all stages of the development process. C. All departments within the company work together at the same time, but to maintain a high level of secrecy, no one outside the company is involved in simultaneous product development. D. Simultaneous product development actually increases the length of time it takes to get a product to market and the time the product will stay in the growth stage of its product life cycle. E. The Internet has not yet proven to be a useful tool for implementing simultaneous product development. Answer: E Rationale: The Internet has indeed proven a useful tool for this process. 38. _____ is a limited introduction of a product and a marketing program to determine the reactions of potential customers in a market situation. A. Use testing B. Test marketing C. Concept testing D. Laboratory testing E. Discontinuous innovation Answer: B Rationale: After products and marketing programs have been developed, they are usually tested in the marketplace using test marketing, which is the limited introduction of a product and a marketing program to determine the reactions of potential customers in a market situation. 39. When selecting a test market city, a researcher should look for a city: A. where the demographics and purchasing habits mirror the overall market for the product. B. that has media spillover from other cities to increase media alternatives. C. that is as large as possible. D. that has limited distribution. E. where there is no competition. Answer: A Rationale: Test marketing is a limited introduction of a product and a marketing program to determine the reactions of potential customers in a market situation. The market city should be similar to the overall market for accuracy of results. 40. A(n) _____ typically entails showing members of the product’s target market advertising and other promotional materials for several products, including the test product. These people are then taken to a mock or real store, where their purchases are recorded. A. adoption-rate test B. real test market C. concept test D. simulated (laboratory) market test E. consumer juried test Answer: D Rationale: This is a description of a simulated (laboratory) market test. 41. The maker of Purell hand sanitizer tested consumers’ reactions to its Purell Sanitizing Wipes by getting consumers to look through a newspaper with grocery store ads, make out a grocery list, and then “shop” in a mock store filled with real products, including the new product. This is an example of a(n): A. adoption-rate test B. simulated (laboratory) market test C. concept test D. use test E. juried consumer test Answer: B Rationale: Simulated (laboratory) market tests typically entail showing members of the target market advertising for a variety of products and then monitoring purchase behavior in a mock or real store. 42. Which of the following has been found to be an efficient substitute for traditional methods of conducting test markets? A. Product sampling B. Simulated product adoption C. Online test marketing D. Simulated diffusion E. Market concentration Answer: C Rationale: The text describes Proctor & Gamble’s online test marketing. 43. The final stage in the new-product development process is: A. product testing B. market testing C. commercialization D. product prototyping E. simulated marketing Answer: C Rationale: Commercialization is the decision to market a product. 44. During the _____ stage of the new-product development process, production starts, inventories are built up, the product is shipped to distribution points, the sales force is trained, and advertising and promotion begin. A. commercialization B. product testing C. market analysis D. product prototyping E. simulated marketing Answer: A Rationale: The commercialization stage is the decision to market a product and sets several task in motion. 45. Jeni’s Splendid Ice Creams has decided to put its new Beet Cake with Black Walnut ice cream on the market. This product has entered the _____ stage of new-product development. A. business analysis B. development C. test marketing D. commercialization E. research and development Answer: D Rationale: Commercialization is the decision to market a product. 46. The MOST important factor in successful new-product introduction is: A. a good match between the product and market needs. B. strong company leadership. C. an obsession with quantity rather than quality D. project-based team approach to new-product development. E. history of carefully listening to customers. Answer: A Rationale: All of the factors listed are important, but the most important is a good match between the product and market needs—as the marketing concept would predict. 47. The main goal of the global product development process is to: A. come up with a single standard product or product line suitable for all global markets B. expand the number of different new-product offerings so that individual country needs are more likely to be met C. develop every product for potential worldwide distribution and adaptation to other countries D. come up with new-product ideas in the United States and use global markets as test markets E. repeat the steps in the process with marketing teams in each and every foreign country targeted Answer: C Rationale: Multinational companies develop each product for potential worldwide usage, with minor unique requirements incorporated. 48. A product that is perceived as new by a potential adopter, whether the product is “new to the world” or simply new to the individual, is called a(n): A. innovation B. diffusion C. discontinuous product D. laggard E. simulated new product Answer: A Rationale: An innovation is a product perceived as new by a potential adopter. 49. Roger has lots of nice photos of his family on his digital camera and wishes he could display them on his desk at work without having to print them out. A coworker told him that he could purchase a digital photo frame to download and display his digital photos. Because Roger did not know such a product existed, digital photo frames represented a(n) _____ to him. A. heterogeneous shopping product B. diffusion C. market laggard D. product diversification E. innovation Answer: E Rationale: An innovation is a product perceived as new by a potential adopter. 50. The process by which the adoption of an innovation spreads is referred to as: A. diffusion B. circulation C. transmission D. dissemination E. dispersion Answer: A Rationale: This is the definition of diffusion. 51. The process by which the adoption of 3D HD televisions spreads is an example of: A. test marketing B. diffusion C. product dissemination D. innovative dispersal E. marketing communication Answer: B Rationale: Diffusion is the process by which the adoption of an innovation spreads. 52. _____ are eager to try new ideas and products. They typically have higher incomes, are better educated, and are more cosmopolitan than other categories of adopters. A. Early adopters B. Innovators C. Early majority D. Late majority E. Laggards Answer: B Rationale: These are characteristics of innovators, who are almost obsessed with trying new ideas and products. 53. Kathy purchased an Amazon Kindle e-book reader as soon as they came onto the market. She paid $399 for the product, which she bought the first day it was released. Kathy works in Information Technology and is always looking for new electronic products to buy. She would best be described as a(n): A. early adopter B. early majority C. innovator D. diffusers E. lead consumer Answer: C Rationale: Innovators make up the first 2.5 percent of all those who adopt the product, are well-educated, venturesome, have higher incomes, and are self-confident. 54. _____ embrace products relatively early in the product life cycle, are likely to be community oriented, and often are opinion leaders themselves. A. Early adopters B. Innovators C. Early majority D. Late majority E. Laggards Answer: A Rationale: Although early adopters are not the very first, they do adopt early in the product’s life cycle. 55. Joaquin didn’t buy a netbook computer when they first came out, but he did purchase one a year after they were introduced to the market. He is very active in his church and local arts council, and many of his friends asked him which brand to buy when they were considering purchasing a netbook. Joaquin is best described as a(n): A. innovator B. early adopter C. early majority D. leading consumer E. proactive Answer: B Rationale: Although early adopters are not the very first, they do adopt early in the product’s life cycle. 56. One hot August day, Tom saw a can of AriZona Beverage Company’s Arnold Palmer Half-and-Half iced tea and lemonade drink at the local convenience store. He decided to try the product and found that he really liked it. Now, Tom regularly purchases this product to quench his thirst on hot days. Tom is an example of a(n): A. opt-in buyer. B. innovator. C. adopter. D. luddite. E. laggard. Answer: C Rationale: Adopters are consumers who were sufficiently satisfied with their trial experience with a product to proceed with some further usage. 57. Which of the following adopters is MOST likely to become an opinion leader? A. Luddite B. Early majority C. Early adopter D. Proactive adopter E. Innovator Answer: C Rationale: Early adopters are more likely than innovators to become opinion leaders because of their closer affiliation with groups. 58. A dominant characteristic of the late majority is: A. deliberateness B. skepticism C. respect of others D. cheerfulness E. optimism Answer: B Rationale: The late majority tend to be very skeptical of marketing. 59. All of the following statements about how adopters participate in the diffusion process are true EXCEPT: A. early adopters may also be opinion leaders B. laggards are the last consumers to adopt a new product C. there are three categories of consumers who will adopt an innovation D. the dominant characteristic of members of the late majority is skepticism E. innovators are the first to purchase a new product Answer: C Rationale: There are five categories: innovator, early adopter, early majority, late majority, and laggard. 60. All of the following are categories of adopters in the diffusion process of innovations EXCEPT: A. laggards B. innovators C. early majority D. early adopters E. doubters Answer: E Rationale: The five categories of adopters are innovators, early adopters, early majority, late majority, and laggards. 61. A company has learned that members of its target market are likely to collect more information and evaluate more brands than early adopters. They are cautious and thoughtful and do not try the product without asking someone else. The customers’ dominant characteristic is deliberateness. In terms of the diffusion process, these customers are most likely: A. innovators B. early minority C. laggards D. early majority E. late majority Answer: D Rationale: The early majority will collect more information and evaluate more brands than do early adopters. They rely on friends, neighbors, and opinion leaders for information. 62. Liz tends to adopt new products sooner than a lot of other people, but she weighs the pros and cons before buying. She does considerable product research, but her friends don’t really look to her for information. She really deliberates before she makes a purchase. Liz is best described as a(n): A. innovator B. early adopter C. early majority D. late majority E. laggard Answer: C Rationale: These are characteristics of the early majority category of adopters. 63. The _____ adopt a product because most of their friends have already done so, and their adoption is usually the result of pressure to conform because they rely on group norms. A. laggards B. early adopters C. early majority D. innovators E. late majority Answer: E Rationale: The late majority adopt a product because they are influenced by social norms and pressure to conform, waiting until a product is well-established and widely accepted before adopting it themselves. 64. Which category of adopters adopt a new product because most of their friends have already adopted it? A. Innovators B. Early majority C. Early adopters D. Laggards E. Late majority Answer: E Rationale: This describes the late majority, who tend to rely on group norms, and their adoption stems from pressure to conform. 65. Leon is not very quick to purchase innovative products when they come out, but after a while he breaks down and buys if after most of his friends have bought it. He gets most of his information from them when he decides to buy something. He is very skeptical of marketing, so he trusts his friends more than anything. Leon is best described as a(n): A. innovator B. early adopter C. early majority D. late majority E. laggard Answer: D Rationale: This is characteristic of the late majority, who tend to be influenced by group norms. 66. Among the late majority group, adoption of a product primarily results from: A. word-of-mouth communications. B. print advertising. C. advertising in nontraditional media. D. broadcast advertising. E. sales promotions. Answer: A Rationale: This group attains information about new products primarily from word-of-mouth communication rather than the mass media. 67. In the adopter categories, the final 16 percent to adopt are similar to innovators in that they do not rely on the norms of the group but are independent because they are bound to tradition. They tend to have the lowest socioeconomic status, are suspicious of new products, and are alienated from a rapidly advancing society. They are called: A. generics. B. late majority. C. decliners. D. laggards. E. luddites. Answer: D Rationale: By the time laggards adopt an innovation, it has probably become outmoded and been replaced by something else. 68. Which is the last adopter category to adopt an innovation? A. Late majority B. Laggards C. Late diffusers D. Reluctant adopters E. Traditionalists Answer: B Rationale: Laggards represent the final 16 percent to adopt an innovation. By the time they adopt an innovation, it has probably become outmoded and replaced by something else. 69. All of the following are product characteristics influencing the rate of adoption EXCEPT: A. complexity. B. compatibility. C. relative advantage. D. observability. E. distribution. Answer: E Rationale: While it is a marketing factor that may influence the rate of adoption of a new product, distribution is not a product factor influencing the rate of adoption. 70. Which product factor influencing the rate of adoption represents the degree of difficulty involved in understanding and using a new product? A. Complexity B. Compatibility C. Relative advantage D. Observability E. Trialability Answer: A Rationale: The more complex the product, the slower is its diffusion. 71. Oakley, the manufacturer of protective eyewear for sports enthusiasts, has recently introduced Oakley’s Thump, UV sunglasses that allow the wearer to store and listen to 120 different songs. Which of the following product characteristics would likely exert the greatest influence on the market success of these new glasses? A. Compatibility B. Comparability C. Complexity D. Tangibility E. Competitive advantage Answer: C Rationale: Complexity refers to the degree of difficulty involved in using the new product. 72. Which product characteristic affecting the rate of adoption represents the degree to which the new product is consistent with existing values and product knowledge, past experiences, and current needs? A. Complexity B. Compatibility C. Relative advantage D. Observability E. Trialability Answer: B Rationale: Products that are compatible with others will gain from existing product knowledge, past experiences, and current needs. 73. Jeni’s Splendid Ice Creams is famous for its unusual flavors, such as Wildberry Lavender, Goat Cheese with Red Cherries, and Sweet Corn and Black Raspberries. Which of the following product characteristics would most likely influence the market success of these flavors? A. Compatibility B. Comparability C. Complexity D. Tangibility E. Competitive advantage Answer: A Rationale: Is the ice cream consistent with existing product experiences? 74. When the Segway Human Transporter was introduced in 2002, many people expected the product to be a phenomenal success. While the Segway is still on the market, it has never been the success so many expected. A recent Wall Street Journal article suggested that the Segway, while brilliant technologically, seemed impractical to most people since it could not be used to replace their current method of transportation. In other words, the Segway had problems with: A. compatibility. B. relative advantage. C. observability. D. trialability. E. longevity. Answer: A Rationale: Since the Segway could not replace consumer’s current transportation, it was incompatible with consumers’ transportation needs. 75. Many product failures such as Cucumber antiperspirant spray, Toaster Eggs, and Health-Sea sea-sausages most likely did not succeed because they did not conform to the product characteristic of: A. complexity. B. trialability. C. total advantage. D. observability. E. compatibility. Answer: E Rationale: These products most likely failed because they were not compatible with existing products and because customers could not relate to them. 76. The product characteristic affecting the rate of adoption characterized by the degree to which a product is perceived as superior to existing substitutes is: A. compatibility B. complexity C. relative advantage D. product differentiation E. competitive advantage Answer: C Rationale: The relative advantage is the degree to which a product is perceived to be superior to existing substitutes. 77. Biopure Corp. is developing a human blood substitute called Hemopure. Unlike donated human blood that needs to be refrigerated and expires after a short period of time, Hemopure is shelf stable and can be stored for two years or more. Also, it does not need to be matched to a person’s blood type. These factors make it well-suited for trauma cases where blood must be transfused at an accident site or on the battlefield. Which product characteristic influencing the rate of adoption does this represent? A. Complexity B. Compatibility C. Relative advantage D. Observability E. Trialability Answer: C Rationale: Hemopure is superior to existing substitutes in that it does not need to be matched to blood type and can be stored for a longer period of time. 78. Which product characteristic affecting the rate of adoption of a new product represents the degree to which the benefits or other results of using the product can be communicated to target customers? A. Observability B. Diffusion C. Adoption D. Trialability E. Complexity Answer: A Rationale: If target customers can see the benefits, the rate of adoption will be faster. 79. Razor scooters gained instant popularity because walkers and runners frequently saw people on the scooters zipping by them and having fun. This information suggests the adoption rate of the Razor scooter was most affected by which product characteristic? A. Complexity B. Trialability C. Relative advantage D. Observability E. Compatibility Answer: D Rationale: Observability refers to the degree to which the product benefits can be observed. 80. A new medication to eliminate fungal growths on people’s toenails may have a slow rate of adoption because it is not a highly visible item in homes. Also, this item is not often discussed among friends and coworkers. This is an example of how the characteristic of _____ can work to slow the adoption process. A. relative advantage B. observability C. trialability D. complexity E. compatibility Answer: B Rationale: Observability refers to the degree to which the benefits or other results of using a product can be observed by others and communicated to target customers. 81. The degree to which a product can be used on a limited basis represents which product characteristic influencing the rate of adoption? A. Complexity B. Trialability C. Observability D. Relative advantage E. Sampling Answer: B Rationale: Trialability is the degree to which a product can be tried on a limited basis. 82. Which of the following products has the lowest level of trialability? A. An Internet chat room B. Low-fat ice cream C. Surgically implanted hearing aids D. Organic pet food E. A new air freshener Answer: C Rationale: Trialability is the degree to which the product can be tried on a limited basis. Surgically implanted hearing aids would more likely be a permanent procedure. 83. When Coca-Cola North American debuted Gold Peak iced tea, a spokesperson for the company said, “In looking at the ready-to-drink tea market, we found that the one place that nobody seemed to be playing was in the category of classic iced teas like mother used to make. Gold Peak is a quality product that brings back that classic taste. We’re going to do a lot of sampling with this product because we believe that if people try it, they’re going to love it and stick with it.” In other words, its adoption rate will rely on its: A. compatibility B. comparability C. complexity D. trialability E. competitive advantage Answer: D Rationale: Trialability refers to the degree to which a product can be tried on a limited basis. 84. A biological metaphor that traces the stages of a product’s acceptance, from its introduction (birth) to its decline (death) is called the: A. product diffusion process B. product phase C. product adoption stages D. product evolution cycle E. product life cycle Answer: E Rationale: The product life cycle is a useful marketing management diagnostic tool and a general guide for marketing planning in various “life cycle” stages. 85. The stages of the product life cycle, in order, are: A. introduction, maturity, decline B. introduction, growth, maturity, decline C. growth, maturity, plateau, decline D. innovation, early adoption, late adoption, laggard E. category acceptance, category growth, brand acceptance, brand growth Answer: B Rationale: The four stages of the product life cycle are introduction, growth, maturity, and decline. 86. All the brands that satisfy a particular type of need such as shaving products, laundry detergent, soft drinks, and furniture make up a(n): A. product life cycle stage. B. primary group. C. adopter category. D. product category. E. reference group. Answer: D Rationale: This is the definition of a product category. 87. Microwave ovens, staplers, frozen yogurt, and purses are examples of: A. PLC line items. B. line-extendable categories. C. brand classes. D. brand groupings. E. product categories. Answer: E Rationale: A product category includes all brands that satisfy a particular type of need, such as those listed. 88. A brand of iced tea called Gold Peak has high marketing costs as well as high production costs. Promotions for the product are aimed at gaining distribution and informing consumers that this premium product tastes like its home brewed. In which stage of the product life cycle is Gold Peak iced tea? A. Introductory B. Growth C. Maturity D. Decline E. Saturation Answer: A Rationale: These characteristics illustrate the introductory stage of the product life cycle. 89. The length of the introductory stage of the product life cycle for a new product is largely determined by: A. the size of the product B. the product’s marketing mix C. the efficiency of the test marketing D. product characteristics, such as advantages over substitute products E. the size of the market Answer: D Rationale: The introductory stage of the product life cycle is affected by how customers perceive the characteristics of the product. Other factors that affect the length of the introductory stage are the educational effort required to use the item and management’s resource commitment to the item. 90. The phase of the product life cycle in which healthy profits usually begin to appear is the _____ stage. A. growth B. decline C. introductory D. maturity E. commercialization Answer: A Rationale: Because losses often occur during the introductory stage, profits first appear in the growth stage. 91. During the growth stage of the product life cycle: A. there is little emphasis on the distribution strategy B. demand for the product is limited to high-income consumers C. prices increase due to economies of scale D. there is decreased direct competition E. profits peak Answer: E Rationale: During the growth stage of the product life cycle, profits rise rapidly, reach their peak, and begin declining as competition intensifies. 92. At the beginning of the maturity stage of the product life cycle, sales typically: A. become flat and do not change B. increase at an increasing rate C. decrease at a decreasing rate D. decrease at an increasing rate E. increase at a decreasing rate Answer: E Rationale: In the maturity stage, sales are still increasing, but the rate of increase has slowed down. 93. Normally, the longest stage of the product life cycle is the _____ stage. A. maturity B. growth C. introductory D. decline E. development Answer: A Rationale: Many products stay in this stage of their life cycles and have been for years. 94. In which stage of the product life cycle do marginal competitors start dropping out of the market? A. Introduction B. Growth C. Maturity D. Decline E. Shake-out Answer: C Rationale: In the maturity stage, sales increase at a decreasing rate, and as prices and profits continue to fall, marginal competitors start dropping out of the market. 95. A long-run drop in sales signals the beginning of which stage in the product life cycle? A. Laggard B. Saturation C. Deterioration D. Decline E. Decay Answer: D Rationale: This is a characteristic of the decline stage of the product life cycle. 96. Which of the following products is most likely to be in the decline stage of its product life cycle? A. Cookie mixes B. Wireless mobile phones C. Programmable CD-ROMs D. Funeral caskets E. Prerecorded audiocassette tapes Answer: E Rationale: Sales are falling, and the product is not being supported by marketing efforts even though there are still some cassette tape players in use. 97. The rate of sales decline during the decline stage of the product life cycle is largely governed by: A. the amount of money spent on advertising. B. the number of competitors in the market. C. the intensity of the competition’s marketing efforts. D. the amount of shelf space allotted to the product. E. how rapidly consumer tastes change or substitute products are adopted. Answer: E Rationale: Changing consumer tastes can force a product into decline, particularly if an alternative is available. 98. You are a product manager for a manufacturer of trampolines. You have determined that the trampoline is in the maturity stage of the product life cycle. To keep your product from entering the decline stage, you should: A. use promotion to increase primary demand for trampolines B. abandon peripheral target markets for trampolines C. raise the price of your trampolines because customers are not price sensitive in this stage D. eliminate all unnecessary marketing expense E. promote trampolines heavily to dealers and customers Answer: E Rationale: Review Learning Outcome 5. In the maturity stage of the product life cycle, competition is strong, and market saturation can lead to stagnation or decline. Heavy promotion helps maintain or increase market share by stimulating sales and maintaining visibility among dealers and customers. 99. Washing machines and refrigerators are in the _____ stage of the product life cycle. A. maturity B. growth C. introductory D. decline E. development Answer: A Rationale: Many major household appliances are in this stage of their life cycles and have been for years. 100. Which of the following statements about the product life cycle (PLC) is true? A. The PLC concept encourages marketing managers to think reactively. B. Marketers do not yet understand the relationship between the adopter categories and the stages of the PLC. C. There are as many new buyers in the maturity stage of the PLC as in the growth stage. D. The PLC is especially useful as a predicting tool. E. Just as there are five adopter categories, there are five PLC stages. Answer: D Rationale: The PLC concept encourages marketing managers to think proactively about where a product will go in its life cycle. Fitbit The pedometer is getting kicked up a notch with the new Fitbit device. The Fitbit is a $99 motion-detecting sensing device that can measure the user’s activity even when asleep. It digitally records the distance a user walks or runs, right down to the number of steps, and calories burned. The device wirelessly sends data to its Web site, fitbit.com, giving users minute-by-minute details. Users can add consumption information at the Web site, allowing an accurate picture of calorie consumption and expenditures. Unlike other devices, like the Nike + iPod Sport Kit that monitors walking and running and sells for $29 or the Zeo Personal Sleep Coach that records brainwaves while the user is sleeping and sells for $399, the Fitbit combines both functions. The first-time Fitbit setup isn’t very easy for users, but once over that hurdle, it’s relative easy to use. 101. Refer to Fitbit. The company marketing the Fitbit does not currently market a product like this. Which of the following best represents the category of new product the Fitbit is for this company? A. New-to-the-world product B. New product line C. Addition to existing product lines D. Repositioned product E. Incremental product Answer: B Rationale: The category of new product lines represents products that the firm has not previously offered. 102. Refer to Fitbit. The idea for the Fitbit was generated from an employee retreat in which everyone was encouraged to think of unlimited product ideas for the fitness-minded consumer. Participants did not critique any of the ideas, as the goal at that point was just to generate ideas. What idea-generating technique does this illustrate? A. Brainstorming B. Brain dumping C. Snowball sampling D. Laddering E. Entropy Answer: A Rationale: The goal of brainstorming is to get a group to think of unlimited ways to vary a product or solve a problem. Members avoid criticism of an idea no matter how ridiculous it may seem. 103. Refer to Fitbit. In terms of the new-product development process, Fitbit’s availability in retail stores and online shows that it is in the _____ stage. A. idea screening B. idea generation C. business analysis D. test marketing E. commercialization Answer: E Rationale: The final stage in the new-product development process is commercialization, the decision to market the product. 104. Refer to Fitbit. The process by which the adoption of this innovative product spreads is called the: A. diffusion process B. product life cycle C. consumption process D. viral process E. two-stage process Answer: A Rationale: The diffusion process is the process by which the adoption of an innovation spreads. 105. Refer to Fitbit. The fact that Fitbit can monitor motion while awake and asleep makes it different from competing products that might increase its rate of adoption. Which product characteristic affecting the rate of adoption does this illustrate? A. Complexity B. Compatibility C. Relative advantage D. Observability E. Trialability Answer: C Rationale: Fitbit has a relative advantage over the Nike + iPod and the Zeo devices because it monitors what both of them do separately. Rapala VMC Corporation Finland-based Rapala VMC Corporation is the world’s largest fishing lure manufacturer. It developed its first fishing lure––a cigar-shaped minnow––in 1962. The annual market for fishing lures in terms of retail sales is approximately $600 million. Rapala VMC garnered one-third of that amount in 2008. New lures must be introduced each year in order to stay competitive in this industry. Like other lure manufacturers, Rapala focuses on two questions in developing and making new lures: Does it work, and does it look good? Developing a new lure takes two or three years and involves extensive field testing by tournament professionals and fishing guides. At the annual fishing industry trade show in 2008, Rapala unveiled the “dream lure for the 21st century.” 106. Refer to Rapala. The “dream lure for the 21st century” will more than likely be an example of a(n): A. new product line B. addition to existing product line C. discontinuous innovation D. improvement of existing product E. repositioned product Answer: B Rationale: The firm makes lures and is simply adding a new one to its product mix. 107. Refer to Rapala. The testing of new lures by tournament professionals and fishing guides would take place in which stage of new-product development? A. Commercialization B. Idea generation C. Idea screening D. Concept testing E. Product development Answer: E Rationale: These lures would be prototypes. 108. Refer to Rapala. After the trade show, the newest lure made by Rapala will enter which stage of the new-product development process? A. Commercialization B. Idea generation C. Idea screening D. Concept testing E. Product development Answer: A Rationale: The final stage of the new-product development process is commercialization, the decision to market a product. 109. Refer to Rapala. Because buyers of fishing lures want to have the newest, most technologically improved lure, most of Rapala’s customers would more than likely fall into the _____ category of adopters. A. laggards B. prompt diffusers C. early adopters D. innovators E. early majority Answer: D Rationale: Innovators are the first 2.5 percent of all those who adopt the product. 110. Refer to Rapala. The degree to which the newest lures are perceived as superior to earlier models refers to their _____, a characteristic used to predict the rate of adoption. A. trialability B. observability C. relative advantage D. differentiation capability E. complexity Answer: C Rationale: Relative advantage is the degree to which a product is perceived as superior to existing substitutes. 111. Refer to Rapala. Fishing lures are most likely in the _____ stage of their product life cycle. A. saturation B. maturity C. growth D. innovation E. decline Answer: B Rationale: One of the characteristics of products in the maturity stage is the addition of annual models. Going to the Dogs Demand for pet services is greatly increasing across the United States. In fact, Americans will spend nearly $44 billion dollars on their pets this year. Many American dog owners are seeking out “doggy day cares” that are more like a resort or spa than an ordinary kennel for their canine companion. Day care services for dogs now include toy rooms, outside play areas, doggie massages, treadmills, swimming pools, and cushy beds for nap time. Pet owners can watch their pets via a webcam, and some doggy day cares have a phone where owners can talk to their dogs when they feel their pets need to hear their voices. Rates vary from $20 to $45 a day per pet, depending on the amenities that the pet owner chooses. 112. Refer to Going to the Dogs. The doggy day care center is more like a dog resort than a dog kennel. A dog day care represents what type of new product? A. Competitive innovation B. Discontinuous innovation C. New product line D. Revision of existing product E. Higher-priced product Answer: D Rationale: The dog day care is an improvement on the standard dog-boarding kennel and, as such, is a revision of an existing product. 113. Refer to Going to the Dogs. Suppose that before opening the It’s a Dog’s Life doggy resort the owners gathered together a group of ten dog owners to discuss what they would want to see in the ideal dog day care center. This would be an example of: A. idea screening B. diffusion C. a focus group interview D. basic research E. observation Answer: C Rationale: An objective of focus group interviews is to stimulate insightful comments through group interaction. 114. Refer to Going to the Dogs. Before opening the It’s a Dog’s Life doggy resort, the potential company owners projected the costs involved in providing the services the target market (dog owners) appears to desire. Unfortunately, a decision was made to not proceed with the doggy resort, as the owners felt the cost to provide the services was too high considering the rates the target market would be willing to pay. This would be an example of: A. idea screening. B. concept testing. C. business analysis. D. commercialization. E. test marketing. Answer: C Rationale: In the business analysis stage preliminary figures for demand, cost, sales, and profitability are calculated 115. Refer to Going to the Dogs. Laura felt a little bad about leaving her dog, Toby, at home all day while she was at work. She remarked to a coworker one day about her concerns and was surprised when the coworker told her about the local “doggy daycare” business, which she had never heard of before. Because Laura did not know such a service existed, “doggy daycare” represented a(n) _____ to her. A. heterogeneous shopping product B. diffusion C. market laggard D. product diversification E. innovation Answer: E Rationale: An innovation is a product perceived as new by a potential adopter. 116. Refer to Going to the Dogs. For many dog owners, it may be hard to understand the difference between a traditional kennel and a dog day care, especially given the price differential. As a result, dog day cares may offer special low rates for first-time users in an effort to reduce which of the following issues concerning the rate of diffusion? A. Compatibility B. Complexity C. Observability D. Innovativeness E. Relative advantage Answer: C Rationale: By lowering the price for first-time users, the dog day care will make it easier for dog owners to try the facility and experience its benefits. 117. Refer to Going to the Dogs. One challenge to adoption of the dog day care is the fact that dog owners are used to paying a somewhat minimal fee for boarding their dogs in a kennel, while a day care center may demand many times as much for the same time span. This suggests that the _____ of the new product may affect the rate of diffusion. A. compatibility B. complexity C. relative advantage D. observability E. trialability Answer: A Rationale: Compatibility refers to the degree to which the new product is consistent with existing values and product knowledge. 118. Refer to Going to the Dogs. Central Bark Doggie Day Care currently has 31 locations, with a new Gainesville, Florida, location coming soon. With the opening of competition in the form of Camp Bow Wow and others, it appears that the dog day care product is moving into the _____ stage of the product life cycle. A. commercialization B. introductory C. growth D. maturity E. decline Answer: C Rationale: A competitor entering the market is an indicator of the growth stage in the product life cycle. ESSAY 1. Name and describe the six categories of new products. Answer: NEW-TO-THE-WORLD PRODUCTS (DISCONTINUOUS INNOVATIONS) create an entirely new market. New-to-the-world products represent the smallest category of new products. NEW PRODUCT LINES are products the firm has not offered in the past but will introduce into an established market. ADDITIONS TO EXISTING PRODUCT LINES are new products that supplement a firm’s established line. Complementary products can also supplement a firm’s offerings. IMPROVEMENTS OR REVISIONS OF EXISTING PRODUCTS are usually minor changes that may entail addition or deletion of ingredients. Many of these products are labeled “new and improved.” REPOSITIONED PRODUCTS are existing products targeted at new markets or market segments. LOWER-PRICED PRODUCTS are those that provide similar performance to competing brands at a lower cost. Lower cost may result from technological advantages, economies of scale in production, or lower marketing costs. 2. List the seven steps of the new-product development process. Answer: 1. New-product strategy 2. Idea generation 3. Idea screening 4. Business analysis 5. Development 6. Test marketing 7. Commercialization 3. Name five sources of new-product ideas. Which two techniques are considered the most useful for generating creative thinking for new-product ideas? Answer: The chapter discusses several sources of ideas: customers, employees, distributors, vendors, competitors, research and development, and consultants. The two approaches considered most useful for generating new-product ideas are brainstorming and focus group exercises. 4. After successful business analysis of a new product, the development stage begins. Describe the development process and explain the difference between laboratory testing and use testing. Answer: While a prototype product is being physically developed by the research and development or engineering department, the marketing department begins to develop marketing strategies. Packaging, branding, and labeling decisions are made, while preliminary promotion, price, and distribution strategies are developed. Technical feasibility of manufacturing the product at a reasonable cost is also examined. Prototype product models are often laboratory and use tested during the development stage. Laboratory tests subject products to much more severe treatment than is expected by end users. Laboratory testing can ensure products conform to established safety standards. Use tests place the prototype in consumers’ homes or businesses for trial. 5. Describe the idea screening stage of the new-product development process. Describe a common test used during the screening stage. Answer: The screening process is used as an initial filter to eliminate new-product ideas that are inconsistent with the organization’s new-product strategy or are inappropriate for some other reason. The review is usually done by a group formally appointed to screen new-product ideas. Most of the ideas are rejected at this stage. Concept tests are often used at the screening stage to evaluate product concept alternatives. These tests can be good predictors of early trial and repeat purchases for line extensions. They are not as accurate in predicting the success of new products that create new consumption patterns and require major changes in consumer behavior. 6. After products and marketing programs have been developed for new products, they are usually tested in the marketplace. What is test marketing? How may a company benefit from the use of such marketing? How can test marketing cause problems for a company? Answer: Test marketing is a limited introduction of a product and a marketing program to determine the reactions of potential customers in a market situation. An advantage of test marketing is that it allows management to evaluate alternative strategies and see how well the various aspects of the marketing mix fit together. Test marketing may reduce risk by allowing modification of a marketing mix before national introduction or by withdrawal of a product with failure characteristics. However, test markets have several disadvantages: costs of test markets are high, it reveals the product to competitors, and a product’s success in a test market does not guarantee it will be a nationwide hit. 7. Discuss alternatives to traditional test marketing. Answer: Supermarket scanner testing (single-source research) is one test marketing alternative. Another alternative is simulated (laboratory) market testing. These tests entail showing people promotional materials for several products, including the test products. These people then shop at a mock or real grocery store, where purchases are recorded. Another alternative is online test marketing. Many firms are finding that the Internet offers a fast, cost-effective way to conduct test marketing. 8. What is the final stage in the new-product development process? What tasks are set in motion during this stage? Answer: The final stage in the new-product development process is commercialization. The decision to commercialize sets several tasks in motion: ordering production materials and equipment, starting production, building inventories, shipping the product to field distribution points, training the sales force, announcing the product to the trade, and advertising to potential customers. 9. Global R&D is important for two reasons. What are they? Answer: Global R&D is important for two reasons. First, large companies have become global and no longer are focused only on one market. Global R&D is needed to connect with customers in different parts of the world. Second, companies want to tap into the world’s best talent. 10. Describe how a multinational corporation can most efficiently and effectively meet the needs of the global market. Answer: A firm that adopts a worldwide new-product development strategy is better able to develop products with specifications that are marketable in multiple countries. Ideally, products should be developed for potential worldwide distribution, and unique multinational market requirements are incorporated into the base product whenever technically feasible. A company could also design products to meet regulations and other key requirements in its major market and then meet smaller markets’ requirements on a country-by-country basis. The main goal of the global product development process is not to develop a standard product or product line, but to build adaptability into products that are expected to achieve worldwide appeal. 11. There are five categories of adopters participating in the diffusion process. Name and briefly describe each of these five categories in the correct order from first adopter to last adopter. Answer: INNOVATORS represent the first 2.5 percent of adopters. They are venturesome and eager to try new products. They have higher incomes, better education, and more self-confidence than non-innovators and rely less on group norms. Moreover, they are active outside their communities. Innovators obtain information from scientific sources and experts. EARLY ADOPTERS represent the next 13.5 percent of adopters. They are reliant on group norms and values, oriented to the local community, and likely to be opinion leaders. THE EARLY MAJORITY is the next 34 percent to adopt. They are deliberate in their information collection and are likely to be the friends and neighbors of opinion leaders. THE LATE MAJORITY is the next 34 percent to adopt. They adopt to conform to social pressure. They tend to be older and below average in income and education. They rely on word-of-mouth communication rather than the mass media. Their dominant characteristic is skepticism. LAGGARDS are the final 16 percent to adopt. They are tradition bound and do not rely on group norms. Laggards have the lowest socioeconomic status, are suspicious of new products, and are alienated from a rapidly advancing society. Marketers typically ignore laggards. 12. Hemopure is a blood substitute developed by Biopure Corporation. It does not have to be refrigerated or blood typed like donated human blood that is currently being used in blood transfusions. It comes in bags just like donated blood, so it can be used with intravenous tubes and needles just like donated human blood. As a result, medical personnel will not have to make any adjustments if they adopt this product. Name and describe the five product characteristics influencing the rate of adoption of new products, and explain how each factor will influence the rate of adoption of a blood substitute like Hemopure. Answer: COMPLEXITY refers to the degree of difficulty involved in understanding and using a new product. The more complex the product, the slower its diffusion. Hemopure is administered the same way as donated blood, so the complexity will be low for this product and will increase the rate of adoption. COMPATIBILITY refers to the degree to which the new product is consistent with existing values and product knowledge, past experiences, and current needs. Incompatible products diffuse more slowly than compatible products. Hemopure is very compatible with how medical personnel currently meet patients’ need for blood transfusions, so the rate of adoption will increase. RELATIVE ADVANTAGE is the degree to which a product is perceived to be superior to existing substitutes. Superiority increases the diffusion rate. Because Hemopure does not have to be refrigerated or blood typed, it is superior to donated human blood. This will increase the rate of adoption. OBSERVABILITY refers to the degree to which the benefit or other results of using the product can be observed by others and communicated to target customers. A higher degree of observability enhances diffusion. Medical professionals can see the benefit of the product by looking at the results of clinical trials. TRIALABILITY is the degree to which a product can be tried on a limited basis. Demonstrations and sampling help trialability. Products with low levels of trialability experience slower diffusion. Hospitals could be given the blood to test, which would likely increase the rate of adoption. 13. Draw the sales line and the profit line of the product life cycle in the following diagram, and label each line. Then indicate the names of the four stages of the product life cycle in the blanks provided. Answer: See Exhibit 11.2. The sales line should start at zero sales at the beginning of the introductory stage, accelerate during the growth stage, peak in the maturity stage, and decrease during the decline stage. The profit line should start in the negative range during the introductory stage, break even at the start of the growth stage, peak during the growth stage, and reach a near-zero asymptote during the maturity and decline stages. At no point should the profit line be above the sales line. 14. List the four stages of the product life cycle and discuss the typical characteristics of each stage. Answer: The INTRODUCTORY STAGE is characterized by a high failure rate, little competition, frequent product modification, limited distribution, high marketing and production costs, negative profits, and promotion that stimulates primary demand. The GROWTH STAGE is characterized by increased sales, new competitors, healthy profits that peak, aggressive brand promotion, expanded distribution, price reductions, and possible acquisitions. The MATURITY STAGE is characterized by a peak in sales, lengthened product lines, style modifications, price reductions, falling profits, competitor turnover, heavy promotion, and brand “wars.” The DECLINE STAGE is characterized by a long-run drop in sales and profits, less demand, widespread competitor failure, reduction of advertising costs, and possible elimination of the product. Chapter 12—Services and Nonprofit Organization Marketing TRUE/FALSE 1. A barber shop is an example of a service business. Answer: True Rationale: A barber shop provides a service (haircuts), making it an example of a service business. 2. Services tend to exhibit more search qualities than do tangible goods. Answer: False Rationale: A search quality is a characteristic that can be easily assessed before purchase, and tangible goods tend to exhibit more search qualities than do intangible goods. Services exhibit more experience and credence qualities. 3. Services are considered inseparable because most services cannot be felt or touched in the same way most goods can be sensed. Answer: False Rationale: This describes the characteristic of intangibility. 4. Greta, the owner of House Mouse Cleaning Services, must be physically present when she cleans a house or office. The need for her presence as the service is performed is an example of the intangibility of services. Answer: False Rationale: The need for her presence and the customer’s presence is an example of how services are produced and consumed simultaneously (inseparability). 5. Heterogeneity of services means the quality of a service may not be consistent. Answer: True Rationale: Heterogeneity in services refers to variability in quality due to factors like human involvement, making consistent quality challenging. 6. Hotels will often offer deep discounts on weekends and during the off-season; for the same reason, airlines will adopt a similar pricing strategy during off-peak hours. These services cannot be stored, warehoused, or inventoried because they are perishable. Answer: True Rationale: Hotels and airlines use dynamic pricing strategies to adjust prices based on demand fluctuations, particularly during off-peak times, as their services are perishable and cannot be stored or inventoried like physical goods. 7. Responsiveness is the service quality component most valued by customers. Answer: False Rationale: Reliability has been found to be the component most important to consumers. 8. Karen has a gluten allergy, which means she is unable to eat certain foods. The chef at her favorite restaurant understands Karen’s situation and is always happy to modify his recipes for her when she dines there. The chef is providing assurance to Karen. Answer: False Rationale: The chef is exhibiting empathy—caring, individualized attention—to Karen. 9. The gap model of service quality identifies five gaps that can cause problems in service delivery and influence customer evaluations of service quality. Answer: True Rationale: The gap model of service quality identifies five gaps (such as knowledge gap, policy gap) that can lead to discrepancies between customer expectations and service delivery, affecting perceived service quality. 10. The only type of service processing required for a comedian entertaining at a local club is information processing. Answer: False Rationale: Comedians require mental stimulus processing. 11. The core service is the most basic benefit the consumer is buying. Answer: True Rationale: The core service is the fundamental benefit or service a consumer seeks when purchasing a product or service. 12. Most service organizations market only one specific service Answer: False Rationale: Most service organizations market more than one service. 13. Vera is designing a promotional strategy for a company that provides premium boarding services for pampered animals. Her promotions should use personal information sources and stress the tangible cues associated with the service. Answer: True Rationale: Promotional strategies for premium boarding services for pampered animals should utilize personal information sources (such as testimonials or personalized messages) and stress tangible cues (such as luxurious facilities or personalized care) to appeal to the target market's desire for high-quality service. 14. Revenue-oriented pricing focuses on maximizing the surplus of income over costs. Answer: True Rationale: Revenue-oriented pricing aims to maximize the difference between revenue and costs, ensuring a surplus of income over expenses, which aligns with maximizing profitability. 15. Many businesses have found that it is more expensive to hang on to the customers they have than to focus only on attracting new ones. Answer: False Rationale: Many businesses have found that it is more cost-effective to hang on to the customers they have than to focus only on attracting new ones. 16. Chesapeake Energy Corporation was recognized by Fortune Magazine as one of the top companies to work for. Fortune mentioned that Chesapeake Energy provides its employees many perks, including the opportunity to take free scuba-diving lessons in Chesapeake’s on-site Olympic-size pool. This example illustrates relationship marketing in action. Answer: False Rationale: Chesapeake Energy is actually practicing internal marketing. Relationship marketing is focused on customers, not employees. 17. U.S. service firms such as financial institutions and construction, engineering, and insurance companies will have a difficult time expanding to global markets because competing foreign firms possess many competitive advantages. Answer: False Rationale: These U.S. firms have the greatest potential for globalization because of their existing competitive advantages. 18. A nonprofit organization is an organization that exists to achieve some goal other than the usual business goals of profit, market share, or return on investment. Answer: True Rationale: Nonprofit organizations are defined by their primary goal of achieving social, educational, charitable, or other non-business-related objectives rather than maximizing profit. 19. Nonprofit organizations do not charge prices for the services they provide. Answer: False Rationale: Nonprofit organizations do charge for the services they provide, though they often use terms such as fees, donations, tuition, fares, fines, or rates. 20. Nonprofit managers do not need to worry about developing marketing strategies. Answer: False Rationale: Although nonprofit organizations have a different purpose than for-profit businesses and operate in different environments, managers of these organizations need to develop marketing strategies to bring about mutually satisfying exchanges with target markets. 21. One marketing issue unique to nonprofit organizations is that these organizations must often target those who are apathetic about or strongly opposed to receiving their services. Answer: True Rationale: Nonprofit organizations often face the unique challenge of persuading individuals who may be indifferent or even opposed to their mission to support or utilize their services. 22. Nonprofit organizations may or may not require special facilities for distribution of their services. Answer: True Rationale: Nonprofit organizations may require special facilities for distributing their services, depending on the nature of the services they provide and the target populations they serve. 23. The advertising budgets of most nonprofits are too small to pay for the running of public service advertisements to promote their programs, activities, or services. Answer: False Rationale: The sponsor of a public service advertisement does not pay for the time or space. Instead, it is donated by the medium. MULTIPLE CHOICE 1. A(n) _____ is the result of applying human or mechanical efforts to people or objects. A. application processor B. profit intermediary C. tangible product D. service E. nonprofit organization Answer: D Rationale: This is the definition of a service. 2. “Girls Just Wanna Have Funds” is a Washington, DC, support group that consists mostly of young women who offer tips on budgeting and debt relief. As they state on their Web site, their goal is to help women break financial ceilings “one stiletto at a time.” Girls Just Wanna Have Funds is providing a _____ that is helpful to women who want to be financially savvy. A. synergy B. tangible resource C. tangible product D. service E. good Answer: D Rationale: A service is the result of applying human or mechanical efforts to people or objects. 3. Xoom.com is a San Francisco-based online money transfer company that provides consumer remittance services. With the Xoom bank deposit service, consumers can send money directly to bank accounts in several countries around the world. Xoom.com would be classified as a: A. good. B. tangible resource. C. tangible product. D. service. E. nonprofit organization. Answer: D Rationale: A service is the result of applying human or mechanical efforts to people or objects. 4. All of the following are unique characteristics that distinguish services from goods EXCEPT: A. intangible. B. inseparable. C. searchable. D. heterogeneous. E. perishable. Answer: C Rationale: Services are intangible, inseparable, heterogeneous, and perishable. 5. A service cannot be touched, seen, tasted, heard, or felt in the same manner in which goods can be sensed and, therefore, is referred to as being: A. impervious. B. extraneous. C. synergistic. D. perishable. E. intangible. Answer: E Rationale: The basic difference between services and goods is that services are intangible performances. 6. A(n) _____ is a characteristic that can be easily assessed prior to purchase, such as the softness of a mattress or the color of curtains. A. search quality B. intangible attribute C. experience quality D. credence quality E. heterogeneity feature Answer: A Rationale: Search quality is a characteristic that can be easily assessed before purchase. 7. Maribeth and Asher are trying to think of someplace to take their kids this weekend. Asher thinks it might be fun to take the kids to Coco Key, an indoor waterpark located in a nearby city. However, since no one in the family has ever been to Coco Key before, they will be unable to assess the _____ of the waterpark until they actually visit. A. tangible attribute B. experience quality C. creative quality D. credence quality E. heterogeneity feature Answer: B Rationale: An experience quality is a characteristic that can only be assessed after use. 8. “Girls Just Wanna Have Funds” is a Washington, DC, support group that consists mostly of young women who offer tips on budgeting and debt relief. Since you cannot evaluate the quality of the financial advice until after you have received it, this is an example of the _____ characteristic of service. A. reliability B. heterogeneous satisfaction C. search quality D. temporal quality E. experience quality Answer: E Rationale: Experience quality is described as a characteristic that can be assessed after use. 9. A characteristic that consumers may have difficulty assessing even after purchase because they do not have the necessary knowledge or experience is referred to as _____ quality. A. search B. experience C. credence D. abstract E. inert Answer: C Rationale: Medical and consulting services are examples of services that exhibit credence qualities. 10. Alec had his gall bladder removed, but he was unconscious during the operation. In fact, even though he has an incision, he really has no way of knowing if the service was actually performed even after it was allegedly performed. That is because medical services such as this exhibit _____ qualities. A. credence B. perishable C. experience D. search E. homogeneous Answer: A Rationale: A credence quality is a characteristic that consumers may have difficulty assessing even after purchase because they do not have the necessary knowledge or experience. 11. Which of the following services would be most likely to exhibit strong credence qualities? A. A math tutorial service B. The repair of a leaky drain C. The preparation of a dead body for burial D. Tax return preparation E. A landscaping service that mows lawns Answer: C Rationale: A credence quality is a characteristic that cannot easily be assessed even after purchase and experience. 12. Which unique characteristic of services means that consumers must be present during the production? A. Intangibility B. Inseparability C. Heterogeneity D. Perishability E. Connectivity Answer: B Rationale: Inseparability is the inability of the production and consumption of a service to be separated. 13. Auto repair, manicures, and landscaping are all services that are produced and consumed at the same time. All of these services exhibit the service characteristic of: A. inseparability. B. intangibility. C. heterogeneity. D. perishability. E. variability. Answer: A Rationale: Inseparability is the inability of the production and consumption of a service to be separated. 14. It is difficult for most of us to talk about mistakes we have made. Unfortunately, for organizations like Consumer Credit Counseling to help you work through your financial issues, you must be willing to honestly discuss your financial condition. In other words, for these organizations to help, you must be actually involved in the process of developing your financial plan, which points to the _____ nature of services. A. intangible B. inseparable C. heterogeneous D. perishable E. flexible Answer: B Rationale: Inseparability is when services or goods are produced and consumed at the same time. 15. Which unique characteristic of services is the variability of the inputs and outputs of services, which causes services to tend to be less standardized and less uniform than goods? A. Intangibility B. Inseparability C. Heterogeneity D. Perishability E. Flexibility Answer: C Rationale: Because services have greater heterogeneity, or variability of inputs and outputs, they tend to be less standardized. 16. It is difficult to achieve consistency and standardization of services because of which service characteristic? A. Customization B. Simultaneous production and consumption C. Intangibility D. Perishability E. Heterogeneity Answer: E Rationale: Heterogeneity means that a service tends to be less standardized and uniform than goods. 17. A reading service for the visually impaired requires each reader applicant to prepare and submit a one-hour interview tape of material chosen by the service to determine whether the reader has pleasing vocal characteristics and is accent free so that it is not necessary to use the same reader every time. The reading service is trying to limit problems associated with the service characteristic of: A. tangibility. B. credence quality. C. heterogeneity. D. simultaneous production and consumption. E. flexibility. Answer: C Rationale: Heterogeneity means that a service tends to be less standardized and uniform than goods. 18. The consistency and reliability of a service can be increased by: A. making sure that consumers are present during the production of the service. B. eliminating credence quality. C. standardization and training, D. Simultaneous production and consumption E. raising prices. Answer: C Rationale: Mechanization can also increase consistency and reliability. 19. Due to service _____, services cannot be stored, warehoused, or inventoried. A. tangibility B. variability C. intangibility D. perishability E. heterogeneity Answer: D Rationale: Perishability is the inability of services to be stored, warehoused, or inventoried. 20. David and Kathy like to take their young son, Chaz, to Moe’s for lunch after church on Sundays. While they like to eat at Moe’s at any time, Sunday is particularly good because it’s “kids-eat-free” day at Moe’s southwestern grill. Until Moe’s began the reduced pricing program, Sundays were very slow. Now it is one of the busiest days of the week. This price reduction was a way to contend with the service characteristic of: A. variability. B. perishability. C. intangibility. D. inseparability. E. simultaneous production and consumption. Answer: B Rationale: Differential pricing tries to even out demand. This is important because services cannot be stored, inventoried, or warehoused. 21. One of the reasons consumers can purchase cheap flights or hotel rooms on Web sites such as Expedia.com and Hotels.com is due to the fact that airlines cannot sell the seat on a specific flight after the plane takes off and hotels cannot recoup the revenue from that room for that night once the night passes. Which unique characteristic of services does this illustrate? A. Intangibility B. Inseparability C. Perishability D. Simultaneous production and consumption E. Variability Answer: C Rationale: Services cannot be stored, inventoried, or warehoused. 22. All of the following are service components customers use to evaluate service quality EXCEPT: A. validity. B. empathy. C. assurance. D. responsiveness. E. reliability. Answer: A Rationale: Customers evaluate service quality through five components: reliability, responsiveness, assurance, empathy, and tangibles. 23. Which component of a service is the ability to perform the service dependably, accurately, and consistently? A. Responsiveness B. Reliability C. Assurance D. Empathy E. Tangibles Answer: B Rationale: Reliability is performing the service right the first time. 24. Canyons ski resort is so positive that guests will enjoy their visit that they offer a money-back guarantee for special lift tickets purchased at the resort Web site. Canyons’ “You’ll-Love-It-Or-It’s-Free-Guarantee” provides consumers which of the following service quality aspects? A. Tangibility B. Responsiveness C. Sympathy D. Responsibility E. Reliability Answer: E Rationale: Reliability is the ability to perform the service dependably, accurately, and consistently. 25. Virtual Bellhop is a company that ships awkward sporting goods to vacation destinations so its customers do not have to worry about lugging them through airports or having them damaged in transit. Thus far, its record for getting the sporting equipment to the correct destination for the vacationer is perfect. This indicates that the company excels at which component of service quality? A. Tangibles B. Empathy C. Reliability D. Flexibility E. Assurance Answer: C Rationale: Reliability is the ability to perform the service dependably and consistently. 26. Which component of service quality represents the ability to provide prompt service? A. Reliability B. Responsiveness C. Assurance D. Empathy E. Tangibles Answer: B Rationale: This describes responsiveness. 27. Jonathon had a problem when submitting an exam in his online marketing class and he received a score of zero, so he sent an e-mail to the professor explaining his problem. He was really worried because the exam was due that evening, and he was very pleased when the professor replied within an hour telling him that he scored a 90 on the exam. The professor’s prompt reply to his inquiry is an example of which service quality component? A. Reliability B. Responsiveness C. Assurance D. Empathy E. Tangibles Answer: B Rationale: Responsiveness is the ability to provide prompt service. 28. Which service quality component is the knowledge and courtesy of employees and their ability to convey trust? A. Reliability B. Responsiveness C. Assurance D. Empathy E. Tangibles Answer: C Rationale: This is a description of assurance. Skilled employees treat customers with respect and make customers feel that they can trust the firm to exemplify assurance. 29. Buying a car through eBay may result in a low price, but how do you know you will be getting a good car? One nice feature provided by eBay is the “feedback rating” that provides ratings and comments from people who have made purchases from that seller before. Which service quality component does the feedback rating address? A. Empathy B. Assurance C. Tangibles D. Flexibility E. Responsiveness Answer: B Rationale: Assurance refers to the knowledge and courtesy of the employees and their ability to convey trust. Customer ratings would provide this information. 30. Service personnel who exhibit caring, individualized attention to customers are addressing which service quality component? A. Reliability B. Responsiveness C. Assurance D. Empathy E. Tangibles Answer: D Rationale: Empathy is providing caring, individualized attention to customers. 31. Emily’s nephew is on the autism spectrum, and she has read stories on the Internet about how vaccines supposedly cause autism. Though Emily knows this theory has been debunked and that vaccines are safe, she is surprised that she feels a bit worried when she takes her own child into the pediatrician for her shots. Emily is glad her pediatrician does not think her fears are silly and will answer all of her questions without making her feel stupid for asking them. By which of the following components of service quality is Emily most likely to rate her pediatrician? A. Empathy B. Assurance C. Tangibles D. Reliability E. Responsiveness Answer: A Rationale: Empathy refers to providing caring and individualized attention to customers. 32. The physical evidence of a service, including the physical facilities, tools, and equipment used to provide the service, represents which component of service quality? A. Reliability B. Responsiveness C. Assurance D. Empathy E. Tangibles Answer: E Rationale: Tangibles are the physical evidence of the service. 33. James recently went to a new health clinic because he had a sore throat that wouldn’t go away. He was not familiar with this clinic and was a bit surprised when the doctor came in and was wearing blue jeans, a flannel shirt, and tennis shoes. The doctor and nurse took proper care of James, but it seemed strange to him that the doctor was dressed that way. Which component of service quality does this illustrate? A. Reliability B. Responsiveness C. Assurance D. Empathy E. Tangibles Answer: E Rationale: Tangibles are the physical evidence of the service. 34. The model of service quality that identifies five disparities that can cause problems in service delivery and influence customer evaluations of service quality is referred to as the _____ model. A. gap B. theory of reasoned action C. attribution D. dual-coding E. Fishbein Answer: A Rationale: The gap model identifies five gaps that can cause problems in service delivery and influence customer evaluations of service quality. 35. Starbucks management has been trying to refocus on the things that originally made the company so successful. For example, Starbucks stores across the United States recently shut down for a short period of time for retraining, and the company has worked to make sure you can smell the fresh ground coffee aroma when you enter the store. However, when questioned, consumers tend to focus on the need for pricing incentives such as a frequent purchasing program. According to the _____, there may be a difference between what customers want and what Starbucks management thinks customers want. A. gap model of service quality B. service pyramid C. four dimensions of service quality D. service paradigm E. service delineation matrix Answer: A Rationale: The gap model identifies five gaps that can cause problems in service delivery and influence customer evaluation of service quality. 36. First Community Bank spent a considerable amount of money updating its lobby with plush sofas, a large-screen television, and a refreshment counter with popcorn and soft drinks. However, very few customers spend time in the lobby. In fact, most customers want to be in and out of the bank, with their financial transaction completed as quickly as possible. The gap model of service quality would suggest that a gap exists between: A. the service quality specifications and the service that is actually provided. B. what the company provides and what the customer is told it provides. C. the service customers receive and the service they want. D. what customers want and what management thinks customers want. E. what management thinks customers want and the quality specifications management develops to provide the service. Answer: D Rationale: Management thinks people want to spend time in a really nice bank lobby, but consumers seem much more concerned with timely service, which is characteristic of gap 1 in the gap model. Thus, what the customers really want seems different from what management thinks they want. 37. To improve its friendliness, the First National Bank hired Joy Taylor as a greeter. Joy’s job is to sit at a desk near the front lobby and greet customers as they enter with a warm, friendly smile along with a “Good Morning” or “Good Afternoon.” Unfortunately, Joy is also required to answer the bank’s main telephone line. Since she spends most of her day on the telephone, Joy rarely gets to flash her smile or greet anyone who is entering the bank. In fact, Joy is on the phone so much that she doesn’t really even seem friendly. This is an example of a gap between: A. the service quality specifications and the service that is actually provided. B. what the company provides and what the customer is told it provides. C. the service customers receive and the service they want. D. what customers want and what management thinks customers want. E. what management thinks customers want and the quality specifications management develops to provide the service. Answer: A Rationale: The gap model identifies five gaps that can cause problems in service delivery and influence customer evaluation of service quality. This is an example of gap 3––the gap between the service quality specification and the service that is actually provided. 38. Purchasing your clothes online can be fun, allowing you to shop from companies around the world without leaving your home. You can also often get great prices on the items you purchase. However, you could get stuck with a product that doesn’t look nearly as nice in reality as it does on your computer screen. Lands’ End takes this concern away with its “Guaranteed Period” promise. Lands’ End will happily return your purchase price on any item you purchase for any reason. If Lands’ End ever failed to follow through with this promise, it would represent a gap between: A. what management thinks customers want and the quality specifications that management develops to provide the service. B. what the company tells the customer it provides and what is actually provided. C. service quality specifications and employee training. D. what customers want and what management thinks customers want. E. none of these. Answer: B Rationale: This is an example of gap 4––the gap between what the company provides and what the customer is told it provides. 39. If a customer expects to wait three hours for an auto repair but the repair is finished just one hour later, the customer’s evaluation of service quality will be high. However, a two day wait would result in a lower evaluation. The two day wait would illustrate a gap between: A. service quality specifications and the service that is actually provided. B. the service customers receive and the service they want. C. what the company provides and what the customer is told it provides. D. what customers want and what management thinks customers want. E. what management thinks customers want and the quality specifications management develops to provide it. Answer: B Rationale: Gap 5 is the gap between the service that customers receive and the service they want. 40. All of the following are types of service processing that can occur EXCEPT: A. people processing. B. product processing. C. possession processing. D. information processing. E. mental stimulus processing. Answer: B Rationale: Service processes can be placed into one of four categories: people processing, possession processing, mental stimulus processing, and information processing. 41. Which category of service processing takes place when the service is directed at a customer? A. People processing B. Possessions processing C. Information processing D. Mechanical processing E. Personal processing Answer: A Rationale: This is the definition of people processing. Text examples are transportation services and health care. 42. Trina is a physical fitness trainer who tries to get to know her clients and their specific fitness goals. Trina understands that she is in the _____ processing business. A. people B. mechanical C. possession D. mental E. information Answer: A Rationale: People processing takes place when the service is directed at a customer. 43. Which type of service processing occurs when the service is directed at customers’ physical possessions, such as lawn care, dry-cleaning, and veterinary services? A. People processing B. Possession processing C. Mental stimulus processing D. Information processing E. Tangible processing Answer: B Rationale: This is a description of possession processing. 44. Which of the following services is an example of possession processing? A. A theatrical performance B. An x-ray of a broken bone C. An advertising agency D. Marriage counseling E. Heating system repair Answer: E Rationale: Possessions processing occurs when the service is directed to the customers’ physical possessions. 45. TeamBuilds is a service organization that has corporate teams pay $7,500 for an all-day team-building session with a management consultant while they work together on renovating a Habitat for Humanity home. Which category of service processing does TeamBuilds offer? A. Possession processing B. Information processing C. Mental stimulus processing D. People processing E. Homogeneous processing Answer: C Rationale: Mental stimulus processing refers to services directed at people’s minds. 46. Which of the following services is an example of mental stimulus processing? A. Taxi cab service B. Insurance C. A professional tennis match D. Veterinarian care E. A haircut Answer: C Rationale: Mental stimulus processing refers to services directed at people’s minds. 47. Which type of service processing uses technology or brainpower directed at customer’s assets? A. People processing B. Possession processing C. Mental stimulus processing D. Information processing E. Technology processing Answer: D Rationale: This is the definition of the information processing category of service processes. 48. Which of the following services is an example of information processing? A. Drafting a will B. Having a prescription filled C. Getting a down comforter dry cleaned D. Taking a class at the local community college E. Ordering a pizza for delivery Answer: A Rationale: Information processing describes services that use brainpower directed at a customer’s assets. 49. The most basic benefit the consumer is buying in a service is referred to as the: A. primary service B. inherent service C. core service D. tangible service E. complete service Answer: C Rationale: This is the definition of core service. 50. TeamBuilds is a service organization that has corporate teams pay $7,500 for an all-day team-building session with a management consultant while they work together on renovating a Habitat for Humanity home. TeamBuilds’ _____ service is improved work team relationships. A. primary B. core C. niche D. supplementary E. foundation Answer: B Rationale: Core service is the most basic benefit the consumer is buying. 51. Western Union is in the business of providing a medium for international money transfers. Through the services of Western Union, a Mexican working in the United States can transfer a portion of his or her earnings to family members still living in Mexico. The wire transfer of funds is the _____ service the company provides. A. supplementary B. benchmarked C. core D. primary E. fundamental Answer: C Rationale: Core service is the most basic benefit the consumer is buying. 52. A group of services that support or enhance the core service is called: A. secondary services B. peripheral services C. additional services D. supplementary services E. complementary services Answer: D Rationale: This is the definition of supplementary services. 53. Western Union is in the business of providing a medium for international money transfers. Through the services of Western Union, a Mexican working in the United States can transfer a portion of his or her earnings to family members still living in Mexico. The company is now planning to move into areas traditionally handled by banks, such as offering its customers checking accounts and loans. If it does follow through and provide these additional services, Western Union will exemplify _____ services. A. complementary B. secondary C. peripheral D. supplementary E. additional Answer: D Rationale: Supplementary services are a group of services that support or enhance the core service. 54. _____ is the strategy that uses technology to deliver customized services on a mass basis. A. Mass standardization B. Aggregated service C. Technological processing D. Mass customization E. Aggregated marketing Answer: D Rationale: Mass customization can deliver customized services on a mass basis, which results in giving each customer whatever he or she asks for. 55. At the My M&M’s Web site, customers can design their own versions of the famous candy. Customers choose from 25 M&M'S colors; write a message, add clip-art, or even a face to each M&M, and select specific packaging. This is an example of: A. a cooperative service. B. mass customization. C. aggregated standardization. D. a market development strategy. E. mental stimulus processing. Answer: B Rationale: Mass customization is a strategy that uses technology to deliver customized service on a mass basis. 56. TruGreen offers lawn care, shrub care, carpet cleaning, and industrial lawn services. These different services combine to make TruGreen’s: A. service mix. B. standardization strategy. C. component services. D. customized services. E. distribution strategy. Answer: A Rationale: Each part of the service mix should make a different contribution to achieving the firm’s goals. 57. After years of expansion, Starbucks recently closed 600 of its poorer performing stores. Many of the stores closed were near other Starbucks locations. In those areas, management felt that the company had too many outlets, which was boosting the company’s cost unnecessarily. Decisions concerning the number of outlets are a part of the _____ strategy of service organizations. A. distribution B. promotion C. production D. product E. demand/supply Answer: A Rationale: Distribution strategies for service organizations focus on convenience, number of outlets, direct versus indirect distribution, location, and scheduling. 58. All of the following are key factors in the distribution strategy for services EXCEPT: A. customization B. convenience for customers C. number of intermediaries and outlets D. scheduling of service deliveries E. location Answer: A Rationale: Customization is a key factor in the product component of the service marketing mix. 59. All of the following are recommended promotional strategies for services EXCEPT: A. concentrating on franchising B. stressing tangible cues C. using personal information sources D. creating a strong organizational image E. engaging in post purchase communications Answer: A Rationale: Franchising is a distribution decision. 60. The _____ of a service most clearly reveals the relationship between its target market strategy and its distribution strategy. A. convenience B. location C. customization D. promotion E. number of intermediaries and outlets Answer: A Rationale: The location of a service most clearly reveals the relationship between its target market strategy and its distribution strategy. 61. Sir Paul McCartney and Alicia Silverstone are two of the many celebrities serving as spokespeople for the animal rights group People for the Ethical Treatment of Animals (PETA). Both are committed to vegetarianism. In terms of promotion strategy, they will be considered a(n) _____ information source. A. cue control B. non-source-controlled C. impersonal D. remote E. personal Answer: E Rationale: A personal information source is someone consumers are familiar with, such as celebrities. 62. Allstate Insurance says “You’re in good hands with Allstate,” and Prudential Life Insurance shows a rock in its commercials because it wants to convey stability. These companies are using what type of promotion strategy? A. Stressing tangible cues B. Stressing homogeneity C. Using personal information sources D. Providing packaging cues E. Communicating service integration signals Answer: A Rationale: A tangible cue is a concrete symbol, like hands or a rock, of the service offering. 63. The three categories of pricing objectives for services are: A. sales-, quality-, and location oriented B. quality-, value-, and consumer oriented C. revenue-, operations-, and patronage oriented D. turnover-, volume-, and capacity oriented E. equity-, cost-, and revenue oriented Answer: C Rationale: The three categories of pricing strategies are revenue-, operations-, and patronage-oriented pricing. 64. A focus on maximizing the surplus of income over costs is a(n) _____ pricing objective for service firms. A. revenue-oriented B. operations-oriented C. market-share–oriented D. patronage-oriented E. service-quality Answer: A Rationale: This is a description of a revenue-oriented pricing objective. 65. Which of the following is an example of post purchase communication? A. An airline hires an actor to film a series of humorous commercials touting the airline’s service. B. An insurance company designs a new “umbrella” logo to indicate that its customers are well-protected by coverage. C. A fast-food restaurant issues standardized uniforms for all its employees. D. A hotel sends an email survey to a Henri a week after his stay, asking him to comment on the quality of the service he received. E. An income tax preparer offers a 20% discount for any customers who use the service in January. Answer: D Rationale: Post purchase communication refers to the follow-up activities that a service firm might engage in after a customer transaction. 66. Determining costs can be extremely difficult for a service provider who has adopted a(n) _____ pricing objective and may limit the usefulness of the objective. A. market-share-oriented B. operations-oriented C. revenue-oriented D. patronage-oriented E. break-even Answer: C Rationale: Revenue-oriented objectives require calculations of income and costs, which can be difficult for many services. 67. Which service pricing objective seeks to match supply and demand by varying prices? A. Revenue oriented B. Operations oriented C. Patronage oriented D. Externally oriented E. Equilibrium oriented Answer: B Rationale: Operations-oriented pricing seeks to match supply and demand by varying prices (e.g., matching hotel demand to the number of available rooms). 68. Monday and Tuesday nights are traditionally slow nights in the restaurant business. So LaRosa’s Italian Restaurant offers “all-you-can-eat spaghetti night” on Mondays and Tuesdays. For the price of a regular spaghetti dinner, customers can eat all the spaghetti they want on those nights. Because it is focusing on coordinating supply and demand by varying prices, the restaurant is using a(n) _____ pricing objective for service firms. A. revenue-oriented B. operations-oriented C. image-focused D. patronage-oriented E. equity-oriented Answer: B Rationale: A focus on matching supply and demand by varying prices to ensure maximum use of productive capacity at any specific point in time is an operations-oriented pricing objective for service firms. 69. A focus on maximizing the number of customers using a service is a(n) _____ pricing objective for service firms. A. maximization of demand B. patronage-oriented C. profitability-oriented D. operations-oriented E. revenue-oriented Answer: B Rationale: This is the definition of patronage-oriented pricing. 70. A continuing problem for most Minor League Baseball (MiLB) teams is low attendance. Event and venue sponsors want to see fans in the stadiums, and the players appreciate fan support. Given this information, which pricing strategy would you recommend MiLB teams use? A. Operations-oriented pricing B. Peak profitability pricing C. Patronage-oriented pricing D. Supply–demand pricing E. Revenue-oriented pricing Answer: C Rationale: Patronage-oriented pricing tries to maximize the number of customers using the service. 71. Most movie theaters offer discount ticket prices to students and senior citizens. This illustrates the _____ pricing objective. A. subsidized B. operations-oriented C. profitability-oriented D. service quality E. patronage-oriented Answer: E Rationale: A focus on maximizing the number of customers using a service is a patronage-oriented pricing objective for service firms. 72. Any one-at-a-time sale, such as a movie theater ticket, a meal at a restaurant, or public transportation fare, is called a(n) _____ transaction. A. patronage B. interactive C. distinct D. affiliation E. discrete Answer: E Rationale: This defines a discrete transaction. 73. Because many services involve a process of continuous interaction between the service organization and the customer, _____ marketing is an important strategy. A. interactive B. relationship C. patronage D. nonprofit E. affiliation Answer: B Rationale: Relationship marketing is a means of attracting, developing, and retaining customer relationships. 74. One of the main responsibilities of sport marketers is to keep their loyal customers as heavy users and escalate lower-level users into loyal customers. An effective tool for accomplishing this goal would be _____ marketing. A. interactive B. relationship C. patronage D. nonprofit E. affiliation Answer: B Rationale: Relationship marketing involves ongoing interaction between the service provider and the customer. It is a means of attracting, developing, and retaining customer loyalty. 75. At which level of relationship marketing does the firm only use pricing incentives to encourage customers to continue doing business with it? A. Level 1 B. Level 2 C. Level 3 D. Level 4 E. Level 5 Answer: A Rationale: Level 1 using pricing incentives such as frequent-flyer programs. 76. The level of relationship marketing that is least likely to be effective in the long term because its advantage is easily imitated by other firms is based on: A. personal communications B. social bonds C. service delivery heuristics D. pricing incentives E. structural bonds Answer: D Rationale: This level uses pricing incentives that can easily be imitated by other firms. 77. Mountain Express is a company that provides grocery delivery service to several small communities in Idaho. The company gives a 2 percent discount on all food orders to customers after they have used the service ten times. An additional percentage discount is offered for each additional 15 orders up to a maximum of 8 percent. The relationship marketing strategy used by Mountain Express is based on: A. following service delivery paradigms. B. building social bonds. C. building financial bonds. D. opening personal communication channels. E. creating structural bonds. Answer: C Rationale: Building financial bonds includes the use of price incentives. 78. Which level of relationship marketing uses pricing incentives but also seeks to build social bonds with customers? A. Level 1 B. Level 2 C. Primary level D. Secondary level E. Supplemental level Answer: B Rationale: Level 2 designs services to meet customer needs and includes financial incentives but also seeks to build social bonds with customers. 79. A management consulting business stays in touch with its business customers through phone calls and greeting cards. It periodically sends out needs-assessment questionnaires and designs new services to meet the needs revealed in these surveys. This is an example of relationship marketing based on: A. financial bonds B. social bonds C. service delivery D. patronage bonds E. structural bonds Answer: B Rationale: Building social bonds includes staying in touch with customers, learning about their needs, and designing services to meet those needs. 80. Building value-added services that are not readily available from other firms into the delivery system defines relationship marketing based on the creation of: A. financial bonds B. social bonds C. service delivery paradigms D. personal communication channels E. structural bonds Answer: E Rationale: This is level 3 of relationship marketing and includes the financial and social bonds from previous levels but adds value-added services to create structural bonds. 81. Hyatt Regency’s Platinum Card Program allows members to bypass any line at the check-in desk and get immediate check-in service. In this example, _____ are developed by offering value-added services that are not readily available from other hotel chains. A. price incentives B. social bonds C. service delivery paradigms D. personal communication channels E. structural bonds Answer: E Rationale: This example is similar to the example in the text. 82. Relationship marketing programs that are based on _____ bonds have the strongest potential for sustaining long-term relationships with customers. A. financial and individual B. social and structural C. financial, social, and structural D. financial and patronage E. financial and structural Answer: C Rationale: Level 3 marketing programs, which use all three components, are the strongest relationship marketing programs. 83. Treating employees as customers and developing systems and benefits that satisfy their needs is known as _____ marketing. A. organizational quality B. relationship C. internal D. job-faceted E. job-satisfaction Answer: C Rationale: This is the definition of internal marketing. 84. Which of the following is not an example of a program that companies design and institute in order to satisfy its employees? A. On-site day care B. Transportation services C. Flextime D. Concierge services E. Work from home Answer: A Rationale: Many companies design programs in order to retain satisfied employees. Providing transportation to and from work is not an example given in the text. 85. Which country is the world’s largest exporter of services? A. Germany B. Japan C. The United States D. China E. Canada Answer: C Rationale: The United States is the leading exporter of services, but competition in international services is increasing rapidly. 86. To be successful in the global marketplace, service firms must first: A. develop pricing incentives to encourage customers to begin doing business with them. B. design marketing mix elements that account for each country’s unique environment. C. hire employees who are familiar with the local culture. D. determine the nature of their core product. E. build strong social bonds with their foreign customers. Answer: D Rationale: Determining the nature of the core product is the first task of a service firm that hopes to succeed in the global marketplace 87. A _____ seeks to achieve some goal other than the usual business goals of profit, market share, or return on investment. A. nonprofit organization B. synergistic entity C. company with no equity D. service provider E. patronage-oriented organization Answer: A Rationale: This is the definition of a nonprofit organization. 88. People for the Ethical Treatment of Animals (PETA) is the largest animal rights organization in the world. PETA works for better treatment of animals instead of achieving profits, market share, or return on investment. PETA is an example of a: A. patronage-oriented organization B. synergistic entity C. company with no equity D. service provider E. nonprofit organization Answer: E Rationale: A nonprofit organization seeks to achieve some goal other than the usual business goals of profit, market share, or return on investment. 89. Most nonprofit organizations perform all of the following marketing activities EXCEPT: A. identify the customers they wish to serve or attract B. develop, manage, and eliminate programs and services C. decide on prices to charge D. schedule events or programs E. Most nonprofit organizations perform all of the above. Answer: E Rationale: Most nonprofit organizations perform all of these activities. Nonprofits usually charge for their services, though they often use other terms, such as fees, donations, tuition, fares, fines, or rates, rather than prices. They also often refer to their customers as clients, members, or another similar term. 90. The predominant form of nonprofit organization in the United States is: A. farms B. arts museums and programs C. private-sector volunteer organizations D. social service firms E. government Answer: E Rationale: The cost of government (i.e., taxes) is the biggest single item in the American family budget. 91. All of the following are examples of nonprofit organizations EXCEPT: A. GEICO, an insurance company. B. the Muscular Dystrophy Association. C. the Make-a-Wish Foundation. D. Kiwanis International, a service organization. E. the United Auto Workers, a labor union. Answer: A Rationale: GEICO is a for-profit business. 92. _____ refers to the effort of nonprofit organizations to bring about mutually satisfying exchanges with their target markets. A. Autonomous marketing B. Public affairs marketing C. Social targeting D. Social marketing E. Nonprofit organization marketing Answer: E Rationale: This is the definition of nonprofit organization marketing. 93. Habitat for Humanity renovates vacant houses and sells them at no-interest mortgage rates to low-income home buyers. The organization relies on volunteers to provide the labor and funding for its renovations. Habitat for Humanity is best described as a(n): A. image-oriented organization B. for-profit organization C. nonprofit organization D. service branch E. source-based organization Answer: C Rationale: A nonprofit organization exists to achieve some goal other than the usual business goals of profit, market share, or return on investment. 94. Nonprofit organizations face a unique challenge because the target market of the nonprofit organization is often: A. the community where it is located B. the people who provide the funding C. a paid administrator D. apathetic or strongly opposed individuals E. the government Answer: D Rationale: The target market for many nonprofit services is often apathetic, disinterested, or opposed individuals who nonetheless may need the service. Alternatives A, B, and C are as likely to be targeted by for-profits as nonprofits. 95. Many nonprofit organizations are pressured or required to serve the maximum number of people by targeting the average user, which pressures them to adopt which type of targeting strategy? A. Niche B. Undifferentiated C. Multi-segmented D. Uniform E. Unidimensional Answer: B Rationale: Undifferentiated targeting strategies focus on average users. 96. If a benefit is indirect to the customer, the marketer may find a challenge in promoting a product with little: A. benefit complexity B. feature charisma C. benefit strength D. personalization E. benefit exchange Answer: C Rationale: The benefit strength of many nonprofit offerings is quite weak or indirect. 97. Gifts in Kind International is a nonprofit organization that gathers new and used merchandise and distributes it to charities nationwide. Traditional promotional tools may be inadequate to motivate its target markets because of: A. nonprofit product volunteer policies B. the absence of consumer involvement C. lack of prices for the products D. direct distribution E. infrequent use of target markets Answer: B Rationale: Involvement may range from extremely high (join the military, stop smoking) to low (prevent forest fires, don’t litter), which is a much wider range than consumer products usually experience. 98. Susan G. Komen for the Cure is the largest breast cancer organization in the United States. Komen’s annual Race for the Cure series of 5K runs and fitness walks are organized to raise funds for the organization as well as to raise public awareness of the disease. Komen’s Race for the Cure is an example of a _____ strategy. A. target market B. distribution C. product D. promotion E. direct marketing Answer: D Rationale: The organization is using runners to promote its message. 99. _____ is an announcement in mass media for which no charge is made and which promotes programs, activities, or services of federal, state, or local governments or the programs, activities, or services of nonprofit organizations. A. Non-sponsored advertising B. Public service advertising C. Societal marketing D. Cause-related marketing E. Social advertising Answer: B Rationale: Unlike a commercial advertiser, the sponsor of the public service advertisement does not pay for the time or space. 100. The Monroe Job Training for the Disabled program wants to notify the community about its upcoming rummage sale. As an experienced marketer and director of the program, you plan to visit several local radio and televisions stations to request: A. cooperative advertising B. non-sponsored advertising C. primary advertising D. public service advertising E. nonprofit advertising Answer: D Rationale: An announcement in mass media for which no charge is made and which promotes programs, activities, or services of nonprofit organizations is called public service advertising. 101. In a nonprofit organization, _____ are often concerned with partially or fully defraying costs rather than achieving a profit. A. nonfiscal costs B. pricing objectives C. cause-related goals D. equity earnings E. liquidity fees Answer: B Rationale: In nonprofit organizations, pricing objectives are concerned with partially or fully defraying costs and are not focused on achieving a profit like they are in the profit sector. 102. In many nonprofit situations, the consumer is not charged a tangible price yet must absorb time costs, embarrassment costs, or effort costs. These costs are called the organization’s: A. liquidity fees B. financial costs C. fiscal prices D. nonfinancial prices E. retained earnings Answer: D Rationale: The importance of those costs is illustrated by the large number of eligible citizens who do not take advantage of so-called free services for the poor. 103. All of the following are key characteristics distinguishing the pricing decisions of nonprofit organizations from those of the profit sector EXCEPT: A. separation between payers and users B. nonfinancial prices C. indirect payment D. cost allocation E. below-cost pricing Answer: D Rationale: A fifth characteristic is pricing objectives. Boutique Hotels In an industry where guests are tired of cookie-cutter hotels, some consumers are looking for personalized service, which can be found in boutique hotels. Boutique hotels cater to their guests’ sense of their personal image as being discriminating, more sophisticated, and more hip. Frequently, these guests don’t want to be where the crowds are. This is a small but growing market niche. There are no generally recognized rules for boutique hotels, but they tend to be small and service oriented, with high-style decor and top-notch restaurants. Employees are called cast members. Amenities include cordless phones, DVR players hooked up to HD televisions, Aveda brand bath and hair products, and down comforters and pillows. 104. Refer to Boutique Hotels. The service provided by employees at boutique hotels cannot be touched, seen, or felt in the same manner as the hair and bath products and can be described as: A. unknowable B. tangible C. intangible D. credible E. incomprehensible Answer: C Rationale: Services are intangible because they cannot be touched, seen, or felt in the same manner as goods. 105. Refer to Boutique Hotels. To evaluate the quality provided by boutique hotels, customers can assess the quality only after staying. This is which quality characteristic? A. experience B. relational C. credence D. search E. synergistic Answer: A Rationale: An experience quality is a characteristic that can be assessed only after use. 106. Refer to Boutique Hotels. Since boutique hotels are typically independently owned and/or part of small chains, people who seek out boutique hotels cannot be sure of what type of amenities or specific services they will be provided. In other words, the services provided by boutique hotels tend to be relatively high in: A. instability B. inseparability C. intangibility D. heterogeneity E. perishability Answer: D Rationale: There is no way to standardize the services offered, and in fact, one of the benefits of boutique hotels is the uniqueness of their offerings. 107. Refer to Boutique Hotels. When a guest asks an employee for directions to a local gallery or museum, the service received cannot be stored or warehoused. This service, therefore, is characterized as: A. inseparable B. perishable C. intangible D. unstable E. homogeneous Answer: B Rationale: Services cannot be stored, warehoused, or inventoried and, therefore, are perishable. 108. Refer to Boutique Hotels. Which type of processing occurs most often at a hotel that wants to provide personalized service? A. People processing B. Possession processing C. Information processing D. Equipment processing E. Supplementary processing Answer: A Rationale: People processing takes place when the service is directed at a customer. 109. Refer to Boutique Hotels. Boutique hotels encourage employees to develop personal relationships with guests. The hotels offer value-added services that are not available at traditional hotels. At which level of relationship marketing are these hotels operating? A. First B. Second C. Third D. Fourth E. Fifth Answer: C Rationale: The hotels are operating at the third level of relationship marketing, which involves creating value-added service not available elsewhere. 110. Refer to Boutique Hotels. Boutique hotels not only call their employees cast members, but they compete for their talent, provide them with all the training needed, stress teamwork, and give employees freedom to make decisions. In order to provide the level of service guests require, boutique hotels need to engage in _____ marketing. A. interactive B. relationship C. internal D. nonprofit E. affiliation Answer: C Rationale: Internal marketing is providing the employees with the desire and the ability to offer superior service. Cincinnati Museum Center The Cincinnati Museum Center is home to the Cincinnati History Museum, the Duke Energy Children’s Museum, and the Museum of Natural History and Science. The Cincinnati History Museum displays materials and related aspects of the history of Cincinnati and the surrounding region. Costumed interpreters throughout the museum allow visitors the unique opportunity to make a personal connection with the past. The educational and dramatic exhibits at Duke Energy Children’s Museum allow kids to climb, crawl, explore and learn about themselves and the world around them. And the Museum of Natural History and Science gives children hands-on, inquiry based experiences in science, technology, engineering and math. 111. Refer to Cincinnati Museum Center. The museum charges an admission, but only enough to defray its cost of maintaining the exhibits because it doesn’t have the typical business goals. Cincinnati Museum Center is an example of a(n): A. nonprofit organization B. subgovernment entity C. unprofitable transaction D. service-oriented firm E. social marketing company Answer: A Rationale: Nonprofit organizations exist to achieve some goal other than the usual business goals of profit, market share, or return on investment. 112. Refer to Cincinnati Museum Center. The visitors to the Cincinnati Museum Center represent the museum’s: A. service entity B. promotional tools C. target market D. benefit strength E. benefit complexity Answer: C Rationale: The people the museum wishes to attract are its target market. 113. Refer to Cincinnati Museum Center. The _____ makes it difficult for the Cincinnati Museum Center to prioritize its objectives and evaluate its performance. A. creation of a benefit strength B. lack of a financial objective C. inability to promote the exhibits D. absence of service qualities E. presence of intangible factors Answer: B Rationale: Prioritizing objectives and performance evaluation are more difficult without a financial objective. Service qualities are present in the exhibits themselves, and there is no reason why the Museum Center cannot promote the exhibits. 114. Refer to Cincinnati Museum Center. The product decisions associated with the Museum Center exhibits: A. deal with benefits that are both short term and simplistic. B. illustrate the unimportance of service quality. C. are very high involvement. D. prove word-of-mouth promotion would be the best communication tool to use. E. suggest nonfinancial objectives will be used to set prices. Answer: C Rationale: The benefits desired are long term, and service quality determines whether the objective is achieved. Word-of-mouth and financial objectives do not deal with product decisions. 115. Refer to Cincinnati Museum Center. A decision to use public service announcements about the various exhibits and attractions and to broadcast them in the Cincinnati area is an example of a _____ decision the Museum Center would need to make. A. product B. service C. distribution D. promotion E. price Answer: D Rationale: Communication with potential target markets is an example of promotion. Got Milk? Just because consumers live in a big city doesn’t mean they can’t have fresh milk right from the farm. Many dairy farms, such as Oberweis Dairy and Smith Brothers Farms, offer home delivery of fresh milk, just like in the old days. While some of these services are established and have been around since the 1920s, New York City–based Manhattan Milk started up in 2008. For $20 or more, depending on what is ordered, consumers anywhere in Manhattan can enjoy organic milk, butter, yogurt, and cheese from Pennsylvania Amish farms and eggs from Vermont. Unlike most store-bought milk, theirs is not homogenized and is free of hormones typically given to cows to produce more milk. 116. Refer to Got Milk? Manhattan Milk is an example of a: A. retail operation B. service C. wholesale intermediary D. facilitator E. manufacturer Answer: B Rationale: A service is the result of applying human or mechanical efforts to people or objects. 117. Refer to Got Milk? Manhattan Milk has a high _____ quality because its quality can only be assessed after its use. A. experience B. credence C. cognitive D. search E. perceptual Answer: A Rationale: An experience quality is a characteristic that can be assessed only after use. 118. Refer to Got Milk? One important aspect of Manhattan Milk’s service is the ability to deliver the milk within 48 hours of being milked from the cow. This service quality component of the ability to perform the service dependably, accurately, and consistently is referred to as: A. assurance B. empathy C. reliability D. tangibles E. consistency Answer: C Rationale: Reliability is the ability to perform a service right the first time. 119. Refer to Got Milk? Oberweis Dairy has been in business for over 80 years, and customers trust this provider. Which component of service quality does this trust exemplify? A. Creativity B. Assurance C. Empathy D. Tangibility E. Reliability Answer: B Rationale: Assurance is the knowledge and courtesy of employees and their ability to convey trust. 120. Refer to Got Milk? Customers place orders at Manhattanmilk.com two days before the delivery date. The Internet allows Manhattan Milk to customize each customer’s order to his or her preferences. This is an example of: A. a supplementary product. B. an intangible product. C. mass customization. D. a promotion strategy. E. internal marketing. Answer: C Rationale: Mass customization is a strategy that uses technology to deliver customized services on a mass basis. The Ritzy Canine The Ritzy Canine Carriage House looks like several other Manhattan boutique hotels. The lobby features a crystal chandelier, brocade-patterned wallpaper, gold-framed mirrors, and antique chairs. Room service and salon service are available as well as exercise facilities. There is also a masseuse on staff. The Ritzy Canine is a high-end doggy “hotel” and “spa” where dogs are the only guests, and they are lavishly cared for. The $175-a-night Windsor Suite even has a DVD player. Without such extras, a one-day visit will cost $40 to $50, depending on the size of the dog. 121. Refer to The Ritzy Canine. Before leaving a dog at The Ritzy Canine, its owner would more than likely inspect the facilities and use _____ qualities to evaluate the establishment. A. experience B. credence C. investigative D. search E. standardized Answer: D Rationale: A search quality is a characteristic that can be easily assessed before purchased. 122. Refer to The Ritzy Canine. While it is difficult for dog owners to evaluate the service their pooch will receive, they are impressed with the upscale design and amenities offered. To cope with the problems of service _____, the facilities at The Ritzy Canine are upscale in design to give the impression of excellent treatment. A. perishability B. homogeneity C. intangibility D. inseparability E. heterogeneity Answer: C Rationale: Intangibility is the inability of services to be touched, seen, tasted, heard, or felt in the same manner that goods can be sensed. 123. Refer to The Ritzy Canine. Research has shown that consumers evaluate service quality on five components. When customers rate the people who supervise the dog activities as being “dog people” who have an understanding of the needs of dogs and their owners, they are concerned with the _____ component. A. reliability B. responsiveness C. standardization D. empathy E. flexibility Answer: D Rationale: Empathy represents caring, individualized attention to customers. 124. Refer to The Ritzy Canine. Which category of service processing is used at The Ritzy Canine? A. Possession B. Mental stimulus C. Physical stimulus D. Information E. People Answer: A Rationale: The dogs are physical possessions of the customers. 125. Refer to The Ritzy Canine. Supervised doggy care is the _____ service product, and dog massages would be _____ service products. A. central; peripheral B. primary; secondary C. core; supplementary D. essential; superfluous E. supplementary, core Answer: C Rationale: The core service is the most basic benefit the consumer is buying (dog care), whereas the supplementary services are a group of services that support or enhance the core service (special treatment). 126. Refer to The Ritzy Canine. What type of promotion strategy would you expect The Ritzy Canine to use? A. Engaging in post purchase research B. Forging prepurchase commitments C. Using motivational behavior D. Creating a strong organizational image E. Using outsourced information sources Answer: D Rationale: The company creates an image by managing its environment. See the description of its lobby. Grizzly Watching Have you ever wanted to spend your vacation up close and personal with a grizzly bear? Great Bear Nature Tours is one of the many lodges that provide such a service. Great Bear, located in British Columbia, is actually a lodge built on a floating barge. The lodge can handle up to ten guests in what the Wall Street Journal calls rustic but upscale accommodations in double rooms that begin at a rate of $1,418 per night, including bear-watching tours. Although bear hunting still brings tourists to Canada, bear watching is a growing attraction. On a bear-watching tour, the guide, who is often unarmed, will take you close enough to get a good view of a grizzly in the wild. In fact, one of the favorite tours allows you to watch bears swipe at salmon as they swim upstream to spawning grounds. While bear watching sounds dangerous, bear attacks are not common. However, just to be careful, many tour guides carry pepper spray. Tour guides say they are able to keep tour members safe by watching the bears and reading their body movements. 127. Refer to Grizzly Watching. The bear-watching vacation experience can vary greatly depending on many factors, including the lodge itself and the tour guide. For example, lodging can run from a “rustic” room with little more than a bed to a plush room with all the modern amenities. This is an example of the _____ characteristic of services. A. intangible. B. inseparable. C. heterogeneous. D. perishable. E. homogeneous. Answer: C Rationale: Heterogeneity refers to the variability of the inputs and outputs of services, which causes services to tend to be less standardized and less uniform than goods. 128. Refer to Grizzly Watching. If Great Bear Nature Tours rated and rewarded its tour guides based on the number of bears seen by guests, the tour guides might be willing to accept more risk in seeking out the bears than guests would really want. This could result in which service quality gap? A. A gap between what customers want and what management thinks customers want B. A gap between what management thinks customers want and the quality specification that management develops to provide the service C. A gap between the service quality specifications and the service that is actually provided D. A gap between what the company provides and what the customer is told it provides E. A gap between the service that customers receive and the service they want Answer: B Rationale: The lodge has set a quality specification based on what it thinks guests want, number of bears seen, when guest want to see bears, but not in unsafe situations. 129. Refer to Grizzly Watching. The height of the bear-watching season is in the fall during salmon spawning season. Tourists love to watch bears capturing salmon as the salmon swim upstream to spawn. If a bear-watching tour lodge raises its prices during this season, it is practicing: A. revenue-oriented pricing B. operations-oriented pricing C. target market pricing D. patronage-oriented pricing E. cumulative pricing Answer: B Rationale: Operations-oriented pricing seeks to match supply and demand by varying prices. 130. Refer to Grizzly Watching. Instead of pricing guests separately for their room, breakfast, tours, and so on, Great Bear Nature Tours charges a _____ price of $1,418 per night, including bear-watching tours. A. bundled B. supplementary C. core service D. intangible E. unbundled Answer: A Rationale: Prices that reflect multiple aspects of related services are termed bundled prices. 131. Refer to Grizzly Watching. At Great Bear Nature Tours, the owners are careful to treat both guests and employees very well. In fact, some employees have commented that the employees are treated like they were guests. This is an example of: A. knowing your target market B. interior/exterior marketing C. internal marketing D. gap theory E. unbundled marketing Answer: C Rationale: Internal marketing means treating employees as customers. ESSAY 1. What is a service? Describe the importance of services to the U.S. economy and what the demand for services is projected to be in the coming years. Answer: A service is the result of applying human or mechanical efforts to people or objects. Services involve a deed, a performance, or an effort that cannot be physically possessed. Today, the service sector substantially influences the U.S. economy, accounting for 81 percent of both U.S. gross domestic product and U.S. employment. The demand for services is expected to continue. According the BLS, service occupations will be responsible for nearly all net-job growth through the year 2016. Much of this demand results from demographics. An aging population will need services provided by nurses, home health care professionals, therapists, and social workers. Two-career families need services provided by child-care workers, housecleaners, and landscapers. Also increasing will be the demand for information managers, such as computer engineers and systems analysts. 2. Compare and contrast search quality, experience quality, and credence quality. Describe their significance for the marketing of services. Answer: SEARCH QUALITIES are characteristics that can be easily assessed before purchase. The emphasized tangible characteristics of a service would be evaluated by the consumer in this case. Automatic versus standard transmission in a car, the color of the new paint on a house, or the size of a hamburger at a fast-food restaurant are all examples of search components. EXPERIENCE QUALITIES are those that can only be discerned after use. Services such as restaurants, housecleaning, and haircuts would fit in this category. CREDENCE QUALITIES are associated with specialized services for which the consumer might not have the experience or expertise to evaluate service quality even after consumption. Legal, tax, and medical services have credence qualities. These characteristics make it harder for marketers to communicate the benefits of an intangible service than to communicate the benefits of tangible goods. Thus, marketers often rely on tangible cues to communicate a service’s nature and quality. 3. Services have four unique characteristics that distinguish them from goods. Name and briefly define each of these four characteristics. Use the example of an airline to help describe each of the four characteristics. Answer: INTANGIBILITY. Services are intangible; that is, they cannot be touched, seen, tasted, heard, or felt in the same manner in which goods can be sensed. An airline provides travel services. Although there are some aspects that are tangible (such as the plane, the staff, and the food), the service benefit of fast travel is intangible. INSEPARABILITY. Services are often sold and then produced and consumed at the same time. For airline service, first the ticket is sold. At some later time, the service is produced and consumed simultaneously as the airplane flies to the destination point. HETEROGENEITY. Consistency and quality control are often difficult to achieve in a service because services are dependent on their labor force, and services are produced and consumed at the same time. In airline travel, a passenger cannot get exactly the same service experience from flight to flight. The length of the flight, flight comfort, flight attendants’ service, food, neighboring passengers, and many other factors will vary. PERISHABILITY. Services cannot be stored, warehoused, or inventoried. An empty airline seat produces no revenue and cannot be saved for the next flight. However, by overbooking flights, passengers can be saved up and used to fill later flights. 4. Describe the five gaps identified in the gap model of service quality that can cause problems in service delivery and influence customer evaluations of service quality. Discuss ways that marketers can close each gap. Answer: GAP 1 is the gap between what customers want and what management thinks customers want. An important step in closing this gap is to keep in touch with what customers want by doing research on customer needs and customer satisfaction. GAP 2 is the gap between what management thinks customers want and the quality specifications that management develops to provide the service. This gap is the result of management’s inability to translate customers’ needs into delivery systems within the firm, so management should concentrate on implementing service elements to better meet customers’ needs. GAP 3 is the gap between the service quality specifications and the service that is actually provided. Management needs to ensure that employees have the skills and the proper tools to perform their jobs, and training, encouraging teamwork, and hiring employees with the proper attitude can assist in this goal. GAP 4 is the gap between what the company provides and what the customer is told it provides. This is clearly a communications gap, so to close this gap, companies need to create realistic customer expectations through honest, accurate communication about what the firms can provide. GAP 5 is the gap between the service that customers receive and the service they want. This gap can be positive or negative, and providers need to have a better understanding of what customers want and what they actually deliver. 5. Define core services and supplementary services. Give an example of each for the following services: a wedding consultant, a lawn-care service, and tutoring service. Answer: The service is made up of a bundle of activities that include the core service, which is the most basic benefit being purchased, and supplementary services that support or enhance the core service. A wedding consultant’s core service would be planning a wedding. A supplementary service might be promising an error-free wedding. The wedding consultant could also include additional services like honeymoon planning. The core service for a lawn-care service is a green, mowed yard. Supplementary services could include cleaning the driveway, planting and maintaining flower borders, and watering when the home owner is out of town. The core service for a tutoring service is education. Supplementary services could include mentoring, improvement in other classes than the one for which tutoring is needed, and a better self-image. Students, of course, should generate many additional answers. 6. What are the five key issues to remember when developing distribution strategies for service organizations? Answer: • Convenience to customers • Enough outlets to satisfy customers’ needs • Whether to provide direct or indirect distribution • Location of service • Scheduling of when services will be available 7. There are four promotional strategies for dealing with the unique features of services. Name these strategies and give examples of each, using American Airlines (or another air carrier you are familiar with) as your example service. Answer: STRESSING TANGIBLE CUES. The most obvious tangible cue is the aircraft itself because this is an equipment-based service. Other tangible cues can be stressed during in-flight service, such as special meal and beverage services, provision of promotional items such as playing cards featuring the American Airlines (AA) logo, or other methods of making intangible services more tangible. USING PERSONAL INFORMATION SOURCES. Personal sources of information can help to reduce a customers’ perceived risk in choosing a service. An endorsement of a famous actor or political figure who prefers to use AA would be beneficial. AA should also seek to stimulate positive word-of-mouth communication among present and prospective customers. CREATING A STRONG ORGANIZATIONAL IMAGE. Service marketers should attempt to create strong organizational images. One way to create an image is to manage the evidence of the service. This would mean that AA should create a good appearance of the flight crews and other employees, keep the aircraft clean, and provide other tangible goods (items featuring the AA logo). ENGAGING IN POSTPURCHASE COMMUNICATION. Postpurchase communication can be accomplished with postcard surveys, in-flight brochures, or personal interviews. Customers should be shown that their feedback is sought and their patronage is appreciated. 8. Describe the two unique challenges faced by service providers as they decide on a pricing strategy to use. Answer: (1) In order to price a service, it is important to define the unit of service consumption. For example, should pricing be based on completing a specific service task or should it be time based? (2) For services that are composed of multiple elements, the issue is whether pricing should be based on a bundle of elements or whether each element should be priced separately. 9. Name and briefly define the three categories of pricing objectives for services. Answer: REVENUE-ORIENTED PRICING focuses on maximizing the surplus of income over costs. A limitation of this approach is that for many services, determining costs can be difficult. OPERATIONS-ORIENTED PRICING seeks to match supply and demand by varying prices. Prices can be increased during peak times and decreased during slow times. PATRONAGE-ORIENTED PRICING tries to maximize the number of customers using the service. Pricing according to different market segments’ ability to pay and offering methods of payment that increase the likelihood of purchase are strategies used for this pricing objective. 10. What is the difference between internal marketing and relationship marketing? Why are these types of marketing important to service marketing? Answer: INTERNAL MARKETING means treating employees as customers and developing systems and benefits that satisfy their needs. A firm that engages in internal marketing practices activities such as competing for talent, offering a vision, training employees, stressing teamwork, giving employees more freedom to make decisions, measuring and rewarding quality, and knowing employees’ needs. The quality of a firm’s employees is an important part of service quality. Employees who like their jobs and are happy with their company are more likely to deliver high-quality service. Internal marketing provides a strong foundation for the external marketing of services. RELATIONSHIP MARKETING is a means for attracting, developing, and retaining customer relationships. Satisfied customers who are loyal will buy additional services from the firm and are unlikely to switch to a competitor. Satisfied customers are also likely to engage in positive word-of-mouth communications, thereby bringing in new customers. Because many services involve a process of continuous interaction between the service organization and the customer, relationship marketing is an important strategy. It is often more cost-effective to hang on to existing customers than to attract new ones. 11. Relationship marketing can be practiced at three levels, with each level adding a new kind of bond with the customer. Name and describe each of these levels, using a legal service to illustrate each level. Answer: LEVEL 1: FINANCIAL BONDS. This level of relationship marketing is based mainly on pricing incentives that encourage customers to continue doing business with a firm. A legal service could offer discounts to long-term customers or to customers who use the service frequently or regularly. LEVEL 2: SOCIAL BONDS. This level also uses pricing incentives but additionally seeks to build social bonds with customers. This means staying in touch with clients, learning about their needs, and designing the service to meet those needs. For example, the legal service could send thank-you cards or call clients to see if they were satisfied and ask about other desired services. LEVEL 3: STRUCTURAL BONDS. This level of relationship marketing uses financial and social bonds but adds structural bonds. Structural bonds are developed by building value-added services into the service that are not readily available from other firms. This might include exclusive after-hours phone numbers for special clients; transportation services to and from legal hearings or the legal offices; fax, computer, or express document services; or permanent office space reserved for special clients’ use. 12. Briefly explain what service firms must do to be successful in the global marketplace. Answer: To be successful in the global marketplace, service firms must first determine the nature of their core product. Then the marketing mix elements (additional services, place, promotion, pricing, distribution) should be designed to take into account each country’s cultural, technological, and political environment. 13. What is nonprofit organization marketing? How does nonprofit marketing affect economic activity in the United States? Answer: Nonprofit organization marketing is the effort by nonprofit organizations to bring about mutually satisfying exchanges with target markets by using the same marketing activities as for-profit businesses. Unlike for-profit organizations, nonprofits do not have a bottom-line orientation. Nonprofit organizations account for over 20 percent of the economic activity in the United States. Government organizations collect revenues that amount to more than a third of the U.S. gross domestic product. 14. What is a nonprofit organization? Discuss the marketing activities performed by nonprofit organizations. Answer: A nonprofit organization is an organization that exists to achieve some goal other than the usual business goals of profit, market share, or return on investment. Nonprofit organizations include a wide variety of entities ranging from the government to private, not-for-profit churches. Although nonprofit organizations differ substantially in size and purposes and operate in quite different environments, most perform common marketing activities: • Identify customers (such as clients, patients, members, or sponsors) they wish to serve or attract • Explicitly or implicitly specify objectives • Develop, manage, and eliminate programs and services • Decide on prices (such as fees, donations, tuition, fares, or rates) to charge • Schedule events or programs and determine where they will be held or where services will be offered • Communicate their availability through brochures, signs, public service announcements, or advertisements 15. Discuss three issues relating to target markets that are unique to nonprofit organizations. Provide an example of each issue. Answer: APATHETIC OR STRONGLY OPPOSED TARGETS. While private-sector organizations develop market segments that are most responsive to their offerings, nonprofit organizations must develop marketing programs aimed at relatively unresponsive targets or people strongly opposed to receiving their services. This includes targets for services such as vaccinations, family planning, aid for substance abuse, and psychological counseling. PRESSURE TO ADOPT UNDIFFERENTIATED SEGMENTATION STRATEGIES. Some nonprofit organizations fail to recognize the advantages of segmentation, or they use an undifferentiated approach for apparent economies of scale and low per-capita costs. Other organizations are required to serve the maximum number of people by targeting the average user. Unfortunately, there are few “average” users, and most nonprofit organizations benefit from differentiated strategies. COMPLEMENTARY POSITIONING. Nonprofit organizations strive to provide services to those individuals or groups who are not adequately served by private-sector organizations. Thus, the goal is to complement rather than compete with the efforts of others. For example, a nonprofit organization would seek to identify underserved market segments (low-income families) and develop marketing programs that match their needs (low-cost health care or free vaccinations). 16. Discuss the three product-related distinctions between business and nonprofit organizations. Answer: BENEFIT COMPLEXITY. Nonprofit organizations market complex behaviors and ideas, such as the need to exercise or to stop smoking. The benefits are complex, long term, and intangible. BENEFIT STRENGTH. The benefit strength of many nonprofit offerings is quite weak or indirect. Most private-sector service organizations can claim direct, personal benefits. INVOLVEMENT. Nonprofit organizations market products that elicit a wide range of involvement levels, from very low involvement (don’t litter) to very high involvement (stop smoking). Traditional promotional tools may be inadequate to motivate product adoption in either of these extreme cases. 17. Discuss the limitations nonprofit marketers face when making promotion decisions and describe how they overcome these limitations. Answer: Many nonprofit organizations are prohibited from advertising, thus limiting their range of promotion options. Most federal agencies fall into this category. Other nonprofit organizations do not have the resources to retain advertising agencies, promotion consultants, or marketing staff members. Despite these limitations, nonprofit organizations sometimes have access to professional volunteers or donated media time or space. Professionals at advertising agencies or market research firms can donate services. Sales promotion activities that make use of other existing services or resources can be used. Public service announcements are similar to advertisements, but the media donate the time or space. Finally, some nonprofit agencies have been successful in offering forums for people to share experiences, which is an application of peer-to-peer communications. 18. List and describe the five key characteristics that distinguish the pricing decisions of nonprofit organizations from those of the profit sector. Answer: PRICING OBJECTIVES. Nonprofit organizations are concerned with revenue production but only to defray costs, not to achieve profits for distribution to stockholders. Income redistribution takes the form of equitable allocation among individuals and households or across geographic or political boundaries. NONFINANCIAL PRICES. Consumers often absorb nonmonetary costs rather than paying a monetary price. These nonmonetary costs may consist of the cost of time, embarrassment costs, or effort costs. INDIRECT PAYMENT. Indirect payment is received to cover a broad range of services. Indirect payment may take the form of taxes or other dues that help pay for free services provided by libraries, police departments, or fire departments. SEPARATION BETWEEN PAYERS AND USERS. Services distributed to one group are largely paid for by those in another group (usually those in a better financial situation). BELOW-COST PRICING. Often, products and services are provided below cost. Nonprofit organizations do this to remain socially responsible. Chapter 13—Supply Chain Management TRUE/FALSE 1. Supply chain management is completely producer driven. Answer: False Rationale: Supply chain management is completely customer driven. 2. Companies that use supply chain management techniques typically also use a push promotional strategy. Answer: False Rationale: They typically use a pull strategy. 3. Supply chain management is a key means of differentiation for a firm and a critical component in marketing and corporate strategy. Answer: True Rationale: Supply chain management plays a crucial role in differentiating a firm and is integral to both marketing and corporate strategy. 4. Management practices that reflect a highly coordinated effort between supply chain partners are said to be “integrated.” Answer: True Rationale: Integrated management practices in supply chain involve close coordination and collaboration among supply chain partners to optimize processes and achieve common goals efficiently. 5. Supplier integration occurs when multiple firms in a supply chain coordinate their activities and processes so that they are seamlessly linked to one another in an effort to satisfy the customer. Answer: True Rationale: Supplier integration involves coordinating activities and processes across multiple firms in a supply chain to enhance customer satisfaction. 6. The demand management process seeks to align supply and demand throughout the supply chain by anticipating customer requirements at each level of creating demand-related plans of action prior to actual customer purchasing behavior. Answer: True Rationale: The demand management process involves forecasting and planning to synchronize supply and demand across the supply chain by anticipating customer needs and creating plans accordingly, aiming to minimize mismatches between supply and customer demand. 7. The goals of the manufacturing flow management process are centered on generating, filing, delivering, and providing on-the-spot service for customer orders. Answer: False Rationale: These activities are associated with the order fulfillment process, not the manufacturing flow management process. 8. RFID tags are not one of the components of a logistics information system. Answer: False Rationale: RFID tags are one of several components of a logistics information system. 9. EDI is often used to replace paper documents with electronic documents containing the same information. Answer: True Rationale: Electronic Data Interchange (EDI) commonly replaces paper documents with electronic versions containing the same information. 10. Managing inventory from the supplier to the manufacturer is called distribution resource planning (DRP). Answer: False Rationale: This is materials requirement planning (MRP). 11. The goal of a materials-handling system is to handle products as infrequently as possible Answer: True Rationale: The goal of a materials-handling system is to minimize the handling of products to reduce costs and improve efficiency. 12. The six criteria important in selecting a transportation mode are relative cost, transit time, reliability, capability, accessibility, and traceability. Answer: True Rationale: The criteria for selecting a transportation mode include relative cost, transit time, reliability, capability, accessibility, and traceability. 13. Water transportation is by far the most expensive mode of transportation, but it is also the most reliable. Answer: False Rationale: It is the least expensive and least reliable transportation mode. See Exhibit 14.1. 14. Traceability with respect to transporting modes refers to the relative ease with which a shipment can be located and transferred. Answer: True Rationale: Traceability in transportation modes refers to the ability to track and locate shipments throughout the transportation process, ensuring efficient transfer and delivery. 15. Both air transportation costs and traceability are high when compared with any other transportation mode. Answer: True Rationale: See Exhibit 14.1. Air transportation is often associated with high costs and provides high traceability compared to other transportation modes due to its speed, reliability, and tracking capabilities. 16. Another name for outsourcing is contract logistics. Answer: True Rationale: Outsourcing and contract logistics are often used interchangeably to refer to the practice of hiring external parties to manage certain aspects of a company's logistics and supply chain operations. 17. Digital distribution is a distribution technique that includes any kind of product or service that can be distributed electronically. Answer: False Rationale: This is known as electronic distribution. 18. Experiments with three-dimensional printing have not been successful in industries such as auto parts. Answer: False Rationale: Experiments with three-dimensional printing have indeed been successful in certain industries. 19. Manufacturers who require suppliers to meet certain environmental standards are implementing sustainable supply chain management. Answer: True Rationale: Manufacturers that impose environmental standards on their suppliers are promoting sustainable practices within their supply chain, which is a key aspect of sustainable supply chain management. 20. One of the most critical global supply issues for importers of any size is coping with the legalities of trade in other countries. Answer: True Rationale: Coping with the legalities of trade in other countries is a critical issue for importers, as they must navigate various international laws, regulations, and trade agreements to ensure compliance and avoid legal risks. MULTIPLE CHOICE 1. The connected chain of all of the business entities, both internal and external to the company, that perform or support the logistics function is known as the: A. channel of distribution. B. intermediary link. C. physical distribution integration. D. reseller network. E. supply chain. Answer: E Rationale: A company’s supply chain includes all of the companies involved in all of the upstream and downstream flows of products, services, finances, and information, from initial suppliers (point of origin) to the ultimate customer (point of consumption). 2. _____ coordinates and integrates all of the activities performed by channel members into a seamless process, from the source to the point of consumption, resulting in enhanced customer and economic value. A. Yield management B. Channel power C. Materials handling D. Supply chain management E. Physical distribution Answer: D Rationale: This is the definition of supply chain management. 3. Peachtree Windows does not make a window until an order is received. It can promise delivery in five working days because of the excellent relationships it has established with its suppliers. The partnering relationship that Peachtree Windows has established with its suppliers indicates the use of: A. supply chain management B. yield management C. channel networks D. horizontal marketing channels E. intensive distribution strategy Answer: A Rationale: Supply chain management coordinates and integrates all the activities from raw materials to product consumption. 4. A large automotive after-market business wanted to improve its current situation, which is characterized by excessive inventory, incomplete information, high logistics costs, slow reactions to environmental change, and lost profits at the retail level. Would supply chain management be of any help to the business? A. Yes, supply chain management is designed to coordinate and integrate all the activities from raw materials to product consumption. B. No, supply chain management is only applicable to distribution strategies for products being sold to the consumer market. C. Yes, supply chain management would be very useful because it relies so heavily on benchmarking. D. Yes, supply chain management would alleviate all channel conflict. E. No, supply chain management requires an expensive investment of time and resources to make it worthwhile. Answer: A Rationale: The correct answer contains the definition of supply chain management. 5. All of the following accurately describe supply chain management EXCEPT: A. physical flow process. B. customer driven. C. provides enhanced customer and economic value. D. plays the role of communicator of customer demand. E. relies on a “push” strategy. Answer: E Rationale: In earlier days, standardized products were “pushed” down through the supply channel to the consumer, but now the system relies on customers to “pull” the product through the channel. 6. All of the following are benefits of supply chain management EXCEPT: A. increased flexibility of supply chain activities. B. improved customer service. C. reduced supply chain costs. D. decreased cash flow. E. decreased inventory. Answer: D Rationale: Cash flow would likely be increased due to higher revenues. 7. Which of the following is a key principle of supply chain management that means multiple firms working together to perform tasks as a single unified system rather than as several individual companies acting in isolation? A. Pareto principle B. 80/20 principle C. Systems approach D. Win–win approach E. Integrative approach Answer: C Rationale: This describes a systems approach. 8. For true supplier integration to occur, all of the following must occur EXCEPT: A. firms must choose their suppliers carefully B. supply chain partners must interact in ways that minimize waste and redundancy C. firms need to adopt a “push” system rather than a “pull” system of supply chain management D. firms must monitor each supplier’s strategic goals and operational capabilities E. both firms must have a stake in the outcomes of their interactions Answer: C Rationale: In earlier days, standardized products were “pushed” down through the supply channel to the consumer, but now the system relies on customers to “pull” the product through the channel. 9. _____ occurs when all the firms and business units participating in the supply chain appear as though they are reading from the same script. A. Strategic channel operation B. Supply chain integration C. Strategic MRP D. Demand management E. Extensive distribution integration Answer: B Rationale: Supply chain integration occurs when multiple firms in a supply chain coordinate their activities and processes so that they are seamlessly linked to one another in an effort to satisfy the customer. 10. When the Boeing Co. announced it would delay the introduction of its new 787, the CEO blamed the problem on the company’s supply chain. Airplanes use thousands of individual parts, and in an attempt to relieve the smaller, individual supply chain quandary, Boeing used major suppliers to construct large pieces of the plane. Boeing would have benefited from more _____ integration. A. channel B. supply chain C. MRP D. functional E. tactical distribution Answer: B Rationale: Supply chain integration occurs when multiple firms in a supply chain coordinate their activities and processes so that they are seamlessly linked to one another in an effort to satisfy the customer. 11. _____ integration is the ability of two or more companies to develop social connections that serve to guide their interactions when working together. A. Relationship B. Social C. Customer D. Materials and service supplier E. Technology and planning integration Answer: A Rationale: This is the definition of relationship integration. 12. Which type of integration implies the performance assessment of the supply chain should be taken as a whole while also holding each member of the supply chain accountable for meeting its own goals? A. Accountability B. Benchmarking C. Social D. Planning E. Measurement Answer: E Rationale: Measurement integration reflects the idea that performance assessments should assess the performance of the supply chain as a whole while holding each individual firm or business unit accountable for meeting its own goals. 13. Which of the following is an example of one of the six types of integration that are sought by firms interested in providing top-level service to their customers? A. Strategic integration B. Hierarchical integration C. Material and service supplier integration D. Global integration E. All of the above Answer: C Rationale: The five types of integration are relationship integration, measurement integration, technology and planning integration, material and service supplier integration, and customer integration. 14. In terms of firm-to-firm social interactions, _____ integration provides important benefits. A. relationship B. measurement C. customer D. production E. technology and planning Answer: A Rationale: Relationship integration provides benefits to firms regarding their social interactions with one another. 15. By achieving _____ integration across their supply chains, firms can use hardware and software systems to gain the information needed to execute short-term and long-term planning, and thereby make better operational decisions. A. relationship B. social C. functional D. technology and planning E. internal operations Answer: D Rationale: Technology and planning integration is the creation and maintenance of information technology systems that connect managers across and through the firms in the supply chain, supporting supply chain activities that best satisfy customer demands. 16. Through the use of automatic shipping notices, Ace Hardware is effective at removing unnecessary costs from its supply chain. According to a VP of inventory for the retailer, “Information gathering that used to take three or four phone calls now is available immediately.” This is an example of the use of _____ integration. A. relationship B. social C. functional D. technology and planning E. internal operations Answer: D Rationale: By achieving technology and planning integration across their supply chains, firms can gain the information needed to execute short-term and long-term planning. The process is the creation and maintenance of information technology systems that connect managers across and through the firms in the supply chain, with the goal of supporting supply chain activities that best satisfy customer demands. 17. Food producers love big boxes because they serve as billboards on store shelves. Walmart wants to change this practice and promises suppliers that their shelf spaces won’t shrink even if their boxes do. As a result, some of its vendors have reengineered their packaging. This is an example of _____ integration. A. relationship B. social C. functional D. operational E. material and service supplier Answer: E Rationale: Both the retailer and its suppliers have a common vision of the total value creation process. 18. _____ integration holds that performance assessments should be transparent and measurable across the borders of different firms. A. Measurement B. Social C. Functional D. Relationship E. Material and service supplier Answer: A Rationale: Measurement integration reflects the idea that performance assessments should be transparent and measurable across the borders of different firms 19. Nabisco has linked seamlessly with its parts manufacturers and outsourced customer call centers to provide streamlined work processes and a high-quality customer experience. This seamless linking of Nabisco’s strategic business units toward a common vision is an example of _____ integration. A. relationship B. social C. functional D. hierarchical E. Material and service supplier Answer: E Rationale: Material and service supplier integration requires firms to link seamlessly to those outsiders that provide goods and services to them so that they can streamline work processes and thereby provide smooth, high-quality customer experiences. 20. Sobha Developers has begun to develop distinctive value-added offerings to its customers—starting with the oldest and most consistent ones. Sobha is working toward stronger _____ integration. A. relationship B. social C. functional D. operational E. customer Answer: E Rationale: Customer integration is a competency that enables firms to offer long-lasting, distinctive, value-added offerings to those customers who represent the greatest value to the firm or supply chain. 21. Highly integrated supply chains (those that are successful in achieving many or all of these types of integration) have been shown to be better at: A. satisfying customers. B. managing costs. C. delivering high-quality products. D. enhancing productivity. E. All of the above Answer: E Rationale: Highly integrated supply chains have been shown to be better at satisfying customers, managing costs, delivering high-quality products, and enhancing productivity, all of which translate into greater profitability for the firms and their partners working together in the supply chain. 22. _____ is (are) composed of bundles of interconnected activities that stretch across firms in the supply chain. A. ABC segmentation strategies B. Activity-based costing C. Customer relationship management D. Business processes E. Customer service activities Answer: D Rationale: This is the definition of business processes. 23. There are eight critical business processes on which supply chain managers must focus. Which of the following is one of those processes? A. Returns management B. Sales promotion activities C. Distribution intensity D. Public relations activities E. Environmental assessments Answer: A Rationale: Returns management is one of the eight critical business processes on which supply chain managers must focus. 24. Of the eight critical business processes on which supply chain managers must focus, which process allows companies to prioritize their marketing focus on different customer groups according to each group’s long-term value to the company or the supply chain? A. Customer relationship management B. Returns management C. Product development and commercialization D. Order fulfillment E. Demand management Answer: A Rationale: Customer relationship management is designed to identify and build relationships with good customers. 25. Of the eight critical business processes on which supply chain managers must focus, the _____ process presents a multi-company, unified response system to the customer whenever complaints, concerns, questions, or comments are voiced. A. manufacturing flow management B. customer service management C. product development and commercialization D. order fulfillment E. demand management Answer: B Rationale: The customer service management process is designed to ensure that customer relationships remain strong. 26. By working closely with its suppliers and listening to its customers, Target has improved the service encounter to a level that was not thought possible in mass retailing. This is an example of which of the eight critical processes performed by supply managers? A. Manufacturing flow management B. Customer service management C. Product development and commercialization D. Order fulfillment E. Demand management Answer: B Rationale: Of the eight critical business processes on which supply chain managers must focus, the customer service management process presents a multi-company, unified response system to the customer whenever complaints, concerns, questions, or comments are voiced. 27. Which business process is based on the activities involved in customer data collection, forecasting of future demand, synchronization of supply and demand by comparing production capacity to forecasts, and the development of activities to “smooth out” demand? A. Manufacturing flow management B. Customer service management C. Product development and commercialization D. Order fulfillment E. Demand management Answer: E Rationale: The demand management process is the alignment of supply and demand throughout the supply chain. 28. The activities that enable the demand management process to work include: A. customer data collection. B. forecasting of future demand. C. sales and operations planning. D. the development of activities that tend to “smooth out” demand. E. All of the above. Answer: E Rationale: All of the listed activities are necessary for the demand management process to work properly. 29. All of the following are activities related to the manufacturing flow process EXCEPT: A. improve manufacturing output in terms of quality, delivery speed, and flexibility B. updating account status following shipment C. leveraging the capabilities held by multiple members of the supply chain D. creating flexible agreements with suppliers and shippers E. producing of goods and service Answer: B Rationale: This is not an activity related to the manufacturing flow process. 30. The amount of time between order placement, receipt of the customer’s payment, and order shipment is known as the: A. 3PL cycle B. product life cycle C. purchase cycle D. order-to-cash cycle E. float Answer: D Rationale: When the order fulfillment process is managed diligently, the amount of time between order placement and receipt of the customer’s payment after order shipment, (known as the order-to-cash cycle), is minimized as much as possible. 31. A firm that provides functional logistics services to others is called a(n): A. third-party logistics firm (3PL) B. intermediate distributor C. industrial distributor D. merchant wholesaler E. ABC firm Answer: A Rationale: Because many firms do not view order fulfillment as a core competency, they often outsource this function to a third-party logistics firm (3PL) that specializes in the order fulfillment process. 32. ProLog is a firm that specializes in warehousing and fulfilling orders for firms as diverse as those supplying parts for manufacturers of complicated missile defense systems to those manufacturing pet-care products. Firms contract ProLog to manage part or all of their order fulfillment process. ProLog is an example of a: A. merchant wholesaler B. retailer C. distributor D. third-party logistics firm (3PL) E. manufacturing flow process manager Answer: D Rationale: Third-party logistics firms (3PLs) are contracted to manage part or all of another firm’s order fulfillment process. 33. Which business process ensures that firms in the supply chain have the resources they need? A. Manufacturing flow management B. Customer service management C. Product development and commercialization D. Order fulfillment E. Demand management Answer: A Rationale: The manufacturing flow management process is concerned with ensuring that firms in the supply chain have the needed resources to manufacture with flexibility and to move products through a multi-stage production process. 34. Which business process supports manufacturing flow by identifying and maintaining relationships with highly valued suppliers? A. Supplier relationship management B. Customer service management C. Product development and commercialization D. Order fulfillment E. Demand management Answer: A Rationale: The supplier relationship management process is closely related to the manufacturing flow management process and contains several characteristics that parallel the customer relationship management process. 35. Specialty Blades designs precision cutting devices for varied applications. On one recent project, they worked closely with a medical device company, developing a combination scalpel and stapler that slices tissue very neatly close to the staples, which results in less traumatic procedures and quicker patient recoveries. This is an example of which of the eight critical processes performed by supply chain managers? A. Supplier relationship management B. Customer service management C. Product development and commercialization D. Order fulfillment E. Demand management Answer: C Rationale: The product development and commercialization process includes the group of activities that facilitates the joint development and marketing of new offerings among a group of supply chain partners. 36. Mattel, the world’s biggest toymaker, ordered 18 million toys made in China to be recalled because of hazards such as the use of lead paint. Mattel’s _____ process allowed it to handle the product recall efficiently. A. order fulfillment B. returns management C. product commercialization D. demand management E. product fulfillment Answer: B Rationale: The returns management process enables firms to manage the returned merchandise while minimizing returns-related costs. 37. Companies operating under a demand-supply integration philosophy are better at their business because all of the different divisions within the company _____. A. work to minimize complexity B. have their own management systems C. compete with each other to earn revenue D. play from the same sheet of music E. operate under a strict hierarchy Answer: D Rationale: Under the DSI philosophy, functional areas communicate frequently and are synchronized. 38. New product idea generation and testing are related to which of the following processes? A. Demand management B. Order fulfillment C. Customer service management D. Product development and commercialization E. Customer relationship management Answer: D Rationale: The product development and commercialization process includes the group of activities that facilitates the joint development and marketing of new offerings among a group of supply chain partners. 39. Returns average as much as _____ percent of sales volume in certain cases. A. 2 B. 6 C. 10 D. 25 E. 40 Answer: E Rationale: Returns reach up to 40 percent of sales volume in certain industries such as apparel e-retailing. 40. The goal of most sourcing and procurement activities is to: A. reduce the cost of raw materials and supplies. B. provide satisfying customer service C. make sure the right amount of product is kept on hand D. identify, sort, and label goods E. move products from supplier to producer and from producer to buyer Answer: A Rationale: This is the goal of most sourcing and procurement activities. 41. All of the following are logistics components of the supply chain EXCEPT: A. trade promotions B. production C. inventory control D. warehousing and materials handling E. order processing Answer: A Rationale: The logistics function consists of several interrelated and integrated logistical components: (1) sourcing and procurement of raw materials and supplies, (2) inventory control, (3) order processing, (4) production, (5) warehousing and materials handling, and (6) transportation. 42. Which of the following provides the technological link connecting all of the logistics components of the supply chain? A. Logistics information system B. Materials requirement plan C. EDI D. RFID chips E. Distribution resources strategy Answer: A Rationale: The components of the system include software for materials acquisition and handling, warehouse-management and enterprise-wide solutions, data storage and integration in data warehouses, and so on. 43. One-quarter of Ace’s 60,000 SKUs are vendor managed through software called the “Collaboration” program. This software allows certain vendors to go online to get information to build their forecasts. The result of this application is fewer shipments, fuller pallets, increased pallets per truck, and a reduction in freight. Collaboration is part of Ace’s: A. logistics information system B. materials requirement plan C. EDI D. RFID chips E. distribution resources strategy Answer: A Rationale: A logistical information system provides the technological link connecting all of the logistics components of the supply chain 44. The supply chain team does all of the following EXCEPT: A. communicating, coordinating, and cooperating extensively. B. embracing all parties who participate in moving the product to market. C. typically cutting across organizational boundaries. D. providing financing for distributors. E. orchestrating the movement of goods, services, and information from the source to the consumer. Answer: D Rationale: The supply chain team does all of these, but it does not provide financing for distributors. 45. Which of the following is a member of a supply chain team that moves just-picked cotton to finished clothing and then to the consumer of that clothing? A. The warehouse in which the cotton was temporarily stored B. The ad agency that develops the promotion for the clothing C. The farmer who picks the cotton D. The trucking company that hauls the cotton to the clothing manufacturer E. All of the choices Answer: E Rationale: The supply chain team is composed of everyone who participates in the movement from raw material to consumer, including all external participants like ad agencies and transportation companies. 46. In which logistical component of the supply chain will you find electronic data interchange a common feature? A. Production scheduling B. Transportation C. Order processing D. Materials handling E. All of these Answer: C Rationale: Order processing is becoming more automated through the use of computer technology known as electronic data interchange (EDI). 47. _____ is information technology that replaces the paper documents that usually accompany business transactions. A. Business prioritizing B. Electronic data interchange C. Just-in-time inventory D. Real-time inventory E. Direct sourcing Answer: B Rationale: Electronic data interchange (EDI) is information technology that replaces the paper documents that usually accompany business transactions. 48. _____ is a method of developing and maintaining an adequate assortment of materials or products to meet a manufacturer’s or a customer’s demand. A. The materials-handling system B. An inventory control system C. Direct sourcing D. Contract logistics E. Warehousing Answer: B Rationale: This is the definition of an inventory control system. 49. Which of the following is the inventory control system that manages the replenishment of raw materials, supplies, and components from the supplier to the manufacturer? A. Electronic data interchange (EDI) B. Distribution resource planning (DRP) C. Third-party logistics firm (3PL) D. Materials requirement planning (MRP) E. Activity-based costing (ABC) Answer: D Rationale: This is the definition of materials requirement planning (MRP), which is also referred to as materials management. 50. The goal of inventory management is to: A. minimize inventory levels while maintaining an adequate supply of goods to meet customer demands. B. maximize inventory levels to prevent any possibility of stockouts. C. determine order timing and order quantity. D. estimate proper usage rates. E. integrate JIT to reduce risks of obsolescence, theft, and damage. Answer: A Rationale: Inventory decisions affect physical distribution costs and service levels. The goal is to balance minimum inventory levels while meeting customer needs for stock. 51. In which logistical component of the supply chain will you find distribution resource planning (DRP) a common feature? A. Production scheduling B. Transportation C. Order processing D. Materials handling E. Inventory control Answer: E Rationale: Distribution resource planning (DRP) is an inventory control system that manages the replenishment of goods from the manufacturer to the final consumer. 52. ZuTech makes sensitive monitors for measuring exposure to hazardous materials. ZuTech has suffered substantial losses when moving products from the assembly line to the warehouse and shipping docks. ZuTech should consider reducing the number of times an item is moved in the warehouse by installing a(n): A. inventory control system. B. materials-handling system. C. safety procedure to manage SKUs. D. order-processing system. E. outsourcing system. Answer: B Rationale: The goal of the materials-handling system is to move items quickly with minimal handling. 53. All of the following are types of inputs used in both materials requirement planning and distribution resource planning EXCEPT: A. the source of funding. B. lead times. C. the mode of transportation to be used. D. outstanding orders. E. sales forecasts. Answer: A Rationale: Both inventory systems use various inputs, such as sales forecasts, available inventory, outstanding orders, lead times, and mode of transportation to be used, to determine what actions must be taken to replenish goods at all points in the supply chain. 54. _____ provides time utility to buyers and sellers and aids manufacturers in managing supply and demand. A. Containerization B. Distribution C. Storage D. Direct sourcing E. Channelization Answer: C Rationale: Storage is necessary to match supply with demand and to provide a level of safety stock. 55. Which of the following moves inventory into, within, and out of a warehouse? A. An inventory control system B. A multichannel system C. An ERP control system D. A freight forwarder E. A materials-handling system Answer: E Rationale: A materials-handling system is a method of moving inventory into, within, and out of the warehouse. 56. The goal of any effective materials-handling system is to: A. reduce the time of the order-processing system B. optimize the inventory control system C. increase safety procedures D. move items quickly with minimal handling E. reduce the length of time a product is in the transportation subsystem Answer: D Rationale: Each time a product is handled, the cost and risk of damage increase. 57. All of the following are criteria used to choose a mode of transportation EXCEPT: A. diversity B. cost C. transit time D. reliability E. traceability Answer: A Rationale: Criteria used include cost, transit time, reliability, capability, accessibility, and traceability. 58. The mode of transportation that provides the most accessibility is: A. pipelines B. air freight C. truck freight D. water transport E. railroad Answer: C Rationale: Motor carriers (trucks) can go to more locations than any of the other modes of transportation. See Exhibit 14.1. 59. The relative ease with which a shipment can be located and transferred is: A. dependability B. traceability C. capability D. transit time E. observability Answer: B Rationale: This defines traceability. 60. The growers of Vidalia onions have determined that their chief priority for choosing a transportation mode within the United States is transit time. However, this must be tempered by practical cost considerations. In light of these considerations, Vidalia onion growers should use _____ to ship their onions. A. railroads B. motor carriers (trucks) C. airplanes D. water transportation E. pipelines Answer: B Rationale: Motor carriers are not as fast as airplanes, but they are less expensive, particularly when dealing with a bulky product like onions. See Exhibit 14.1. 61. One major advantage of pipelines over other modes of transportation is: A. availability B. points served C. flexibility D. speed E. reliability Answer: E Rationale: Pipelines are rarely interrupted by weather, traffic, or labor disputes. See Exhibit 14.1. 62. Which of the following describes a disadvantage associated with water transportation? A. Cannot be used for international trade B. High cost C. Capability D. Lack of speed E. Product weight restrictions Answer: D Rationale: Shipping via water is the slowest mode of transportation. See Exhibit 14.1. 63. Which of the following transportation modes can most easily determine the location of a package en route between shipper and receiver? A. Air B. Truck C. Rail D. Pipe E. Water Answer: A Rationale: In addition to speed, air transport offers an important additional benefit to shippers, that being the ease of tracing shipments. See Exhibit 14.1. 64. The Canadian lobster harvest requires special handling coupled with rapid transportation to ensure the cargo reaches destinations alive. The favored transportation mode would be: A. air B. water C. truck D. pipeline E. rail Answer: A Rationale: Air transportation ensures the speed necessary for lobster delivery. (Air Canada has developed special handling and shipping techniques for the lobsters.) See Exhibit 14.1. 65. All of the following are business trends currently having a strong influence on the job of the supply chain manager EXCEPT: A. bar coding and radio frequency technology. B. the advancement of computer technology. C. the outsourcing of logistical functions. D. replenishment benchmarking. E. electronic distribution. Answer: D Rationale: This is a fictitious term. 66. A manufacturer’s or supplier’s use of an independent third party to manage an entire function of the logistics system, such as transportation, warehousing, or order processing, is called: A. outsourcing. B. mediation. C. intermediary distribution. D. benchmarking. E. disintermediation. Answer: A Rationale: This is the definition of outsourcing, also called contract logistics. 67. Papa John’s pizza chain uses UPS trucks to carry supplies to its franchises. Papa John’s does not own the trucks but has contracted with UPS to have all of the trucks it uses in its deliveries to Papa John’s establishments painted with the Papa John’s logo and company colors. Papa John’s is using: A. a horizontal distribution channel. B. a selective distribution channel. C. outsourcing. D. multi-sourcing. E. a portable inventory control system. Answer: C Rationale: Papa John’s supplier is an outsourced, independent third party. 68. Volvo, working with FedEx, set up a warehouse in Memphis with a complete stock of truck parts. A dealer needing a part for an emergency phones a toll-free number, and the part is delivered by FedEx within 24 hours. This is an example of how manufacturers are using: A. outsourcing. B. cross-docking. C. disintermediation. D. piggybacking. E. an integrated demand channel. Answer: A Rationale: With outsourcing, a company would use an independent third party to operate some or all of its logistics system. 69. A manufacturer of automobile component parts that has turned over all transportation functions to an independent third party in its supply chain has engaged in: A. creating a dependent materials-handling system. B. outsourcing. C. removing entry barriers. D. task deployment. E. channel rotation. Answer: B Rationale: Outsourcing (or contract logistics) is a manufacturer’s or supplier’s use of an independent third party to manage an entire function of the logistics system, such as transportation, warehousing, or order processing. 70. When Rick purchased tickets to a concert on the Internet, it was an example of: A. single-sourcing distribution. B. one-way channelization. C. electronic distribution. D. mass customization. E. aggregated marketing. Answer: C Rationale: Electronic distribution includes any kind of product or service that can be distributed electronically. Tickets are just one of many types of products and services that can be distributed electronically. 71. Pepisco and Waste Management have developed a plan to improve the reverse supply chain for individual beverage bottles not most commonly thrown in trash cans and not recycled. Together Pepsico and Waste Management have plans to deploy thousands of “Dream Machines” in both permanent and temporary locations that will take in recycled bottles and provide rewards to consumers who participate. These companies are practicing: A. sustainable supply chain management B. contract logistics C. intermediary distribution D. eco-marketing E. environmental mediation Answer: A Rationale: Sustainable supply chain management is the integration of environmentally conscious thinking into all phases of key supply chain management processes. 72. All of the following are examples of sustainable supply chain management EXCEPT: A. revising routes taken by delivery drivers to minimize fuel consumption B. buying goods from a supplier located in a developing country in Africa C. limiting excess packaging materials from shipments D. shipping raw materials to point of use rather than point of manufacture E. instructing drivers to limit the amount of time trucks sit idling Answer: B Rationale: Buying local goods reduce carbon pollution to the air from fuel. This outcome is not achieved when goods are purchased thousands of miles away. Fishy Business DNA testing has revealed that sushi bar customers are not always getting what they pay for. Scientists at the American Museum of Natural History tested fish from 31 sushi bars and found that the tuna coveted by many diners is often a cheaper substitute, a banned fish, or an endangered species. This could be the result of an honest mistake in the supply chain, or it could be a deliberate attempt to deceive buyers. However, researchers have uncovered a DNA “bar code” that can accurately identify the type of fish. While still a ways off, the scientist who developed the tuna DNA bar code envisions a small device that can provide instantaneous species identification, which will result in restaurants ultimately providing what their customers are demanding and paying for. 73. Refer to Fishy Business. The way a restaurant coordinates and integrates the activities performed by its supply chain members into a collection of seamless end-to-end processes is an example of: A. yield management. B. channel power. C. materials handling. D. supply chain management. E. physical distribution. Answer: D Rationale: Supply chain management coordinates and integrates all of the activities performed by channel members into a seamless process, from the source to the point of consumption, resulting in enhanced customer and economic value 74. Refer to Fishy Business. If all the firms involved with getting the fish from the ocean onto consumers’ plates work together to perform tasks as a single unified system, which of the following are they using? A. A synergistic approach B. A systems approach C. A primary approach D. A networked approach E. An interdependent approach Answer: B Rationale: A systems approach is a key principle of supply chain management and involves multiple firms working together to perform tasks as a single, unified system rather than as several individual companies acting in isolation. 75. Refer to Fishy Business. In the future, a device will allow each member of the supply chain for fresh fish to verify accountability for providing the species of fish the customer ordered. Which element of supply chain integration is this when each individual firm or business unit is accountable for meeting its own goals regarding providing the fish it says it is providing? A. Benchmark integration B. Measurement integration C. Planning integration D. Management integration E. Leagile integration Answer: B Rationale: Measurement integration is the performance assessment of the supply chain as a whole that also holds each individual firm or business unit accountable for meeting is own goals. 76. Refer to Fishy Business. In the future, suppliers of fresh fish will be able to use electronic bar codes to verify the fish species for members of the supply chain. This information can be incorporated into the information technology used in the inventory management and control business process. This information technology is called: A. activity-based costing (ABC). B. electronic data interchange (EDI). C. benchmarking. D. a materials-handling system. E. procurement. Answer: B Rationale: Electronic data interchange (EDI) is the information technology that replaces the paper documents that usually accompany business transactions, such as purchase orders and invoices, with electronic transmission of the needed information to reduce inventory levels, improve cash flow, streamline operations, and increase the speed and accuracy of information transmission. 77. Refer to Fishy Business. Tsunami, a sushi bar, takes every precaution to make sure it does not purchase any species of fish that have been classified as endangered. It partners with its suppliers to verify—as much as is possible—the species of the fish it purchases. This is an example of: A. sustainable supply chain management B. contract logistics C. intermediary distribution D. eco-marketing E. environmental mediation Answer: A Rationale: Sustainable supply chain management is the integration of environmentally conscious thinking into all phases of key supply chain management processes. Whirlpool and Lowe’s Every week, Whirlpool and Lowe’s have an online conference call to discuss what appliances are selling in the stores and what capacity Whirlpool has to make product. Often, the discussion revolves around a particular model that is selling at a higher than expected rate in Lowe’s. This often results in Whirlpool quickly flexing its supply chain to make more of the high-selling product and delivering it to Lowe’s customers. However, sometimes the answer is that Whirlpool and/or its suppliers do not have the capacity to make more of the product in question. It then becomes a question of demand shaping for Lowe’s. What promotions, in-store displays, and sales incentives can Lowe’s implement to shift demand from the capacity-constrained model to one that the supply chain has more capacity to deliver? In this way, Lowe’s, Whirlpool, and their suppliers manage supply and also demand. 78. Refer to Whirlpool and Lowe’s. A tremendous amount of _____ exists between Lowe’s and Whirlpool, which is evident by the long-lasting, value-added offerings Whirlpool provides to Lowe’s. A. channel manipulation B. customer integration C. resources integration D. yield management E. corporate socialization Answer: B Rationale: Customer integration is a competency that enables Whirlpool and Lowe’s to offer long-lasting, distinctive, value-added offerings to those customers who represent the greatest value to the firm. 79. Refer to Whirlpool and Lowe’s. Which business process is Whirlpool and Lowe’s using when they work together to align supply and demand throughout the supply chain? A. Dependent materials handling B. Outsourcing C. Warehousing D. Demand management E. Channel rotation Answer: D Rationale: Demand management seeks to anticipate customer requirements at each level and create demand-related plans of actions such as Lowe’s promotions. 80. Refer to Whirlpool and Lowe’s. Whirlpool contracts with another firm to manage its order fulfillment process with Lowe’s. This contracted firm is referred to as a(n): A. third-party logistics firm (3PL) B. drop shipper C. mediator D. independent agent E. interloper Answer: A Rationale: A third-party logistics firm (3PL) is contracted to manage part or all of another firm’s order fulfillment process. 81. Refer to Whirlpool and Lowe’s. Given that accessibility and reliability are keys to Lowe’s marketing strategy, what mode of transportation is most likely used in this relationship? A. Air B. Water C. Truck D. Pipeline E. Rail Answer: C Rationale: Trucks excel at providing users with product accessibility and reliability. ESSAY 1. How has the use of supply chain management resulted in a radical reformulation of traditional marketing, production, and distribution functions? Answer: The use of supply chain management has resulted in a customer-driven view of traditional marketing, production, and distribution functions. It has reversed the flow of demand from a “push” to a “pull.” Through the channel partnering of suppliers, manufacturers, wholesalers, and retailers along the whole supply chain who work together toward the common goal of creating customer value, supply chain management allows companies to respond with the unique product configuration and mix of services demanded by the customers. Thus, supply chain management communicates customer demands from the point of sale all the way to the supplier. In addition, supply chain management engineers the timely and cost-effective flow of goods through the entire supply pipeline. 2. What is relationship integration? Describe a supply chain that has achieved high relationship integration. Answer: Relationship integration is the ability of two or more companies to develop social connections that serve to guide their interactions when working together. When multiple firms in a supply chain have achieved high relationship integration, member firms are able to perform at a higher level than would be possible for any of the partners working alone. Relationally integrated supply chains tend to be managed by social guidelines. In addition, relationally integrated supply chains tend to be open to information sharing across firm/business unit boundaries so that everyone involved in the supply chain can clearly see the problems and opportunities that arise when firm-to-firm collaboration is taking place. 3. Explain what is meant by supply chain integration and list the five types of integration sought by firms interested in providing top-level service to customers. Answer: Supply chain integration is when multiple firms in a supply chain coordinate their activities and processes so that they are seamlessly linked to one another in an effort to satisfy the customer. Five types of integration are sought by firms interested in providing top-level service to customers: (1) relationship integration, (2) measurement integration, (3) technology and planning integration, (4) material and service supplier integration, and (5) customer integration. 4. List the eight critical business processes on which supply chain managers must focus. Answer: 1. Customer relationship management 2. Customer service management 3. Demand management 4. Order fulfillment 5. Manufacturing flow management 6. Supplier relationship management 7. Product development and commercialization 8. Returns management 5. What is the customer service management process? List five activities that may be considered part of this process. Answer: The customer service management process presents a multi-company, unified response system to the customer whenever complaints, concerns, questions, or comments are voiced. It includes such activities as taking customer call or requests for information, developing response procedures, amending orders, aiding in installation, measuring customer satisfaction with the service encounter, and checking warranty status. Students may have other equally correct ideas about this process. 6. Describe the manufacturing flow management process and discuss the goals of this process. Answer: The manufacturing flow management process is a process that ensures that firms in the supply chain have the resources they need. The goals of the manufacturing flow process are centered on leveraging the capabilities held by multiple members of the supply chain to improve overall manufacturing output in terms of quality, delivers speed, and flexibility, all tied to increasing profitability. 7. Discuss the financial implications of the returns management process. Answer: The returns management process enables firms to manage volumes of returned product efficiently while minimizing returns-related costs and maximizing the value of the returned assets to the firms in the supply chain. These activities can also impact the firm’s financial position negatively if this process is not efficiently handled. Some students may recognize that returns management creates additional marketing and customer service touch points that can be leveraged for added customer value above and beyond normal sales and promotion-driven encounters. 8. List the components of the logistics function in the supply chain. Answer: 1. Sourcing and procurement 2. Inventory control 3. Order processing 4. Production 5. Warehousing and materials handling 6. Transportation 9. What is an inventory control system? Discuss the objective of inventory management, and describe materials requirement planning (MRP) and distribution resource planning (DRP). Answer: An inventory control system develops and maintains an adequate assortment of products to meet a manufacturer’s or a customer’s demands. The objective of inventory management is to balance minimum inventory levels (to reduce costs) while maintaining an adequate supply of goods to meet customer demand. Managing inventory from the supplier to the manufacturer is called materials requirement planning (MRP). This is an inventory control system that manages the replenishment of raw materials, supplies, and components from the supplier to the manufacturer. The system that manages the replenishment of goods from manufacturer to end user is commonly referred to as distribution resource planning (DRP). Both inventory systems use inputs, such as sales forecasts, available inventory, outstanding orders, lead times, and mode of transportation to be used, to determine what actions must be taken to replenish goods at all points in the supply chain. 10. Distribution managers select different transportation modes based on several distinct criteria. List and briefly define these criteria, and name the best and worst transportation mode for each criterion. Answer: COST. The total amount a specific carrier charges to move the product from the point of origin to the destination. Airways have the highest cost, while water is the cheapest mode. TRANSIT TIME. The total time a carrier has possession of goods. Water is the slowest, requiring the most transit time, and air is the fastest mode. RELIABILITY. The consistency with which the carrier delivers goods on time and in an acceptable condition. Pipelines are the most reliable, while water transport is the least reliable. CAPABILITY. The ability of the carrier to provide the appropriate equipment and conditions for moving specific kinds of goods. Water has the most capability, while pipelines have the least. ACCESSIBILITY. A carrier’s ability to move goods over a specific route or network. Trucks have the best accessibility, and pipelines have the most limited accessibility. TRACEABILITY. The relative ease with which a shipment can be located and transferred. Air is the best mode, and pipeline is the worst. 11. Describe outsourcing. How does outsourcing affect the efficient deployment of supply chain management? Answer: With outsourcing, or contract logistics, a company turns over the entire function of buying and managing transportation to a third party, usually an independent carrier. Third-party carriers have economies of scale because of consolidating transportation modes and can also offer extra services. Many corporations use contract logistics due to budget constraints, mergers and divestitures, and increased globalization. Outsourcing can also apply to contract warehousing. Outsourcing allows companies to cut inventories, locate stock at fewer plants and distribution centers, and provide the same service level or better. Outsourcing often leads to exclusive partnerships between suppliers, retailers, or manufacturers and carriers, warehousing experts, or logistics management suppliers. These partnerships allow better inventory management, the ability to meet delivery dates, efficiency in filling emergency orders, and high accuracy in filling orders. Thus, outsourcing is becoming increasingly important in efficient deployment of supply chain management. Chapter 14—Marketing Channels and Retailing TRUE/FALSE 1. A marketing channel is a set of interdependent organizations that ease the transfer of ownership as products move from producer to business user or consumer. Answer: True Rationale: A marketing channel comprises a network of organizations that work together to facilitate the transfer of ownership as products move from producers to end consumers or business users. 2. As products move through the marketing channel, channel members provide specialization and division of labor, overcome discrepancies, and provide contact efficiency. Answer: True Rationale: Channel members in the marketing channel specialize in tasks, reduce discrepancies between supply and demand, and enhance efficiency in reaching customers, contributing to the overall effectiveness of the channel. 3. Consider a scenario in which there are five manufacturers, no intermediaries, and four consumers. Twenty transactions would be required for each consumer to receive products from each manufacturer. The introduction of one intermediary reduces the required number of transactions to four and demonstrates the idea of contact efficiency. Answer: False Rationale: Introducing one intermediary reduces the required number of transactions to nine. See Exhibit 14.1. 4. Carl represents a manufacturer of floor coverings and gets paid a commission for finding buyers and linking them up with the manufacturer. Carl is an example of a merchant wholesaler. Answer: False Rationale: This defines an agent or broker. 5. Transactional functions include contacting potential customers and assuming the risk of owning inventory. Answer: True Rationale: Transactional functions in marketing include activities such as contacting potential customers (selling) and assuming the risk associated with owning inventory (holding inventory). 6. The direct channel is used more often in consumer markets than in business and industrial markets. Answer: False Rationale: The direct channel is used more often in business and industrial markets than in consumer markets. 7. Metallurgical Designs makes gold and silver charms for necklaces and bracelets. It markets its charms to a few retailers in any one region and promotes them intensively to those retailers. This is an example of intensive distribution. Answer: False Rationale: Intensive distribution means the product is available in every outlet the potential customer might want to buy it, not to just a few retailers. 8. Exclusive distribution increases direct competition among a great number of retailers. Answer: False Rationale: An exclusive distribution arrangement establishes a dealer and enables it to be the main beneficiary of the manufacturer’s promotion efforts in that geographic area. 9. Highly integrated channel relationships are loosely bonded relationships characterized by informal arrangements that explicitly define the relationships of the channel members. Answer: False Rationale: Highly integrated channel relationships are closely bonded relationships characterized by formal arrangements that explicitly define the relationships of the channel members. 10. Reciprocal conflict occurs among channel members on the same level, such as two or more different wholesalers or two or more different retailers that handle the same manufacturer’s brands. Answer: False Rationale: This is referred to as horizontal conflict. 11. Many regard vertical conflict as healthy competition. Answer: False Rationale: Vertical conflict occurs between different levels in a marketing channel and is much more serious than horizontal conflict among channel members on the same level. 12. Gray marketing channels assist brand name manufacturers in marketing their products more efficiently. Answer: False Rationale: “Gray” marketing channels use unauthorized channel intermediaries. 13. When discussing customer-oriented service industries, you will often hear channel members discussing the need to minimize wait times. Answer: True Rationale: Customer-oriented service industries often focus on minimizing wait times to enhance customer satisfaction. 14. A retailing establishment that sells a wide variety of shopping and specialty goods under one roof would be classified as a department store. Answer: True Rationale: A retail establishment that offers a wide range of shopping and specialty goods under one roof is typically classified as a department store. 15. Emily has just opened a health food store that sells organic foods, herbal supplements, and environmentally friendly products. Because it will be a specialty store, she should have a broad product assortment with moderately low prices. Answer: False Rationale: Specialty stores have narrow product assortments and moderate to high prices. 16. Jennifer owns a greeting card store where she also sells women’s purses, Crocs shoes for the whole family, Jibbitz charms, cell phone skins, Sandy Lion stickers, Yankee candles, Godiva chocolates, Precious Memories figurines, beaded jewelry, and stuffed animals. Jennifer uses multi-brand merchandising. Answer: False Rationale: This would be an example of scrambled merchandising because it involves offering nontraditional goods. There is no such thing as a multi-brand. 17. Supercenters are more than just larger versions of traditional “markets.” In addition to foodstuffs, supercenters have pharmacies, dry cleaners, portrait studies, photo finishing, hair salons, and restaurants. Answer: True Rationale: Supercenters offer a variety of services beyond traditional markets, including pharmacies, dry cleaners, portrait studios, photo finishing, hair salons, and restaurants. 18. Specialty discount stores differ from other discount stores. While most other discounters offer a predictable assortment of merchandise, the styles and brands offered at specialty discount stores change frequently and cost 25 percent or more below traditional department store prices. Answer: False Rationale: Off-price retailers are being described here, not specialty discount stores. 19. Marketers who use non-store retailing no longer have to worry about the “place” element of the four Ps. Answer: False Rationale: Non-store retailing refers to shopping without visiting a store; however, products must still be distributed to end users, whether by vending, in-home retailing, or direct marketing. 20. The person who called Alisa last night and tried to sell her double-paned windows for her home was using outbound telemarketing. Answer: True Rationale: Outbound telemarketing involves calling potential customers to promote products or services. 21. The two types of franchising are target market franchising and geographic franchising. Answer: False Rationale: The two types of franchising are product and trade name franchising and business format franchising. 22. The six Ps of the retailing mix are product, place, price, promotion, personnel, and planning. Answer: False Rationale: Presentation, not planning, is one of the six Ps of the retailing mix. 23. Every October through December, Hickory Farms opens a temporary store in many malls to sell products during the Christmas season without the long-term commitment of a more expensive retail lease. This is an example of a pop-up shop. Answer: True Rationale: Hickory Farms opening temporary stores during the holiday season in malls without a long-term commitment is an example of a pop-up shop strategy. 24. Color can be used by retailers to create a mood or focus customer attention on a particular product. Answer: True Rationale: Color is a powerful tool in retail environments, capable of influencing mood and directing customer attention towards specific products, thereby impacting purchasing decisions. 25. Dillard’s, Target, and Walmart use data mining to determine which products to stock at what price, how to manage markdowns, and how to advertise to draw target customers. Answer: True Rationale: Retail giants like Dillard’s, Target, and Walmart leverage data mining techniques to optimize inventory management, pricing strategies, markdowns, and targeted advertising efforts, based on detailed customer data analysis. 26. M-commerce is a type of managed commerce in which both buyer and seller end the transaction with a win–win feeling. Answer: False Rationale: M-commerce, or mobile e-commerce, enables consumers using wireless mobile devices to connect to the Internet and shop. MULTIPLE CHOICE 1. A _____ is a set of interdependent organizations that eases the transfer of ownership as products move from producer to business user or consumer. A. facilitating agency or place member B. marketing mix intermediary C. selective distribution channel D. marketing channel or channel of distribution E. transportation channel or channel of movement Answer: D Rationale: This is the definition of a marketing channel, also called the channel of distribution. 2. Kayak.com is a company that allows customers to efficiently search databases to find the best airline and hotel deals all over the Web. Kayak does not make bookings but rather provides recommendations for the best travel plans. Travel providers pay Kayak a commission when customers click through to their sites. Kayak.com, travelers, the airlines, and the hotels are all part of a: A. facilitating agency B. marketing mix intermediary C. selective promotion channel D. marketing channel or channel of distribution E. transportation channel or channel of movement Answer: D Rationale: A marketing channel is a business structure of interdependent organizations that reaches from the point of product origin to the consumer. 3. Jones Soda is positioned as the anti-Coke. In the early years, few mainstream retailers sold Jones Soda. Fans had to get their Jones fix in surf shops, tattoo parlors, and bookstores, adding to the brand’s mystique. Jones Soda used its _____ to create a competitive advantage. A. channel ascendancy B. distribution channel C. channel conflict D. channel focus E. vertical integration Answer: B Rationale: Its unusual retail venues added to the brand’s mystique. 4. Marketing channels make distribution simpler by reducing the number of transactions required to get products from manufacturers to consumers. This is called: A. forward integration B. contact efficiency C. elimination of temporal discrepancies D. sorting E. reciprocity Answer: B Rationale: Marketing channels provide contact efficiencies by reducing the number of stores customers must shop in to complete their purchases. 5. The major characteristic that is used to differentiate among types of intermediaries is whether they: A. install exchange barriers such as location, time, and quantity B. create specialization of labor C. create economies of scale D. take title to the products they sell E. raise profit margins for independent middlemen Answer: D Rationale: Taking title means they own the merchandise and control the terms of the sale. 6. W.W. Grainger, Inc. is one of the world’s largest business-to-business distributors of equipment, component parts, and supplies in the United States and Canada. It has ownership title to over 220,000 products, which are stocked in one national and nine regional warehouses to guarantee product availability and quick service to the many manufacturers who are its customers. W.W. Grainger is an example of a(n): A. agent or broker B. merchant wholesaler C. retailer D. consumer market E. hypermarket intermediary Answer: B Rationale: A merchant wholesaler is an institution that buys goods from manufacturers and resells them to other wholesalers and retailers. Merchant Wholesalers receive and take title to the goods, own warehouses, and ship the products they sell. 7. _____ are intermediaries who facilitate the sales of a product from producer to end user by representing retailers, wholesalers, or manufacturers and providing little input as to the terms of the sale. A. Marketing facilitators B. Channel cooperatives C. Agents and brokers D. Merchant wholesalers E. Channel functionaries Answer: C Rationale: Agents and brokers simply facilitate the sale of a product from producer to end user. 8. _____ considerations affecting the wholesaler choice include how often a product is purchased and how long a customer is willing to wait to receive the product. A. Market B. Transactional C. Product D. Buyer E. Logistic Answer: D Rationale: The question describes buyer considerations. 9. The three basic functions channel intermediaries perform are: A. transactional, logistical, and facilitating B. contacting, negotiating, and ownership C. promoting, distributing, and bulk-breaking D. assorting, accumulating, and allocating E. financing, mediating, and storing Answer: A Rationale: Retailing and wholesaling intermediaries in marketing channels perform essential transactional, logistical, and facilitating functions. 10. Transactional channel functions include all of the following activities EXCEPT: A. explaining product benefits B. making buyers aware of existing products C. explaining product features, advantages, and benefits D. contacting and communicating with prospective buyers E. physically distributing and sorting products Answer: E Rationale: Physically distributing and sorting products is not a transactional channel function. 11. Which of the following are examples of facilitating functions performed by wholesaling intermediaries? A. Sorting and storing B. Risk taking and promotion C. Assorting, accumulating, grading, and allocating D. Researching and financing E. Financial management and storing Answer: D Rationale: The third basic channel function, facilitating, includes research and financing. 12. Serenity is a manufacturer of outdoor fountains that are popular in gardens. Even though fountains represent a product category that does not sell year-round due to inclement weather, Serenity’s sales remain steady all year because it sells to wholesale distributors that stock the product. Serenity sells to wholesale distributors that perform _____ functions for the manufacturer. A. financial B. transactional C. facilitating D. logistical E. promotional Answer: D Rationale: Storing for later sale to retailers is a logistical function. 13. The only way that Jim Keeler in New Mexico can get a box of Carolyn Popwell’s Festive Holiday Truffles from Washington is to order it through the mail. Popwell, who makes the candy by hand, uses a(n) _____ exclusively. A. exclusive distribution system B. conventional channel C. vertical marketing system D. reciprocal channel E. direct channel Answer: E Rationale: The manufacturer sells directly to the consumer. 14. Vutek manufactures printing machines used to print high-resolution graphics for billboards, bus cards, banners, and posters. For distribution, you would expect Vutek to use a: A. network of facilitating agents B. horizontally integrated channel C. reciprocal channel D. direct channel E. vertical marketing system Answer: D Rationale: Direct channels are typical in business and industrial markets. 15. A _____ channel is commonly used for low-cost consumer items that are frequently purchased, such as candy, cigarettes, and magazines. A. retailer B. agent/broker C. industrial distributor D. producer E. wholesaler Answer: E Rationale: Manufacturers sell their products to wholesalers in large quantities; the wholesalers then break the large quantities into smaller quantities to satisfy individual retailer orders. 16. Nontraditional channel arrangements: A. tend to make a firm’s product seem the same as the competition. B. usually broaden a brand’s coverage. C. can give a producer serving a niche market a way to gain market access without having to establish channel intermediaries. D. are not usually useful for larger firms. E. All of the above are true. Answer: C Rationale: Nontraditional channel arrangements can give a producer serving a niche market a way to gain market access without having to establish channel intermediaries 17. Customers can purchase Hewlett-Packard computers from retail stores like Best Buy and Office Depot, online directly from HP, and through various catalogs. HP is using a(n) _____ distribution arrangement. A. intensive B. multiple C. exclusive D. cumulative E. aggregated Answer: B Rationale: Multiple distribution occurs when a producer selects two or more different channels to distribute the same products to target markets. 18. Selfridges is the second-largest department store in Great Britain. It works with House of Frasier, one of its competitors, to reduce operating costs by sharing channels of distribution to ship goods from 1,500-plus suppliers. Selfridges and House of Frasier are engaged in: A. vertical conflict B. an integrated supply chain C. a strategic channel alliance D. an information-based distribution channel E. a distribution cooperative Answer: C Rationale: A strategic channel alliance is a cooperative agreement between businesses to use each other’s already-established distribution channels. 19. Which of the following statements concerning how market factors affect channel decisions is true? A. Industrial customers tend to buy in larger quantities and require more customer service. B. Geographically concentrated target markets should be served with an indirect sales force. C. Widely dispersed markets require fewer intermediaries. D. New firms in extremely competitive markets will be more successful if they use indirect channels. E. A very large market requires fewer intermediaries. Answer: A Rationale: Industrial customers’ buying habits are very different from those of consumers, and they tend to buy in larger quantities and require more customer service. 20. Canesta Company has developed a virtual keyboard out of light to be used with cell phones and PDAs. The product beams an image of a keyboard on a desk, allowing the user to type on the image. The words are picked up by the user’s digital device. When introduced to the market, the device will sell for less than $50. If, in choosing its channel, Canesta is most concerned about its lack of financial, managerial, and marketing resources to support the product’s introduction, then its choice of channels will largely be influenced by: A. factors of ownership. B. market factors. C. producer factors. D. product factors. E. internal environmental characteristics. Answer: C Rationale: Financial, managerial, and marketing issues are producer factors. 21. Chewing gum and soft drinks are sold in grocery stores, service stations, convenience stores, drugstores, discount stores, and motel vending machines. This is a an example of a(n) _____ distribution strategy. A. exclusive B. reciprocal C. selective D. horizontal E. intensive Answer: E Rationale: Intensive distribution is distribution aimed at maximum market coverage. It is used for many convenience goods that need to be available in every outlet where the potential customer might want to buy them. 22. _____ distribution is achieved by screening dealers to eliminate all but a few in any single geographic area. Shopping goods and some specialty products that consumers are willing to search for are sold this way. A. Intensive B. Selective C. Exclusive D. Dual E. Controlled Answer: B Rationale: This is a description of selective distribution. 23. Canesta Company has developed a virtual keyboard out of light to be used with cell phones and PDAs. The product beams an image of a keyboard on a desk, allowing the user to type on the image. The words are picked up by the user’s digital device. When introduced to the market, the device will sell for less than $50. What level of distribution intensity should the company use? A. Exclusive B. Controlled C. Extensive D. Reciprocal E. Selective Answer: E Rationale: Screening dealers will promote a quality image for the product. Price is also a factor in its level of distribution intensity. It is not a convenience good. 24. Which level of distribution intensity is the most restrictive and entails establishing only one or a few dealers within a given geographic area? A. Selective B. Intensive C. Exclusive D. Dual E. Premium Answer: C Rationale: This is the definition of exclusive distribution. 25. Caterpillar, the manufacturer of tractors and other earthmoving equipment, has an extremely high market share, which means customers seek the dealers out regardless of where they are. There are approximately 50 Caterpillar dealers in the United States––one in each state. From this information, you can surmise that Caterpillar uses _____ distribution. A. selective B. premium C. intensive D. exclusive E. inclusive Answer: D Rationale: Exclusive distribution, the most restrictive form of distribution, entails establishing one or a few dealers within a given geographic area. Because buyers need to be willing to search or travel to acquire the product, this form of distribution is limited to consumer specialty goods, a few shopping goods, and major industrial equipment. 26. This summer, college student Pat is planning on selling kites at Panama City Beach. He has found some suppliers and has preordered all of the kites he thinks he can sell this summer. What type of a distribution channel relationship does Pat have with his suppliers? A. Arm’s-length B. Functional C. Cooperative D. Hierarchical E. Integrated Answer: A Rationale: The arm’s-length relationship is considered to be temporary. 27. With the _____ channel relationship, both parties retain their independence and pursue their own interests while attempting to benefit from the goods or services provided by the other. A. Arm’s-length B. functional C. cooperative D. hierarchical E. integrated Answer: A Rationale: These types of relationships are often referred to as “Arm’s-length” relationships due to the company’s unwillingness or lack of ability to develop a closer type of relationship. 28. One of the earliest, largest, and most famous examples of vertical integration was the Carnegie Steel Company. The company controlled not only the mills where the steel was manufactured but also the mines where the iron ore was extracted, the coal mines that supplied the coal, the ships that transported the iron ore, and the railroads that transported the coal to the factory. Carnegie Steel had a(n) _____ channel relationship. A. Arm’s-length B. functional C. cooperative D. hierarchical E. integrated Answer: E Rationale: Integrated relationships are tightly connected, with linked processes across and between firm boundaries and high levels of trust and commitment. 29. Which channel relationship is less ambiguous than an Arm’s-length relationship but without the longer-term and/or capital investment required to achieve full integration? A. Ersatz B. Cooperative C. Functional D. Integrated E. Supplemental Answer: B Rationale: Cooperative relationships are between companies that take the form of informal partnership with moderate levels of trust and information sharing as needed to further each company’s goals. 30. The capacity of a particular marketing channel member to control or influence the behavior of other channel members is known as: A. channel power. B. channel conflict. C. channel control. D. channel dominance. E. channel inversion. Answer: A Rationale: Channel power is a channel members’ ability to control or influence the behavior of other channel members. 31. A situation that occurs when one marketing channel member intentionally affects another member’s behavior is called: A. channel power. B. channel control. C. channel conflict. D. channel dominance. E. channel inversion. Answer: B Rationale: Channel control is when one channel member actually affects another. 32. DeBeers sells most of the diamonds it produces to industry. The company controls over 90 percent of the market and has great power over its distributors. In its marketing channel, DeBeers would be considered a: A. channel authority. B. channel member. C. channel captain. D. channel gatekeeper. E. power broker. Answer: C Rationale: The channel captain is a member of a marketing channel that exercises authority and power over the activities of other channel members. DeBeers has control over delivery, inventory, and so on. 33. Channel conflict: A. always has an adverse effect on the members of the channel B. is not caused by ideological differences, because such differences would prevent the members from ever operating as a channel C. is often caused by an inability of some channel members to keep up with the changing times D. can appear as either internal or external E. does not occur when multiple distribution channels are used Answer: C Rationale: In a broad context, channel conflict can be good for a channel. It is sometimes caused by ideological differences. It appears as vertical or horizontal. 34. Honda uses just-in-time manufacturing to build its Odyssey minivans. Five companies that make component parts for the Honda Odyssey relocated to Alabama and states bordering Alabama when Honda announced plans to build a new Odyssey manufacturing plant in Alabama. This relocation in order to provide efficient delivery of goods indicates that Honda is a: A. channel captain B. horizontal integrator C. distribution champion D. distribution ombudsman E. demand maverick Answer: A Rationale: A channel captain exercises authority and power over the other channel members. 35. Horizontal channel conflict occurs most often when manufacturers practice: A. dual or multiple distribution B. trade loading C. promotional pricing D. direct distribution E. channel distribution Answer: A Rationale: Horizontal conflict occurs when more than one channel is used to handle the same brand. 36. Channel conflict that occurs between different levels in a marketing channel is referred to as: A. horizontal conflict B. primary conflict C. vertical conflict D. secondary conflict E. parallel conflict Answer: C Rationale: This describes vertical conflict, which typically occurs between the manufacturer and wholesaler or between the manufacturer and retailer. 37. _____ is the joint effort of all channel members to create a channel that serves customers and creates a competitive advantage. A. Selective partnering B. Direct distribution C. Channel partnering D. Intensive integration E. Closed channel distribution Answer: C Rationale: This is the definition of channel partnering, which is also called channel cooperation. 38. You are responsible for physical distribution of your company’s service and should focus on: A. ensuring the intangibility of the service so that physical distribution becomes a less important factor. B. minimizing wait times, managing service capacity, improving service delivery, and establishing channel-wide network coherence. C. making sure production and consumption are simultaneous. D. setting quality standards, choosing faster transportation modes, and using safety stock. E. customer-oriented order processing and inventory control. Answer: B Rationale: Service industries are customer oriented and must manage intangible services by minimizing wait times, managing service capacity, improving service delivery, and establishing channel-wide network coherence. 39. Paul operates Kayak.com, a company that allows customers to efficiently search company databases and find the best airline and hotel deals all over the Web. Kayak does not make bookings but rather provides recommendations for the best travel plans. Travel providers pay Kayak a commission for most customers who click through to their sites. Paul wants his users to have as short and sweet an experience on Kayak as possible. Time is of the essence in responding to any customer question, especially in this day where airfare can increase dramatically in just a couple of hours. Paul is devoted to: A. improving service delivery. B. maximizing queues. C. reducing discrepancies of assortment. D. reducing wait time. E. managing service capacity. Answer: A Rationale: The Internet is an alternative channel for delivering these deals. Kayak is a service organization. 40. _____ is defined as all activities directly related to the sale of goods and services to the ultimate consumer for personal, nonbusiness use or consumption. A. Wholesaling B. Retailing C. Business D. Franchising E. Distribution Answer: B Rationale: This is the definition of retailing. 41. A Target store is an example of a(n): A. merchant wholesaler B. retailer C. broker D. agent E. exporter Answer: B Rationale: Retailers are firms that sell mainly to consumers. 42. Which of the following intermediaries sell mainly to consumers? A. Retailers B. Merchant wholesalers C. Agents D. Brokers E. Specialists Answer: A Rationale: Retailers are firms that sell mainly to consumers. 43. Retail establishments are generally classified according to all of the following EXCEPT: A. ownership B. level of sales C. level of service D. price E. product assortment Answer: B Rationale: The four elements used to classify retail establishments are its ownership, level of service, product assortment, and price. 44. Around the world, most retailers are _____, operating one or a few stores in their community, owned by a single person or partnership and not operated as part of a larger retail institution. A. independent B. chain stores C. franchise outlets D. specialty clubs E. product assortment stores Answer: A Rationale: Independent retail outlets are owned by a single person or partnership and not operated as part of a larger retail institution. 45. Jim’s Bike Shop sells bicycles for the serious racer to the casual cyclist. Jim’s also sells biking gear and clothing. Jim is the sole owner of the store, so in terms of ownership, his store would be classified as a(n): A. independent retailer. B. freestanding store. C. franchise. D. chain store. E. common retailer. Answer: A Rationale: Independent retailer stores are owned by a single person or partnership and not operated as part of a larger retail institution. 46. Which type of retail ownership is owned and operated by an individual but is licensed by a larger supporting organization? A. Independent retailer B. Chain store C. Franchise D. Cooperative E. Secondary retailer Answer: C Rationale: This describes a franchise. 47. The number of different varieties of fishing lures carried by the Angler’s Store refers to the store’s: A. supply standard. B. complete retail offering. C. retail mix. D. volume. E. assortment depth. Answer: E Rationale: Breadth and depth refers to product assortment. 48. _____ shows how much the retailer makes as a percentage of sales after the cost of goods sold is subtracted. A. Net income B. Retained earnings C. Profitability D. Net equity E. Gross margin Answer: E Rationale: This is the definition of gross margin. 49. Housing several departments under one roof, a _____ carries a wide variety of shopping and specialty goods, including apparel, cosmetics, housewares, electronics, and sometimes furniture. A. supermarket B. specialty store C. convenience store D. super club E. department store Answer: E Rationale: Department stores house several departments under one roof. 50. Each department in a department store is usually treated as a different _____ center and central management sets broad policies about the types of merchandise carried and prices. A. franchise B. financial C. comptroller D. human resources E. buying Answer: E Rationale: Each department in a department store is treated as a separate buying center 51. Which type of retail store specializes in a given type of merchandise? A. An independent store B. A department store C. A discount store D. A specialty store E. A first-level store Answer: D Rationale: This is the definition of a specialty store. 52. James wants to open a small store that caters to the model railroad hobbyist. The store would sell model trains, scenery, accessories, and books on the subject of model railroading. Which of the following types of stores would most likely support an effective launch of his business idea? A. An off-price retailer B. A specialty store C. A full-discount store D. A general store E. A warehouse club Answer: B Rationale: Specialty stores specialize in a given type of merchandise. 53. Which of the following statements about specialty stores is true? A. Specialty stores find it easy to compete on price with big-box stores. B. Customers at specialty stores are not concerned with the distinctiveness of the merchandise or the store’s physical appearance. C. Customers of specialty stores usually consider price to be secondary. D. Specialty stores are ineffective test markets. E. A typical specialty store carries a wider assortment of specialty merchandise than department stores. Answer: C Rationale: Customers usually consider price to be of secondary importance in their selection of a specialty store. Customers at specialty stores are very concerned with the distinctiveness of the merchandise or the store’s physical appearance. Specialty stores provide a low-risk testing ground for new products. A typical specialty store carries a narrower assortment of specialty merchandise than department stores. 54. Fairway is a chain of retail outlets in Iowa, Minnesota, Nebraska, and Illinois that are described as large, departmentalized, self-service stores that specialize in food and limited non-food items. Fairway stores are: A. off-price retailers B. discount stores C. wholesale clubs D. convenience stores E. supermarkets Answer: E Rationale: The key factors are the emphasis on food and the size of the stores. 55. Kroger is the second-largest supermarket chain in the United States. Which of the following is the most likely reason why Kroger supermarkets sell all types of foodstuffs as well as non-food products including a wide range of eyewear and accessories? A. Consumer demand for one-stop shopping often leads to scrambled merchandising. B. The fulfillment of the wheel of retailing hypothesis C. Inflationary economic climate D. Repositioning as a discounter E. Growth of convenience merchandising Answer: A Rationale: In many cases, supermarkets offer a wide variety of nontraditional goods and services under one roof (a strategy called scrambled merchandising) to respond to consumers who want convenience. 56. _____ carry a limited line of high-turnover, high-priced goods and resemble miniature supermarkets. These self-service stores are typically located near residential areas and are open long hours, seven days a week. A. Specialty stores B. Wholesale stores C. Convenience stores D. Factory outlets E. General stores Answer: C Rationale: This describes convenience stores. 57. Prices are usually higher in convenience stores than in supermarkets because convenience stores offer: A. more specialty goods B. a wider variety of products C. a higher level of customer services D. better location, longer hours, and faster service E. higher-quality goods Answer: D Rationale: Convenience stores carry a limited amount of high-turnover, high-priced goods and resemble miniature supermarkets. Customers patronize these stores because of convenience. 58. _____ are retailers that compete on the basis of low prices, high turnover, and high volume. A. Convenience outlets B. Discount stores C. General stores D. Department stores E. Specialty stores Answer: B Rationale: This is the definition of discount stores. Convenience stores are miniature supermarkets, carrying only a limited line of high-turnover convenience goods. 59. _____ is the retailing strategy of offering consumers very limited service and carrying a broad assortment of well-known, nationally branded “hard goods.” A. Specialty store positioning B. Merchandise positioning C. M-store retailing D. The wheel of retailing E. Full-line discounting Answer: E Rationale: This describes full-line discounters. 60. A retail operation that has a 90,000-square-foot facility and houses a florist, baker, bank, craft shop, photo finisher, dry cleaner, and pharmacy under the same roof as grocery and household items would be called a: A. membership warehouse club. B. wholesale outlet. C. factory outlet. D. super center. E. mass merchandiser. Answer: D Rationale: Super centers offer an array of products and services, as well as groceries. 61. Toys “R” Us sells a huge assortment of toys and children’s books at greatly reduced prices. It offers customers few services and competes on the basis of moderate to low prices on the large quantities of merchandise it stocks. Toys “R” Us is an example of a: A. full-line discount store. B. specialty discount store. C. general store. D. department store. E. wholesale store. Answer: B Rationale: A specialty discount store (also called a category killer) offers a nearly complete selection of single-line merchandise and uses self-service, discount prices, high volume, and high turnover to its advantage. 62. _____ sell a limited selection of brand name appliances, household items, and groceries, usually in bulk on a cash-and-carry basis to members only. A. Super outlets B. Factory outlets C. Wholesale discounters D. Off-price discount retailers E. Warehouse membership clubs Answer: E Rationale: This describes warehouse membership clubs. 63. Off-price retailers: A. usually don’t ask for return privileges B. very often offer a consistent assortment of merchandise. C. seldom stock brand name merchandise. D. are owned by the manufacturer of the products they sell. E. offer very deep assortments. Answer: A Rationale: Off-price retailers usually don’t ask for return privileges. 64. A _____ is an off-price retailer that is owned and operated by a manufacturer and carries one line of merchandise––its own. A. mass merchandiser B. factory outlet C. wholesale club D. discount store E. bargain basement store Answer: B Rationale: This describes a factory outlet. CB&E Model Distribution 65. _____ refers to shopping without visiting a store and is currently growing faster than in-store shopping because of the consumer demand for convenience. A. Non-store retailing B. Customerization C. Hyper shopping D. Niche marketing E. Specialty merchandising Answer: A Rationale: Non-store retailing is selling to consumers through other means than by visiting a store. 66. _____ is the most pervasive retail business in the United States. Consumers get products out of automated machines, and it accounts for billions of dollars worth of goods each year in the United States. A. Automatic vending B. Party-plan selling C. Direct marketing D. A pyramid scheme E. Professional selling Answer: A Rationale: Automatic vending is the use of machines to offer goods for sale. 67. Southwestern Company recruits and trains college students to sell educational reference books, software, and children's books door-to-door. This form of non-store retailing is called: A. direct retailing B. personal marketing C. direct marketing D. automatic vending E. electronic retailing Answer: A Rationale: Direct retailing is the selling of products by representatives who work door-to-door, office-to-office, or at home sales parties. 68. _____ refers to the techniques used to get consumers to buy from their home. Those techniques include direct mail, catalogs and mail-order, telemarketing, and electronic retailing. A. Non-store vending B. In-home retailing C. Franchising D. Direct marketing E. Direct retailing Answer: D Rationale: Direct marketing includes techniques, such as catalogs and telemarketing, to get consumers to make a purchase from their home, office, or other nonretail setting. 69. Marketers can precisely target their customers according to demographics, geographics, and psychographics with: A. direct mail. B. vending machines. C. franchise stores. D. specialty retail outlets. E. electronic selling. Answer: A Rationale: With direct mail, marketers can precisely target their customers according to demographics, geographics, and even psychographics. 70. Telemarketing can consist of either unsolicited _____ sales calls, or _____ sales calls, which allow companies to receive orders through toll-free telephone numbers. A. order placement; lead generation B. push strategy; pull strategy C. systematic; synergistic D. qualifying; finalizing E. outbound; inbound Answer: E Rationale: Outbound telemarketing is generally used for unsolicited selling, while inbound calls are from customers placing orders. 71. _____ is a type of shopping available to consumers with access to the Internet. A. Shop-at-home networks B. Online retailing C. Franchising D. Electronic wholesaling E. Mobile vending Answer: B Rationale: Online retailing, also called e-tailing, is a type of shopping available to consumers with access to the Internet. 72. The Cedar Store carries a full line of cedar swings, gliders, garden benches, and tables. All of its products can be purchased through its Web site. The site has done well since the Cedar Store’s products do not need to be touched or smelled before purchase. The Cedar Store is engaging in: A. online retailing. B. shop-at-home networking. C. mobile vending. D. franchising. E. electronic wholesaling. Answer: A Rationale: Online retailing is a two-way interactive service offered to users with personal computers. 73. A(n) _____ is a continuing relationship in which an individual or business grants business rights to operate or sell a product to another individual or business. A. leasing contract B. alliance for profit C. brokered partnership D. franchise E. countertrade Answer: D Rationale: A franchise is a continuing relationship in which a franchiser grants to a franchisee the business rights to operate or sell a product. 74. The party in a franchise relationship that is an individual or business granted the right to sell another party’s product is called the: A. franchisor B. franchisee C. receiver D. renter E. owner/operator Answer: B Rationale: The franchisee sells the franchisor’s products. 75. Chattanooga Bottling Company is licensed to bottle Coke products. The Chattanooga Bottling Company is an example of a: A. product and trade name franchisee. B. redistributor franchisor. C. business format franchisee. D. licensed franchisor. E. relationship franchisee. Answer: A Rationale: In product and trade name franchising, a dealer agrees to sell certain products provided by a manufacturer or wholesaler. 76. Domino’s pizza, Hyatt Corporation, and Wendy’s are all examples of _____ because the franchisee buys the right to use the franchisor’s approach to doing business. A. product and trade name franchisors B. redistributor franchisors C. business format franchisors D. licensed franchisors E. relationship franchisors Answer: C Rationale: Business format franchising is an ongoing relationship between a franchisor and a franchisee in which the franchisor sells a franchisee the rights to use a format or approach to doing business. 77. The first task of developing a retail strategy is to: A. create a buying organization. B. decide what to buy. C. define the target market. D. create a promotional strategy. E. define the pricing policies. Answer: C Rationale: The target market’s wants and needs form the foundation for a successful retail strategy. 78. Retailers such as Dillard’s and Sears are responding to consumer needs by changing product mixes, hours of operation, locations, and prices. These stores are changing aspects of their: A. merchandising groupings. B. retailing mixes. C. product offerings. D. retail trade areas. E. store positioning strategies. Answer: B Rationale: Retailers are altering several aspects of the six Ps of the retailing mix. See Exhibit 14.6. 79. Which of the following statements about a retailer’s promotion strategy is true? A. The design of the promotion strategy would be done separately from the creation of the retailing mix. B. The goal of a retail store’s promotion mix is to position the store in consumers’ minds. C. Most advertising for retailers is carried out at the national level. D. Retailers find direct mail marketing inefficient. E. Retail promotion strategy does not include public relations activities. Answer: B Rationale: Promotion is part of the retailing mix and includes advertising, public relations and publicity, and sales promotions. Most advertising for retailers is carried out at the local level. Retailers have been very successful with direct marketing. 80. When opening a new retail operation, the retailer needs to consider all of the following factors EXCEPT: A. traffic flow. B. employee density. C. geographic region. D. growth potential. E. zoning regulations. Answer: B Rationale: Employee density is concerned with the amount of employees per 1,000 square feet in the store––not in the neighborhood. The other four factors are to be considered when deciding on a location. 81. Large retailers such as Target and Walmart and sellers of shopping goods such as automobiles and furniture often use _____ for their locations. A. remote, rural locations B. office complexes C. factory outlets D. freestanding stores E. regional malls Answer: D Rationale: Freestanding stores are often used by large retailers. 82. Maggie Moo’s has decided that consumers will drive out of their way for the store’s yummy ice cream offerings. The owner needs to keep his overhead costs (such as rent) low and wants to avoid locating near competitors. For a location, Maggie Moo’s should open in a: A. factory outlet. B. strip center. C. freestanding store. D. shopping center. E. regional mall. Answer: C Rationale: A freestanding store has the advantage of low site costs and will be a benefit if consumers are willing to seek it out. 83. Another name for an anchor store is a: A. mass merchandiser. B. generator store. C. host store. D. shopping center pillar. E. destination store. Answer: B Rationale: Anchor stores or generator stores are usually located at opposite ends of the mall to create heavy foot traffic. 84. The newest generation of shopping centers is the: A. regional mall. B. shopping business district. C. lifestyle center. D. franchised center. E. strip mall. Answer: C Rationale: Lifestyle centers are open-air shopping centers targeted to upper-income shoppers with an aversion to “the mall.” 85. The Halloween Super Store is only open from September until November, usually in an otherwise vacant store. This is an example of a: A. temp store. B. m-commerce. C. convenience retailing. D. pop-up shop. E. flash store. Answer: D Rationale: Pop-up shops are temporary shops that allow a flexible location without a long-term commitment. 86. All of the following are factors in creating a store’s atmosphere EXCEPT: A. employee type and density. B. fixture type and density. C. sound. D. price. E. odors. Answer: D Rationale: Influential factors in creating a store’s atmosphere include employee type and density, merchandise type and density, fixture type and density, sound, odors, and visual factors. 87. The _____ of retail stores is a key factor in their success; the goal is to use all space in the store effectively, including aisles, displays, and even non-selling areas. A. merchandise density B. layout C. target strategy D. merchandise mix E. promotional strategy Answer: B Rationale: The layout of retail stores is described here. 88. A common practice among most retailers that seeks to broaden customers’ original purchases with related items is called: A. suggestion selling. B. trading up. C. bait and switch. D. comprehensive selling. E. interactive selling. Answer: A Rationale: Suggestion selling is common in helping customers recognize true needs. 89. When Irvin went into Lowe’s to buy his wife a small tool set she said she wanted, the sales clerk suggested a larger set with a tool kit on wheels that included a router and circular saw. In this example, the sales clerk engaged in: A. trading up. B. suggestion selling. C. bait and switch. D. customer relationship retailing. E. service-disguised selling. Answer: A Rationale: Trading up means persuading customers to buy a higher-priced item than the one they intended to buy. 90. Whenever a customer opens an account at First Southern National Bank, a teller or account representative asks the customer whether he or she needs a CD, online banking, or other services. In other words, the account representative engages in: A. interactive selling. B. bait and switch. C. trading up. D. customer relationship retailing. E. suggestion selling. Answer: E Rationale: Suggestion selling seeks to broaden customers’ original purchase with related items. 91. One of the new developments in retailing is m-commerce. The “m” in m-commerce stands for: A. motivated B. multimedia C. marketing D. managed E. mobile Answer: E Rationale: M-commerce, or “mobile” e-commerce enables consumers using wireless mobile devices to connect to the Internet and shop. 92. _____ uses complex mathematical models to help retailers make better product mix decisions. A. Inbound telemarketing B. Data mining C. Blogs D. Spamming E. Online auctions Answer: B Rationale: This is the definition of data mining. Redbox Movie DVD sales represent a $16 billion market, and rentals make up another $7.5 billion. Naturally, production studios would prefer consumers purchase DVDs rather than rent them. Production studios like Twentieth-Century Fox, Warner Bros., and General Electric refused to sell new releases to Redbox, a DVD vending machine company, until almost a month after new releases arrived in stores. Redbox, the ubiquitous DVD rental red kiosks found in and outside of convenience stores, grocery stores, drugstores, fast-food restaurants, and Walmart, is cutting in on production companies’ profits. These studios are tangled in lawsuits with Redbox. Sony, Paramount, and Lionsgate, on the other hand, permit distribution through Redbox, and Disney allows third-party distribution to Redbox. With more than 20,000 kiosks now in operation, Redbox ranks fifth in DVD rental revenues, which is impressive considering the rental fee is only $1.00. But Blockbuster is trying to steal vending market share by allowing NCR Corporation, known for ATM machines, to license its name to place Blockbuster Express kiosks in similar types of locations. 93. Refer to Redbox. Warner Bros., Blockbuster, Redbox, Walmart, and third-party DVD distributors are part of a: A. distribution cooperative B. marketing mix C. transportation channel D. marketing channel E. logistics system Answer: D Rationale: A marketing channel is a set of interdependent organizations that facilitate the transfer of ownership as products move from producer to business user or consumer. 94. Refer to Redbox. Redbox offers DVDs from several production studios, such as Disney, Sony, and Paramount. Consumers can rent from Redbox instead of obtaining the DVD from the various production studios if they want a children’s movie from Disney and an action movie from Paramount. Thus, one kiosk provides _____ for both consumers and the production studios. A. contact efficiency B. integration C. synergy D. distribution efficiency E. distribution intensity Answer: A Rationale: Marketing channels provide contact efficiencies by reducing the number of stores customers must shop in to complete their purchases. 95. Refer to Redbox. Video stores that distribute DVDs usually also sell popcorn, candy, soft drinks, and entertainment publications. Basically, they overcome discrepancies of quantity and assortment by combining products into collections or assortment that consumers want available in one place. Which channel function does this describe? A. Logistical functions B. Transactional functions C. Facilitating functions D. Purchasing functions E. Secondary functions Answer: A Rationale: Logistical functions include physically distributing, storing, and sorting. Sorting overcomes discrepancies of quantity and assortment. 96. Refer to Redbox. Production studios distribute DVDs in grocery stores, drugstores, discount stores, entertainment stores, and vending machines like Redbox. Which level of distribution intensity does this represent? A. Intensive B. Extensive C. Product focused D. Market focused E. Exclusive Answer: A Rationale: Intensive distribution is aimed at maximum market coverage. 97. Refer to Redbox. Production studios are potentially producing _____ among its existing distributors by allowing Redbox to rent their DVDs for $1.00. A. vertical conflict B. cognitive dissonance C. supply management divergence D. horizontal conflict E. disintermediation Answer: D Rationale: Horizontal conflict occurs among channel members on the same level. Walmart Walmart, the world’s largest retailer by revenue, announced that it will get back to its founder’s roots by initiating more price cuts. Critics claimed that the giant was getting away from its original focus because it posted relatively high gross profit margins. CEO Mike Duke assuaged critics by saying Walmart’s $400 billion in sales will become even larger because the price cuts will attract more customers and it will lower its costs of goods sold. 98. Refer to Walmart. In terms of ownership, Walmart would be classified as a(n): A. independent retailer. B. chain store. C. franchise. D. discount store. E. department store. Answer: B Rationale: Chain stores are owned and operated as a group by a single organization. While Walmart is a discount store, in terms of ownership, it is a chain store. 99. Refer to Walmart. Walmart’s gross margin is the: A. total sales from all of its stores worldwide. B. amount of money it makes as a percentage of sales after the cost of goods sold is subtracted. C. amount of money it makes as a percentage of sales before taxes. D. amount of money it makes as a percentage of sales after all costs are subtracted. E. net profit it earns after all expenses and taxes are subtracted. Answer: B Rationale: Gross margin is the money the retailer makes as a percentage of sales after the costs of goods sold is subtracted. 100. Refer to Walmart. Which type of pricing strategy describes Walmart’s commitment to offer consistently lower prices? A. High–low pricing B. Everyday low pricing C. extreme value pricing D. Prestige pricing E. Marginal pricing Answer: B Rationale: Everyday low pricing offers consumers a low price all the time rather than holding periodic sales on merchandise. 101. Refer to Walmart. Many Walmart stores have a full line of groceries and general merchandise with a wide range of services, such as a pharmacy, portrait studio, optical shop, and bank. Walmart is an example of which type of discount store? A. Hypermarket B. Supermarket C. Extreme-value retailer D. Super center E. Category killer Answer: D Rationale: Super centers combine a full line of groceries and general merchandise with a wide range of services––all in one location. Walmart operates over 7,200 super centers worldwide. 102. Refer to Walmart. Most Walmart stores are not located in shopping centers but rather are buildings all by themselves. This type of location is referred to as a: A. primary outlet. B. freestanding store. C. pop-up store. D. single-unit retailer. E. strip store. Answer: B Rationale: An isolated, freestanding location can be used by large retailers like Walmart. Diaz John Diaz immigrated to Tallahassee from Cuba in the 1980s. He set up a coffee shop called Diaz in a small upper-middle-class neighborhood. It rapidly became popular because of the wonderful coffee it brewed. Soon Diaz was selling the finest coffees from around the world, plus coffee-making necessities such as grinders and brewers. Within ten years, Diaz was operating a lucrative mail-order business in addition to his coffee shop. Upon entering the shop, the first thing the customer sees is a countertop crowded with all of the machinery needed for making a perfect cup of coffee. Marble-topped tables are set in cozy nooks with overstuffed chairs. Bookshelves on one wall hold books about coffee for patrons to read while they sip. The smell is intoxicating to a real coffee lover. Coffee drinkers can take home a pound when they leave. The store carries all types of coffee from $8 per pound Brazilian Cerrado to $300 per pound Kopi Luwak from Sumatra. 103. Refer to Diaz. In terms of ownership, Diaz’s coffee shop would be classified as a(n): A. franchise. B. wholesaler. C. independent retailer. D. agency. E. chain store. Answer: C Rationale: It is owned by a single person. 104. Refer to Diaz. The type of merchandise carried by Diaz is deep but with a narrow assortment. The store would most likely be classified as a: A. convenience store. B. mass merchandiser. C. specialty store. D. category killer. E. membership club. Answer: C Rationale: A specialty store specializes in a given type of merchandise with a deep but narrow assortment. 105. Refer to Diaz. Customers of Diaz’s coffee shop can purchase coffee, coffee makers, mugs, chocolate, music CDs, and all sorts of unique gift items. Diaz is practicing: A. m-commerce. B. interactive merchandising. C. lifestyle merchandising. D. scrambled merchandising. E. pop-up shopping. Answer: D Rationale: Scrambled merchandising is the tendency to offer a wide variety of nontraditional goods and services under one roof. 106. Refer to Diaz. If Diaz wanted to grow his business and granted another owner the rights to use his format and approach to doing business, he would be involved in: A. trading up. B. m-commerce. C. direct marketing. D. product and trade name franchising. E. business format franchising. Answer: E Rationale: Business format franchising is a business relationship in which the franchisor (Diaz) “sells” a franchisee the rights to use the franchisor’s format or approach to doing business. 107. Refer to Diaz. The smell of fresh coffee, the fact it is made where drinkers can watch the process, and the comfortable chairs all are used to create the store’s: A. atmosphere. B. cultural impact. C. target strategy. D. merchandise mix. E. promotional strategy. Answer: A Rationale: The main element of a store’s presentation is its atmosphere––how the store’s physical layout, decor, and surroundings convey an overall impression. Apple Stores Apple, Inc. opened its first Apple Store in 2001 and currently has hundreds of stores spanning several countries. Many of the Apple Stores are inside malls, but Apple also has several freestanding stores, which they call their flagship stores, with each looking a little different on the outside. These are located in New York City, Boston, Los Angeles, Chicago, San Francisco, Montreal, Tokyo, and Osaka. The store in New York City is a glass cube with a glass cylindrical elevator and spiral staircase that leads to an underground store. Inside, they have long tables displaying products ranging from the iMac, MacBook Air, iPods, printers, cameras, and other accessories. All these products are accessible to customers who can try them out and ask various questions of informed associates. The walls are mostly metallic, and the signs are all backlit. All stores have a Genius Bar where customers can receive technical advice or set up service and repair for their products. To address the needs of the many iPod users, some stores, such as the New York store, have set up a separate iPod Bar where customers can get their own technical assistance. They also have an iMac station for kids who want to try out games and learning products and a theater for workshops, product training sessions, and special presentations. Apple introduced the handheld Easy Pay system where customers don’t have to wait in line to purchase products. With a simple click of a paperless, handheld credit card scanner, the employee brings the cash register to the customer. Items can be rung up anywhere on the show room floor. Apple plans to open several more stores in the years to come. Apple also has its online store. At its Web site, a customer can order any Mac or iPod product, plus a large variety of accessories. In addition to its product offerings, a customer can watch a demonstration of how products work or check out the latest Mac ads or news. 108. Refer to Apple Stores. In terms of ownership, which of the following best describes Apple stores? A. Independent retailer B. Chain stores C. Franchises D. Pop-up shops E. Department stores Answer: B Rationale: Chain stores are owned and operated as a group by a single organization. 109. Refer to Apple Stores. Of the major types of retail operations, Apple stores are: A. department stores. B. specialty stores. C. Superstores. D. convenience stores. E. discount stores. Answer: B Rationale: A specialty store specializes in a deeper but narrower assortment of merchandise. The Apple Store’s specialty product is electronics. 110. Refer to Apple Stores. What method of non-store retailing does Apple take advantage of through the use of its Web site? A. Telemarketing B. Automatic vending C. Direct mail D. Catalogs and mail orders E. Online retailing Answer: E Rationale: Online retailing is a type of shopping available to consumers with personal computers and access to the Internet. 111. Refer to Apple Stores. Some shoppers purposely plan to visit the Apple store on Fifth Avenue in New York City, which is characteristic of: A. generator stores. B. lifestyle stores. C. interactive stores. D. m-stores. E. destination stores. Answer: E Rationale: Destination stores are ones that consumers purposely plan to visit. 112. Refer to Apple Stores. Apple has the Genius Bar, the backlit signs, the Apple products themselves, the modern metallic walls and spiral staircases in its stores. These are all work together to convey the store’s: A. impression. B. environment. C. atmosphere. D. surroundings. E. elements. Answer: C Rationale: Atmosphere is the overall impression given by a store’s physical layout, decor, and surroundings. ESSAY 1. What is a marketing channel? What type of businesses make up a marketing channel? Answer: A marketing channel, or channel of distribution, is a business structure of interdependent organizations that eases the transfer of ownership as products move from producer to business user or consumer. It encompasses the processes involved in getting the right product to the right place at the right time. Marketing channels represent “place” in the marketing mix. Channel members include wholesalers, distributors, agents and brokers, and retailers. 2. One important reason channel members are included between producers and users is the concept of specialization and division of labor. Define these concepts and explain their roles in channels of distribution. Answer: Specialization and division of labor break down a complex task into smaller, simpler ones and allocate them to specialists, which results in much greater efficiency. Marketing channels achieve economies of scale through specialization and division of labor. Some producers do not have the interest, financing, or expertise to market directly to end users or consumers. These producers use channel members to perform functions and activities that the producers are not equipped to perform or that these intermediaries are better prepared to perform. Channel members can perform some functions and activities more efficiently than producers, and they enhance the overall performance of the channel because of their specialized expertise. 3. One important reason channel members are included between producers and users is to increase contact efficiency. How do channel members increase contact efficiency? Use the following two scenarios to illustrate your answer: (1) Assume that there are 2,000 business students who need to buy textbooks from eight textbook publishers. How many transactions must take place? (2) Assume the first scenario, but now one campus bookstore is acting as an intermediary. Now how many transactions must take place? Answer: Channels make distribution simpler and more efficient by reducing the number of transactions required to get products from manufacturers to consumers. In the provided example, the first scenario requires 16,000 transactions (2,000 students 8 publishers). In the second scenario, only 2,008 transactions are necessary (1 bookstore × 8 publishers + 2,000 students × 1 bookstore). Clearly, a significant reduction in transactions takes place if intermediaries are added to the system. 4. Intermediaries must perform three basic functions. Name these functions and describe the activities involved in each function. Answer: TRANSACTIONAL FUNCTIONS include contacting and promoting (contacting potential customers, promoting products, soliciting orders), negotiating (determining how many goods or services to buy and sell, transportation types, delivery times, and payment method and timing), and risk taking (assuming the risk of owning inventory). LOGISTICAL FUNCTIONS include physical distribution (transporting and storing goods to overcome temporal and spatial discrepancies), storing (maintaining inventories and protecting goods), and sorting (overcoming discrepancies of quantity and assortment by sorting out, accumulating, allocating, and assorting). FACILITATING FUNCTIONS include researching (gathering information about other channel members and consumers) and financing (extending credit and other financial services to facilitate the flow of goods through the channel to the final consumer). 5. What are the three factors affecting channel choice? Briefly discuss how each factor might influence a company to implement a direct channel of distribution. Answer: MARKET FACTORS. Industrial customers tend to buy in larger quantities and require more customer service. Conversely, consumers usually buy in very small quantities and sometimes do not mind if they get no service at all. Geographic location and size also determine channel choice. A geographically concentrated market is appropriate for direct selling through a direct sales force. Markets that are more widely dispersed may need more intermediaries. Generally, a very large market requires more intermediaries. Also, if there is lots of existing competition, choosing a direct channel may be less difficult and more profitable. PRODUCT FACTORS. Products that are more complex, customized, and expensive tend to benefit from shorter and more direct marketing channels. Product life cycle also plays a role. In fact, the choice of channels may change over the life of the product. As products become more common and less intimidating to potential users, producers tend to look for alternative channels. Finally, fragile and perishable products require a shorter channel that involves less handling. PRODUCER FACTORS. Producers with large financial, managerial, and marketing resources are better able to use channels that require fewer intermediaries. More direct channels may also be used by producers who wish to control pricing, positioning, brand image, and customer support. 6. A retail clothing store in a small northwest Georgia community is the sole distributor of Spanx brand hosiery in the region. How does this form of market coverage benefit the retailer? How does it benefit the manufacturer? Answer: Exclusive distribution entails only one or a few dealers within a given market area. It is the most restrictive form of distribution intensity. Retailers may be unwilling to commit the time and money necessary to promote a product unless the manufacturer guarantees them an exclusive territory. The arrangement shields the retailer from direct competition and enables it to be the main beneficiary of the manufacturer’s promotion within the geographic area. For the manufacturer, exclusive distribution can serve to project an exclusive image for the product. 7. Define channel power, channel control, channel leadership, and channel partnering and explain how these four terms are related. Answer: Power, control, leadership, conflict and partnering are the main social dimensions of marketing channel relationships. CHANNEL POWER refers to the capacity of one channel member to control or influence the behavior of other channel members. CHANNEL CONTROL occurs when one channel member intentionally affects another member’s behavior. CHANNEL LEADERSHIP is the exercise of authority and power. CHANNEL PARTNERING is the joint effort of all channel members to create a channel that serves customers and creates a competitive advantage. Collaborating channel partners meet the needs of consumers more effectively by ensuring the right products reach shelves at the right time and at a lower cost, boosting sales and profits. They are all components of channel relationship management. 8. Discuss the four main areas of importance in service distribution. Answer: MINIMIZING WAIT TIMES. Minimizing the amount of time a customer waits for service is a key factor in maintaining the quality of service. People tend to overestimate the amount of time they spend waiting in line, and unexplained waiting seems longer than explained waiting. MANAGING SERVICE CAPACITY. If service firms don’t have the capacity to meet demand, they must either turn down dome prospective customers, let service slip, or expand capacity. For instance, at tax time, a tax preparation firm may have so many customers desiring its services that it has to either turn business away or add temporary offices or prepares. IMPROVING SERVICE DELIVERY. Like manufacturers, service providers are now experimenting with different distribution channels. These new channels can increase the time that service is available (such as the Internet) or add to consumer convenience (plan a vacation cruise). ESTABLISHING CHANNEL-WIDE NETWORK COHERENCE. Because services are to some degree intangible, service firms also find it necessary to standardize their service offering quality across different geographic regions in order to maintain brand image. Network coherence means that suppliers, service processes and customer service have quality standards that are maintained regardless of where the service is purchased or consumed. 9. Define retailing and list the factors that are used to classify retail operations. Answer: Retailing is defined as all the activities directly related to the sale of goods and services to the ultimate consumer for personal, nonbusiness use. Retail establishments can be classified according to ownership, level of service, product assortment, and price, and retailers use the latter three variables to position themselves in the competitive marketplace. 10. There are several types of retail stores, each offering a different product assortment, service level, and price level, according to the shopping preferences of its customers. Name eight types of retailers. For each type, indicate the level of service, product assortment, and level of price. Answer: See Exhibit 14.4. 11. A specialty store is not only a type of store but also is a method of retail operations. Describe the specialty store strategy. Then name two examples of specialty stores. Answer: Specialty stores specialize in a given type of merchandise such as children’s clothing, baked goods, and pet supplies. Specialty stores carry a narrow assortment of merchandise but a very deep product selection. Specialty stores typically offer more customer service and more knowledgeable salesclerks. Prices and gross margins tend to be high, but this is of secondary importance to the customer who strongly values distinctive merchandise, attractive stores, and quality personnel. Examples of specialty stores include The Children’s Place, Gadzooks, Williams-Sonoma, and Foot Locker. 12. Discount stores are retail chains that compete on the basis of low prices, high turnover, and high volume. Name and briefly describe four types of discounters, and give an example of each type. Answer: FULL-LINE DISCOUNTERS. These stores are similar to traditional department stores, except that they carry a much broader assortment of well-known, nationally branded “hard goods” and offer consumers very limited service. Most full-line discounters are national chains such as Kmart, Walmart, and Target. Supercenters combine a full line of groceries and general merchandise with a wide range of services. Extreme-value retailers cater to low-income consumers; their stores are generally rather small, they offer a narrow selection of merchandise emphasizing day-to-day necessities, and they stress rock-bottom prices. SPECIALTY DISCOUNT STORES. Single-line specialty discount stores offer consumers a nearly complete selection of one line of merchandise and use self-service, discount prices, high volume, and high-turnover merchandise in their retailing strategies. These stores are also called “category killers” because they dominate their narrow segment. Toys “R” Us (toys), Home Depot (home improvement), Office Depot (office supplies), and Best Buy (electronics) are examples. WAREHOUSE MEMBERSHIP CLUBS. These are usually warehouse outlets that allow members to buy items on a cash-and-carry basis. Examples include Costco and Sam’s Club. OFF-PRICE RETAILERS. These retailers buy manufacturers’ overruns, irregular merchandise, unsold end-of-season output, and goods from bankrupt stores. The merchandise is then sold at large discounts from traditional department stores. Examples include T.J.Maxx, Ross Stores, Marshalls, and Tuesday Morning. A special case of this type is the single-price store, such as Family Dollar, Dollar General, and the One Price Clothing Stores. Factory outlets are owned and operated by manufacturers and carry only their own line of merchandise. Outlets are used to sell surplus goods, factory seconds, return merchandise, and closeouts. Most manufacturers place these outlets in locations that do not conflict with their normal retailer. Manufacturers with factory outlets include Liz Claiborne, J. Crew, Calvin Klein, West Point Pepperel, Oneida, and Lands’ End. 13. Retailing opportunities can take place without customers shopping at a store. Name and briefly discuss four forms of non-store retailing. Answer: Answers will vary, and may include: AUTOMATIC VENDING. Vending machines for items such as soft drinks, snacks, and coffee are an important form of non-store retailing. Vending is the most pervasive retail business in the United States, with about 11.5 million vending machines selling billions of dollars worth of goods annually. Consumers are willing to pay higher prices for products from a vending machine than for the same products in a traditional retail setting. DIRECT RETAILING. This form of non-store retailing is characterized by sales transactions in a home setting. This includes door-to-door selling and party plan selling. Direct retailers are also using the Internet as a channel to reach more customers and increase sales. DIRECT MARKETING. In this case, consumers buy from their homes. Direct marketing (sometimes called direct response marketing) refers to a variety of techniques such as telemarketing, direct mail, and catalogs and mail order. TELEMARKETING is the use of the telephone to sell directly to consumers. It consists of outbound sales calls, usually unsolicited, and inbound calls—that is, orders through toll-free 800 numbers or fee-based 900 numbers. SHOP-AT-HOME TELEVISION NETWORKS are specialized forms of direct response marketing. These shows display merchandise, with the retail price, to home viewers. Viewers can phone in their orders directly on a toll-free line and shop with a credit card. ONLINE RETAILING, or e-tailing, is a type of shopping available to consumers with personal computers and access to the Internet. ELECTRONIC RETAILING. This form of non-store retailing includes the 24-hour, shop-at-home television networks and online retailing, and mobile retailing. The best-known television networks are HSN and QVC. Online retailing, or e-tailing, is a type of shopping available to consumers over the Internet. Mobile retailing, or m-commerce, involves consumers using wireless mobile devices to connect to the Internet and shop 14. What is the first task of developing a retail strategy? What is involved in this task? Answer: The first and foremost task of developing a retail strategy is defining the target market the prospective retailer wishes to serve. This process begins with market segmentation and a thorough knowledge of the segment targeted. Target markets in retailing are often defined on demographic, geographic, and psychographic dimensions. Different combinations of the elements in the retail marketing mix enable a retailing establishment to position itself for its particular targeting strategy. 15. When retailers decide on location, they have two basic site options: the freestanding store and the shopping center. Briefly describe these location alternatives and discuss the advantages and disadvantages of each location alternative. Answer: FREESTANDING STORES. This alternative is an isolated, freestanding location that customers will seek out (a destination store). Advantages include low site cost or rent and no direct competitors close by. Disadvantages are that it may be difficult to attract customers, and there are no shared costs. SHOPPING CENTERS. These include strip centers, community shopping centers, malls, and lifestyle centers. An advantage of a strip store is customer convenience. A disadvantage is crowding, lack of parking, and old buildings. Community shopping centers and malls provide several advantages, including a planned shopping environment, anchor stores, good parking, a unified image, and special activities. Disadvantages include expensive leases, common promotional expenses, not attracting customers to a particular store, lease restrictions on merchandise and hours of operation, domination by anchors, and possibility of direct competition within the same facility. Lifestyle centers are newer, open-air shopping centers targeted to upper-income shoppers with an aversion for “the mall” and seek to create an atmosphere that is part neighborhood park and part urban shopping center. 16. Compare and contrast trading up and suggestion selling. Answer: These are two common selling techniques, and they should always help shoppers recognize true needs rather than sell them unwanted merchandise. TRADING UP means convincing the customer to buy a higher-priced item than he or she originally intended to buy. To avoid selling customers something they do not need or want, however, salespeople should take care when practicing trading-up techniques. SUGGESTION SELLING seeks to broaden the customer’s original purchase with related items. For example, if a consumer buys a new printer, the sales representative will ask if he or she would like to purchase paper, a USB cable, and/or extra ink cartridges. Chapter 15—Marketing Communications TRUE/FALSE 1. Promotion is communication by marketers that informs, persuades, and reminds potential buyers of a product in order to influence their opinion or elicit a response. Answer: True Rationale: Promotion encompasses various marketing communications aimed at informing, persuading, and reminding potential buyers about a product, ultimately influencing their purchasing decisions. 2. Black & Mild FT are the only cigars on the market made with “a skillful blend of premium pipe tobaccos and a filter tip for extra smoothness.” This feature creates a competitive advantage for the product. Answer: True Rationale: Black & Mild FT's unique feature of using a blend of premium pipe tobaccos and a filter tip provides a distinctive product offering that can create a competitive advantage by appealing to consumers looking for a smooth smoking experience that combines elements of both cigars and pipes. 3. A U.S. manager is working with some Indonesians who do not speak English. In attempting to exchange information, they are using symbols to assign meanings to the facts the manager hopes to convey. This is an example of communication. Answer: True Rationale: Communication involves the exchange of information and meaning. Even when language barriers exist, communication can occur through symbols that convey intended messages and meanings effectively. 4. Nature Valley uses popular magazines, radio, and cable television to promote its brand of trail mix. Nature Valley uses mass communication. Answer: True Rationale: Nature Valley's use of popular magazines, radio, and cable television for promoting its trail mix brand constitutes mass communication, reaching a broad audience through widely accessible media channels. 5. One way of conveying a message that the receiver will hear properly is to use abstract words and pictures. Answer: False Rationale: Concrete words and pictures should be used, not abstract. 6. Nutri-Foods is concerned that its advertising messages are not getting through to the target market as intended. Its radio ads contain music that its target market does not like, and its spokesperson has a very nasal voice that sounds as though he has a perpetual cold. These are examples of noise in the communications process. Answer: True Rationale: In the communication process, noise refers to any interference that distorts or disrupts the message being conveyed. In Nutri-Foods' case, factors like disliked music in radio ads and an unappealing spokesperson voice act as forms of noise, hindering effective message delivery to the target audience. 7. Marketers targeting consumers in foreign countries must worry about the translation and possible miscommunication of their promotional messages by other cultures. Answer: True Rationale: Marketers targeting consumers in foreign countries need to consider translation and cultural differences to ensure that their promotional messages are effectively understood and do not lead to miscommunication or misunderstanding among different cultures. 8. If Charles Schwab Mutual Funds Investment Broker were to study its promotional strategies by looking at sales trends and market research, it would be using direct feedback to evaluate its promotional strategies. Answer: False Rationale: Charles Schwab relies on indirect feedback. Direct feedback would imply immediate interaction. 9. Informative promotion is used to keep the product and brand name in the public’s mind. Answer: False Rationale: This is the function of reminder promotion. 10. All promotions are designed to inform, persuade, or remind the target audience. Answer: True Rationale: All promotional activities—whether advertising, personal selling, sales promotions, or public relations—are designed to inform, persuade, or remind the target audience about a product or service. 11. Maddie’s Beverage Company has recently introduced Wateroos, eight-ounce servings of water in juice-style containers with straws, to encourage children to drink more water. Since this product is in the introductory stage of the product life cycle and as yet has no direct competition, Maddie’s promotional objective should be persuasion. Answer: False Rationale: Early in the life cycle, the primary focus of promotions should be to inform consumers of the product. 12. It’s been a few years since odor-free paint hit the market, and a paint manufacturer estimates the paint to be in the growth stage of its product life cycle. With all the competition in the industry today, the manufacturer should use promotions that persuade buyers of odor-free paint to purchase its brand over all others. Answer: True Rationale: In the growth stage of the product life cycle, competitive pressures are high, so persuasive promotions are crucial to differentiate a brand and persuade customers to choose it over competitors. 13. The promotional mix is the combination of promotional tools used to reach the target market and fulfill the organization’s overall goals. Answer: True Rationale: The promotional mix refers to the blend of promotional tools—such as advertising, personal selling, sales promotion, and public relations—that a company uses to achieve its marketing objectives and reach its target audience effectively. 14. Advertising is impersonal, one-way mass communication about a product or organization that is paid for by a marketer. Answer: True Rationale: Advertising involves one-way, paid communication aimed at promoting a product or organization to a mass audience, making it an impersonal form of communication. 15. Benedict and Taapo are arguing about newspaper publicity. Benedict says favorable publicity is free––hence the phrase, “free publicity.” Taapo says that publicity is not free because costs were incurred in developing the public relations materials used by the newspapers. Benedict has a more accurate grasp of the definition of publicity. Answer: False Rationale: Publicity is not free. Preparing news releases, staging special events, and persuading media to broadcast them cost money. 16. Publicity is free. Answer: False Rationale: Preparing news releases, staging special events, and persuading media to broadcast them cost money. 17. Sales promotion consists of all marketing activities other than personal selling, advertising, and public relations that stimulate consumer buying and dealer effectiveness. Answer: True Rationale: Sales promotion includes all promotional activities besides personal selling, advertising, and public relations that are designed to stimulate consumer buying and improve dealer effectiveness. 18. Communication via telephone is not considered personal selling because it is not face-to-face. Answer: False Rationale: Selling over the phone is a form of personal selling. It does allow for immediate interaction. 19. AIDA is an acronym for attention, interest, desire, and action. Answer: True Rationale: AIDA stands for Attention, Interest, Desire, and Action, which are the stages in the consumer decision-making process that marketing communications aim to move consumers through. 20. A person buying a Valentine’s Day gift for a significant other is more likely to go through all of the steps in the AIDA process than a person buying a gift certificate for his or her mail carrier. Answer: True Rationale: The present for Valentine’s Day is more likely to be a high-involvement purchase, and people engaged in high-involvement purchases typically go through all four stages of the AIDA process. 21. The AIDA concept can be used to explain how all promotions influence purchase decisions. Answer: False Rationale: The order of stages in the model, as well as whether consumers go through all its steps, has been much debated. 22. In terms of the AIDA process, public relations has its greatest impact in gaining attention and interest for a company, good, or service. Answer: True Rationale: Public relations activities, such as media coverage or events, are often effective in capturing attention and generating interest in a company, product, or service as part of the AIDA model. 23. Allyson Brown is trying to develop a promotional mix for her firm’s new product, a sophisticated modular sun porch designed for use all year. She reasons that because her product is complex and carries high financial risks for consumers, the mix should concentrate on personal selling. Answer: True Rationale: Personal selling is effective for complex products with high financial risks because it allows for direct communication, relationship-building, and addressing specific consumer concerns or questions. 24. From the consumer’s point of view, a company’s communications are already integrated, no matter how the company itself defines them. Answer: True Rationale: From the consumer's perspective, they interact with a company's various communication channels holistically, regardless of how the company internally organizes or defines these channels. 25. One of the reasons for the growing popularity of integrated marketing communications is the proliferation of thousands of new media choices. Answer: True Rationale: Integrated marketing communications (IMC) integrates various promotional tools and media channels to deliver a consistent message to consumers, which is increasingly important due to the diverse and numerous media options available today. 26. All promotion, especially advertising, is reduced as a product enters the decline stage. However, personal selling and sales promotion efforts might be maintained, especially at the retail level. Answer: True Rationale: During the decline stage of a product's life cycle, overall promotion, especially advertising, is typically reduced. However, personal selling and sales promotion efforts might still be maintained, especially at the retail level. 27. Lansky sells 60 different knife and garden tool sharpeners. It uses aggressive personal selling and sales promotions to encourage intermediaries to carry and sell its sharpener. Lansky is using a push strategy. Answer: True Rationale: Lansky's use of aggressive personal selling and sales promotions to encourage intermediaries to carry and sell its sharpeners represents a push strategy. 28. Marketers typically use either a push or a pull strategy exclusively. Answer: False Rationale: Companies rarely use one or the other exclusively. The strategy that best supports their promotional objectives (usually in combination) is what is selected. MULTIPLE CHOICE 1. _____ is communication by marketers that informs, persuades, and reminds potential buyers of a product in order to influence an opinion or elicit a response. A. Distributive communication B. Perceptual communication C. Statistically provable advertising D. Promotion E. Publicity Answer: D Rationale: This is the definition of promotion. 2. Rosarita is responsible for developing a plan for the optimal use of advertising, personal selling, sales promotion, and public relations. Rosarita is developing a: A. promotional strategy. B. set of promotion goals. C. marketing mix. D. communication mix. E. selling plan. Answer: A Rationale: Promotional strategy is a plan for the optimal use of the elements of promotion: advertising, public relations, personal selling, and sales promotion. 3. In 2008, the NBC television network used advertising, personal selling, public relations, and sales promotion to communicate with its target audience about its new show My Own Worst Enemy. The television network’s _____ described its plan on how to use these tools most effectively. A. mass communication mix B. promotional strategy C. selling plan D. marketing mix E. strategic goals Answer: B Rationale: Promotional strategy is a plan for the optimal use of the elements of the promotion mix. 4. The main function of a promotional strategy is to: A. convince the target customers that a firm’s products offer competitive advantages over those of its competition. B. find a niche in the marketplace for the firm and its products. C. provide the firm with research information about the success of its marketing effort. D. create efficient distribution channels. E. guarantee control over the length of the stages of the product life cycle. Answer: A Rationale: The function of a promotional strategy is to convince the target customers that the goods and services offered provide a competitive advantage over the competition. 5. The National Pork Board wants to create more consumer demand for pork products. The board has developed a(n) _____ that sets out how it will use all of the elements of the promotional mix to create a coordinated plan that will become an integral part of the organization’s marketing strategy. A. promotional strategy B. tactic for communication C. mass aggregation strategy D. communication objective E. informative strategy Answer: A Rationale: Promotional strategy is a plan for the optimal use of the elements of the promotional mix. 6. A _____ is defined as a unique aspect of an organization that causes target consumers to patronize that firm rather than competitors. A. comparative differentiation B. competitive advantage C. marketing mix D. special benefit E. promotional plan Answer: B Rationale: This is the definition of competitive advantage. 7. People who love to fish prefer American Magic Lures that are cut from ash or cedar and then hand painted and hand tied with a feather skirt. Advertising for American Magic Lures emphasizes how its lures, unlike others, are all handcrafted. This is its: A. publicity objective. B. marketing differentiator. C. competitive advantage. D. one-to-one marketing advantage. E. sales promotional theme. Answer: C Rationale: A competitive advantage is the unique set of features of a company and its products that are perceived by the target market as significant and superior to the competition. It is not the result of publicity; it is design and production based. Since this is advertising, the sales promotion theme may or may not agree; there is no indication of integrated marketing communications. 8. The ads for Don Francisco’s ground coffee simply state, “The highest quality coffee doesn’t have to be expensive.” This is an example of how a product communicates its _____ with promotion. A. marketing mix B. comparative differentiation C. competitive advantage D. researchable objective E. promotional theme Answer: C Rationale: A competitive advantage is the unique set of features of a company and its products that are perceived by the target market as significant and superior to the competition. It may be based on a scientific discovery, but the company didn’t create it by saying it. 9. Vegetarians sometimes have trouble making sure they get enough protein in their diets. Protons are a new, poppable snack made with super-crunchy tofu. They are bite-sized nuggets that come in both sweet and savory flavors: Original Crunchy, Backyard Bar-B-Q, Tomato Herb Garden, Cinnamon Toast, and Lemon Meringue Pie. Protons’ features, including that they are organic, vegan, and cholesterol free, give the new product its: A. differential parity. B. synergistic goal. C. competitive advantage. D. comparative benefit. E. tactical objective for implementing strategy. Answer: C Rationale: A competitive advantage is one or more unique aspects of an organization that cause target customers to patronize that brand rather than others. 10. _____ is the process by which we exchange or share meanings through a common set of symbols. People also assign meanings to feelings, ideas, facts, and attitudes. A. Feedback B. Promotion C. Advertising D. Publicity E. Communication Answer: E Rationale: This describes communication. 11. While talking to Mr. and Mrs. Knighton, the appliance salesperson described one of the refrigerators as “a real money saver” and “a true bargain,” but he also kept looking down at his feet and shuffling around. His nonverbal cues: A. create subservient selling. B. create differential disadvantages. C. are part of how he communicated to the Knightons. D. are a type of marketing mix feedback. E. will not affect interpersonal communications. Answer: C Rationale: Communication is the process by which we exchange or share meanings through a common set of symbols. 12. Interpersonal communication is: A. nonpaid information such as publicity. B. paid communication placed in personal media. C. long-distance communication between a business and its target market. D. direct face-to-face communication between two or more people. E. noise-free communication. Answer: D Rationale: Interpersonal communication is direct, face-to-face communication between two people. 13. The two major categories of communications are: A. verbal and nonverbal. B. direct and indirect. C. long term and short term. D. mass and interpersonal. E. informative and persuasive. Answer: D Rationale: Communication can be divided into two major categories: interpersonal communication and mass communication. 14. Usborne Books consultants sell books directly to customers via presentations in customers’ homes, at book fairs and community events, and at craft shows. Usborne consultants are engaged in: A. telemarketing. B. interpersonal communication. C. publicity. D. mass communication. E. crowd selling. Answer: B Rationale: Interpersonal communication is direct, face-to-face communication between two or more people. 15. Communication to large audiences, usually through a medium such as television or a newspaper, is called _____ communication. A. mass B. feedback-proof C. referential D. interpersonal E. public Answer: A Rationale: This describes mass communication. 16. USA cable television uses billboard, television, magazine, and newspaper advertising to inform television viewers about new episodes of Monk, a popular television series. USA uses _____ communication. A. referential B. mass C. factual D. interpersonal E. public Answer: B Rationale: Mass communication involves communicating a concept or message to large audiences. 17. Which of the following is NOT a disadvantage a firm using mass communication might experience? A. Inability to assess customer reactions to the promotion immediately B. Clutter from competitors’ messages C. Distractions around the consumer at the time of receipt of the message D. Inability to know exactly who is receiving the message E. Great expense on a per person exposed basis Answer: E Rationale: Mass media reaches so many people that even though the initial investment is large, cost per person is small. 18. The communication process itself consists of: A. message, media, and transmittal. B. source, receiver, and channel. C. sender, receiver, and message. D. encoding, decoding, channel, sender, and receiver. E. comprehension, noise, and feedback. Answer: D Rationale: See Exhibit 15.2. The communication process involves encoding (message creation), decoding (message interpretation), the channel through which the message is transmitted, the sender (source of the message), and the receiver (recipient of the message). 19. The _____ is the originator of the message in the communication process. A. communicator B. encoder C. channeler D. sender E. receiver Answer: D Rationale: This is the definition of a sender. 20. For its new Jeep Compass, DaimlerChrysler launched a campaign targeting young, hip consumers. To reach its target, Jeep’s advertising agency used music, talking bobbleheads, and other nontraditional marketing tactics to capture consumers’ interest. The agency devised a host of bobblehead characters that represented the wide variety of potential Compass buyers. DaimlerChrysler was the _____ of the message in this communication process. A. sender B. decoder C. channeler D. receiver E. communicator Answer: A Rationale: The sender is the originator of the message in the communication process. 21. The manufacturer of Meridian yam products spent $25,000 to create an ad that ran in the Thanksgiving issue of a popular cooking magazine. Because it was the originator of the communication process, Meridian acted as a: A. channel. B. noise filter. C. receiver. D. decoder. E. sender. Answer: E Rationale: The sender is the originator of the message in the communication process. 22. The maker of Frank’s RedHot pepper sauce spent $25,000 to create an ad that was targeted to consumers in the southwestern U.S. market. Because it created the ad itself and did not use an ad agency, Frank’s RedHot sauce engaged in: A. channeling. B. creating noise. C. receiving. D. decoding. E. encoding. Answer: E Rationale: Encoding is the conversion of a sender’s ideas and thoughts into a message, usually in the form of words or signs. 23. _____ is the interpretation of the language and symbols sent by a source through a channel. A. Development B. Envelopment C. Processing D. Decoding E. Encoding Answer: D Rationale: This defines Decoding. 24. Encoding is the: A. creation of the original ideas and thoughts of a message. B. conversion of the sender’s ideas and thoughts into a message. C. transmission of a message. D. receipt and comprehension of a message. E. deciphering and understanding of a message. Answer: B Rationale: Encoding is the conversion of a sender’s ideas and thoughts into a message, usually in the form of words or signs. 25. For its new Jeep Compass, DaimlerChrysler has launched a new campaign targeting young, hip consumers. To reach its target, DaimlerChrysler’s advertising agency is using music, talking bobbleheads, and other nontraditional marketing tactics to capture consumers’ interest. The agency has devised a host of bobblehead characters that represent the wide variety of potential Compass buyers. The ad agency is acting as the _____ of the message. A. decoder B. sender C. encoder D. receiver E. channeler Answer: C Rationale: Encoding is the conversion of the sender’s ideas and thoughts into a message. 26. A channel in communication is: A. the only source of noise. B. the same as in distribution. C. usually an ad agency or public relations firm. D. someone who places advertising into media. E. any communication medium. Answer: E Rationale: A channel is the transmitter of the message such as a voice, gesture, radio, newspaper, or any other communication medium. 27. Campbell’s soup has been promoted with television commercials, radio spots, newspaper coupons, and magazine advertisements. In the communication process, these media served as _____ for transmitting the message. A. senders B. decoders C. encoders D. channels E. receivers Answer: D Rationale: A channel is the transmitter of the message such as a voice, gesture, radio, newspaper, or any other communication medium. 28. When Castrol Syntec, a premium motor oil, sought a new way to reach its young, car-passionate audience, the brand found its mark with video games. In its first-ever attempt at in-game advertising, Castrol Syntec found a way to show consumers (in a virtual world) the brand’s power and performance. In the communication process, the game served as a(n) _____ for transmitting the message. A. sender B. decoder C. encoder D. channel E. receiver Answer: D Rationale: A channel is the transmitter of the message such as a voice, gesture, radio, newspaper, or any other communication medium. 29. The marketing campaign for Alpo dog food uses posters in veterinarians’ offices, radio and television ads, and a Web site to promote benefits of the dog food. In terms of the communication process, Alpo uses several different: A. source of feedback. B. channels. C. encoders. D. decoders. E. noise interrupters. Answer: B Rationale: Channels are media of communication used for transmitting a message. 30. _____ is anything that interferes with, distorts, or slows down the transmission of information. A. Media communication B. Feedback C. Static D. Nonpaid communication E. Noise Answer: E Rationale: This is the definition of noise. 31. Just as Grace was in sight of a billboard for the Riverbend Antique Mall, a police car drove by with its lights flashing. Grace was so concerned with watching where the police car went that she totally missed seeing the mall’s billboard. In terms of the communication process, the police car acted as: A. media interruptus. B. negative feedback. C. static. D. explicit communication. E. noise. Answer: E Rationale: Noise is anything that interferes with, distorts, or slows down the transmission of information. 32. Within the communication process for a car dealer’s ad, which of the following would be an example of noise? A. Competing ads B. News stories in the newspaper C. The physical surroundings in which the individual reading the ad is located D. The presence of other people while the ad is being read E. All of these Answer: E Rationale: Noise is anything that interferes with, distorts, or slows the transmission of information. 33. For its new Jeep Compass, DaimlerChrysler came out with a new campaign targeting young, hip consumers. To reach its target, DaimlerChrysler used music, talking bobbleheads, and other nontraditional marketing tactics to capture consumers’ interest. These communications were directed at the _____, the persons who would decode the message. A. senders B. receivers C. encoders D. channels E. communicators Answer: B Rationale: Customers are the receivers who decode the message. 34. _____ is the interpretation of the language and symbols sent by the source through a channel. A. Encoding B. Perceiving communication C. Decoding D. Channeling E. Semiotics Answer: C Rationale: This is the definition of decoding. 35. For communication to be effective: A. marketing managers should use both interpersonal and mass communications. B. all steps of the communications process model must take place. C. marketing managers must ensure a proper match between the message to be conveyed and the target market’s attitudes and ideas. D. advertising messages must be general enough to cover all ages, social classes, and education levels. E. marketers should use televised instead of print communication. Answer: C Rationale: Common understanding between two communicators is required for effective communication. 36. Ernest has three engineering degrees. He has hired an interior decorator to update his home. He is tremendously frustrated when he asks what he thinks are simple questions about weight-bearing walls and insulation requirements that the designer is unable to understand. Yet Ernest has the same problems when the designer starts talking about mauve, cerise, and magenta. Why is the communication process not working in this instance? A. The designer is not intelligent enough to understand engineering. B. Noise is interfering with both encoding and decoding. C. The wrong medium has been used. D. The sender and receiver do not share overlapping frames of reference. E. Personal selling should not be used to market this type of product. Answer: D Rationale: Common understanding between two communicators, or a common frame of reference, is required for effective communication. In this case, the designer has no knowledge base in engineering, and Ernest has no knowledge base in the designer’s colors. 37. The receiver’s response to a message is _____ to the source. A. decoded B. noise C. feedback D. channeled E. static filled Answer: C Rationale: In interpersonal communication, the receiver’s response to a message is direct feedback to the source. 38. When Becca saw the ad suggesting Subway sandwiches as a dinner meal, she was offended by the spokesperson and wondered why anyone would want to do business with a company that thought being obnoxious was a good business practice. In terms of the communication process, Becca: A. correctly encoded the message. B. created a longer than normal communication channel. C. improperly decoded the message. D. had no feedback. E. improperly encoded the message. Answer: C Rationale: Decoding is interpreting the language and symbols sent by the source through a channel. 39. Vera ordered the Chihuahua Christmas ornament she saw advertised in the Sunday coupon supplement by phoning The Danbury Mint and placing her order. In terms of the communication process, Vera’s purchase was an example of: A. feedback. B. noise. C. implicit communication. D. channeled communication. E. message encoding. Answer: A Rationale: The receiver’s response to a message is feedback to the source. 40. Because marketers do not control consumers’ comments on social media sites such as Twitter and Facebook: A. there is a chance that comments and postings will be negative. B. any comments consumers leave are virtually useless to marketers. C. filtered feedback channel has been created. D. most consumers do not trust the comments they read there. E. any feedback marketers receive there will be impersonal. Answer: A Rationale: Because there is no control over social media, there is chance that comments and postings will be negative. 41. Bag Media is a British company that sells space on paper bags to advertisers who want to drive consumers to their Web sites. A Bag Media representative made a presentation to several Internet entrepreneurs who are interested in getting traffic to their sites. When one of the entrepreneurs ordered its Web address printed on 25,000 bags, it was an example of: A. feedback. B. decoding. C. Noise. D. indirect transmission. E. nonverbal communication. Answer: A Rationale: The receiver’s response to a message is feedback to the source. 42. What are the four basic tasks of promotion? A. Informing, persuading, instituting and convincing B. Persuading, convincing, remixing, and reminding C. Informing, providing, convincing, and reminding D. Informing, persuading, reminding, and connecting E. Persuading, implementing, comparing, and reminding Answer: D Rationale: Promotion can perform one or more of four tasks: inform the target audience, persuade the target audience, remind the target audience, or connect with the target audience. 43. Which of the following is one of the basic tasks typically performed by promotion? A. Managing B. Organizing C. Rewarding D. Persuading E. Perceiving Answer: D Rationale: Persuasive promotion is designed to stimulate a purchase or an action. 44. Informative promotion is generally used: A. when memory stimulus is needed. B. when the brand name is well-known to consumers. C. during the early stages of the product life cycle. D. during an attempt to gain the immediate action of a consumer. E. when advertising a simple, nontechnical, mature product. Answer: C Rationale: Informative promotion is generally more prevalent during the early stages of the product life cycle. 45. The Hollymatic Patty Machine grinds and shapes meat into perfectly sized patties using a revolutionary new process that decreases shrinkage. The advertising agency is writing copy for the Hollymatic Patty Machine and has decided on a(n) _____ format because the product is so technical. A. influencer B. persuasive C. reminder D. reseller E. informative Answer: E Rationale: Informative promotion is a necessary ingredient for a highly technical product category. 46. When Heinz introduced its ketchup globally, it first had to deal with the fact that ketchup is not a household staple outside the United States. Heinz had to show foreign users how ketchup can be used. Globally, Heinz has had to rely primarily on _____ advertising. A. reminder B. persuasive C. informative D. compulsive E. influential Answer: C Rationale: Informative promotion seeks to convert an existing need into a want or to stimulate interest in a new product, generally during the early stages of the product life cycle. 47. Maddie’s Beverage Company has recently introduced Wateroos, eight-ounce servings of water in juice-style containers with straws, to encourage children to drink more water. The goal of promotion developed for this new healthy alternative beverage for children would be to _____ consumers. A. inform B. remind C. prompt D. influence E. encourage Answer: A Rationale: Informative promotion seeks to stimulate interest in a new product. 48. The American Plastics Council wants to stimulate more consumer demand for products that are packaged in plastic. The ads will explain how plastics protect families from food spoilage and contamination. The ads will also describe how tamper-evident plastic seals provide an extra measure of product safety assurance. By trying to change customers’ perception of plastic packaging, the council is using _____ promotion. A. reminder B. persuasive C. declarative D. diffusive E. mass Answer: B Rationale: The council is trying to stimulate demand for consumer products packaged in plastics. 49. The goal of persuasive promotion is to: A. stimulate a purchase or an action. B. increase brand awareness. C. describe available services. D. remind the consumers of where to buy the product. E. maintain top-of-mind consumer awareness. Answer: A Rationale: Persuasion generally attempts to motivate a consumer to purchase a product. 50. Persuasion normally becomes the primary promotion goal when: A. a firm is trying to increase brand awareness. B. the product enters the growth stage of the product life cycle. C. selling a highly technical product. D. reminding consumers where to buy the product. E. new products are in early stages of the product life cycle. Answer: B Rationale: Persuasion attempts to motivate a consumer to purchase a particular product. It is useful after the product has been made known through informational advertising during introduction. 51. Innovyx is an e-mail marketing service provider. It has a new ad campaign aimed at changing negative perceptions of e-mail. The ads explain how much cheaper e-mail advertising is than traditional advertising and what a high return on investment it can generate if used properly. Finally, the message ends by suggesting that companies wanting to promote their products give e-mail a chance to show what it can do. This promotion has the task of: A. persuading. B. informing. C. reminding. D. suppressing. E. rewarding. Answer: A Rationale: This promotion is aimed at persuading consumers to change their attitudes and try e-mail. 52. _____ promotion is used to keep a familiar brand name in the public’s mind and is prevalent during the maturity stage of the product life cycle. A. Influence B. Amusement C. Informative D. Persuasive E. Reminder Answer: E Rationale: This describes reminder promotion. 53. Hasbro, Inc. hired actress Jamie Lee Curtis as its spokesperson to promote the company’s board games. As part of the campaign, Curtis appeared in television spots dressed as characters from various Hasbro games such as Battleship, Monopoly, and Clue to promote the games as great Christmas gifts to parents who played the games as children. This promotion was intended to: A. amuse consumers. B. influence consumers to switch to Hasbro games. C. act as a reminder advertisement. D. persuade nonplayers of games to try them. E. act as a persuasive device for the product category. Answer: C Rationale: Reminder promotion is used to keep a familiar brand name in the public’s mind and is prevalent during the maturity stage of the product life cycle. 54. For many years Jell-O gelatin has run advertisements that tell consumers there is “always room for Jell-O.” This is an example of _____ promotion. A. influence B. amusement C. informative D. prevalent E. reminder Answer: E Rationale: Reminder promotion is used to keep a familiar brand name in the public’s mind and is prevalent during the maturity stage of the product life cycle. 55. Which of the following products is MOST likely to use reminder promotion? A. Samsung 3-D television B. Brother electronic typewriter C. iPhone D. Crest toothpaste E. All of the above. Answer: D Rationale: Reminder promotion is used to keep a familiar brand name in the public's mind and is prevalent during the maturity stage of the product life cycle. 56. Social media include all of the following EXCEPT: A. blogs B. LinkedIn C. a pop-up ad on the ESPN Web site D. YouTube E. All of the above are examples of social media. Answer: C Rationale: A pop-up ad on a Web site would be classified as an advertisement, not as social media. 57. The promotional mix consists of: A. advertising, publicity, direct marketing, and personal selling. B. public relations, direct marketing, personal selling, and publicity. C. product, promotion, price, and place. D. advertising, public relations, sales promotion, personal selling, and social media. E. advertising, telemarketing, public relations, and sales promotions. Answer: D Rationale: The promotional mix is the combination of promotional tools––including advertising, public relations, social media, personal selling, and sales promotion––used to reach the target market and fulfill the organization’s overall goals. 58. The five major promotional tools (advertising, personal selling, sales promotion, social media, and public relations) are known collectively as the: A. communication model. B. advertising campaign. C. marketing mix. D. publicity four. E. promotional mix. Answer: E Rationale: The promotional mix includes the five major promotional tools of advertising, personal selling, sales promotion, public relations, and social media. 59. _____ is any form of sponsor-identified, impersonal paid mass communication. A. Advertising B. Publicity C. Promotion D. Public relations E. Nonpaid communication Answer: A Rationale: Advertising is any form of impersonal paid communication in which the sponsor or company is identified. 60. To increase its revenues, US Airways has decided to sell space on airsickness bags to companies that would like to use this unique channel for their promotional messages. In terms of the promotional strategy, the airsickness bags will be used for: A. implicit communications. B. publicity. C. sales promotion. D. public relations. E. Advertising. Answer: E Rationale: Advertising is a form of sponsor-paid, impersonal one-way mass communication. 61. Which of the following statements about advertising is true? A. The total costs of advertising are typically low. B. The signs on the outsides of buses and taxis are not a form of advertising. C. The cost per contact in advertising is low. D. Innovative media are not used in advertising. E. Advertising is any form of communication in which the sponsor is identified. Answer: C Rationale: The total costs of advertising are typically high even though the cost of contact is low. To be advertising, the communication must be paid for. Innovative media are used in advertising. The signs are a form of advertising because they are paid for and they identify their sponsors. 62. The marketing function that evaluates public attitudes, identifies areas within the organization that the public may be interested in, and executes a program of action to gain public understanding is called: A. public relations. B. advertising. C. implicit communications. D. personal selling. E. sales promotion. Answer: A Rationale: This is the definition of public relations. 63. Marketers use public relations to: A. earn public understanding and acceptance B. communicate with the community in which they operate C. educate the public about company goals D. introduce new products E. do all of these things Answer: E Rationale: Public relations is used to achieve all of these things. 64. In 2006, Atlanta once again hosted the Peachtree Road Race, a running event that attracts many world-caliber racers. This year, race officials also sanctioned a race conducted in Iraq so that soldiers from Georgia would not have to miss the annual event. The winners of the Mideast race as well as scenes of the actual race were televised. In terms of a promotional mix, this Iraqi Peachtree Race was as example of: A. advertising and personal selling efforts. B. strategic product promotions and resulting sales. C. a target marketing strategy. D. sales promotion efforts. E. a public relations strategy and resulting publicity. Answer: E Rationale: The public relations activities involved in the race in Iraq led to publicity in the form of news media reporting. 65. Every year, the Discovery Channel has what it calls Shark Week, a weeklong marathon of programs on sharks. In New York, Discovery Channel street teams disguised as Surfers, Bight University faculty, and Bight University “chewleaders” will attack city streets in July visiting morning television shows, landmarks, and high-traffic areas to promote the series. An article in the New York Times on Shark Week and the Discovery Channel’s street teams would be an example of: A. a sales promotion. B. publicity. C. advertising. D. implicit communications. E. a personal sales presentations. Answer: B Rationale: Publicity is public information about a company, product, or issue appearing in the mass media as a news item. 66. Public information about a company, good, or service appearing in the mass media as a news item is called: A. personal selling. B. advertising. C. mass communications. D. publicity. E. sales promotion. Answer: D Rationale: This is the definition of publicity. 67. Publicity: A. will never damage a company because it performs the information task of promotion. B. is free communication. C. is not persuasive among customers. D. has to be purchased from the mass media. E. has internal costs to the company associated with it. Answer: E Rationale: A firm may have to hire a public relations firm to prepare new releases and persuade media personnel to print or broadcast the news. This can be expensive. 68. The Steel Recycling Institute is an industry association that promotes and sustains the recycling of all steel products. An article in BusinessWeek magazine about how the institute plans to stimulate consumer demand would be an example of: A. direct selling. B. news marketing. C. publicity. D. event marketing. E. direct marketing. Answer: C Rationale: Publicity is public information in mass media about a company and its products. 69. Which of the following statements about sales promotions is true? A. Public relations cannot effectively be used with sales promotions. B. Sales promotion is a type of direct marketing. C. Marketers view sales promotions as more effective when they are created as long-run stimulation tools. D. Trade shows, coupons, premiums, and vacation giveaways are types of sales promotions. E. All sales promotions must be aimed outside the organization. Answer: D Rationale: Sales promotions can be used to improve the effectiveness of advertising and public relations activities. They are most effective as short-run tools. Sales promotions are commonly directed at the organization’s employees as well as to consumers and dealers. 70. _____ consists of all marketing activities that stimulate consumer purchasing such as coupons, contests, free samples, and trade shows. A. Sales promotion B. Publicity C. Personal selling D. Advertising E. Sponsorship Answer: A Rationale: Sales promotions include free samples, contests, premiums, trade shows, vacation giveaways, and coupons. 71. A $1-off coupon for Carnation Instant Breakfast powder is a form of: A. public relations. B. sales promotion. C. personal selling. D. Advertising. E. Publicity. Answer: B Rationale: A sales promotion is a marketing activity that stimulates consumer buying and dealer effectiveness. 72. During a recent summer, Volvo Cars of North America drove traffic to its dealerships via a sweepstakes tied to Walt Disney’s summer blockbuster Pirates of the Caribbean: Dead Man’s Chest. This was an example of: A. a sales promotion. B. publicity. C. advertising. D. implicit communications. E. a personal sales presentation. Answer: A Rationale: A sales promotion is a marketing activity that stimulates consumer buying and dealer effectiveness. 73. All of the following are examples of sales promotion EXCEPT: A. coupons. B. transit advertising. C. free samples. D. contests. E. trade shows. Answer: B Rationale: Transit advertisements on buses and taxis are not examples of sales promotion. 74. Java Jacket is a company that designs and prints ads on the paper jackets that go around hot coffee cups sold in coffee shops. To find clients to advertise on the coffee cup jackets, the company sent a representative to companies like Warner Brothers, eBay.com, and the Wall Street Journal to tell them how their ads on coffee cup jackets would give them inexpensive exposure to a large number of potential customers. Java Jacket’s activities can best be described as: A. mass communication. B. implicit communication. C. personal selling. D. public relations. E. telemarketing. Answer: C Rationale: Personal selling is the planned, face-to-face presentations in a conversation with one or more prospective purchasers for the purpose of making sales. 75. To increase its revenues, US Airways has decided to sell space on airsickness bags to sponsoring companies as a different medium for their promotional messages. The most effective promotional method for US Airways to use to find companies that would like to pay to have their promotional messages delivered to the hundred or so travelers on each of its flights would be: A. push advertising. B. personal selling. C. direct advertising. D. sales promotions. E. publicity. Answer: B Rationale: Personal selling is a purchase situation involving a personal, paid-for communication between a buyer and a seller. 76. Which of the following statements about the characteristics of the elements in the promotional mix is true? A. The speed of feedback is delayed in sales promotion, advertising, and personal selling. B. Message flexibility is greatest with advertising. C. The message flow is two-way in public relations, sales promotions, and personal selling. D. Important promotional ingredients tend to receive the most funding. E. The sponsor is identified in public relations, advertising, and personal selling. Answer: D Rationale: The more funds allocated to each promotional ingredient and the more managerial emphasis placed on each technique, the more important that element is thought to be in the overall mix. 77. Which of the following statements about the characteristics of the elements in the promotional mix is true? A. Control over message content is greatest when public relations is used. B. A large audience is best reached with personal selling. C. The sponsor or company is identified in advertising. D. The mode of communication for sales promotion is usually direct and personal. E. The message flow with advertising is two-way. Answer: C Rationale: Advertising is any form of impersonal paid communication in which the sponsor or company is identified. 78. GM uses blogging to communicate with customers online. GM’s vice chairman Bob Lutz—an auto industry veteran and the company’s director of product development—began an online journal about the company in 2005. Once readers realized it really was Lutz and not an advertising ploy, they quickly warmed to the idea and appreciated his candid comments about GM and its competition. This is an example of: A. a blog. B. an interactive podcast. C. narrowcasting. D. social media. E. answers A and D. Answer: E Rationale: A blog is an online journal, a type of social media. Social media are promotion tools used to facilitate conversations among people online. 79. Which of the following outlines a sequential four-step process for achieving promotional goals? A. The communication model B. The hierarchy of impacts C. The AIDA concept D. Maslow’s hierarchy of needs E. Schramm’s model Answer: C Rationale: The AIDA acronym stands for attention, interest, desire, and action. 80. The AIDA concept: A. proves that promotional effectiveness is an insignificant abstract term. B. demonstrates that buyers go through nine stages on the way to making a decision. C. is a model effectively showing that advertising can move people to the purchase stage. D. is a model for reaching promotional objectives that outlines a sequential process for effective promotion. E. is a budget plan based on the effectiveness of various promotional mixes in achieving certain objectives. Answer: D Rationale: This describes the model called the AIDA concept. 81. AIDA stands for: A. attitude, interest, demand, activity. B. attention, interest, desire, action. C. awareness, intent, demand, action. D. avoidance, interest, desire, acceptance. E. attitudes in developing acquisitions. Answer: B Rationale: The AIDA acronym stands for attention, interest, desire, and action. 82. According to the AIDA concept, the first step an advertiser must take is to gain the consumers’: A. attention. B. attitude alignment. C. action. D. adaptation. E. adoption. Answer: A Rationale: The advertiser must first gain the attention of the target market. 83. Lyndon went to a Web site that sells barbeque grills and read reviews of several different models. He has requested more information about a Weber grill that appeared to meet his needs, but he is not yet willing to give up his search to find the perfect grill. As far as Weber is concerned, Lyndon is in the _____ step of the AIDA concept. A. action B. attention C. liking D. interest E. desire Answer: D Rationale: Lyndon is aware of the product, but he is not yet convinced it is the one he wants to purchase. 84. When Vincent saw Mike’s new Iron Horse Yakuza Aniki mountain bike, Vincent immediately knew he wanted to own one just like it someday. According to the AIDA concept, Vincent entered the _____ stage. A. action B. attention C. liking D. interest E. desire Answer: E Rationale: The Yakuza Aniki mountain bike has created a brand preference. 85. Point-of-purchase displays in grocery stores, coupons, premiums, and trial-size packages are most useful when the consumer is near the _____ stage of the AIDA model. A. liking B. awareness C. action D. preference E. knowledge Answer: C Rationale: These extras are the sales promotion techniques that often push the complacent consumer into the purchase stage. 86. For which of the following purchases is Gordo most likely to pass through all four stages of the AIDA process? A. A pair of white socks B. A two-pack of light bulbs C. A loaf of bread D. Gear for mountain climbing E. A six-pack of beer Answer: D Rationale: Most buyers involved in high-involvement purchases pass through the four stages of the AIDA model. The climbing gear would be the alternative most likely to create a high level of psychological risk. 87. Which of the following statements about the AIDA model is true? A. The AIDA model explains why public relations is the most important element in the promotional mix. B. Advertising is most effective at the purchase stage of the AIDA model. C. Certain promotional tools are more effective at certain stages of the AIDA model. D. Sales promotion is not helpful during the attention stage of the AIDA model. E. The promotional mix should not be influenced by the AIDA model. Answer: C Rationale: A helpful concept for marketing managers is that as the customer moves from awareness to purchase, different promotional tools are more effective at certain points. 88. The AIDA concept: A. assumes that the second step in the purchase-decision process is inertia. B. does not explain how all promotions influence purchase decisions. C. assumes that consumers regularly go through each stage of the process during all purchases. D. indicates that the purchase decision is the same for high-involvement and low-involvement products. E. is accurately described by none of the choices. Answer: B Rationale: The AIDA concept does not explain how all promotions influence purchase decisions. The model suggests that promotional effectiveness can be measured in terms of consumers progressing from one stage to the next. However, the order of stages in the model, as well as whether consumers go through all steps, has been much debated. 89. Protons are new, poppable snacks made with super-crunchy tofu. They are bite-sized nuggets that come in both sweet and savory flavors: Original Crunchy, Backyard Bar-B-Q, Tomato Herb Garden, Cinnamon Toast, and Lemon Meringue Pie. Which elements of the promotional mix would be most useful in attracting attention to this new snack food? A. Public relations and sales promotion B. Advertising and direct marketing C. Direct marketing and personal selling D. Public relations and direct marketing E. Advertising and public relations Answer: E Rationale: Advertising is useful in gaining attention for goods or services, and public relations has its greatest impact in gaining attention for a company, good, or service. 90. To provide more delivery service to the consumer market, UPS created UPS stores. According to the AIDA model, to create attention for this more convenient and less expensive way to mail packages through UPS, its marketing department should have relied on which element of the promotional mix? A. Public relations B. Sales promotion C. Event sponsorship D. Personal selling E. Direct marketing Answer: A Rationale: Public relations is best at drawing attention to a product or company. 91. In terms of the AIDA concept, sales promotion is most effective at creating: A. strong desire and purchase intent. B. positive attitudes toward a brand. C. awareness of a product. D. long-term interest. E. awareness of a product and purchase intent. Answer: A Rationale: Sales promotion’s greatest strength is in creating strong desire and purchase intent. 92. Entrepreneur Krissa Fernandes has developed flour made from ground almonds that she sells under the Platinum Nut brand for use in baking. Everyone who has baked with the new flour loves it, but she is having trouble getting potential consumers to the desire stage of the AIDA concept. Which of the following elements of the promotional mix would be most likely to move people to look for the product and buy it? A. Flyers placed under windshield wipers at crafts festivals B. An article in local newspaper about entrepreneurs C. Increasing the number of stores carrying Platinum Nut D. A $2-off coupon for a five-pound bag of almond flour E. A classified ad Answer: D Rationale: Sales promotions (coupons) are very effective at creating desire and action. 93. In January 2005, Burger King sponsored the opener of the third season of The Apprentice. The Apprentice sponsorship was an important element in Burger King’s overall marketing strategy to build social currency for the brand through contests, advertising, and other sales promotion. That is, the brand set out to reverse negative perceptions of Burger King through the use of a consistent and coordinated promotional message. Burger King used: A. coordinational promotion. B. promotional mixing. C. integrated marketing communications. D. creative selling. E. processed marketing. Answer: C Rationale: Integrated marketing communications is the careful coordination of promotional messages for a product to assure the consistency of messages at every contact point where a company meets the consumer. 94. _____ is the careful coordination of all promotional activities to produce a consistent, unified message that is customer focused. A. Interpersonal and mass communications (IMC) approach B. Promotional mixing C. Integrated marketing communications D. Creative selling E. Relationship marketing Answer: C Rationale: Integrated marketing communications is the careful coordination of promotional messages for a product to assure the consistency of messages at every contact point where a company meets the consumer. 95. Molson beer was produced in Canada. Coors was manufactured in the United States. A merger of the two breweries gave each brand access to a significantly larger market. To effectively reach both markets, the merged company needed to coordinate its promotional mix to produce a consistent, unified, and customer-focused message. In other words, the brewery needed to use: A. coordinational promotion. B. integrated marketing communications. C. transformational marketing. D. creative selling. E. transactional communications. Answer: B Rationale: Integrated marketing communications is the careful coordination of promotional messages for a product to assure the consistency of messages at every contact point where a company meets the consumer. 96. Why is the concept of integrated marketing communications (IMC) growing in popularity? A. The proliferation of thousands of media choices beyond the traditional media has made promotion a complicated task. B. The mass market has fragmented. C. Critics of promotion have suggested that uncoordinated promotion is wasteful and inefficient. D. Marketers have slashed their advertising spending in favor of promotional techniques that generate immediate sales responses and have more easily measured effects. E. All of these statements help to explain why IMC is growing in popularity. Answer: E Rationale: All of these statements help to explain the growing popularity of integrated marketing communications. 97. The various Survivor shows have been promoted through personal appearances by show participants, Web sites, and merchandise with the Survivor logo as well as print and broadcast advertising. To make sure that all of the promotional messages are coordinated, CBS (the network on which the show is broadcast) should use the: A. integrated marketing communications approach. B. integrated communications management approach. C. AIDA concept. D. interpersonal and mass communications approach. E. coordinated market management approach. Answer: A Rationale: Integrated marketing communications is the careful coordination of all promotional messages for a product or service to assure the consistency of messages at every contact point where a company meets the consumer. 98. Which of the following is LEAST likely to be a factor that would determine the nature of the promotional mix? A. Stage in the product life cycle B. Target market characteristics C. Funds available for promotion D. Use of a push or pull strategy E. Size of the company Answer: E Rationale: Size of the company may or may not affect the promotional mix, depending on other factors such as available funds or product type. 99. Frigo Design has developed easy-to-install panel sets to update any refrigerator, dishwasher, or compactor made since 1942. As Frigo Design plans its promotion for its new chalkboard panels, which of the following factors will be likely to affect its promotional mix? A. Its chalkboard panels are a new product. B. Its chalkboard panels are targeted to families with school-age children. C. Its chalkboard panels are easy to install. D. Its panels fit any refrigerator, dishwasher, or compactor made since 1942. E. All of these factors will affect Frigo’s promotional mix. Answer: E Rationale: Nature of the product, target market, and product life cycle stage are all factors that would affect the choice of promotional mix elements. 100. The promotional mix elements that should be emphasized for products moving into the growth stage of the product life cycle are: A. public relations and reminder advertising. B. sales promotion and public relations. C. personal selling and persuasive advertising. D. sales promotion and personal selling. E. informational advertising and publicity. Answer: C Rationale: During the growth stage, advertising and public relations remain strong, sales promotion declines in importance, and personal selling is used. Advertising is persuasive. 101. Because automobile tires are in the maturity stage of their product life cycle, the promotional mix element _____ should NOT be emphasized in the marketing of this product. A. public relations B. persuasive advertising C. sales promotion D. reminder advertising E. personal selling Answer: A Rationale: During the maturity stage, persuasive and reminder advertising are emphasized, as well as sales promotion. Personal selling is maintained, while the usefulness of public relations is doubtful. 102. For products like compact discs (which are in the decline stage of the product life cycle): A. persuasive advertising is commonly used. B. heavy public relations is used to maintain brand loyalty. C. personal selling should be used to obtain distribution. D. sales promotion is used to build market share. E. none of these is true. Answer: E Rationale: All promotion, especially advertising, is reduced as the product enters the decline stage. 103. Fans of Ale-8, a ginger-flavored soft drink, are able to explain why Ale-8 is superior to any other type of cherry cola. They are scattered throughout eastern Kentucky and southern Ohio and are extremely brand loyal. Though Ale-8 is available in bottles or cans, most connoisseurs prefer glass bottles. Ale-8 “traditionalists” even insist on drinking the beverage from the old-style “long-neck” bottles as opposed to the newer, short-neck design. The characteristics of this target market indicate that a promotional mix with _____ should be used. A. less publicity and sales promotion and more personal selling B. more advertising and sales promotion and less personal selling C. more sales promotion and personal selling and less advertising D. less sales promotion and more advertising and personal selling E. more advertising and less sales promotion and public relations Answer: B Rationale: A target market characterized by widely scattered customers, highly informed buyers, and brand-loyal repeat purchases generally requires a promotional mix with more advertising and sales promotion and less personal selling. 104. When a product is in the growth stage of its product life cycle: A. all elements of the promotional mix are equally important. B. advertising and public relations are particularly important. C. persuasive and reminder advertising is decreased. D. personal selling is ineffective. E. personal selling and sales promotions are the major elements of the promotional mix used. Answer: B Rationale: Although advertising and public relations continue to be major elements of the promotional mix, sales promotion can be reduced because consumers need fewer incentives to purchase. 105. What is the most effective promotional mix to use with products that are convenience goods and purchased routinely? A. Personal selling and advertising B. Public relations and personal selling C. Personal selling and sales promotion D. Advertising and public relations E. Sales promotion and advertising Answer: E Rationale: For routine buying decisions, the most effective promotional tools are advertising and sales promotion. 106. Which of the following elements of the promotional mix would be most effective for helping highly involved consumers make complex buying decisions? A. Personal selling B. Reminder advertising C. Sales promotion D. Public relations E. Informative advertising Answer: A Rationale: Consumers making complex buying decisions are more extensively involved. Personal selling is most effective in helping these consumers decide. 107. _____ is probably the most important factor in determining the promotional mix. A. The advertising agency B. The need for feedback C. Money D. Media availability E. Message flexibility Answer: C Rationale: Without available funds, the promotional mix is severely limited. 108. The use of aggressive personal selling and trade advertising by a manufacturer to convince a wholesaler or retailer to carry and sell particular merchandise is known as a _____ strategy. A. strong-arm B. kinetic C. pull D. foot-in-the-door E. push Answer: E Rationale: This is the definition of a push strategy. 109. Petra McCoy has developed a product for cleaning grout called Grout-Chee. Everyone who has tried the product really appreciates how quickly it works, but McCoy is having trouble getting retail stores to carry the product. She has decided to send retailers free samples and use other trade promotions to encourage stores to carry her product. McCoy is using a(n) _____ promotional strategy. A. kinetic B. inert C. push D. advertorial E. pull Answer: C Rationale: This describes a push promotional strategy. 110. An ad in a trade magazine targeted to veterinarians urges them to prescribe Hepato for cats and dogs with damaged livers. This ad was placed by the product’s manufacturer. This ad is an example of how manufacturers use a(n) _____ strategy. A. kinetic B. pull C. inertia D. advertorial E. push Answer: E Rationale: A push strategy is a marketing strategy that uses trade advertising to convince a retailer (in this case the veterinarian) to carry and sell particular merchandise. 111. Slatescape sells crushed slate to be used as a decorative, permanent mulch to keep weeds out of flower beds. Initially, Slatescape was marketed heavily to home owners, but after years of radio and television advertising, Slatescape has decided to put a majority of its promotion budget into personal selling and sales promotion to lawn and garden supply stores and large discount stores such as Walmart. This new strategy is a _____ strategy. A. kinetic B. push C. pull D. foot-in-the-door E. reinforcement Answer: B Rationale: The use of aggressive personal selling and trade advertising by a manufacturer to convince a wholesaler or retailer to carry and sell its merchandise is known as a push strategy. 112. Shannon Murphey wanted to increase sales at his Tyler, Texas, jewelry store, Murphey the Jeweler. After analyzing his sales figures, he decided to give away $50 gift certificates for local businesses to give to their employees. “Many people used the gift certificates as credit toward more expensive items,” explains Murphey. Murphey used a(n) _____ strategy. A. pull B. push C. kinetic D. inertia E. cash-flow Answer: A Rationale: A pull strategy is a marketing strategy that stimulates consumer demand to obtain product distribution. 113. The American Plastics Council wants to stimulate more consumer demand for products that are packaged in plastic. The ads will be run in consumer magazines and will hopefully cause customers to ask retailers to carry more products packaged in plastic. The American Plastics Council is planning to use a(n) _____ strategy. A. kinetic B. pull C. inertia D. advertorial E. push Answer: B Rationale: A pull strategy is a marketing strategy that stimulates consumer demand to obtain product distribution. 114. A manufacturer using the _____ promotional strategy focuses its promotional efforts on the consumer. A. reinforcement B. personal selling C. push D. pull E. kinetic Answer: D Rationale: A pull strategy is a marketing strategy that stimulates consumer demand to obtain product distribution. 115. An ad for a prescription-only antihistamine that does not make its users sleepy like many other allergy medications was run in several health and fitness magazines. The ad said that if you were interested in learning more about the new product or trying it out, you should ask your physician. The manufacturer of the drug was using a _____ promotional strategy. A. reinforcement B. personal selling C. push D. pull E. kinetic Answer: D Rationale: A manufacturer using the pull promotional strategy focuses its promotional efforts on the consumer. 116. Procter & Gamble uses cents-off campaigns, couponing, and free samples to increase sales of Tide detergent. In other words, it uses a _____ promotional strategy. A. push B. personal selling C. pull D. reinforcing E. kinetic Answer: C Rationale: A manufacturer using the pull strategy focuses its promotional efforts on the consumer. 117. To get more retailers to stock its products, Rust-Oleum paint developed a strong advertising campaign aimed at consumers. The ads show creative ways to use paint to make homes more attractive and emphasize that the paint doesn’t chip, peel, or fade. Rust-Oleum is using a _____ promotional strategy. A. push B. channel C. reinforcement D. pull E. kinetic Answer: D Rationale: A pull strategy involves ad campaigns aimed at consumers in hopes of getting consumers to demand the product from retailers who in turn will demand it from manufacturers. Cable TV It wasn’t so long ago that cable swaggered around the television industry as the upstart threatening the broadcast networks. Now, cable is old news. The antagonist is the satellite dish with the capability of delivering hundreds of channels and offering an array of movies and sports events. The technology is known as direct broadcast satellite, or DBS. The new challenger has awakened the “sleeping giant” cable industry, which has hit the airwaves with some of the most aggressive advertisements the television industry has seen. The ads emphasize the downside of disconnecting cable and choosing to purchase a satellite dish. One commercial opened with a man on a couch joined by his wife with a large bowl of popcorn. They snuggled in front of their television set, hooked up to DBS. She seemed excited about their new system and exclaimed, “So this is it?” She then asked how much the satellite dish cost. When her husband answered, the excitement was off. “There’s no monthly fee?” she asked. “No more than cable,” he responded. When she asked to watch the news, he sheepishly said, “Uh, can’t get local news.” At the end of the commercial, the spokesperson for the cable industry declared, “These days when everyone is promising you the future of television, isn’t it nice to know you already have it?” 118. Refer to Cable TV. The television advertising utilized by cable operators represents the means by which the company communicates to current users to influence an opinion or elicit a response. This is called: A. distribution. B. price. C. research and development. D. promotion. E. publicity. Answer: D Rationale: Promotion is communication by marketers that informs, persuades, and reminds potential buyers of a product in order to influence an opinion or elicit a response. 119. Refer to Cable TV. The cable industry’s ad campaigns are designed to communicate to a large audience via television and radio. It uses _____ communication. A. mass B. clutter C. interpersonal D. intrapersonal E. public Answer: A Rationale: Mass communication involves communicating to large audiences, usually through a mass medium such as television or newspaper. 120. Refer to Cable TV. The cable industry is the originator of this ad campaign against satellite dishes. The industry represents the _____ in the communication process. A. communicator B. encoder C. sender D. channeler E. receiver Answer: C Rationale: The sender is the originator of the message in the communication process. 121. Refer to Cable TV. Television represents the _____ in the communication process used by the cable industry to transmit its message. A. feedback loop B. channel C. encoder D. receiver E. communicator Answer: B Rationale: A channel is any communication medium utilized to transmit a message. 122. Refer to Cable TV. Keisha is watching television and sees an ad for DBS. Two ads later, she sees the new television ad for her local cable system. When questioned an hour later about advertising she has seen most recently, Keisha remembers a “cable” ad but cannot recall the details. Keisha’s inability to recall the ad is most likely caused by _____ in the communication process. A. feedback B. the use of a channel C. a shared frame of reference D. cognitive dissonance E. noise Answer: E Rationale: Noise is anything that interferes with, distorts, or slows down the transmission of information. Competing ads can be a source of noise. RugRatReaders Greg Martin has developed his own line of children’s books. The books are written for children ages 3 to 6 and are produced using cotton fiber as pages, rendering them “nearly” indestructible. This unique feature makes Martin’s books superior to the competition. His products are called RugRatReaders. Martin has decided to advertise the books to his target market (women with children ages 3 to 6) via radio. He does not have a formal advertising budget, but he will spend whatever money he has in the bank after producing the books each month. His message will focus on the product and its unique features. Martin has been having problems getting retailers to stock his new product. It is Martin’s hope that consumers will go to their local bookstores and ask for the RugRatReaders. 123. Refer to RugRatReaders. In all of Martin’s ads, he will emphasize the books’ indestructibility. This represents the product’s: A. tactical advantage. B. competitive advantage. C. feedback. D. promotional result. E. objective strategy. Answer: B Rationale: A competitive advantage is the unique set of features of a company and its products that are perceived by the target market as significant and superior to the competition. 124. Refer to RugRatReaders. The promotional campaign Martin is utilizing is designed to accomplish which basic promotional task? A. Rewarding B. Persuading C. Informing D. Influencing E. Reminding Answer: C Rationale: Informative promotion is important for a new product and more prevalent during the early stages of the product life cycle. 125. Refer to RugRatReaders. Martin’s choice of radio as his promotional tool is based on the fact that it is a one-way mass communication vehicle. Radio is most commonly used with which promotional mix element? A. Public relations B. Advertising C. Direct marketing D. Sales promotion E. Sponsorship Answer: B Rationale: Advertising is any form of impersonal, one-way communication paid for by the sponsor. Radio is an example of an advertising medium. 126. Refer to RugRatReaders. According to the AIDA model, the first step Martin must achieve with his target market is: A. attention. B. interest. C. liking. D. desire. E. action. Answer: A Rationale: The first step in AIDA is to draw customers’ attention to the product. 127. Refer to RugRatReaders. In attempting to get consumers to ask the retailer for his product, Martin is using a _____ promotional strategy: A. push B. pull C. kinetic D. publicity E. reinforcement Answer: B Rationale: A pull strategy stimulates consumer demand to obtain product distribution. Methodist Church In 2001, the United Methodist Church, the second-largest Protestant denomination in the United States, launched a four-year, $20 million national ad campaign. The theologically gentle but culturally edgy ads are aimed at Americans who are unchurched––people who know the name “Methodist” but don’t know anything else about it. The “Igniting Ministries” ads have appeared on CBS and cable television. The purpose of the ads is to draw more people through Methodist Church doors in a time when mainline Protestant denominations are struggling. 128. Refer to Methodist Church. The ad campaign will use _____ communication. A. mass B. interpersonal C. direct D. hierarchical E. functional Answer: A Rationale: Mass communication involves communicating a concept or message to large audiences. 129. Refer to Methodist Church. In terms of the communication process, the unchurched who see the ads while watching television are expected to be: A. channels and encoders. B. senders and receivers. C. receivers and decoders. D. channels and decoders. E. receivers and channels. Answer: C Rationale: The receiver is the person who decodes a message. The decoder interprets the language and symbols sent by the source through a channel. 130. Refer to Methodist Church. The church is hoping to see an increase of 3 percent in visitors to Methodist churches as a result of this ad. In terms of the communication process, this is the desired: A. noise. B. encoding. C. reception. D. channel. E. feedback. Answer: E Rationale: Feedback is the response to the promotional message. 131. Refer to Methodist Church. A news article about the Methodist Church that appeared in the Rome News-Tribune was an example of which promotional mix element? A. Advertising B. Sales promotion C. Pull strategy D. Public relations E. Personal selling Answer: D Rationale: Public relations is the marketing function that evaluates public attitudes, identifies areas within the organization the public may be interested in, and executes a program of action to earn public understanding and acceptance. 132. Refer to Methodist Church. According to the AIDA concept, the ads must first: A. create interest in the Methodist Church. B. draw attention to the Methodist Church. C. act on unfulfilled needs of the unchurched. D. augment current knowledge. E. create a desire to attend the Methodist Church. Answer: B Rationale: AIDA is attention, interest, desire, and action. 133. Refer to Methodist Church. How might the United Methodist Church benefit from use of integrated marketing communications (IMC)? A. Ads built using the IMC concept are not as closely regulated by the Federal Trade Commission. B. Noise can be eliminated from the communication channel with the use of IMC concepts. C. Correct decoding is guaranteed with the application of IMC concepts. D. The application of IMC concepts allows marketers to reach a more fragmented market. E. None of these statements explains how the United Methodist Church could benefit from the use of IMC. Answer: D Rationale: Instead of promoting a product just through mass media options, like television and magazines, promotional messages today can appear in many varied sources. Amtrak Amtrak is taking the gloves off in advertisements and belittling its major competition, which is airline shuttles. Amtrak’s new name for its Metroliner is The Smart Shuttle. Gone are ads that were inwardly directed and sold the Metroliner on its leisurely pace. In their place are ads showing rapid shots of the sleek train and business executives conducting no-nonsense meetings “eyeball-to-eyeball.” The campaign, which includes television, radio, print, billboards, and posters in commuter railroad cars, targets business travelers, who, Amtrak says, incorrectly believes the train takes far longer than the airplane to travel between two cities. One television spot opens with a shot of airplanes stacked up in a tarmac traffic jam. Another shows a business traveler lumbering into the airport during a snowstorm. The announcer says, “Schedule a meeting. Take the shuttle. Then cross your fingers.” The poor soul looks up at the departure board and sees a row of “Canceled” signs. The announcer continues, “Schedule a meeting. Take the Metroliner. Then cross your legs.” 134. Refer to Amtrak. The means by which Amtrak communicates with business travelers about the merits and characteristics of the Metroliner is called: A. distribution. B. direct marketing. C. promotion. D. publicity. E. personal selling. Answer: C Rationale: Promotion is the means by which businesses communicate with their target audiences. 135. Refer to Amtrak. The plan to optimally utilize television, radio, print, and out-of-home vehicles to target business travelers is called a: A. promotional strategy. B. marketing mix. C. promotion goal. D. selling plan. E. publicity plan. Answer: A Rationale: A promotional strategy is a plan for the optimal use of the elements of promotion. 136. Refer to Amtrak. Amtrak’s campaign stresses that the Metroliner is more comfortable and more likely to be on time and run in inclement weather. The promotional campaign focuses on Amtrak’s: A. superior skill set B. special feature C. tactical strategy D. differential parity E. competitive advantage Answer: E Rationale: A competitive advantage is the unique set of features of a company and its products. 137. Refer to Amtrak. Amtrak’s campaign is designed to communicate to a large audience via television, radio, and print. This is an example of _____ communication. A. mass B. clutter C. interpersonal D. intrapersonal E. public Answer: A Rationale: Mass communication involves communicating to large audiences, usually through a mass medium such as television or newspaper. 138. Refer to Amtrak. Amtrak originated the new campaign. Amtrak represents the _____ in the communication process. A. promotional impetus B. decoder C. sender D. channeler E. receiver Answer: C Rationale: The sender is the originator of the message in the communication process. 139. Refer to Amtrak. Television, radio, print, and out-of-home media represent the _____ in the communication process used by Amtrak. A. feedback loop B. channel C. encoder D. receiver E. communicator Answer: B Rationale: A channel is any communication medium utilized to transmit a message. 140. Refer to Amtrak. Amtrak’s careful coordination of all of its promotional activities to present a single, focused communication to its consumers is an example of: A. coordinational promotion. B. promotional mixing. C. integrated marketing communications. D. creative selling. E. processed marketing. Answer: C Rationale: Integrated marketing communications is the careful coordination of promotional messages for a product to assure the consistency of messages at every contact point where a company meets the consumer. ESSAY 1. What is promotion? Why is it a vital part of a company’s marketing mix? Answer: Promotion is communication by marketers that informs, persuades, and/or reminds potential buyers of a product in order to influence an opinion or elicit a response. It is a vital part of the marketing mix because few goods or services, no matter how well developed, can survive in the marketplace without effective promotion. It also informs the consumer of the product’s benefits, thus positioning it in the marketplace. The main function of promotion is to convince target customers that the goods and services offered provide a competitive advantage over the competition. 2. What is the basic definition of communication? What are two forms of communication? Give specific examples of these two forms and indicate how a company could use each of them. Answer: Communication is the process by which meanings are exchanged or shared though a common set of symbols. Individuals assign meanings to feelings, ideas, facts, and attitudes. INTERPERSONAL COMMUNICATION is direct, face-to-face communication between two or more people. With this communication, people know whom they are communicating with and can respond to the other person’s reaction almost immediately. An example of this is a salesperson speaking directly with a client. MASS COMMUNICATION is the communication of a concept or message to large audiences, usually through a mass medium such as television, radio, magazine, or a newspaper. In this case, the company does not know individually the people with whom it is communicating, and there is no way to respond immediately to the consumers’ reactions. 3. You are the promotions director for Sunland Farm cholesterol-free eggs. Describe the steps in the communication process in your promotion campaign, using specific examples of the actions that will take place in promoting the eggs. Answer: 1. The sender encodes ideas into a promotional message. In this case, the promotions director at Sunland Farm is the sender of the message and may hire an advertising agency to create or encode an advertising message. 2. The message is transmitted through a channel, or communication medium. Sunland Farm’s promotion may be carried by television, radio, magazines, newspapers, billboards, or other media. 3. The message may or may not be received by the intended target market. The audience may not be exposed to the channel. 4. Receivers decode the message by interpreting the contents of the promotion. Messages may or may not be properly decoded. When people receive messages, they tend to manipulate, alter, and modify the messages to reflect their own biases, needs, knowledge, and culture. 5. The receiver’s response to a message is feedback to the source. In mass communications, feedback is indirect rather than direct. Sunland Farm may need to conduct marketing research to see if the audience can recognize, recall, or state that they have been exposed to the message. 6. Any phase of this process may be hindered by noise, such as competitors’ advertisements. 4. What are the four basic tasks of promotion? For each of these four tasks, give two specific promotions examples of how that task might be accomplished. Answer: INFORM • Increase awareness of a new product or brand • Inform the market of new product attributes • Suggest new uses for a product • Reduce consumers’ anxieties • Tell the market about a price change • Describe available services • Correct false impressions • Explain the way a product or service works • Build a company image PERSUADE • Build brand preference • Encourage brand switching • Change customers’ perceptions of product attributes • Influence consumers to buy now • Persuade customers to receive a sales call REMIND • Remind consumers that the product may be needed in the near future • Remind consumers where to buy the product • Keep the product in consumers’ minds during off times • Maintain consumer awareness CONNECT • Share ideas, information and feedback • Facilitate an exchange of information • Create brand advocates that promote the brand • Use social networks, social games, social publishing tools, and social commerce 5. The promotional mix is made up of a blend of five promotional tools. Name and briefly define each of these five tools. Answer: ADVERTISING is impersonal, one-way mass communication about a product or organizer that is paid for by the marketer. Advertising is transmitted by different media, including television, radio, newspapers, magazines, books, direct mail, billboards, and transit cards. PUBLIC RELATIONS is the marketing function that evaluates public attitudes, identifies areas within the organization the public may be interested in, and executes a program of action to earn public understanding and acceptance. A solid public relations program can generate favorable publicity. SALES PROMOTION consists of all marketing activities that stimulate consumer purchasing and dealer effectiveness. Examples include free samples, contests, premiums, trade shows, vacation giveaways, and coupons. PERSONAL SELLING is a purchase situation involving a personal, paid-for communication between two people in an attempt to influence each other. It involves a planned face-to-face presentation to one or more prospective purchasers for the purpose of making sales. Personal selling was initially more prevalent in the industrial goods field but now plays an important role in relationship marketing. SOCIAL MEDIA are promotion tools used to facilitate conversations among people online. When used by marketers, these tools facilitate consumer empowerment. Social media include blogs, microblogs, podcasting, vodcasts, and social networks. 6. You are the promotions manager for a new weight-loss program that is based on a low-carbohydrate diet. Describe how you might use all five tools of the promotional mix to promote the product. Answer: Promotion tools should be directed toward a weight-conscious target audience. General examples of tools are: ADVERTISING could take place in many different media, including television, radio, newspapers, magazines, books, direct mail, billboards, and transit cards. PUBLIC RELATIONS could be used to generate publicity for the new product in the form of news items, feature articles, or sporting event sponsorship. SALES PROMOTION could include free samples, contests, premiums, trade shows, vacation giveaways, and coupons. PERSONAL SELLING may be needed to encourage wholesalers and/or retailers to carry the product. However, personal selling may not be feasible to reach a large target audience. SOCIAL MEDIA could include online videos posted on YouTube that inform consumers of the new product; a Twitter and/or Facebook account where customers can interact with the company; and a corporate blog that talks about various aspects of the product and invites consumer responses. 7. The ultimate objective of any promotion is a purchase or some other activity. A model for reaching promotional objectives is called the AIDA concept. What does this acronym stand for? Describe what marketing actions might take place in each stage of this model. Answer: AIDA stands for attention, interest, desire, and action. A promotion manager attracts a person’s attention by a greeting and approach (personal selling) or loud volume, unusual contrasts, bold headlines, movement, bright colors, and so on (advertising and sales promotion). Second, a good sales presentation, demonstration, or promotional copy creates interest in the product. Third, desire is created by illustrating how the product’s features will satisfy the consumer’s needs. Finally, a special offer or a strong closing sales pitch may be used to obtain purchase action. 8. The ultimate objective of any promotion is a purchase or some other activity. One model for reaching promotional objectives is called the AIDA concept. This model proposes that consumers are moved through four stages by the promotional mix. Suppose you are the promotional manager for the new B-Gone squirrel-proof bird feeders. (Before the B-Gone, there were no feeders on the market that were really squirrel proof.) This bird feeder is guaranteed to prevent squirrels from reaching and eating its contents. Describe the promotional activities you would recommend in each stage of AIDA model for the B-Gone feeder. Assume that personal selling will NOT be used. Answer: ATTENTION. General awareness could be created with heavy publicity, advertisements on television during nature shows, and print ads in bird-watching magazines. INTEREST. Simple awareness generally does not result in a sale. The target market should be informed about the product’s benefits. Print advertisements could detail features of the B-Gone, including price, availability, construction material, installation instructions, and so forth. These ads could also explain how the feeder keeps squirrels out. To generate a favorable attitude, bird-watching clubs could be sent a sample B-Gone and then be asked to recommend the feeders in television, radio, and magazine advertisements. Managers of stores catering to nature lovers could also be sent free samples and then be asked to recommend the B-Gone to their customers. DESIRE. In this stage, promotion should create brand preference. Promotions should tell bird lovers how much better off the birds will be if they install a B-Gone feeder. Promotions should also emphasize that bird lovers will spend fewer aggravating hours trying to chase squirrels away. The fact the feeder is squirrel proof should be promoted as the competitive advantage. ACTION. Catchy displays in stores targeted to nature lovers, coupons in bird-watching magazines, premiums (such as free seed or bird food), a display of how a B-Gone feeder works in some public park, or other forms of sales promotion could be used to encourage a consumer to purchase. 9. What is integrated marketing communications (IMC)? How is IMC implemented? Answer: Integrated marketing communications (IMC) is the careful coordination of all promotional activities to produce a consistent, unified message that is consumer focused. To implement IMC, marketing managers carefully work out the roles that the various promotional elements will play in the marketing mix. Timing of promotional activities is coordinated, and the results of each campaign are carefully monitored to improve future use of the promotional mix tools. Typically, one individual in the firm is given the overall responsibility for integrating the company’s marketing communications. 10. List three reasons why the concept of integrated marketing communications has been growing in popularity. Answer: 1. The proliferation of thousands of media choices beyond traditional media has made promotion a more complicated process. 2. The mass market has become increasingly more fragmented. 3. Marketers have slashed their advertising spending in favor of promotional techniques that generate immediate sales responses and those that are more easily measured. 4. The interest in IMC is largely a reaction to the scrutiny that marketing communications has come under and particularly to suggestions that uncoordinated promotional activities are wasteful and inefficient. 11. Several factors affect the choice of promotional mix. Name five of these factors. Answer: • Nature of the product (consumer or industrial) • Stage in the product life cycle • Target market characteristics • Type of buying decision • Funds available for promotion • Whether a push or pull strategy will be used 12. Several factors inherent to the product itself influence the promotional mix. Name and briefly describe three of these factors. Answer: NATURE OF THE PRODUCT. If the product is a business product, it is less likely to be well suited for mass communication. Since they are often custom-tailored, business products are better suited for personal selling. Consumer products are designed to appeal to a large audience, so advertising and sales promotion are more appropriate. STAGE IN THE PRODUCT LIFE CYCLE. During the introduction stage, emphasis is placed on advertising and public relations as well as some sales promotion and personal selling. During growth, sales promotion efforts are reduced, while advertising and public relations continue. At maturity, sales promotion and advertising become the focus. All promotion is reduced during the decline stage. TARGET MARKET CHARACTERISTICS. A target market characterized by widely scattered potential customers, highly informed buyers, and brand-loyal repeat purchasers generally requires a promotional mix with more advertising and sales promotion and less personal selling. TYPE OF BUYING DECISION. The promotional mix depends on the type of buying decision—for example, a routine decision or a complex decision. Advertising and sales promotion are the most effective promotional tools for routine purchase decisions, such as buying toothpaste. A decision that is neither routine nor complex may be influenced by advertising and public relations that create awareness of a product. For complex, high-involvement decisions, personal selling and print advertising are effective and provide the information needed. AVAILABLE FUNDS. Money, or the lack of it, may easily be the most important factor in determining the promotional mix. When funds are available to permit a mix of promotional elements, a firm will generally try to optimize its return on promotion dollars while minimizing the cost per contact, or the cost of reaching one member of the target market. 13. List the four stages of the product life cycle. Then describe the promotional strategies used in each stage. Answer: INTRODUCTION. During the introduction stage, emphasis is placed on general product class factors and informative messages. Heavy advertising and public relations are used to build awareness, while sales promotion can be used to induce trial. Personal selling is used in this stage to obtain distribution. GROWTH. In this stage, promotion emphasizes the product’s competitive advantage over the competition. Heavy advertising and public relations are used to build brand loyalty. Sales promotion decreases, while personal selling is used to maintain distribution. MATURITY. With increased competition, advertising may decrease slightly, and the advertising used is more often the persuasive and/or reminder type. Sales promotion increases to build market share, and personal selling is used to maintain distribution. DECLINE. In general, all products decline, and promotional messages may be focused on other firm products. Because of expense, advertising and public relations are dramatically decreased. Sales promotion and personal selling may be maintained at low levels. 14. You are the promotions manager for a small, financially strapped company. Describe how a lack of funds affects the use of each of the promotional tools. Describe the trade-offs that occur between funds available and other aspects of promotion. Answer: A firm with limited funds can rely heavily on publicity if the product is unique. If personal selling is necessary, the firm may use manufacturers’ agents who work on a commission basis. Some sales promotions can also be inexpensive. Although advertising is very expensive, it has low cost per contact, which may be necessary for a large potential market. There is usually a trade-off among the funds available, the number of people in the target market, the quality of communication needed, and the relative costs of the various promotional elements. 15. You are the promotions manager for Pyrolave, a company that turns lava rock into counter tile. The company’s counter tile is easy to clean and resistant to scratches but is more expensive than other materials used for countertops. You need to decide whether to implement a push or a pull promotional strategy. Define and describe both the push and the pull strategy, and describe what promotions might be used in each strategy. Answer: PUSH STRATEGY is when manufacturers use aggressive personal selling and trade advertising to convince a wholesaler or a retailer to carry and sell their merchandise. The wholesaler, in turn, must push the merchandise forward by persuading the retailer to handle the goods. The retailer then uses advertising, displays, and so on to convince the consumer to buy the pushed products. PULL STRATEGY stimulates consumer demand to obtain product distribution. In this case, the manufacturer focuses its promotional efforts on final consumers, usually with mass media. As consumers demand the product, the retailer orders the merchandise from the wholesaler. As the wholesaler is confronted with rising demand, it places orders for the merchandise from the manufacturer. Stimulating final consumer demand pulls the product through the distribution channel. Consumer advertising, cents-off campaigns, and couponing are part of a pull strategy. Chapter 16—Advertising, Public Relations, and Sales Promotion TRUE/FALSE 1. New brands with a small market shares tend to spend proportionately more for advertising and sales promotion than do those with large market shares. Answer: True Rationale: New brands with smaller market shares often allocate a higher proportion of their budgets to advertising and sales promotion compared to brands with larger market shares. 2. The advertising response function is a phenomenon in which spending for advertising and sales promotion increases sales or market share up to a certain level but then produces diminishing results. Answer: True Rationale: The advertising response function describes how increased spending on advertising and sales promotion initially boosts sales or market share, but beyond a certain point, further spending may result in diminishing returns, where additional expenditures do not yield proportional increases in sales or market share. 3. The goal of advertising is to change people’s values. Answer: False Rationale: Advertising cannot change a person’s basic value system, moral code, or cultural influences. Advertising instead tries to change consumers’ attitudes about brands. 4. On Subway napkins, nutritional information for their 6-inch subs is compared to McDonald’s Big Mac and Burger King Whopper. This is an example of competitive advertising. Answer: False Rationale: Comparative advertising specifically compares two or more products. 5. The goal of competitive advertising is to influence demand for a specific brand. Answer: True Rationale: Competitive advertising aims to influence demand for a specific brand by positioning it favorably against competitors. 6. The DAGMAR approach is one method of setting advertising objectives. Answer: True Rationale: DAGMAR (Defining Advertising Goals for Measured Advertising Results) is indeed a method used for setting advertising objectives, focusing on defining clear and measurable goals to ensure advertising effectiveness and success. 7. An appeal to vanity and egotism is the reason celebrity spokespeople are used so often in advertising. Answer: False Rationale: This common advertising appeal is admiration. See Exhibit 16.1. 8. An insurance company’s advertisement poses the question, “Will your family survive if you die uninsured?” This is an example of a health appeal. Answer: False Rationale: This is an example of a fear appeal. See Exhibit 16.1. 9. There are many snack foods for children, but only Nature’s Path makes an organic snack designed for a child’s palate. The product’s unique selling proposition is that it is completely organic. Answer: True Rationale: Nature’s Path's unique selling proposition (USP) for its organic snack designed for children is its completely organic nature, which differentiates it in the market. 10. Humorous executional styles are more often used in radio and television advertising than in print or magazine advertising. Answer: True Rationale: Humorous styles are more prevalent in radio and television advertising because these mediums allow for audiovisual storytelling and comedic timing, which can engage audiences effectively. 11. One of the most popular approaches for Internet advertising is search engine ads. Answer: True Rationale: Search engine ads are highly popular in Internet advertising due to their targeted nature, appearing when users search for specific keywords related to products or services. 12. Advergaming is an Internet advertising format where companies put ad messages in Web-based or video games to advertise or promote products. Answer: True Rationale: Advergaming involves integrating advertising messages into web-based or video games, leveraging the interactive and engaging nature of gaming platforms to promote products subtly. 13. Mobile advertising may be one of the most promising tactics for contacting consumers when they are thinking about a specific product. Answer: True Rationale: Mobile advertising capitalizes on reaching consumers at opportune moments when they are actively engaged with their mobile devices, making it a promising tactic for targeted marketing and influencing purchasing decisions. 14. Publicity is the element in the promotional mix that evaluates public attitudes, identifies issues that may elicit public concern, and executes programs to gain public understanding and acceptance. Answer: False Rationale: The statement describes public relations. Publicity is the outcome of these public relations efforts. 15. An example of product placement is when the judges on American Idol are shown sipping Coca-Cola during the broadcast. Answer: True Rationale: Product placement refers to integrating branded products or messages into entertainment content, such as television shows or movies, to subtly promote them to viewers or audiences. 16. Influencing legislators and government officials is an example of a public relations activity. Answer: True Rationale: Public relations activities include influencing legislators and government officials to create a favorable environment or legislation for a company or industry. 17. Companies increasingly are using the Internet in their public relations strategies. Answer: True Rationale: Companies are increasingly utilizing the internet for various public relations strategies, such as social media campaigns, online press releases, and influencer partnerships. 18. In our free-press environment, publicity is not easily controlled. Answer: True Rationale: Publicity, which is a part of public relations, involves information spread through media that is not directly controlled by the company, making it difficult to control the message in a free-press environment. 19. Sales promotion offers an incentive to buy. Answer: True Rationale: Sales promotion involves offering incentives, such as discounts, coupons, or contests, to stimulate immediate consumer purchasing. 20. The objectives of a promotion depend on the general behavior of target consumers. Answer: True Rationale: The objectives of a promotion strategy are tailored to influence the behaviors of target consumers, such as increasing sales, generating leads, or enhancing brand awareness. 21. Research shows that coupons tend to increase the amount of a product bought. Answer: True Rationale: Research indicates that coupons encourage consumers to purchase larger quantities of a product than they might otherwise buy. 22. When cosmetics company Estée Lauder gives customers a bag of small-sized cosmetics with the purchase of a full-priced item, this is an example of a premium offer. Answer: True Rationale: Offering customers a free bag of small-sized cosmetics with the purchase of a full-priced item is an example of a premium offer, which is a sales promotion technique designed to add value and encourage purchases. 23. Point-of-purchase promotions are seldom used as a sales promotion tool because most purchase decisions are made before consumers ever enter a store. Answer: False Rationale: Point-of-purchase promotions are an extremely important sales promotion tool because between 70 and 80 percent of all retail purchase decisions are made in the store. 24. A trade allowance is a price reduction offered by retailers to customers who are buying homogeneous shopping goods. Answer: False Rationale: A trade allowance is a price reduction offered by manufacturers to intermediaries. 25. Trade association meetings, conferences, and conventions are an important aspect of sales promotion and a growing, multi-billion-dollar market. Answer: True Rationale: Trade association meetings, conferences, and conventions play a crucial role in sales promotion by providing opportunities for networking, knowledge sharing, and showcasing products or services, contributing to a significant and expanding market in the business events industry. MULTIPLE CHOICE 1. _____ is any form of impersonal, sponsor-paid, one-way mass communication. A. Publicity B. Advertising C. Promotion D. Public relations E. Direct marketing Answer: B Rationale: This defines advertising. 2. Kraft’s marketing efforts for its Macaroni & Cheese include television commercial and magazine inserts. These are all activities associated with: A. advertising. B. publicity. C. public relations. D. sales promotions. E. direct marketing. Answer: A Rationale: Advertising is any form of impersonal, paid communications that are paid for by a marketer. 3. In 2011, advertising expenditures: A. increased. B. decreased. C. Equaled those in 2010. D. equaled those in 2012. E. were not tracked. Answer: A Rationale: Expenditures increased in 2011 and were expected to grow by about 5.5 percent globally in 2012. 4. Why do many brands with a large market share spend proportionally less on advertising compared to brands with small market share? A. Beyond a certain volume of promotion, diminishing returns set in. B. Certain industries have a practice of spending a low amount of dollars, relative to sales, on advertising. C. There is no minimum level of exposure for advertising to have an effect on sales. D. Advertising will not stimulate economic growth for the industry. E. The firms with large market share do not have to advertise anymore. Answer: A Rationale: Sales and market share improvements slow down and eventually decrease no matter how much is spent on advertising and sales promotion. 5. The advertising response function helps marketers: A. create ads that will be noticed by consumers each time the ad runs. B. use their advertising budgets wisely. C. calculate the break-even points for each product being advertised. D. determine the reach and frequency of specific media. E. select an executional appeal appropriate to the product being advertised. Answer: B Rationale: The advertising response function is a phenomenon in which spending for advertising and sales promotion increases sales or market share up to a certain level but then produces diminishing results. 6. Why do companies with new brands that have a small market share tend to spend proportionately more for advertising and sales promotions compared to those with a large market share for existing products? A. As a result of experiential marketing research B. To create the appropriate level of institutional advertising C. To determine which executional appeal is the most effective D. To achieve a certain minimum level of exposure to measurably affect purchase habits E. To prevent product cannibalization Answer: D Rationale: There are two reasons why companies with new brands with small market share tend to spend proportionately more for advertising and sales promotions compared to those with existing products and large market share. The first reason is in response to the advertising response function. The second reason is to create product awareness in the minds of the target audience. 7. When Gillette introduced its Venus razor for women several years ago, it spent $100 million on advertising and other promotions and successfully boosted its market share. As an advertising consultant, you would advise Gillette to now: A. double its advertising budget in order to double its market share. B. maintain the current advertising budget and continue to focus on brand awareness and market share gains. C. slowly increase the amount spent on advertising and promotion to offset competitive advertisements. D. shift the budget dollars from advertising to sales promotion and personal selling due to a change in the product life cycle stage. E. focus on maintaining market share and spend proportionately less on advertising because additional expenditures would reap diminishing benefits. Answer: E Rationale: The advertising response function shows that beyond a certain level of spending for advertising and sales promotion, diminishing returns set in. As a mature, successful company, Gillette can choose to spend only as much as needed to maintain market share. 8. How can advertising affect consumers? A. It can change cultural practices and conventions B. It can change strongly held values C. It can manipulate society against its will D. It can change negative attitudes to positive ones E. It can make people buy things they do not want Answer: D Rationale: Advertising can affect attitudes but cannot alter values or force people to do things they do not want to do or believe are wrong. 9. _____ advertising is designed to enhance a company’s image rather than promote a particular product. A. Publicity B. Institutional C. Pioneering D. Selective E. Image Answer: B Rationale: This is the definition of institutional advertising. 10. Tobacco company Philip Morris runs television ads where they promote quitting smoking and direct viewers to a Web site for support and help. These ads are examples of _____ advertising. A. product B. institutional C. pioneering D. environmental E. comparative Answer: B Rationale: Institutional advertising is a form of advertising designed to enhance a company’s image rather than promote a particular product. 11. DuPont has an advertisement that shows police officers describing how they were shot at close range in the line of duty. The advertisement explains that the lives of these public servants were saved by Kevlar bullet-proof vests and that Kevlar was invented by DuPont. This is an example of _____ advertising. A. publicity B. attribute C. image D. pioneering E. institutional Answer: E Rationale: Institutional advertising is designed to enhance the image of the company and asks for no action except maintaining a favorable attitude toward the advertiser and its goods and services. 12. Which form of advertising involves an organization expressing its views on controversial issues or responding to media attacks? A. Advocacy B. Persuasive C. Issue D. Comparative E. Image Answer: A Rationale: This is the definition of advocacy advertising. 13. Exxon Mobil has launched a series of ads promoting its commitment to alternative energy sources during a time when the company is recording record profits while consumers struggle with high gas prices and environmentalists are concerned that burning fossil fuels is contributing to the acceleration of global warming. This is an example of _____ advertising. A. global B. homogeneous C. pioneering D. advocacy E. image Answer: D Rationale: Advocacy advertising is a means for organizations to express their viewpoints on various controversial issues or respond to media attacks. 14. Unlike advertising that establishes or maintains a company’s identity, _____ advertising touts the benefits of a specific good or service. A. selective B. cooperative C. advocacy D. image E. product Answer: E Rationale: This is the definition of product advertising. 15. Kia Motors runs television advertisements that encourage the purchase of their Kia Soul compact car. This is an example of _____ advertising. A. pioneering B. circumstantial C. product D. advocacy E. comparative Answer: C Rationale: Product advertising touts the benefits of a specific product or service. 16. Pioneering advertising, competitive advertising, and comparative advertising are all types of: A. institutional advertising B. product advertising C. primary advertising D. commercial advertising E. commissioned advertising Answer: B Rationale: Product advertising promotes the benefits of a specific good or service, and the types used are pioneering, competitive, and comparative advertising. 17. Instead of sending artfully arranged flower bouquets, an entrepreneur has developed fresh fruit bouquets and is marketing them under the name Edible Arrangements. The premium fruit is cut to mimic flowers, arranged in a basket, and carefully delivered to the recipient’s home. One of the problems is convincing people that it is not a novelty product, but an attractive way to show how you feel about someone. Edible Arrangements will be using _____ advertising to promote its fruit bouquets. A. pioneering B. innovative C. introductory D. start-up E. focused Answer: A Rationale: Pioneering advertising is heavily utilized during the introductory stage of the product life cycle. 18. Cranium, the Seattle-based toy company, has recently introduced Giggle Gear, a toy designed to let kids’ imagination run wild. With the face and headpieces that make up Giggle Gear, kids can change themselves into aliens, fairies, bugs, and robots. What type of advertising will this company be using to promote this new toy? A. Pioneering B. Innovative C. Promotional D. Start-up E. Comparative Answer: A Rationale: Pioneering advertising is intended to stimulate demand for a new product. 19. Which form of advertising is designed to influence demand for a specific brand? A. Institutional B. Primary C. Competitive D. Direct E. Growth Answer: C Rationale: Competitive advertising is used when a product enters the growth phase of the product life cycle and other companies begin to enter the marketplace. 20. An emphasis on branding generally begins as firms use _____ advertising. A. comparative B. differentiational C. pioneering D. institutional E. competitive Answer: E Rationale: The goal of competitive advertising is to influence demand for a specific brand. Such advertising is less informational and more emotional. The emphasis is on brand name recall and a favorable attitude toward the brand. 21. _____ advertising compares two or more specifically named or shown competing brands on one or more specific attributes. A. Contrasting B. Comparative C. Pioneering D. Superlative E. Differentiational Answer: B Rationale: This is the definition of comparative advertising. 22. An ad in which Dunkin’ Donuts tells consumers that more “hard-working” people prefer their coffee than the high-priced, “elitist” coffee sold at Starbucks is an example of _____ advertising. A. competitive B. institutional C. comparative D. image E. pioneering Answer: C Rationale: This comparative advertising names Dunkin Donut’s competitor, Starbucks, and distinguishes the differences in the network coverage. 23. Around election time every year, we see ads on television claiming that one candidate is superior to another in terms of experience, capability, and/or ethics. Sometimes opponents are named and even vilified in these _____ ads. A. differential B. cooperative C. advocacy D. comparative E. image Answer: D Rationale: Comparative advertising compares two or more specifically named or shown competing brands on one or more specific attributes. 24. Which of the following statements about comparative advertising is true? A. Comparative advertising is highly effective in Arabic countries. B. The FCC is the only federal agency that has any regulatory power over comparative advertising. C. Comparative advertising is often used for products experiencing strong growth. D. Comparative advertising is regulated by the FTC in the United States. E. Comparative advertising is illegal in the United States. Answer: D Rationale: Federal rulings prohibit advertisers from falsely describing competitors’ products and allow competitors to sue if ads show their products or mention their brand names in an incorrect or false manner. FTC rules also apply to advertisers making false claims about their own products. 25. A series of related advertisements focusing on a common theme, slogan, and set of advertising appeals is referred to as a(n): A. advertising campaign. B. message strategy. C. continuity program. D. flight. E. creative program. Answer: A Rationale: This is the definition of an advertising campaign. 26. An advertising campaign: A. may contain a wide variety of themes and slogans. B. extends for a defined period of time. C. continues for the life of the product. D. starts with determining which media will be used. E. rarely has a specific budget. Answer: B Rationale: The advertising campaign is a series of related advertisements that focus on a common theme, slogan, and set of advertising appeals and extends for a defined period of time. 27. A(n) _____ is a specific communication task that a campaign should accomplish for a specified target audience during a specified period. A. advertising objective B. marketing mix C. media schedule D. advertising life span E. promotional mix Answer: A Rationale: This is the definition of an advertising objective. 28. Before any creative work can begin on an advertising campaign, it is important to: A. determine what goals or objectives the advertising should achieve. B. set the marketing budget. C. decide what executional style to use in the ads. D. see if sales promotion is more important. E. hire a spokesperson. Answer: A Rationale: The first step in any advertising campaign is to set advertising objectives. 29. One method used for setting advertising objectives is: A. campaign budgeting. B. objective and task. C. the DAGMAR approach. D. percentage of sales. E. contribution margin. Answer: C Rationale: According to the DAGMAR approach, all advertising objectives should precisely define the target audience, the desired percentage change in some specified measure of effectiveness, and the time frame in which that change is to occur. 30. DAGMAR is an acronym for: A. Defining and Achieving Goals Means Advertising Reach. B. Developing Aggressive Markets. C. Designing Advertising with Market Research. D. Defining Advertising Goals for Measured Advertising Results. E. Developing Aggressive Growth Markets and Revenues. Answer: D Rationale: The DAGMAR approach (Defining Advertising Goals for Measured Advertising Results) is a method of setting objectives. 31. A feature of a product is called a(n): A. asset B. appeal C. attribute D. element E. benefit Answer: C Rationale: Attributes are features, while benefits are what consumers will receive or achieve by using the product. 32. Ads for Yoplait Greek Yogurt highlight that the product has twice the protein and three times the calcium of other yogurts. This advertising highlights: A. product attributes. B. the creation of brand loyalty. C. how negative consumers’ attitudes are modified. D. product benefits. E. loyalty rewards. Answer: A Rationale: The calcium and protein content are considered product attributes. 33. Which of the following is what consumers will receive or achieve by using a product? A. Attribute B. Benefit C. Compensation D. Market share E. Functionality Answer: B Rationale: Benefits might be such things as convenience, pleasure, savings, or relief. 34. The UPS Store’s advertising “What Brown Can Do for You” whiteboard campaign tells business consumers that when it comes to getting their products delivered somewhere, UPS can help. The advertising is focusing on: A. creating advertising appeal. B. production implementation. C. a product mission. D. creating product loyalty. E. a product benefit. Answer: E Rationale: A benefit is what customers will receive or achieve by using the product or service. 35. A newspaper ad for General Mills cereal highlights its Box Top Redemption program in which General Mills donates cash to schools when the schools redeem box tops from General Mills cereal. This ad focuses on: A. a product attribute. B. production implementation. C. a product mission. D. creating product loyalty. E. a product benefit. Answer: E Rationale: The ad answers the customer’s question, “What’s in it for me?” 36. In advertising, the goal is to sell the _____, not the _____ of a product. A. benefits; attributes B. characteristics; appeals C. values; traits D. competitive edge; virtues E. differential advantage; properties Answer: A Rationale: A well-known rule of thumb in advertising is “Sell the sizzle, not the steak.” 37. An advertising _____ identifies a reason for a person to buy a product. A. profile B. execution C. format D. execution E. appeal Answer: E Rationale: This is the definition of an advertising appeal. 38. All of the following are common types of advertising appeals EXCEPT: A. profit. B. health. C. admiration. D. fear. E. commitment. Answer: E Rationale: See Exhibit 16.1. Commitment is not typically considered a common type of advertising appeal. The other options—profit, health, admiration, and fear—are commonly used in advertising to appeal to different consumer motivations or emotions. 39. A print ad for Kraft Capri Sun drink packs says “take a sip of antioxidant protection.” What kind of an advertising appeal is this ad using? A. Convenience B. Fun and pleasure C. Health D. Profit E. Admiration Answer: C Rationale: See Exhibit 16.1. Health appeals to those who want to be healthy. 40. Gucci, a very expensive brand of clothing and accessories, show beautiful women wearing its products in magazine ads. What kind of an advertising appeal is this? A. Environmental consciousness B. Vanity and egotism C. Health D. Profit E. Admiration Answer: B Rationale: See Exhibit 16.1. Showing beautiful women wearing Gucci products in magazine ads appeals to vanity and egotism, as it associates the brand with ideals of beauty, status, and personal attractiveness. 41. Ads for Brita water filters describe how one Brita pitcher filter can reduce waste by effectively replacing as many as 300 standard plastic water bottles. What kind of appeal is this ad using? A. Environmental Consciousness B. Vanity and Egotism C. Health D. Profit E. Admiration Answer: D Rationale: See Exhibit 16.1. This ad is using a profit appeal by emphasizing cost savings and economic benefits (reducing waste by replacing plastic water bottles with a Brita pitcher filter). 42. An ad for Crystal Geyser bottled water shows why and how the bottler supports the reforestation of America’s forests. What kind of an advertising appeal is this ad using? A. Environmental Consciousness B. Concern C. Health D. Profit E. Admiration Answer: A Rationale: See Exhibit 16.1. The ad for Crystal Geyser bottled water supporting reforestation of America's forests appeals to environmental consciousness, highlighting the brand's commitment to environmental sustainability and conservation efforts. 43. Allstate Insurance has mostly ditched its famous “in good hands” ads in favor of a character called “Mayhem,” who shows up in your life at random — and trashes it. What kind of advertising appeal were these ads using? A. Fear B. Consequences C. Health D. Vanity and egotism E. Consciousness Answer: A Rationale: See Exhibit 16.1. Allstate Insurance's "Mayhem" ads use a fear appeal by depicting unexpected and chaotic situations that could occur without insurance, emphasizing the need for protection against potential risks. 44. Many advertisements use celebrities who are liked by many people in hopes that consumers will buy the product that the celebrity endorses. Using celebrities to influence consumers to buy the product is an example of which advertising appeal? A. Consciousness B. Love or romance C. Fun and pleasure D. Admiration E. Resonance Answer: D Rationale: See Exhibit 16.1. Using celebrities to influence consumers to buy the product is an example of an admiration appeal, leveraging the positive feelings and admiration consumers have for celebrities to enhance the perceived value or desirability of the product. 45. A(n) _____ is a desirable, exclusive, and believable advertising appeal selected as the theme for a campaign. A. executional framework B. creative message strategy C. benefit D. attribute E. unique selling proposition Answer: E Rationale: The advertising appeal selected for the campaign becomes what advertisers call its unique selling promotion, and it usually becomes the campaign slogan. 46. IBM runs television ads that show how their information technology services can help a small Internet start-up prepare for and solve any problems that may arise in its daily operations. This is an example of a: A. unique selling proposition. B. advertising objective. C. media profile. D. basis for comparative advertising. E. basis for product identification. Answer: A Rationale: A unique selling proposition is a desirable, exclusive, and believable advertising appeal selected as a theme for an advertising campaign. 47. M&M’s famous slogan, “Melts in your mouth, not in your hand,” is an example of a(n): A. unique selling proposition. B. advertising objective. C. basis for lifestyle strategy. D. promotional mix. E. basis for comparative advertising. Answer: A Rationale: A unique selling proposition is a desirable, exclusive, and believable advertising appeal selected as a theme for an advertising campaign. 48. Ads for M&M candy that prominently feature the M&M candies in real-life situations are using a _____ style. A. slice-of-life B. lifestyle C. animated product symbol D. demonstration E. scientific Answer: C Rationale: The real/animated product symbol executional style creates a character that represents the product in advertisements. 49. _____ is the way the advertisement portrays its information. A. Message execution B. Appeal formation C. Focus selection D. Audience selection E. Benefit determination Answer: A Rationale: The style in which the message is executed is one of the most creative elements of an advertisement. 50. Which of the following statements is NOT true about post-campaign evaluation? A. Marketers spend little time studying advertising effectiveness because there are so many variables that shape the effectiveness of an ad. B. Testing ad effectiveness can be done either before or after the campaign. C. Most advertising campaigns aim to create an image for the product instead of asking for action, so their real effect is unknown. D. Even if an ad campaign has been particularly successful, advertisers will typically conduct a post-campaign evaluation. E. Evaluating an advertising campaign can be the most demanding task facing advertisers. Answer: A Rationale: Even though there are so many variables that shape the effectiveness of an ad, marketers spend a considerable amount of time studying advertising effectiveness and its probable impact on sales, market share, or awareness. 51. The _____ is the channel used to convey a message to a target market. A. medium B. product network C. attribute D. appeal E. executor Answer: A Rationale: This is the definition of a medium, which is a major decision for advertisers. 52. _____ is the series of decisions advertisers make regarding the selection and use of media, allowing the marketer to optimally and cost-effectively communicate the message to the target audience. A. Promotional implementation B. Marketing mix planning C. Media planning D. Media reach and frequency strategy E. Media targeted selection Answer: C Rationale: This is the definition of media planning. 53. Television is an example of an advertising: A. medium. B. network. C. attribute. D. appeal. E. execution. Answer: A Rationale: A medium is the channel used to convey a message to a target market. 54. Which of the following statements about the selection of media for advertising is true? A. Creative planning for an ad that goes on a billboard is the same as the planning for an ad that runs on television. B. The media selection is typically unrelated to the advertising objectives. C. The appeal and executional style of the ad often affect the media selection. D. Creative work is finished before the media are selected. E. Media selection is the first step in developing an advertising campaign. Answer: C Rationale: In many cases, the advertising objectives dictate the medium and the creative approach to be used. 55. Outdoor advertising is: A. inflexible and high-cost. B. inflexible and low-cost. C. flexible and high-cost. D. flexible and low-cost. E. not an actual medium. Answer: D Rationale: Outdoor (or out-of-home) advertising is a flexible, low-cost medium that may take a variety of forms. Outdoor advertising reaches a broad and diverse market and is therefore ideal for promoting convenience products and services as well as directing consumers to local businesses. 56. _____ advertising has the advantages of being both timely and geographically flexible. A. Television B. Newspaper C. Outdoor D. Magazine E. Catalog Answer: B Rationale: Newspapers are often used by local retailers because they are current and flexible and directed to a local but wide market. 57. _____ advertising is an arrangement in which the manufacturer and the retailer split the costs of advertising the manufacturer’s brand. A. Advocacy B. Comparative C. Institutional D. One-to-one E. Cooperative Answer: E Rationale: This is the definition of cooperative advertising. 58. An ad for the new Magnifique perfume by Lancome shows that it can be purchased at Macy’s. This ad is most likely an example of _____ advertising. A. comparative B. institutional C. cooperative D. advocacy E. strategic Answer: C Rationale: Cooperative advertising is an arrangement in which the manufacturer and the retailer split the costs of advertising the manufacturer’s brand. 59. Advertising in the form of a 30-minute advertisement that resembles a television talk show is called a(n): A. Infomercial. B. extended sales pitch. C. mega-mercial. D. ad expander. E. prolonged advertisement. Answer: A Rationale: Infomercials are an attractive advertising vehicle for many marketers because of the relatively inexpensive airtime and the lower production costs. 60. The many uses of OxiClean products are shown in a program-length television commercial. This is a(n) _____ form of advertising. A. mega-mercial B. ad expander C. infomercial D. prolonged ad E. extended sales pitch Answer: C Rationale: Advertising in the form of a 30-minute advertisement that resembles a television talk show is called an infomercial. 61. Placing advertising messages in Web-based or video games to advertise or promote a product, service, organization, or issue is known as: A. embedded message. B. subliminal advertising. C. advergaming. D. blogging. E. social networking. Answer: C Rationale: This is the definition of advergaming. Sometimes the entire game amounts to a virtual commercial; other times advertisers sponsor games or buy ad space for product placement. 62. Arkadium creates games adults and kids can play on the Internet. While people play these games, advertising creatively appears in or around the game. Their clients include General Motors, NBC, Warner Bros., and more. Arkadium is creating _____ to reach target markets for its clients. A. an infomercial B. a videomercial C. a relationship tool D. advergaming E. an interconnect Answer: D Rationale: Advergaming is the placing of advertising messages in Web-based or video games to advertise or promote a product. 63. The U.S. Army created a video game called America’s Army to boost its recruiting efforts. The game is an example of: A. an interconnect. B. a videomercial. C. advergaming. D. a relationship tool. E. an infomercial. Answer: C Rationale: Advergaming is the placing of advertising messages in Web-based or video games to advertise or promote a product––in this case, the U.S. Army. 64. Ads on shopping carts at grocery stores and in screen savers are examples of: A. communications networks. B. alternative media. C. distribution channels for ads. D. outdoor advertising. E. direct advertising. Answer: B Rationale: Alternative media are those other than the traditional mediums of television, newspaper, outdoor, radio, and magazines and include things like shopping carts in grocery stores, computer screen savers, DVDs, CDs, interactive kiosks in department stores, and so on. 65. When an advertiser decides how much space and/or time will be placed in each advertising medium the company wants to use, the advertiser is determining its: A. marketing mix. B. media mix. C. promotional plan. D. advertising campaign. E. reach objectives. Answer: B Rationale: The media mix is the combination of media to be used for a promotional campaign. 66. All of the following are used for making media mix decisions EXCEPT: A. media schedule. B. cost per contact. C. reach. D. frequency. E. audience selectivity. Answer: A Rationale: The media schedule will be the result AFTER the media mix decisions are made, based on the other four factors. 67. _____ is the cost of reaching one member of the target market. A. Cost per contact B. Cost per thousand C. Gross rating D. GRP E. Effective reach Answer: A Rationale: This is the definition of cost per contact, and advertisers use this to compare media vehicles, such as television versus radio or magazine versus newspaper. 68. _____ measures the number of target consumers exposed to a commercial at least once during a specific period, usually four weeks. A. Noise level B. Frequency C. Reach D. Synchronization E. Cost per contact Answer: C Rationale: This is the definition of reach. 69. _____ is a measure of the number of times an individual is exposed to a brand message. It is used to measure the intensity of a specific medium’s coverage. A. Synchronization B. Reach C. Cost per contact D. Media scheduling E. Frequency Answer: E Rationale: Advertisers use average frequency to measure the intensity of a specific medium’s coverage. 70. The ability of an advertising medium to reach a precisely defined market is called: A. audience selectivity B. market singularity C. geographic selectivity D. noise filtering ability E. life span Answer: A Rationale: This is the definition of audience selectivity. 71. Media vehicles like People magazine and The Chicago Tribune appeal to a wide cross section of the population. In contrast, Self magazine and The Food Network have high levels of: A. longevity. B. audience selectivity. C. geographic selectivity. D. flexibility. E. market singularity. Answer: B Rationale: Some media vehicles, like general newspapers and network television, appeal to a wide cross section of the population. Others appeal to very specific groups. 72. Prices on items sold by supermarkets change frequently, and they often offer special sales to match or beat the prices of their competitors. Therefore, _____ would be a major consideration when a supermarket selects an advertising vehicle. A. audience duplication B. noise level C. life span D. cost per contact E. flexibility Answer: E Rationale: Flexibility of the medium refers to the ability to change the message to fit changing market conditions. 73. Billboard advertisements often have to compete for a driver’s attention with traffic, passengers, radio, or other billboards. Billboard advertising would be described as: A. lacking any geographic selectivity. B. the most flexible advertising medium. C. having a high noise level. D. having a low noise level. E. being a great channel for informative ads. Answer: C Rationale: Noise level is the amount of distractions––other advertisements, people, competing sounds in the environment, or news stories––involved with a type of medium. Outdoor advertising such as billboards suffers from a great deal of competing noise. 74. Advertisers are often concerned with the noise level in a medium. The medium that is assumed to have the lowest noise level is: A. radio. B. television. C. direct mail. D. newspaper. E. magazines. Answer: C Rationale: Direct mail is a private medium with a low noise level. 75. Because Samuel Cabot, Inc., a manufacturer of premium-grade wood stains, wants to spend its promotional budget on timely advertisements that target a large out-of-home audience and have low unit and production costs, it should use _____ advertising. A. newspaper B. radio C. television D. magazine E. creative Answer: B Rationale: Radio has several strengths as an advertising medium: selectivity and audience segmentation, a large out-of-home audience, low unit and production costs, timeliness, and geographic flexibility. 76. In the 1960s, there were only three television networks and most consumers only received these three channels plus a local channel. Now, consumers have more than 1,600 channels to choose from. This is an example of which problem facing media planners? A. Media fragmentation B. Flighted media C. Effective frequency D. Effective reach E. Media consolidation Answer: A Rationale: Media planners have hundreds more media options today than they had when network television reigned. 77. A(n) _____ designates the medium or media to be used, the specific vehicles, and the insertion dates of the advertising. A. advertising plan B. media schedule C. promotion strategy D. ad outline E. promotional profile Answer: B Rationale: This is the definition of a media schedule. 78. Which type of advertising is used to stimulate demand for a new product or product category? A. Comparative B. Innovative C. Focused D. Image E. Pioneering Answer: E Rationale: This is the definition of pioneering advertising, which is heavily utilized during the introductory stage of the product life cycle. 79. Meek’s Hunting Emporium is a retail store that carries all the products hunters need or think they need. It has set up an advertising plan that allows it to advertise every day prior to the opening of quail-, deer-, and turkey-hunting seasons on four local radio stations, as well as in the local newspapers. This is an example of a: A. message execution plan. B. media profile. C. reach program. D. media schedule. E. frequency timetable. Answer: D Rationale: The media schedule designates when and where advertising will appear. 80. Products in the latter stages of the product life cycle, which are advertised on a reminder basis, use a(n) _____ media schedule. A. intermittent B. bursting C. continuous D. trailing E. unremitting Answer: C Rationale: A continuous schedule allows the advertising to run steadily throughout the advertising period, which is good for advertising on a reminder basis. 81. Pepsi-Cola uses reminder advertising that runs steadily all year. This is an example of a _____ media schedule. A. seasonal B. pulsing C. flighted D. consecutive E. continuous Answer: E Rationale: A continuous media schedule allows advertising to run steadily throughout the advertising period. 82. The Swine Palace Theater runs ads in the entertainment section of the local newspaper every first weekend of each month. This is a _____ scheduling plan. A. flighted B. continuous C. bursting D. seasonal E. replication Answer: A Rationale: Flighted media scheduling schedules ads heavily for a period, then drops them, and then repeats them. 83. A(n) _____ media schedule combines continuous scheduling throughout the year with a flighted schedule during the best sales periods. A. pulsing B. bursting C. unremitting D. rhythmic E. vibrating Answer: A Rationale: A retail department might use a pulsing media schedule by advertising on a year-round basis but place more advertising during certain sale periods such as holidays. 84. The Tween Scene is a retail store catering to 8- to 12-year-old boys and girls. It advertises regularly both on radio and in the daily newspaper. The month prior to the beginning of each new school year, the store buys extra advertising space and time. This is an example of a(n) _____ scheduling plan. A. continuous B. replication C. interval D. seasonal E. pulsing Answer: E Rationale: A pulsing media schedule combines continuous scheduling with an extra “pulse” during heavy sale periods. 85. Which of the following products is most likely to use a pulsing media schedule? A. Romance novels B. Gasoline C. Band instruments D. Baking soda E. Ceiling fans Answer: C Rationale: Band instruments would be most heavily advertised just prior to the opening of the new school year. 86. Which of the following products is most likely to have a seasonal media schedule? A. Aromatherapy candles B. Masking tape C. Airplane tickets D. Charcoal E. Soft drinks Answer: D Rationale: Seasonal products like charcoal are not advertised year-round because they are not used year-round. 87. The main premise of _____ planning is that advertising works by influencing the brand choice of people who are ready to buy. A. stealth B. guerrilla C. pulsing D. recency E. seasonal Answer: D Rationale: Recency planning is commonly used for scheduling television advertising for frequently purchased products such as Coca-Cola or Tide detergent. 88. Every day around lunchtime, Rascal’s Deli sends text messages advertising daily specials to customers on their GPS-enabled mobile phones. This is an example of: A. recency planning. B. guerrilla marketing. C. pioneering advertising. D. executional style. E. a seasonal media schedule. Answer: A Rationale: Recency planning’s main premise is that advertising works by influencing the brand choice of people who are ready to buy—such as urging hungry consumers to stop into Rascal’s Deli around lunchtime. 89. Products such as snowblowers, chocolate Easter eggs, mittens, and sunscreen lotion are most likely to be advertised with a _____ media schedule. A. flighted B. pulsing C. seasonal D. continuous E. consecutive Answer: C Rationale: Products that are used more during certain times of the year tend to follow a seasonal strategy. 90. _____ is the element in the promotional mix that evaluates public attitudes, identifies issues that may elicit public concern, and executes programs to gain public understanding and acceptance. A. Personal selling B. Advertising C. Mass communications D. Public relations E. Sales promotion Answer: D Rationale: This is a description of public relations. 91. A news story on the resignation of Steve Jobs as CEO of Apple—complete with descriptions of the numerous innovations he pioneered during his tenure—is an example of: A. a sales promotion. B. free advertising. C. demarketing. D. newspaper advertising. E. publicity. Answer: E Rationale: Publicity can be either positive or negative. 92. Public information about a company, good, or service appearing in the mass media as a news item is: A. personal selling B. advertising C. mass communications D. publicity E. sales promotion Answer: D Rationale: Publicity can be either positive or negative. 93. Tools for the public relations manager include all of the following EXCEPT: A. press relations. B. product publicity. C. lobbying. D. sales promotions. E. corporate communication. Answer: D Rationale: Sales promotions are not part of public relations. 94. An article in BusinessWeek about the reengineered Honda CRV is an example of: A. a sales promotion. B. free advertising. C. direct marketing communications. D. newspaper advertising. E. publicity. Answer: E Rationale: Publicity is the effort to capture media attention. 95. Publicity: A. will never damage a company because it performs the information task of promotion. B. is free communication. C. is not persuasive with customers. D. has to be purchased from the mass media. E. has many internal costs to the company associated with it. Answer: E Rationale: Preparing news releases, staging special events, and persuading media personnel to broadcast or print publicity messages costs money. 96. A Wall Street Journal headline is “Best Buy Wins iPhone Lineup.” This article represents _____ for Best Buy and Apple, Inc. A. marketing research B. sales promotion C. personal selling D. advertising E. publicity Answer: E Rationale: Publicity can help advertisers explain what’s different about their new product by prompting free news stories or positive word of mouth about it. 97. An article in the Wall Street Journal about GM’s turnaround strategy and new upcoming product, the Volt, would be an example of: A. direct selling B. news marketing C. publicity D. event marketing E. direct marketing Answer: C Rationale: Publicity can help advertisers explain what’s different about their new product by prompting free news stories or positive word of mouth about it. 98. Which of the following is a public relations strategy that involves getting a product, service, or company name to appear in a movie or television show? A. Press release B. Product placement C. Product publicity D. Consumer education E. Integrated marketing communications Answer: B Rationale: This is product placement, which also involves getting a product to appear in other venues such as radio programs, magazines, newspapers, and video games, among others. 99. Stormbreaker, a new spy movie, details the story of Alex Rider who, after the death of his uncle, is recruited as an M16 spy. Armed with a special set of gadgets, Rider’s mission is to save millions of lives. In James Bond fashion, the Cross pen serves as one of the key gadgets in the film. The maker of the Cross pen paid to have its pen featured in this movie. This is an example of: A. advertising. B. a sales promotion. C. paid-for public relations. D. product placement. E. lobbying. Answer: D Rationale: Product placement is a public relations tool and is used to create product exposure. 100. BMW of North America sponsored an instructional driving school for teenagers in major cities across the U.S. Teens received a training session that included driving techniques and accident avoidance skills.This is an example of: A. event advocacy B. lobbying C. product placement D. consumer education E. issue sponsorship Answer: D Rationale: Consumer education is a type of public relations in which the user benefits by educating the consumer about the usefulness of its products. 101. Which public relations tool involves a company spending money to support an issue, cause, or event that is consistent with corporate objectives? A. Advocacy B. Lobbying C. Product placement D. Sponsorship E. Underwriting Answer: D Rationale: Corporate objectives for sponsorship include improving brand awareness or enhancing corporate image. 102. The biggest category of sponsorships involves _____, which accounts for almost 68 percent of spending in sponsorships. A. education B. arts and festivals C. political causes D. sports E. music Answer: D Rationale: Sponsorship is a public relations tool that involves a company spending money to support an issue, cause, or event that is consistent with corporate objectives. The biggest category is sports. 103. How do public relations professionals use Internet Web sites? A. To introduce new products B. To promote existing products C. To obtain consumer feedback D. To showcase upcoming events E. To do all of these things Answer: E Rationale: Companies increasingly are using the Internet in their public relations, and it is used for all of these reasons. 104. _____ is used by public relations specialists to handle the effects of unfavorable publicity. A. Media planning B. Crisis management C. Message power D. Damage control E. Communication narrowcasting Answer: B Rationale: Crisis management is the coordinated effort to handle the effects of unfavorable publicity or of another unexpected unfavorable event. 105. _____ is marketing communication activities in which a short-term incentive is offered to induce the purchase of a particular good or service. A. Publicity B. Sales promotion C. Promotion D. Advertising E. Motivation selling Answer: B Rationale: Sales promotion includes marketing activities––other than personal selling, advertising, and public relations––that stimulate consumer buying and dealer effectiveness. 106. Neutrogena offered customers who purchased $30 worth of facial moisturizers, facial cleaners, and facial treatment products a $10 rebate by mail. This is an example of: A. motivational selling. B. a trade promotion. C. a push strategy. D. personal selling. E. a sales promotion. Answer: E Rationale: Sales promotion is an activity in which a short-term incentive encourages the purchase of a particular good or service. 107. What are the two types of sales promotion? A. Consumer and trade B. Informative and reminder C. Start-up and continual D. Adoptive and diffusive E. Personal and impersonal Answer: A Rationale: Sales promotion is usually targeted toward two distinctly different markets: consumers and the trade. 108. All of the following would be a target for a trade sales promotion offered by Rubbermaid, a manufacturer of storage and organization products, EXCEPT: A. a retailer called The Container Store. B. a kitchen department in a large discount store. C. a wholesaler of plastic storage bins. D. a Rubbermaid distributor. E. a consumer who needs to organize her college dorm room. Answer: E Rationale: Trade promotions are directed to members of the marketing channel, such as wholesalers and retailers. 109. About what percentage of consumers uses coupons or discount promotional codes when making purchases online? A. 0 percent B. 25 percent C. 50 percent D. 75 percent E. 100 percent Answer: C Rationale: Nearly 50 percent of consumers who make purchases online use coupons or discount promotional codes. 110. Martina buys cola every week at the grocery store, but she always buys whatever is on sale. Coca-Cola, Pepsi, RC Cola—it doesn’t matter to her. She is happy with whatever costs the least. Martina would be characterized as a: A. loyal customer. B. competitor’s customer. C. price buyer. D. cherry picker. E. freeloader. Answer: C Rationale: Price buyers consistently buy the least expensive brand. See Exhibit 16.4 111. When Rick purchased a Xerox color printer for his law office, he was able to mail in a proof-of-purchase and his cash register receipt to receive a check from Xerox for $200. Rick received a: A. premium. B. trade allowance. C. purchase allowance. D. rebate. E. functional discount. Answer: D Rationale: A rebate is a cash refund given for the purchase of a product during a specific period. 112. A(n) _____ is an extra item offered to the consumer, usually in exchange for some proof that the promoted product has been purchased. A. coupon B. trade sample C. supplement D. premium E. add-on Answer: D Rationale: Probably the best example of a premium is the toy inside a McDonald’s Happy Meal. 113. New subscribers to Men’s Health magazine receive a free copy of Men’s Health Total Fitness Guide. The Fitness Guide is an example of a(n): A. contest. B. premium. C. trade sample. D. product placement. E. loyalty incentive. Answer: B Rationale: A premium is an extra item offered to the consumer, usually in exchange for some proof of purchase of the promoted product. 114. Which type of consumer sales promotion rewards loyal consumers for making multiple purchases of a particular good or service? A. A frequent buyer program B. An 80/20 program C. A net worth program D. A premium program E. A continuity program Answer: A Rationale: These are also called loyalty marketing programs. 115. Jack’s Pet Store offers customers who purchase pet food a “Baker’s Dozen” card. After customers purchase 12 bags of pet food from Jack’s, they get the 13th bag free. The Baker’s Dozen card is an example of a: A. directed coupon mailing. B. sweepstakes. C. loyalty marketing program. D. trade sales promotion. E. sampling program. Answer: C Rationale: Loyalty marketing or frequent buyer programs induce brand loyalty by rewarding customers for multiple purchases. 116. Co-branded credit cards are used in conjunction with: A. self-perpetuating premiums. B. loyalty marketing programs. C. trade consumer promotions. D. consumer discount programs. E. functional consumer discounts. Answer: B Rationale: Co-branded credit cards are an increasingly popular loyalty marketing tool. 117. The Pillsbury Bake-Off requires people to submit a recipe using Pillsbury baking products. Finalists are brought to Pillsbury kitchens to bake their recipes so the winner of a cash prize can be chosen. This is an example of a: A. contest. B. sales promotion game. C. sweepstakes. D. push money deal. E. P-O-P (participant of Pillsbury). Answer: A Rationale: Contests are promotions in which participants use some skill or ability to compete for prizes. 118. Which of the following statements about contests and sweepstakes is true? A. Contests depend on luck, and participation is free. B. Contests are generally effective tools for creating long-term relationships with customers. C. Sweepstakes are promotions that require participants to exhibit some skill. D. Sweepstakes usually draw about ten times more participants than contests. E. Contests and sweepstakes are not useful for creating interest and publicity. Answer: D Rationale: Contests require participants to have some skill. Sweepstakes are based on chance. Neither contests nor sweepstakes can be used if the marketer’s objective is to create a long-term relationship with its customers, though they may generate short-term interest. 119. Club Penguin is an online role-playing game for kids aged 6 to 14 operated by the Walt Disney Company. Players pay a monthly fee to gain access to all of Club Penguin’s feature, although free memberships with limited access to games is also available. Disney offers free membership to Club Penguin in the hope that kids who like the game will ask their parents to pay for the full version. Disney is using which sales promotion tool? A. Sponsorship B. Sampling C. Trade promotions D. In-pack couponing E. Premiums Answer: B Rationale: Sampling allows the consumer to try a product risk-free. 120. _____ are promotional exhibits set up at the retailer’s location to build traffic, advertise the product, or induce impulse buying. They are targeted to consumers. A. Point-of-purchase displays B. Freestanding inserts C. Indirect demonstrations D. Direct demonstrations E. Freestanding kiosks Answer: A Rationale: Point-of-purchase promotion includes any promotional display set up at the retailer’s location to perform these functions. 121. Point-of-purchase promotions work best for: A. high-involvement products. B. purchases that require extensive decision making. C. complex products that require technical knowledge to operate. D. impulse buys. E. expensive products like perfume and jewelry. Answer: D Rationale: Point-of-purchase promotions are most effective for impulse items. 122. Which of the following statements about online sales promotions is true? A. Internet sales promotions are more effective and cost-efficient at generating responses than their off-line counterparts. B. One of the few types of sales promotion that cannot be used online is sampling. C. Online coupons have lower redemption rates than off-line coupons. D. Loyalty marketing programs are ineffective when used at Web sites to encourage traffic. E. There is no way for a marketer to build long-term, mutually beneficial relationships with customers online. Answer: A Rationale: Customer loyalty programs are very effective on Internet sites to build customer loyalty. Sampling is a sales promotion option that can be effectively used by Internet businesses. Online coupons have a higher redemption rate than off-line coupons. 123. Lopez is the largest Hispanic-owned meat processor in the United States. To assist retailers in marketing its products, Lopez provides point-of-sale materials and offers special introductory discounts to retailers. Lopez uses: A. functional marketing. B. relationship selling. C. consumer promotions. D. startup marketing. E. trade promotions. Answer: E Rationale: Trade promotions push a product through the distribution channel, which includes retailers and wholesalers. 124. All of the following are examples of trade sales promotions EXCEPT: A. push money B. store demonstrations C. premiums and coupons D. free merchandise E. trade shows Answer: C Rationale: Premiums and coupons are directed at consumers. 125. General Mills is offering resellers a 10 percent discount on all cases of its granola bars purchased during the month of June. General Mills is offering a: A. trade allowance. B. life cycle extension promotion. C. P-O-P discount. D. push money deal. E. product development deal. Answer: A Rationale: A trade allowance is a price reduction offered by manufacturers to intermediaries, such as wholesalers or retailers, in exchange for performance of specified functions or purchasing during special periods. 126. General Motors is offering the sales force at Meers Furniture Emporium a $100 reward for each Chevy Volt sold in the next 30 days. What type of trade sales promotion is the $100? A. A trade allowance B. Push money C. A selling deal D. Premium payment E. A direct commission Answer: B Rationale: Intermediaries receive push money as a bonus for pushing the manufacturer’s brand. 127. The use of trade shows: A. is not an important aspect of sales promotion. B. can help companies identify new prospects. C. does not help introduce new products. D. does not enhance corporate morale. E. is a disadvantage in that it is usually impossible to display the product itself. Answer: B Rationale: Companies participate in trade shows to attract and identify new prospects, serve current customers, introduce new products, enhance corporate image, test the market response to new products, enhance corporate morale, and gather competitive product information. LeBlanc’s Pecan Company Betty and Orville LeBlanc recently opened LeBlanc’s Pecan Company retail store. In addition to shelled and unshelled pecans, the Texas store sells pecan pies, tortes, fruitcakes, fudge, pralines, chocolate pecan bark, homemade pecan ice cream toppings, and jams and jellies. At this time, there are no other stores of this type in the Houston area. The LeBlancs are considering developing an advertising campaign for their store, focusing on its wide assortment of pecan-related products. Orville LeBlanc has drawn up a list of communication tasks the advertising must accomplish. He used the DAGMAR approach to draw up this list. 128. Refer to LeBlanc’s Pecan Company. The new campaign for the store is designed to stimulate demand for the LeBlanc’s new store, given that the store is in the introductory stage of the product life cycle. The campaign would be considered: A. pioneering. B. comparative. C. institutional. D. exclusive. E. open-ended. Answer: A Rationale: The campaign is pioneering because it is designed to stimulate demand for a new store, given that the store is in the introductory stage of the product life cycle. 129. Refer to LeBlanc’s Pecan Company. When Orville LeBlanc created the list of communication tasks the advertising must accomplish, he was setting his: A. marketing mix. B. media schedule. C. advertising objectives. D. advertising life span. E. media profile. Answer: C Rationale: An advertising objective is a specific communication task that should be accomplished for a specified target market. 130. Refer to LeBlanc’s Pecan Company. One of their advertisements featured Mrs. LeBlanc showing how several of the products sold in the shop can be used when entertaining guests. This is an example of which executional style for advertising? A. Mood or image B. Store demonstration C. Scientific D. Fantasy E. Convenience Answer: B Rationale: The store demonstration executional style for advertising shows consumers the expected benefit. 131. Refer to LeBlanc’s Pecan Company. When Orville LeBlanc decided which media he would use to optimally and cost-effectively reach the store’s target audience, he was engaged in developing: A. a marketing mix. B. a media plan. C. a promotional plan. D. an advertising campaign. E. reach objectives. Answer: B Rationale: Media planning is the series of decisions advertisers make regarding the selection and use of media, allowing the marketer to optimally and cost-effectively communicate the message to the target audience. 132. Refer to LeBlanc’s Pecan Company. A feature article in a Houston newspaper describing the LeBlanc’s Pecan Company store and what it sells would be an example of: A. newspaper advertising B. event sponsorship C. product placement D. competitive advertising E. publicity Answer: E Rationale: Publicity is the effort to attract media attention. Digital Cookware Digital Cookware has developed a new kind of skillet that should make cooking easier for many non-chefs. The skillet has a display on its handle that beeps to alert the cook when the pan’s target temperature is reached. The skillet, with the aid of an accompanying cookbook, will take much of the guesswork out of cooking. Digital Cookware is a small company, and it has invested considerable money in developing this new technology, which it eventually hopes to use in an entire line of pots and pans. It has run some small ads for the new skillet in cooking magazines and has been featured on some morning talk shows. It needs a quick infusion of cash to maintain its current operational level and to develop the rest of the cookware line. Digital Cookware is considering some kind of sales promotion. 133. Refer to Digital Cookware. Why might rebates be a really good sales promotion for Digital Cookware to use? A. They are useful in building long-term relationships with customers. B. They can create a great amount of publicity. C. They create mutually beneficial relationships between the manufacturer and its customers. D. They are good at enticing purchases, but most rebates are not redeemed. E. They can be piggybacked with similar products. Answer: D Rationale: Building long-term relationships describes a loyalty marketing program. Sweepstakes and contests generate publicity. Piggybacked is a term used with sampling. 134. Refer to Digital Cookware. If consumers receive a free spatula with the purchase of Digital Cookware, this is an example of which type of consumer sales promotion? A. A sample B. A premium C. A rebate D. A bonus pack E. An add-on Answer: B Rationale: A premium is an extra item offered to the consumer. 135. Refer to Digital Cookware. The company’s first task is to define its exact sales promotion objective. If the goal of the company is to persuade customers to buy its brand of skillet as opposed to one of the skillets made by its competition, it should: A. use on-pack coupons. B. run a cooking contest that has contestants enter recipes that use the new skillet. C. use price-off packages. D. use bonus packs. E. institute a loyalty marketing program. Answer: B Rationale: The objectives of a promotion depend on the general behavior of targeted customers. See Exhibit 16.4. 136. Refer to Digital Cookware. Digital Cookware is offering retailers a price reduction for allocating space for special displays of its products. This type of trade sales promotion is called: A. push money. B. a trade allowance. C. an event sponsorship. D. a store demonstration. E. a trade show. Answer: B Rationale: Trade allowance is a price reduction offered by manufacturers to intermediaries such as wholesalers and retailers. 137. Refer to Digital Cookware. If Digital Cookware offered the sales associates at a chain of cooking stores a bonus for each digital skillet they sold, it would be an example of: A. push money. B. a trade allowance. C. a trade rebate. D. pull money. E. a functional discount. Answer: A Rationale: Push money is money offered to channel intermediaries to encourage them to “push” products. ESSAY 1. Define advertising and explain why new brands with small market share tend to spend proportionately more for advertising and sales promotion than those with a large market share. Answer: Advertising is impersonal, one-way mass communication about a product or organization that is paid for by a marketer. There are two reasons why new brands with smaller market share tend to spend proportionately more on advertising that those with a larger market share: (1) beyond a certain level of spending for advertising and sales promotion, diminishing returns set in––that is, sales or market share begins to decrease no matter how much is spent on advertising and sales promotion, a phenomenon called the advertising response function; and (2) a certain minimum level of exposure is needed to measurably affect purchase habits. 2. Name and briefly define the two major types of advertising. Answer: INSTITUTIONAL ADVERTISING is used when the goal of the campaign is to build up the image of the company. Institutional advertising promotes the corporation as a whole and is designed to establish, change, or maintain the corporation’s identity. Advocacy advertising is a special form of institutional advertising that allows corporations to express viewpoints on controversial issues. PRODUCT ADVERTISING touts the benefits of a specific product or service. Product advertising can take three forms: pioneer advertising, competitive advertising, and comparative advertising. 3. Name and briefly define three forms of product advertising. Answer: Product advertising touts the benefits of a specific product or service. Product advertising can take three forms: pioneer advertising, competitive advertising, or comparative advertising. PIONEERING ADVERTISING is intended to stimulate primary demand for a new product or product category. It is used during the introductory stage of the product life cycle to offer information about the benefits of the product class. COMPETITIVE ADVERTISING is used to influence demand for a specific brand. This advertising emphasizes the building of brand name recall and favorable brand attitudes. Firms use this type of advertising when a product enters the growth phase of the product life cycle. COMPARATIVE ADVERTISING directly or indirectly compares two or more competing brands on one or more specific attributes. 4. What is the most significant recent trend that has affected television advertising? Describe the trend and how it impacts television networks, and explain how television networks have been dealing with it. Answer: The most significant trend to affect television advertising in recent years is probably the rise in popularity of digital video recorders such as TiVo. Viewers weary of television advertisements have been using ad-skipping DVR technology to avoid interruptions during their favorite shows. If consumers are not watching advertisements, then marketers will spend a greater proportion of their advertising budgets on alternative media, and a major revenue stream for networks will disappear. Some networks have created small icons in the bottom of the screen to promote their own products. Some have also run shorter blocks of advertising to minimize the number of viewers who skip over the ads. TiVo has also offered interactive banner ads to advertisers, making those sponsors’ names visible as their ads are being fast-forwarded. 5. There are several criteria used in selecting media in an advertising campaign. Name and briefly discuss the impact of five of these criteria used to select media. Answer: COST PER CONTACT is the cost of reaching one member of the target market. Lower cost per contact may be appealing to advertisers, but total cost should be considered as well. COST PER CLICK is the cost associated with a consumer clicking on a display or banner ad. REACH is the number of target consumers who are exposed to a commercial at least once during a specific period, usually four weeks. FREQUENCY is the number of times an individual is exposed to a given message during a specific period. TARGET AUDIENCE CONSIDERATIONS. A medium’s ability to reach a precisely defined market is its audience selectivity. Some media vehicles appeal to a wide cross section of the population. Others appeal to very specific groups. FLEXIBILITY is how adaptable the medium is to changes. Magazines, for example, have poor flexibility as they require ad copy months before publication. Radio allows an ad to be changed in one day. NOISE LEVEL is the level of distraction to the target audience in a medium. Billboards may have high noise level, due to driver distraction, whereas direct mail has low noise level. LIFE SPAN is how long the advertising message lasts. A radio or television commercial lasts only for the duration the commercial is aired, whereas a magazine ad may viewed several times by several people. 6. Advertisers must evaluate several qualitative factors involved in media selection. Name at least three of them. Answer: These qualitative factors include such things as (1) attention to the commercial and the program, (2) involvement, (3) lack of distractions, (4) how well the view likes the program, and (5) other audience behaviors that affect the likelihood a commercial message is being seen and absorbed. 7. List and briefly describe the four basic types of media scheduling. Give an example of a product that would be likely to use each of these types of scheduling. Answer: 1. CONTINUOUS MEDIA SCHEDULE. This schedule allows the advertising to run throughout the advertising period. Examples: milk, computers, mobile telephones, soda, bar soap, and detergent. 2. FLIGHTED MEDIA SCHEDULE. This schedule means the advertiser schedules ads heavily every other month or every two weeks to achieve a greater impact with an increased frequency and reach at those times. Examples: new movies, department store sales. 3. PULSING MEDIA SCHEDULE. This schedule combines continuous scheduling with flighting. Continuous advertising is simply heavier during the best sale periods. Examples: sales periods such as Thanksgiving, Christmas, and back-to-school. 4. SEASONAL MEDIA SCHEDULE. This schedule is for products that are used more during certain times of the year. Examples: cough syrup, sunscreen, wood-burning stoves, wood for wood-burning stoves, and holiday decorations. 8. What is public relations? What is publicity? Answer: Public relations is the element in the promotional mix that evaluates public attitudes, identifies issues that may elicit public concern, and executes programs to gain public understanding and acceptance. Public relations is a vital link in a progressive company’s marketing communications mix. The outcome of public relations is publicity––the effort to capture media attention. Public relations and publicity may serve to boost new product introduction and to position or reposition mature products. The two terms are often confused, but publicity is simply one of several public relations tools. 9. List and briefly describe five of the many functions commonly performed by public relations departments. Answer: PRESS RELATIONS. Placing newsworthy information in the news media to attract attention to a product, a service, or a person associated with the firm or institution. PRODUCT PUBLICITY. Publicizing specific products or services. CORPORATE COMMUNICATION. Creating internal and external messages to promote a positive image of the firm or institution. PUBLIC AFFAIRS. Building and maintaining national or local community relations. LOBBYING. Influencing legislators and government officials to promote or defeat legislation and regulation. EMPLOYEE AND INVESTOR RELATIONS. Maintaining positive relationships with employees, shareholders, and others in the financial community. CRISIS MANAGEMENT. Responding to unfavorable publicity or a negative event. 10. Describe some specific ways that companies are using the Internet in their public relations strategies. Answer: Company Web sites are used to introduce new products, provide information to the media including social media news releases, promote existing products, obtain consumer feedback, communicate legislative and regulatory information, showcase upcoming events, provide links to related sites (including corporate and non-corporate blogs, Facebook, and Twitter), release financial information, interact with customers and potential customers, and perform many more marketing activities. In addition, social media is playing a larger role in how companies interact with customers online, particularly through other sites like Facebook, Yelp or Twitter. 11. What is sales promotion? What are the objectives of sales promotion? Answer: Sales promotion is marketing communication activities, other than advertising, personal selling, and public relations, in which a short-term incentive motivates consumers or members of the distribution channel to purchase a good or service immediately, either by lowering the price or by adding value. Sales promotion offers consumers incentives to buy via coupons, premiums, contests, and free samples. Immediate purchase is usually the goal of sales promotion. The objectives depend on the general behavior of the target consumers. Marketers targeting loyal users of their product want to reinforce existing behavior, whereas other promotions may aim to convince consumers to switch brands or try a new product. 12. Oakley Grove and Abraham Enterprises both market oranges and grapefruits. Oakley Grove has been in business for almost 40 years, while Abraham Enterprises has only been selling citrus fruit for about 18 months. Abraham Enterprises would like to take business away from Oakley Grove, and Oakley Grove wants to retain all of its customers. Explain which sales promotions each should emphasize in its marketing mix. Answer: Oakley Grove wants to maintain its current customers. To achieve this objective, it can create a loyalty marketing program. Customers could earn points for each pound purchased, and after acquiring a certain number of points, customers could redeem them for a free gift box. Also, bonus packs would be a good sales promotion to use and would give customers an incentive to stock up. Another possibility is to have customers collect proofs of purchases off of the Oakley Grove packages and redeem those for some premium such as a juicer. Abraham Enterprises would need to use different sales promotions because its goal is to persuade customers to switch from Oakley Grove to its brand of fruit. One method it could try is sampling, especially if it believes that its product is superior to that of Oakley Grove. It could also use a sweepstakes or a contest (such as who makes the best dessert using oranges) to gain publicity for its product and to draw attention to the products it produces. Abraham Enterprises could also use a premium offer, but it would need to be something unique (like the bounty off of one orange tree for a year) so that the premium would make a strong impression on the customers the company wants to gain from its competitors. 13. Consumer sales promotion seeks to reach new users, keep current users, and entice users to stock up. It can also be used to reinforce advertising for the good or service. Name and define five consumer sales promotional tools. Answer: COUPONS are certificates given to consumers entitling them to an immediate price reduction when they purchase the item. REBATES are similar to coupons in that a rebate offers the purchaser a price reduction; however, because the purchaser must mail in a rebate form and usually some proof of purchase, the reward is not as immediate. PREMIUMS are extra items offered to the consumer, usually in exchange for some proof that the promoted product has been purchased. LOYALTY MARKETING PROGRAMS or FREQUENT BUYER PROGRAMS reward loyal consumers for making multiple purchases of a particular good or service. CONTESTS are promotions in which participants compete for prizes based on some skill or ability. SWEEPSTAKES allow anyone to participate and are characterized by chance drawings for prizes. SAMPLING allows consumers to try products risk-free. Trial sizes reduce the risk of trying new products. They also eliminate the problem of being stuck with a large quantity of a disliked product. POINT-OF-PURCHASE PROMOTIONS are special displays set up at retail locations to build traffic, advertise the product, or induce impulse buying. ONLINE SALES PROMOTIONS are a way to boost Web site traffic by giving away free services and/or equipment. Also, these promotions include sites that distribute coupons online as a way to counter the high cost and low redemption rate of coupons in print media. Some Internet sites are also using loyalty programs—users are rewarded with points, which can be redeemed later. Sampling is also possible at some Web sites. 14. You are the sales promotion manager for Vita-Balls, a new children’s vitamin that is designed to be chewed like gum. Your company uses a pull strategy, and you are responsible for recommending sales promotion tools to accomplish this strategy. Describe three specific sales promotion activities you would recommend for Vita-Balls. Answer: Because Vita-Balls uses a pull strategy, consumer sales promotion tools should be used. COUPONS for discounts on Vita-Balls could be mailed directly to parents or could be placed in parenting magazines. PREMIUMS could be offered to purchasers of Vita-Balls, with proof of purchase required. The premium should be related to the product, such as books on common parenting problems, a subscription to a nutrition magazine, or other children’s healthcare products. LOYALTY MARKETING PROGRAMS or FREQUENT BUYER PROGRAMS could also be offered such as offering a free container of Vita-Balls after 12 containers are purchased. CONTESTS could be offered to parents. Entrants could write an ESSAY about Vita-Balls or send in a photograph of their child, with the winning picture appearing on the new package of Vita-Balls. A SWEEPSTAKES could allow anyone to enter a random drawing for a free year’s supply of Vita-Balls. SAMPLING would allow parents to try the product without risk. Free small sizes could be mailed to parents. POINT-OF-PURCHASE PROMOTIONS could be set up at drugstores or grocery stores to attract new customers and encourage current customers to try Vita-Balls. ONLINE SALES PROMOTIONS. The manufacturer of Vita-Balls could set up a Web site where it provided online coupons for the vitamin, which could be redeemed at retail stores, as well as hints on how to get children to eat more nutritiously. It could award points for visiting this site and have the points redeemable for Vita-Balls. 15. Compare and contrast contests and sweepstakes. Answer: Contests and sweepstakes are both types of sales promotions generally designed to create interest in a good or service, often to encourage brand switching. Contests are promotions in which participants use some skill or ability to compete for prizes. A consumer contest usually requires entrants to answer questions, complete sentences, or write a paragraph about the product and submit proof of purchase. Winning a sweepstakes, on the other hand, depends on chance or luck and participation is free. Sweepstakes usually draw about ten times more entries than contests do. Chapter 17—Personal Selling and Sales Management 1. Personal selling becomes a more important promotional tool as product value and complexity and increase. Answer: True Rationale: Generally speaking, personal selling becomes more important as the number of potential customers decreases, as the complexity of the product increases, and as the value of the product grows. See Exhibit 17.1. 2. Dave is in charge of finding sponsors for the 2013 Riverbend Festival. He is meeting with marketing representatives from Wrangler jeans and First Tennessee National Bank, attempting to convince them to sponsor the event. Dave is engaged in personal selling. Answer: True Rationale: Personal selling involves direct interactions between a salesperson and potential buyers to make a sale. Dave's meetings with marketing representatives to convince them to sponsor the event exemplify personal selling. 3. With relationship selling, the objective is to build long-term branded relationships with customers, not just to sell products. Answer: True Rationale: Relationship selling emphasizes building enduring, branded relationships with customers, prioritizing long-term customer satisfaction and loyalty over short-term transactional sales goals. 4. Relationship selling is more typical with selling situations for consumer goods. Answer: False Rationale: Relationship selling is more typical with selling situations for industrial-type goods. 5. With relationship selling, the salesperson would spend most of his or her contact time with the prospect talking about the product because the salesperson does not want to waste the customer’s time. Answer: False Rationale: With relationship selling, the salesperson spends most of his or her time attempting to build a problem-solving environment with the customer. See Exhibit 17.2. 6. Customer relationship management requires that organizations develop relationships with their customers through touch points and data mining. Answer: True Rationale: Customer Relationship Management (CRM) involves using touch points and data mining to understand and serve customers better, enhancing relationships and loyalty. 7. To initiate the CRM cycle, a company must first identify customer relationships within the organization. Answer: True Rationale: Identifying existing customer relationships is crucial for understanding and managing customer interactions effectively, which is the first step in the CRM cycle. 8. Customer-centric is an internal management philosophy similar to the marketing concept. Answer: True Rationale: Both customer-centric philosophy and the marketing concept focus on satisfying customer needs and building long-term relationships for organizational success. 9. Empowerment in a CRM environment is normally a process of collecting customer information through customers’ feedback on products. Answer: False Rationale: This is learning. Empowerment is the delegation of authority to solve customers’ problems quickly. 10. The interaction between a customer and the organization is the foundation on which a CRM system is built. Answer: True Rationale: CRM systems rely on customer interactions to collect data, which is then used to improve customer satisfaction and loyalty. 11. Consumers are reluctant to enter information through point-of-sale interactions because of privacy violation fears. Answer: False Rationale: Many point-of-sale software programs enable customers to easily provide information about themselves without feeling their privacy has been violated. 12. Campaign management is a process by which customer information is centralized and shared in order to enhance the relationship between customers and the organization. Answer: False Rationale: Knowledge management is a process by which customer information is centralized and shared in order to enhance the relationship between customers and the organization. 13. Volvo has a Web site that caters to car enthusiasts who might want to see concept cars evolve into real-life products. Volvo would likely have used data mining to locate people who were car enthusiasts and innovators. Answer: True Rationale: Data mining helps identify specific customer segments, such as car enthusiasts and innovators, enabling Volvo to tailor its marketing and engage these customers effectively. 14. Internet companies use product and customer profiling to reveal cross-selling opportunities while a customer is surfing their sites. Answer: True Rationale: Product and customer profiling helps internet companies identify and suggest additional products that may interest customers, increasing sales through cross-selling. 15. The first step in the selling process is approaching the customer and probing needs. Answer: False Rationale: The first step is generating leads. 16. Tara is a salesperson with several satisfied customers. She asks her customers to recommend her to their business associates. Tara is using cold calling to generate leads. Answer: False Rationale: This is a referral. 17. To qualify a sales lead, a salesperson must identify the person in an organization who has the authority to buy the product and must close the sale with that individual. Answer: False Rationale: Qualifying a sales lead consists of determining the prospect’s need, buying power, and receptivity and accessibility. 18. Before approaching the customer, a salesperson should learn as much as possible about the prospect’s organization and its buyer. Answer: True Rationale: Understanding the prospect's organization and its buyer helps the salesperson tailor their approach and offer solutions that meet the prospect’s specific needs. 19. If a potential customer has objections after the salesperson has made a presentation, that is an indication that the salesperson has done a poor job. Answer: False Rationale: Rarely does a prospect agree to make a purchase right after a presentation. Objections are common and expected in the selling process. 20. Jayson typically negotiates the price with prospective customers because he knows price is the most effective negotiating tool when closing a sale. Answer: False Rationale: Effective negotiators avoid using price as a sales tool. 21. The final selling duty for most successful salespeople is closing the sale. Answer: False Rationale: A salesperson’s responsibilities do not end with a sale; a follow-up is critical to encouraging repeat business. 22. In relationship selling, salespeople focus more time on generating leads than on qualifying leads. Answer: False Rationale: See Exhibit 17.5. In relationship selling, salespeople typically focus more time on qualifying leads rather than just generating them, as the goal is to prioritize quality interactions and build meaningful relationships with potential customers. 23. Meta Singh is expected to sell two surgical laser systems each month. This sales objective is an example of a quota. Answer: True Rationale: A sales quota is a specific sales target set for a salesperson to achieve within a given period, such as selling two surgical laser systems each month. 24. The final stage of sales management is evaluating the effectiveness and performance of the sales force. Answer: True Rationale: Evaluating the effectiveness and performance of the sales force ensures that sales goals are being met and identifies areas for improvement and further training. 25. Sales managers should look for sociability, creativity, and ego strength in applicants. Answer: True Rationale: These traits are important for sales success, as they help salespeople build relationships, think innovatively, and remain confident in challenging situations. 26. Compensation planning is one of the sales manager’s easiest jobs. Answer: False Rationale: This is one of the toughest jobs. MULTIPLE CHOICE 1. When compared to other forms of promotion, personal selling: A. is more important as the number of potential customers increases. B. is more effective in selling frequently purchased products. C. can use a message customized to the immediate needs of the customers. D. is best for selling simple, low-involvement products. E. is less expensive. Answer: C Rationale: Generally speaking, personal selling becomes more important as the number of potential customers decreases, as the complexity of the product increases, and as the value of the product grows. The sales message can be varied according to the motivations and interests of each prospective customer. 2. Macy’s, Nordstrom, Saks Fifth Avenue, and Barneys New York offer free personal shopping, whereby consultants pull clothing they feel will fit the customer’s style and specified need. This free service: A. encourages customers to develop a relationship with the store. B. puts pressure on customers to buy more. C. dissuades customers from shopping at the store. D. is a good way to sell low-cost, low-involvement products. E. Streamlines customer interactions, making sales easier. Answer: A Rationale: This free service encourages customers to continue using personal shoppers and develop a relationship with the store. 3. Personal selling is more important than advertising and sales promotion if: A. the products being sold are standardized. B. there are many customers for the product being sold. C. the product being sold has a low value. D. the buyers of the product are extremely dispersed. E. the products being sold are technically complex. Answer: E Rationale: Personal selling becomes more important as the complexity of the product increases. See Exhibit 17.1. 4. How can personal selling costs be controlled? A. By increasing the time that the sales force spends on personal selling. B. By adjusting the size of the sales force en masse. C. By offering the sales force benefits such as stock options. D. By dropping and replacing the entire sales force. E. By adjusting the size of the sales force in one-person increments. Answer: E Rationale: Personal selling costs can be controlled by adjusting the size of the sales force (and resulting expenses) in one-person increments 5. For which of the following products would its producer be more likely to choose personal selling rather than advertising or sales promotion to market it? A. Garden hose B. HD television C. Dry-cleaning D. Coffee mug E. Breakfast cereal Answer: B Rationale: HD televisions are complex, buyers purchase them only infrequently, and the product has a high value. See Exhibit 17.1. 6. Generally speaking, personal selling becomes more important as the number of potential customers _____ and as the value of the product _____. A. increases; grows B. increases; shrinks C. decreases; shrinks D. decreases; grows E. None of the above. Answer: D Rationale: Personal selling becomes more important as the number of potential customers decreases, as the complexity of the product increases, and as the value of the product grows. See Exhibit 17.1. 7. As a manufacturer of jams and jellies, Smucker’s sells a highly standardized product to consumers all over the United States. You would expect Smucker’s to rely on _____ to promote its product. A. public relations B. personal selling C. advertising and sales promotion D. publicity and direct marketing E. product innovation Answer: C Rationale: With a standard product of low value and widely dispersed customers, the producer should rely on advertising. See Exhibit 17.1. 8. What role does technology play in personal selling? A. Technology used to play a large role in personal selling, but its impact is diminishing. B. Technology is the most important aspect of personal selling. C. Technology plays an important role, especially in regards to social media. D. Technology is completely irrelevant to personal selling. E. Technology plays a small role in personal selling, but experts expect its importance to rise over time. Answer: D Rationale: Technology now plays an important role in personal selling through the use of social media like LinkedIn and Facebook, as well as the use of blogs and Twitter to establish expertise within a field. 9. Boeing is one of the largest aircraft manufacturers in the world. The company sells aircrafts to airlines around the world, such as Delta, China Air, and AirFrance. You would expect Boeing to rely on _____ to promote its vessels. A. publicity B. advertising and sales promotion C. sales promotion D. personal selling E. product innovations and direct marketing Answer: D Rationale: Boeing is selling a complex, high-value product to a few customers. See Exhibit 17.1. 10. Which of the following professions likely requires an individual to make sales? A. Salesperson B. Chemist C. Marketing manager D. Engineer E. All of the above. Answer: E Rationale: An individual may become a plant manager, a chemist, an engineer, or a member of any profession and yet still have to sell. During a job search, applicants must “sell” themselves to prospective employers in an interview. 11. Jefferson Smurfit Company is a multi-billion-dollar supplier of packaging materials. One of its salespeople rearranged production schedules at three different plants to satisfy an unexpected demand for boxes from General Electric. The salesperson’s action is typical of the company’s sales philosophy and indicates an emphasis on: A. consultative promotion. B. relationship selling. C. adaptive selling. D. transformational selling. E. transactional buying. Answer: B Rationale: Relationship selling, or consultative selling, emphasizes personalization and empathy as key ingredients in identifying prospects and developing them as long-term, satisfied customers. 12. When did marketers move away from making one-time sales and toward relationship selling? A. Marketers have always focused on relationship selling. B. During the Industrial Revolution. C. In the wake of the Great Depression. D. Just after World War II. E. Very recently. Answer: D Rationale: Until recently, marketing theory and practice concerning personal selling focused almost entirely on a planned presentation to prospective customers for the sole purpose of making the sale. 13. _____ is a sales practice that involves building, maintaining, and enhancing interactions with customers in order to develop long-term satisfaction through mutually beneficial partnerships. A. Networking B. Adaptive selling C. Stimulus-response selling D. Relationship selling E. Needs-dependent selling Answer: D Rationale: Relationship selling is a multistage process that emphasizes personalization and empathy as key ingredients in identifying prospects and developing them as long-term satisfied customers. 14. Which of the following is not true of traditional personal selling methods? A. They often attempt to persuade the buyer to accept a point of view. B. The objectives of the salesperson are frequently at the expense of the buyer. C. They have disappeared in the wake of relationship selling. D. They entail making a one-time sale and then moving on to the next prospect. E. They sometimes create a win–lose outcome. Answer: C Rationale: Although traditional sales approaches have not disappeared entirely, they are being used less and less often by professional salespeople. 15. Relationship selling is also called: A. long-term selling. B. win–win selling. C. consultative selling. D. experiential selling. E. interactive selling. Answer: C Rationale: Relationship selling is also called consultative selling. 16. Relationship selling is a _____ process that emphasizes _____ as a key ingredient in identifying prospects and developing them as long-term, satisfied customers. A. multistage, empathy B. popular, assertiveness C. short-term, persuasion D. win-lose, personalization E. modern, standardization Answer: A Rationale: Relationship selling is a multistage process that emphasizes personalization and empathy as key ingredients in identifying prospects and developing them as long-term, satisfied customers. 17. Relationship selling: A. is also called adaptive selling. B. is most concerned with making a sale. C. is more typically used when selling low-involvement products in the consumer market. D. is more concerned with developing customer trust than with making a sale. E. is declining in popularity among businesses because the costs are becoming prohibitive. Answer: D Rationale: Relationship selling is also called consultative selling. It is more concerned with developing customer trust than with making a sale. Typically, industrial-type goods are sold using relationship selling. 18. The knowledge that most businesses depend on repeat sales is the basis of: A. traditional personal selling. B. direct marketing. C. all trade promotions. D. relationship selling. E. product-oriented selling. Answer: D Rationale: The goal of relationship selling is to develop long-term satisfied customers. 19. Research has shown that positive customer–salesperson relationships result in: A. win–lose situations. B. decreased customer trust. C. customers who purchase from the company just once. D. higher costs for the company. E. None of the above. Answer: E Rationale: Positive customer–salesperson relationships contribute to trust, increased customer loyalty, and the intent to continue the relationship with the salesperson. 20. All of the following statements characterize the traditional personal selling approach EXCEPT: A. Traditional selling focuses on closing sales. B. Traditional selling uses short-term follow-ups that focus on product delivery. C. Traditional personal selling takes a team approach to the account. D. Salespeople sell products, not advice and assistance. E. Proposals and presentations used emphasize pricing and product features. Answer: C Rationale: Traditional personal selling takes a “lone wolf” approach to the account, whereas relationship or consultative selling takes a team approach. See Exhibit 17.2. 21. Stores carrying Revlon Super Lustrous lipstick have begun using touch-screen televisions that allow customers to see how different styles might look on their own faces without talking to a makeup artist. Which of the following statements does this new development support? A. Salespeople sell products, not advice and assistance. B. Personal selling has taken a technology turn in the last decade. C. Relationship selling promotes a win–win situation for both buyer and seller. D. Personal selling offers several advantages over other forms of promotion. E. Modern views of personal selling emphasize the relationship between a salesperson and a buyer. Answer: B Rationale: Stores that embrace younger shoppers’ love of technology and independence can gain loyal customers. 22. Consultative selling: A. uses limited sales planning. B. uses a “lone wolf” approach to selling. C. sells advice, assistance, and counsel rather than products. D. uses short-term follow-ups, which focus on product delivery. E. emphasizes closing the sale during the sales presentation. Answer: C Rationale: See Exhibit 17.2. Consultative selling involves selling advice, assistance, and counsel rather than just products, focusing on understanding and addressing the customer's specific needs and providing tailored solutions. 23. _____ is the ultimate goal of a new trend in marketing that focuses on understanding customers as individuals instead of as part of a group. A. Organizational optimization B. Consumer relationship marketing (CRM) C. Total quality management (TQM) D. Customer relationship management (CRM) E. Market aggregation Answer: D Rationale: This is the definition of customer relationship management (CRM). 24. Using technology provided by Smart Button software, the WNBA’s Los Angeles Sparks have implemented a new system that gathers information about its best customers, season ticket holders, and rewards those customers in many ways, including free tickets to Sparks home games and cash prizes. This is an example of a(n) _____ system. A. organizational optimization B. corporate relationship marketing (CRM) C. total quality management (TQM) D. customer relationship management (CRM) E. call response marketing (CRM) Answer: D Rationale: CRM is a company-wide business strategy designed to optimize profitability, revenue, and customer satisfaction by focusing on highly defined and precise customer segments. 25. Customer relationship management initially was popularized as _____. A. one-to-one marketing. B. a mass marketing approach. C. total quality management. D. a customer satisfaction program. E. a differentiation strategy. Answer: A Rationale: Customer relationship management initially was popularized as one-to-one marketing. But CRM is a much broader approach to understanding and serving customer needs than is one-to-one marketing. 26. _____ in a CRM environment involves collecting customer information through comments and feedback on product and service performance. A. Uncovering B. Researching C. Learning D. Recording E. Absorbing Answer: C Rationale: This is the definition of learning. 27. To initiate the CRM cycle, a company must first: A. establish marketing objectives. B. capture relevant customer data on interactions. C. identify customer relationships with the organization. D. understand the interactions the company has with current customers. E. decide on a segmentation strategy. Answer: C Rationale: To initiate the CRM cycle, a company must first identify customer relationships with the organization. 28. When Burpee Gardening company collects relevant information from customers, such as date of last communication with the customer and how often the customer contacts the company, this is an example of: A. establishing marketing objectives. B. capturing relevant customer data on interactions. C. identifying customer relationships with the organization. D. understanding the interactions the company has with current customers. E. deciding on a segmentation strategy. Answer: B Rationale: This is an example of a company that uses a CRM approach to capture relevant customer data on interactions. 29. Which of the following statements about a CRM system is true? A. A critical component of a CRM system is the use of the appropriate technology to store and manage customer data. B. A CRM system operates on the theory that all customers are equally important. C. Due to privacy laws, a CRM system only disseminates customer information to those who are actually in day-to-day contact with customers. D. A company using a CRM system must view its customers as bits of data. E. All of these statements about a CRM system are true. Answer: A Rationale: The large volume of data resulting from a CRM initiative can be managed effectively only through technology. Once customer data are collected, the question of who owns those data becomes extremely salient. 30. Best Buy instituted a system in all of its stores that customizes their product offerings for the five key customer segments it has identified: affluent professional males, young entertainment enthusiasts, upscale suburban moms, families who are practical technology adopters, and small businesses with fewer than 20 employees. This focus implies that Best Buy is: A. ethnocentric. B. demand based. C. sales-centric. D. supply based. E. customer-centric. Answer: E Rationale: A customer-centric focus is an internal management philosophy to the marketing concept. 31. Sony wants to build long-lasting relationships by focusing on what satisfies and retains their customers. For example, Sony’s PlayStation Web site is designed to create a community of users who can join PlayStation Underground where they will “feel like they belong to a subculture of intense gamers.” Sony evidently: A. is sales oriented. B. has a standardization focus. C. is ethnocentric. D. has a customer-centric focus. E. is product oriented. Answer: D Rationale: A customer-centric focus is an internal management philosophy similar to the marketing concept. 32. _____ involves delegating authority to solve customers’ problems. A. Consumer learning B. Customerization C. Empowerment D. Autonomy E. Interaction Answer: C Rationale: Empowerment is the delegation of authority to solve customers’ problems quickly––usually by the first person the customer notifies regarding the problem. 33. _____ is the process by which learned information from customers is centralized and shared in order to enhance the relationship between customers and the organization. A. Knowledge management B. Learning C. Database marketing D. Interactive marketing E. Information marketing Answer: A Rationale: This is the definition of knowledge management. 34. When Sony PlayStation users want to access amenities on the Sony Web site, they are required to log in and supply information such as their name, e-mail address, and birth date. They are also given the opportunity to complete a survey that captures much more information about them and their gaming habits. Sony gathers this information and makes it available internally to better serve the customer. This is an example of a company using: A. learned research. B. stimulus/response marketing. C. knowledge management. D. sales-oriented marketing. E. motivational research. Answer: C Rationale: Knowledge management is the process by which learned information from customers is centralized and shared in order to enhance the relationship between customers and the organization. 35. Zappos is an online shoe and apparel retailer that promotes the highest quality of customer service by maintaining the importance of each _____, the point at which customer and store personnel exchange information and develop learning relationships. A. moment of truth B. intervention C. data capture D. response situation E. interaction Answer: E Rationale: An interaction is the point at which a customer and a company representative exchange information and develop learning relationships. 36. Apple, Inc. has stores, a Web site, and a toll-free phone number where consumers can provide valuable information to the company in developing a CRM system. All these possible areas of the company where consumers can communicate with the business are called: A. touch points. B. focus areas. C. data mining. D. information search periods. E. experimental points. Answer: A Rationale: In a CRM system, touch points are all areas of the business where customers have contact with the company and data may be gathered and used to guide and direct the decision making within that business unit. 37. Les Ailes de la Mode, a Quebec retail store that promotes the highest quality of customer service, has not only established itself as a top retailer in the province but also has a credit card and point program company, a cataloger and Internet seller, and even a publisher. Through gathering customer information for its credit cards and point program, through catalog and Internet orders, and through subscriptions to its publications, Les Ailes de la Mode has created several different: A. focus areas. B. touch points. C. interrelationship promotions. D. distribution channels. E. sources of distribution information. Answer: B Rationale: Touch points are all areas of the business where customers have contact with the company and data may be gathered and used to guide and direct the decision making within that business unit. 38. Communications between customers and organizations that occur in stores or at information kiosks are called: A. live interactions. B. point-of-sale interactions. C. empowerment points. D. survey interactions. E. product interactions. Answer: B Rationale: Point-of-sale interactions occur in stores or at information kiosks. 39. Which of the following would be an example of “social” CRM? A. Placing a phone call to a local department store B. Leaving a comment on a company’s blog C. Mailing a survey card back to an organization D. Telling a friend about your experience with an online retailer E. All of the above are examples of “social” CRM Answer: B Rationale: Social CRM incorporates social media with the best of traditional CRM. 40. Which of the following is an example of a channel through which customer data are traditionally gathered? A. Store visits B. Conversations with salespeople C. Interactions via the Web D. Phone conversations E. All of these Answer: E Rationale: Channel interactions include store visits, conversations with salespeople, interactions via the Web, traditional phone conversations, and wireless communications. 41. A(n) _____ allows marketers to track customers’ relationships to the company’s products and services. A. database B. data profile C. algorithm D. byte E. interaction profile Answer: A Rationale: Using transaction and purchase data, a database allows marketers to track customers’ relationships to the company’s products and services and modify the marketing message accordingly. 42. Walmart captures point-of-sale transactions from thousands of stores in several countries and continuously transmits these data to its massive data warehouse. Walmart allows more than 3,500 suppliers to access data on their products and perform data analyses. These suppliers use the data to identify customer buying patterns at the store level. They use this information to manage local store inventory and identify new merchandising opportunities. Both Walmart and its suppliers are engaged in: A. information formatting. B. database enhancement. C. data mining. D. data shading. E. descriptive modeling. Answer: C Rationale: Data mining is a data analysis approach that identifies patterns of characteristics that relate to particular customers or customer groups. 43. Outdoor gear retailer Recreational Equipment, Inc. (REI) in Washington collects a vast amount of data through its Web site, direct mailings, and retail stores. When REI considers new store locations, it examines order data to find places with high concentrations of customers buying online and through the company’s catalogs. REI would be using _____ to identify potential store locations. A. information formatting B. database enhancement C. data mining D. data shading E. customer prediction Answer: C Rationale: Data mining is a data analysis approach that identifies patterns of characteristics that relate to particular customers or customer groups. 44. _____ involves developing product or service offerings customized for the appropriate customer segment and then pricing and communicating these offerings for the purpose of enhancing customer relationships. A. Transaction management B. Campaign management C. Data mining D. Consumerism E. Knowledge management Answer: B Rationale: This is the definition of campaign management. 45. Which of the following is a common CRM marketing database application? A. Campaign management B. Retaining loyal customers C. Cross selling other products and services D. Designing targeted marketing communications E. All of these Answer: E Rationale: Reinforcing customer purchase decisions, inducing product trial by new customers, and increasing effectiveness of distribution channel marketing are other common CRM marketing database applications. See Exhibit 17.4. 46. To target outdoor enthusiasts, Subaru of America added an outdoor life section to its Web site that is organized by area of interest. The site will serve as a platform for co-marketing and promotional programs. This offering of customized products to a customer segment for the purpose of enhancing the auto manufacturer’s customer relationships is an example of: A. campaign management. B. customer aggregation. C. transaction segmentation. D. data mining. E. knowledge management. Answer: A Rationale: Developing customized offerings for the appropriate customer segment is a common tactic in campaign management. 47. Many high-end hotel chains award points for every dollar spent in one of their hotels. Customers who earn a high number of points are given special privileges that may include upgraded hotel rooms or several free nights. This is an example of the leveraging of customer information to: A. retain loyal customers. B. reinforce competitive promotional decisions. C. cross-sell other products and services. D. design targeted marketing communications. E. induce product trial by new customers. Answer: A Rationale: Loyalty programs reward loyal customers for making multiple purchases with the objective of building long-term relationships. 48. When a company retains an additional 5 percent of its customers each year, profits will increase by as much as _____. A. 5 percent B. 15 percent C. 25 percent D. 35 percent E. 45 percent Answer: C Rationale: If a company has identified its best customers, then it should make every effort to maintain and increase their loyalty. When a company retains an additional 5 percent of its customers each year, profits will increase by as much as 25 percent. 49. Every time Barry orders running shoes from Road Runner Sports, the Web site or the operator on the phone always tries to sell him something in addition to the shoes he ordered, such as socks or other apparel. This is an example of: A. data mining. B. cross-selling. C. trading up. D. database enhancement. E. a database channel. Answer: B Rationale: Past purchases on a particular Web site and the site a surfer comes from give online marketers clues about the surfer’s interests and what items to cross-sell. 50. According to the CEO of Allied Office Products, “We’re a head-count business: I know that if you have a 60-person office, you should buy $300 worth of basic office supplies—paper, pens, staples—from us with each order, but if that’s all we get, we stagnate. For us to grow, we have to convince the customer, who already likes our products and service, to buy more than just basic supplies; we have to increase the order by 10, 20, or 30 times.” Allied’s salespeople are trained to push the company’s less traditional, higher-margin lines such as coffee and refreshments, printing and forms management, and office furniture. Allied’s salespeople are engaging in: A. cross-selling. B. trading up. C. buyer empowerment. D. alliance building. E. bundling. Answer: A Rationale: Cross-selling uses past customer buying information to ask for additional sales. 51. After Ruth and Mike had paid in advance for their week’s stay at a Vermont Inn, Ruth felt that they might have chosen poorly and that they were committing themselves to stay at a place they might hate. Then Ruth got a letter from the Inn’s owner stating that she was looking forward to their visit and asking what they most enjoyed for breakfast. The hospitality of the letter dealt with Ruth’s cognitive dissonance by: A. cross-selling. B. disintermediation. C. spamming and flaming. D. implementing a penetration strategy. E. reinforcing her purchase decision. Answer: E Rationale: A database offers marketers an excellent opportunity to reach out to customers to reinforce the purchase decision. 52. The set of steps a salesperson goes through to sell a particular product is called the: A. P-O-S cycle. B. stimulus-response hierarchy. C. sales presentation. D. sales process. E. sales continuum. Answer: D Rationale: The sales process is also called the sales cycle. 53. _____ provide the fuel that keeps the corporate engines humming. Without them, there would be no business. A. Marketing messages B. Sales C. Communications D. Managers E. JIT processes Answer: B Rationale: There is an old adage in business that nothing happens until a sale is made. Without sales there is no need for accountants, production workers, or even a company president. Sales provide the fuel that keeps the corporate engines humming. 54. The first step in the selling process is: A. qualifying leads B. approaching the customer and probing needs C. developing and proposing solutions D. making a sales presentation E. generating leads Answer: E Rationale: The steps in the selling process are: (1) generating leads, (2) qualifying leads, (3) approaching the customer and probing needs, (4) developing and proposing solutions, (5) handling objections, (6) closing the sale, and (7) following up. 55. The seven steps of the selling process closely follow the: A. AIDA concept. B. hierarchy of needs. C. traditional communications model. D. process of forecasting and satisfying industry demand. E. JIT process. Answer: A Rationale: The seven steps attempts to move the customer from attention to interest to desire, and finally to action. 56. _____, or prospecting, is the identification of those firms and people most likely to buy the seller’s offerings. A. Lead generation B. Lead qualification C. Customer mining D. Referral calling E. Cold calling Answer: A Rationale: This is the first step in the selling process. 57. What is needed for sales management to be effective? A. Impersonal relationships with customers. B. A success-oriented sales force. C. Relaxed sales objectives. D. Extensive advertising campaigns. E. Answers B and D. Answer: B Rationale: Effective sales management stems from a success-oriented sales force that accomplishes its mission economically and efficiently. 58. AutoFry is the leading manufacturer of ventless deep fryers for supermarket deli and food-service operations. It has developed a new high-capacity fryer. The company has decided to purchase a mailing list of thousands of food-service managers and to send out brochures with a detachable card that the managers can use to request more information. AutoFry is involved in: A. lead qualification. B. lead generation. C. sales presentation. D. the close procedure. E. moving through the sales continuum. Answer: B Rationale: Lead generation is the identification of those firms and people most likely to buy the seller’s offerings. 59. Joe Hamas sells for Rush Beverages. He is trying to convince retailers to carry his company’s Ginseng Rush, a new all-natural beverage that delivers an energy punch without caffeine. When he gets retailers to agree to stock his product, he asks them for the names of other retail operations that might be interested in carrying it. He is using _____ to get his sales leads. A. networking B. cold calling C. referrals D. direct marketing E. noncompeting sales Answer: C Rationale: A recommendation from a customer or business associate is a referral. 60. When salespeople use price as their go-to tool for negotiation, they may inadvertently: A. decrease the value of the product. B. increase the value of the product. C. render price a nonissue. D. reinforce product advertising. E. None of the above. Answer: A Rationale: Effective negotiators avoid using price as a negotiation tool. Because companies spend millions on advertising and product development to create value, when salespeople give in to price negotiations too quickly, it decreases the value of the product. 61. _____ is a process of finding out about potential clients from friends, business contacts, coworkers, acquaintances, and fellow members in professional or civic organizations. A. People surfing B. Networking C. Co-opting D. Lead intermediation E. Looking for the golden parachute Answer: B Rationale: Networking is using friends, business contacts, coworkers, acquaintances, and fellow members in professional and civic organizations to identify potential clients. 62. Christy recently began selling Arbonne skin-care products on a part-time basis. She began by telling all her friends and relatives about the product. Then she called old friends from her college days and reached out to the friends of her friends. This technique for generating leads is called: A. cannibalizing. B. networking. C. snowballing. D. incremental approach. E. multiplier effect. Answer: B Rationale: Networking uses friends, business contacts, coworkers, and acquaintances as a means of meeting potential clients. 63. _____ involves determining which sales prospects have a recognized need, buying power, and receptivity and accessibility. A. Prospect examination B. Customerization C. Lead reciprocity D. Lead qualification E. Bird-dogging Answer: D Rationale: When a prospect shows interest in learning more about a product, the salesperson has the opportunity to follow up, or qualify, the lead to determine if the prospect has a recognized need and buying power. 64. A _____ is a written document or professional presentation. A _____ is a formal meeting. A. sales proposal; sales performance B. sales performance; sales production C. sales presentation; sales performance D. sales proposal; sales presentation E. sales production; sales proposal Answer: D Rationale: A sales proposal is a written document or professional presentation that outlines how the company’s product or service will meet or exceed the client’s needs. The sales presentation is the formal meeting in which the salesperson has the opportunity to present the sales proposal. 65. Lead qualification involves determining whether a prospect has: A. a recognized need, buying power, and a willingness to see a salesperson. B. any interest in a product. C. a real or imagined need for a product. D. any resources. E. a line membership in the company’s buying center. Answer: A Rationale: Lead qualification involves determining three things about the prospect: a recognized need, buying power, and receptivity and accessibility. 66. Often the task of lead qualification is handled by a telemarketing group or a sales support person who frees the sales representative from the time-consuming task by engaging in: A. prequalification. B. database mining. C. co-opting. D. cold calling. E. Networking. Answer: A Rationale: A telemarketing group or a sales support person often prequalifies the lead for the salesperson. 67. The _____ is a process that describes the homework that must be done by a salesperson before he or she contacts a prospect. A. pre-sales dialogue B. pre-approach C. prospect customerization D. closing preamble E. sale profiling procedure Answer: B Rationale: Before approaching customers, the salesperson should learn as much as possible about the prospect’s organization and its buyers. This process is called the pre-approach. 68. Knowing the industry requires active research by the salesperson. Which of the following is not a way to learn about the industry when conducting a needs assessment? A. Attending industry and trade association meetings B. Reading articles published in industry and trade journals. C. Keeping track of legislation and regulation that affect the industry D. Being aware of product alternatives from competition E. Memorizing a product’s pricing and billing procedures Answer: E Rationale: Memorizing pricing and billing procedures is part of knowing the product or service. Attending industry and trade association meetings, reading articles published in industry and trade journals, keeping track of legislation and regulation that affect the industry, and being aware of product alternatives are all part of knowing the industry. 69. During the pre-approach, the salesperson would: A. ask for referrals. B. learn as much as possible about the prospect’s organization and its buyers. C. create point-of-purchase displays. D. handle pricing objections. E. do all of these things. Answer: B Rationale: The pre-approach is a process that describes the ‘homework” that must be done by a salesperson before he or she contacts a prospect. 70. A salesperson engaged in needs assessment does all of the following EXCEPT: A. researching the industry. B. finding out about the competition. C. knowing everything there is to know about the customer and its needs. D. learning about the product or service. E. handling objections. Answer: E Rationale: Handling objections occurs during the fifth step. 71. _____ means having both the authority to make the purchase decision and the funds to pay for it. A. Purchase management B. Procurement specialization C. Right to acquire D. Buying power E. None of the above. Answer: D Rationale: To avoid wasting time and money, a salesperson needs to identify the purchasing authority and his or her ability to pay before making a presentation. 72. A _____ is a determination of the customer’s specific needs and wants and the range of options the customer has for satisfying them. A. stimulus-response continuum B. needs hierarchy C. needs assessment D. NASIC comparison E. sales probability Answer: C Rationale: This is the definition of a needs assessment. 73. As part of the needs assessment, the consultative salesperson must learn everything there is to know about: A. the product he or she is selling. B. the customers and their needs. C. the competition. D. the industry in which he or she is selling. E. all of the choices. Answer: E Rationale: Using this information, a customer profile is created. 74. Creating a(n) _____ during a sales approach helps salespeople optimize their time and resources. A. sales proposal B. customer profile C. sales presentation D. forecast of needs E. AIDA assessment Answer: B Rationale: This profile is then used to help develop an intelligent analysis of the prospect’s needs in preparation for developing and proposing solutions. 75. Increasingly, sales professionals are using _____ to connect with targeted leads and clients around the world. A. networking sites such as LinkedIn B. online games such as World of Warcraft C. messaging tools such as Google Talk D. delivered memos such as telegrams E. None of these. Answer: A Rationale: Some of LinkedIn’s estimated 30 million users have reported response rates between 50 and 60 percent, versus 3 percent from direct marketing efforts. 76. A _____ is a written document or professional presentation that outlines how a company’s product will meet or exceed the client’s needs. A. customer profile B. needs assessment C. sales proposal D. qualifying document E. forecast of needs Answer: C Rationale: Solutions are typically offered to the client in the form of a sales proposal delivered during a sales presentation. 77. Jel Sert Company makes convenient, durable, and eye-catching packaging. Its salesperson is demonstrating to a manufacturer of sports drinks how Jel Sert’s packaging would serve as a silent salesperson for its products. The salesperson is conducting a _____. A. negotiation B. sales presentation C. follow-up D. sales approach E. closing Answer: B Rationale: The sales presentation often involves a face-to-face presentation of the product or its benefits to a prospect. 78. For a powerful sales presentation, salespeople must do all of the following EXCEPT: A. ask close-ended questions. B. incorporate visual elements that impart valuable information. C. practice. D. use direct eye contact. E. use hand gestures and voice inflections. Answer: A Rationale: Salespeople need to ask open-ended questions. 79. Which of the following statements about handling objections is true? A. A professional salesperson should not anticipate objections. B. A salesperson should view objections as requests for more information. C. Objections should not be used to close the sale. D. A good salesperson dreads having to handle sales objections. E. Objectives can only be handled when they arise, not anticipated beforehand. Answer: B Rationale: A good salesperson should anticipate objections and be prepared to answer them. The way an objection is handled can be used to close a sale. A good salesperson welcomes objections as a legitimate part of the purchase decision. 80. What should the consultative salesperson do when the prospect says, “Are you telling me that your packaging will protect my fragile products better than your competitor’s packaging can?” A. Conduct an unplanned needs assessment. B. Handle the question as an objection. C. Immediately leave the prospect’s office. D. Ask for referrals. E. Modify his or her sales proposal. Answer: B Rationale: The client is objecting to the product. 81. When a salesperson asks for the sale, he or she is: A. closing the sale. B. creating empathy. C. reducing cognitive dissonance. D. creating long-term reciprocal arrangements. E. qualifying the sale. Answer: A Rationale: Closing requires courage and skill. 82. Danita is in charge of new business development for her advertising agency. She recently made a sales presentation to the administration of Northwest Medical System to discuss the creative approaches her agency would use to differentiate Northwest Medical from its competition. Danita ended the presentation by asking, “When would you like our firm to begin working on your new brand awareness campaign?” In which stage of the selling process would such a question most likely be asked? A. Closing the sale B. Follow-up C. Dealing with cognitive dissonance D. Handling rejection E. Qualifying the prospect Answer: A Rationale: Closing the sale means asking for the sale. The request can be placed indirectly, as this one is. 83. _____ is the process during which both the salesperson and the prospect offer special concessions in an attempt to arrive at a sales agreement. A. Conciliation B. Negotiation C. Compromise D. Concession E. Dispensation Answer: B Rationale: Negotiation often plays a key role in the closing of the sale. 84. When a salesperson offers a discount if a prospect places a larger order, he or she is using the process of: A. summary. B. assumption. C. negotiation. D. adaptation. E. follow-up. Answer: C Rationale: Negotiation involves offering special concessions during the closing of the sale. 85. Salespeople ensure that delivery schedules are met, that the good or service performs as promised, and that buyers are trained in the use of the product. All of these functions are part of _____, the final step in the selling process. A. the pre-approach B. closing C. lead qualification D. following up E. the sales presentation Answer: D Rationale: The follow-up is the final step of the selling process and is one of the most important aspects of a salesperson’s job. 86. Which of the following statements about the relative amount of time spent in the selling process by different types of salespeople is true? A. A consultative salesperson would spend a lot of time generating leads. B. A traditional salesperson would spend a lot of time following up the sale. C. A relationship salesperson would spend a lot of time handling objections. D. A relationship salesperson would spend a lot of time qualifying leads. E. A consultative salesperson would spend a lot of time closing the sale. Answer: D Rationale: See Exhibit 17.5. A relationship salesperson would spend a lot of time qualifying leads, as building strong relationships starts with identifying and understanding potential customers who are likely to benefit from the products or services offered. 87. In traditional selling, more time is spent _____ than in relationship selling. A. qualifying leads B. following up C. handling objections D. approaching the customer and probing needs E. designing and proposing solutions Answer: C Rationale: See Exhibit 17.5. In traditional selling, more time is typically spent handling objections compared to relationship selling. Traditional selling often involves a more transactional approach where objections need to be overcome to close the sale. 88. All of the following are responsibilities and decisions for a sales manager EXCEPT: A. developing the product design B. evaluating the sales force C. compensating and motivating the sales force D. recruiting and training the sales force E. determining the sales force structure Answer: A Rationale: Another sales manager responsibility is defining sales goals and the sales process. 89. What trait, meaning the ability to put oneself in someone else’s shoes, is something sales managers should look for in applicants? A. Sympathy B. Ego strength C. Kindness D. Assertiveness E. Empathy Answer: E Rationale: Empathy—the ability to place oneself in someone else’s shoes—enables sales people to understand the client. 90. Effective sales management begins with: A. determining sales goals B. determining the most efficient structure for the sales force C. determination the sales force size D. determining a compensation plan E. recruiting the sales force Answer: A Rationale: Without goals to achieve, salesperson performance would be mediocre at best, and the company would likely fail. 91. A(n) _____ is a statement of sales goals, usually based on sales volume alone. A. breakeven statement B. trend analysis C. order forecast D. account report E. quota Answer: E Rationale: A quota is a statement of the individual salesperson’s objectives, usually based purely on sales volume, but sometimes including other factors 92. Guilden Corporation has instructed its new salesperson that she is responsible for selling five display cases with infrared heating capability per week, plus calling on five current and 10 potential customers. These objectives comprise her: A. promotion goal B. fulfillment C. quota D. contribution E. potentiality Answer: C Rationale: A quota is a statement of sales objectives given to an individual salesperson. 93. Sales departments have traditionally been organized by all of the following EXCEPT: A. geographic region B. product line C. competition D. marketing function E. industry Answer: C Rationale: Another structure is based on the client or account. 94. _____ structures for sales force are gaining popularity in today's competitive selling environment for companies that emphasize relationship selling. A. Geographic-based B. Industry- or market-based C. Product-based D. Customer-based E. Buying center-oriented Answer: B Rationale: Being familiar with one industry or market allows sales reps to become experts in their fields and thereby offer better solutions and service. 95. A new salesperson generally receives training in all of the following EXCEPT: A. company policies and practices B. accounting procedures C. product knowledge D. industry characteristics E. nonselling duties, such as filling out information reports Answer: B Rationale: Accounting procedures and policies are not a typical part of a sale person's domain. 96. All of the following statements about recruiting and training the sales force is true EXCEPT: A. Training should not be limited to just the new sales force members. B. One of the most important traits to look for in a sales job candidate is ego strength. C. Sales training should begin after a brief orientation. D. Aggressiveness is an important trait to look for when recruiting salespeople. E. Salespeople are typically trained to be negotiators. Answer: D Rationale: Effective salespeople are assertive without being overbearing or assertive. 97. Generally, companies with lower levels of compensation: A. suffer higher turnover rates B. experience overall decreased cost C. experience increased effectiveness D. experience lower termination rates E. experience improved customer satisfaction Answer: A Rationale: Companies with lower levels of compensation suffer higher turnover rates, which increases costs and decreases effectiveness. 98. The most popular sales incentives are _____. A. cash awards B. vacations C. plaques and trophies D. stock options E. public recognition in front of family and peers Answer: A Rationale: Cash awards are the most popular sales incentive and are used by virtually all companies. 99. In motivating their sales force, sales managers must be careful not to encourage: A. increased turnover B. unethical behavior C. excessive costs D. lowered morale E. lowered customer satisfaction Answer: B Rationale: Sales managers offer rewards, but they must be careful not to encourage unethical behavior. 100. Sometimes the compensation plan alone is not enough to motivate a sales team to produce the volume of sales or the profit margin required by management. Which of the following can be used to further motivate a sales team? A. vacations B. recognition at annual sales meeting C. pay raises D. cash bonuses E. all of these Answer: E Rationale: All of these are listed as motivational tools. 101. All of the following are typical sales force performance measures EXCEPT: A. size of the sales force B. sales volume C. contribution to profit D. calls per order E. percentage of calls achieving specific goals Answer: A Rationale: Another measure is sales or profits per call. 102. Aside from the usual characteristics, sales managers look for _____ in sales recruits. A. Sociability B. Creativity C. Ego strength D. Empathy E. All of these choices Answer: E Rationale: In addition to the above traits, almost all successful salespeople say their sales style is relationship oriented rather than transaction oriented. 103. How can technology be helpful as a sales tool? A. It can cost-effectively process orders and service requests B. It can free sales reps from tedious administrative tasks C. It can help to improve the relationship between salesperson and customer D. It can give salespeople more time to focus on the needs of their clients E. All of these choices Answer: E Rationale: Technology can make sales management more effective, too. 104. _____, buying, selling, marketing, collaborating with partners, and servicing customers electronically using the Internet, has had a significant impact on personal selling. A. I-marketing B. M-commerce C. E-business D. C-focusing E. None of these choices Answer: C Rationale: Virtually all large companies and most medium and small companies are involved in e-commerce and consider it to be necessary to compete in today’s marketplace. Kraft Foods Kraft Foods has established dedicated teams to focus on their business with many of their largest customers. With “Team Kroger,” Kraft has combined with Nabisco selling organizations in order to bring more unique shopper insights to Kroger supermarkets. This helps Kroger understand their shoppers and customize their programs to grow even more. At the same time, Kraft has strengthened its relationship with Kroger by gaining a better understanding of the supermarket’s needs and how it can help support Kroger’s business objectives concurrently with its own. 105. Refer to Kraft Foods. Which type of selling does this Kraft perform with Kroger? A. Traditional personal selling B. Relationship or consultative selling C. Customer selling D. Interactive selling E. Focused selling Answer: B Rationale: Relationship or consultative selling focuses on improving the customer’s bottom line. See Exhibit 17.2. 106. Refer to Kraft Foods. The AIDA concept is encouraged among the sales teams that focus on other resellers besides Kroger because it can help in the _____ process. A. JIT B. communication C. networking D. selling E. cold calling Answer: D Rationale: The steps of the selling process closely follow the AIDA concept (attention, interest, desire and action). 107. Refer to Kraft Foods. _____ is the identification of those firms and people most likely to buy Kraft products. A. Prospecting B. Lead qualification C. The sales presentation D. The close procedure E. The sales approach Answer: A Rationale: When Kraft identifies the firms most likely to buy its products, it is engaging in lead generation, or prospecting. 108. Refer to Kraft Foods. Each Kraft sales team member is given a statement of his or her individual sales objectives, usually based on sales volume. This is called a(n): A. sales lead B. objective report C. order forecast D. account report E. quota Answer: E Rationale: A quota is a statement of the individual salesperson’s sales objectives, usually based on sales volume alone but sometimes including key accounts, new accounts, repeat sales, and specific products. 109. Refer to Kraft Foods. The sales team in Bentonville, Arkansas, devotes all of its efforts on satisfying the needs of its largest customer, Walmart. This is an example of a sales force organized by: A. geographic region B. product line C. market D. industry E. individual client or account Answer: E Rationale: Wal-Mart is one client or account that Kraft Foods sell to, but an entire sales team is devoted to this client. College Entertainers Most colleges and universities have a campus activity board that brings entertainers and special events to campuses. To locate and hire these entertainers, many campuses send their student activity board members to one of the seven regional conventions or the National Association of Campus Activities (NACA) convention. More than 100 entertainers, talent agencies, and companies that provide promotional services exhibit their products at these conventions in an area similar to a trade show. (Conference attendees call this the Marketplace.) Approximately 20 entertainers are selected to present a short program (called a showcase) in which they demonstrate what they would do if hired to perform on campus. Many other entertainers have videos and CDs, which they hand out to student attendees. Only one or two members of any student group can actually set up a contractual agreement with an entertainer or his or her agency. Other student attendees provide input as to which entertainers best match the demographics of their institutions. 110. Refer to College Entertainers. The entertainers and agency representatives engage in _____ in the Marketplace. A. transformational management B. countertrades C. all types of marketing communications D. personal selling E. loyalty marketing programs Answer: D Rationale: Personal selling is a purchase situation involving a personal, paid-for communication between two people in an attempt to influence each other. 111. Refer to College Entertainers. Every time a talent agency or entertainer exchanges information with an activity board representative at the NACA convention—for example, through a showcase—he or she can claim a successful _____ with that representative. A. interface B. countertrade C. campaign D. interaction E. countertrade Answer: D Rationale: An interaction occurs when a customer and a company representative exchange information and develop learning relationships. 112. Refer to College Entertainers. The student activity board members who attended a NACA convention were approached by a talent agent who represents a group of comedians who have been known to use profane language in their routines. Several of the college students told the agent that this type of language would not be welcome at their college. The agent promised the group that the comedians were able and willing to perform an act that the students could bring their grandmothers to see without being embarrassed. In this example, the agent was: A. getting referrals. B. doing a pre-approach. C. qualifying leads. D. using trade promotions. E. handling objections. Answer: E Rationale: The prospect often raises objections or asks questions about the proposal and the product, and an effective salesperson should anticipate objections. 113. Refer to College Entertainers. The student members might agree to pay lodging and travel expenses in return for the entertainer reducing his or her fees by $100. This is an example of: A. engaging in cross promotion. B. negotiation. C. countertrade. D. a cross-sell. E. creating a value chain. Answer: B Rationale: Negotiation is the process during which both the salesperson and the prospect offer special concessions in an attempt to arrive at a sales agreement. Advanced Bionics Advanced Bionics is a global leader in the development of implantable, high-tech neurostimulation devices used to treat deafness and chronic pain. Just two-and-a- half years after introducing its new stimulators, the firm is doing $200 million in domestic sales. How has Advanced Bionics been so successful? Its success can be attributed to its outstanding products and to three aspects of its sales approach. First, Advanced Bionics hired the right talent for its sales force. Companies tend to use experienced sales reps—often hired away from competing companies—to sell new medical devices. This can be very expensive since experienced reps generally expect as much as $200,000 in annual income. So when Advanced Bionics brought its medical device to market, it hired people with no experience selling medical devices and turned them into sales reps. Hiring inexperienced salespeople created some problems, however, and the company experienced high turnover rates during its first two years in operation. Second, Advanced Bionics provided salespeople with a rigorous training program. Third, Advanced Bionics allowed salespeople to plan their own activities while managers monitored their progress and pushed them to improve. In short, the company has been able to succeed by selecting good people, training them well, and managing them carefully. 114. Refer to Advanced Bionics. Advanced Bionics has cutting-edge technology and its customers are very sophisticated. Salespeople need to persuade physicians, neurosurgeons, and orthopedic doctors to use Advanced Bionics’ products. Which of the following statements is NOT an advantage that personal selling offers the company? A. It can tailor the message to the customer needs. B. It is good for explaining the merits of complex products. C. It is inexpensive on a per contact basis. D. It can be specifically directed to the different types of accounts. E. It is effective in convincing a prospective client. Answer: C Rationale: Personal selling is more expensive than other tools when calculated on a per contact basis. 115. Refer to Advanced Bionics. Personal selling is important for Advanced Bionics because: A. its products are standardized. B. its buyers are not very technologically savvy. C. the complexity of its products has decreased. D. its buyers are extremely dispersed. E. its products are high value. Answer: E Rationale: Customers usually need more information and demonstrations to answer their questions about high-value products. See Exhibit 17.1. 116. Refer to Advanced Bionics. Medical Education Training Associates (META) handles the training of new Advanced Bionics sales reps. One of META’s advanced courses focuses on building networks among the company’s various stakeholders. Networking is often discussed as an approach for developing: A. new product ideas. B. potential new clients. C. strategic resources. D. sales force morale. E. cost reduction strategies. Answer: B Rationale: Networking is using friends, business contacts, coworkers, acquaintances, and fellow members in professional and civic organizations to identify potential clients. 117. Refer to Advanced Bionics. One of the keys to Advanced Bionics’ success has been finding and hiring the right talent for sales positions. Finding and hiring new salespeople is known as _____. A. sales force motivation B. sales force evaluation C. sales force personnel management D. sales force recruiting E. sales force qualification Answer: D Rationale: Sales force recruiting should be based on an accurate, detailed description of the sales task as defined by the sales manager. 118. Refer to Advanced Bionics. District sales managers at Advanced Bionics often spend three days per week in the field coaching and monitoring sales reps. This time in the field with sales reps is an important final task in the management process known as: A. compensation B. networking C. quota determination D. qualifying sales leads E. evaluating performance Answer: E Rationale: The final task of sales managers is evaluating the effectiveness and performance of the sales force. ESSAY 1. List the advantages personal selling offers over other forms of promotion. Answer: • Personal selling provides a detailed explanation or demonstration of the product. This capability is especially needed for complex or new goods and services. • The sales message can be varied according to the motivations and interests of each prospective customer. Moreover, when the prospect has questions or raises objections, the salesperson is there to provide explanations. In contrast, advertising and sales promotion can only respond to the objections the copywriter thinks are important to customers. • Personal selling can be directed only to qualified prospects. Other forms of promotion include some unavoidable waste because many people in the audience are not prospective customers. • Personal selling costs can be controlled by adjusting the size of the sales force (and resulting expenses) in one-person increments. On the other hand, advertising and sales promotion must often be purchased in fairly large amounts. • Perhaps the most important advantage is that personal selling is considerably more effective than other forms of promotion in obtaining a sale and gaining a satisfied customer. 2. Discuss the customer and product conditions that suggest personal selling is more important than advertising and sales promotion. Give some examples of products for which personal selling would likely be the best tactic. Answer: Personal selling is more important if: • the product has a high value • the product is custom-made • there are few customers • the product is technically complex • customers are concentrated Product examples include insurance policies, home remodeling, business jets, automobiles, pharmaceutical drugs, and private communication systems. Students will provide other examples. See Exhibit 17.1. 3. Compare and contrast relationship selling and traditional personal selling. Answer: Traditional selling: • focuses solely on the sales transaction • focuses on closing sales • emphasizes a planned presentation for the sole purpose of making the sale • attempts to persuade buyers to accept a point of view or convince the buyer to take some action • has objectives that are frequently at the expense of the buyer (win–lose outcome) Relationship or consultative selling: • emphasizes the relationship that develops between a salesperson and a buyer • builds, maintains, and enhances interactions with customers to develop long-term satisfaction through mutually beneficial partnerships • develops trust over time • emphasizes solution development over a long-term relationship (a win–win outcome) See Exhibit 17.2. 4. Describe the actions of a company that is referred to as having a customer-centric focus. Answer: Companies that have a CRM system follow a customer-centric focus or model. Customer-centric is an internal management philosophy similar to the marketing concept. Under this philosophy, the company customizes its product and service offerings based on data generated through interactions between the customer and the company. 5. What is knowledge management? What type of companies use knowledge management? Answer: Knowledge management is a process by which learned information from customers is centralized and shared in order to enhance the relationship between customers and the organization. Companies that have a CRM system need to use knowledge management. 6. Explain the importance of interactions in a customer relationship management (CRM) system. Most colleges and universities have some form of CRM system. Describe an interaction that occurs at your institution. Answer: An interaction occurs when a customer and a company representative exchange information and develop learning relationships. There are several examples––probably, the best occurs when the student goes through the admission process. The applying student provides information for the registrar’s office, and the registrar’s office provides information to the student about classes, living arrangements, food plans, and so on. 7. What is a touch point? Give two examples of touch points. Answer: Touch points are all possible areas of a business where customers communicate with that business. Touch points would include sales, requests for information, complaints, warranty applications, and credit applications. Students may develop other equally correct answers. 8. List the seven steps in the personal selling process. Answer: 1. Generating leads 2. Qualifying leads 3. Approaching the customer and probing needs 4. Developing and proposing solutions 5. Handling objections 6. Closing the sale 7. Following up 9. Tremaine Hughes is a salesperson for Allied Pets, a company that sells veterinarian supplies. He is working on a needs assessment for East Athens Veterinary Clinic. What information will he need to find out about East Athens? Answer: A consultative salesperson must know everything there is to know about: • the products that Allied Pets offers • East Athens as a customer and have an understanding of its specific needs • the competition East Athens faces • the industry East Athens operates in 10. The first step in the personal selling process is generating leads. Define lead generation and explain why referral is a superior method for generating leads. Answer: Lead generation, or prospecting, is the identification of those firms and people most likely to buy the seller’s offerings. These firms or people become “sales leads” or “prospects.” Referrals are recommendations from customers or business associates. The advantages of referrals over other forms of prospecting include highly qualified leads, higher closing rates, larger initial transactions, and shorter sales cycles. 11. Shelby is a salesperson for Kohler, a company that sells plumbing fixtures. He has a prospect who is showing interest in learning more about his company’s products, so now he must qualify this sales lead. What will Shelby do? Answer: Lead qualification involves determining whether the prospect has three things: A RECOGNIZED NEED. First, the salesperson must determine if the prospect has a need that is not being satisfied. Preliminary interviews and questioning may help in this process. BUYING POWER. To avoid wasting time and money, the salesperson should identify the purchasing authority before making a presentation and determine that the prospect has the funds to pay for the product. An organization chart can qualify the prospect. Additionally, information on a firm’s credit standing can be obtained from credit and financial reporting services. RECEPTIVITY AND ACCESSIBILITY. The prospect must be willing to see the salesperson and be accessible to the salesperson. 12. Use the chart to indicate the relative amount of time spent in each of the key steps of the selling process for both traditional selling and relationship/consultative selling. The first step has been completed for you. Answer: See Exhibit 17.5. 13. What is automated e-mail follow-up marketing? How does it work? Answer: Automated e-mail follow-up marketing is a combination of sales automation and Internet technology that enhances customer satisfaction as well as brings in more business for some marketers. It works like this: After the initial contact with a prospect, a software program automatically sends a series of personalized e-mail messages to the prospect over a period of time. 14. You are the sales manager for Cone Machinery, a manufacturer of custom-made sawmill machinery. You are responsible for designing the training program for its new salespeople. List four major areas in which you would like your salespeople to receive instruction. Answer: A new salesperson generally receives instruction in company policies and practices, selling techniques, product knowledge, industry and customer characteristics, and nonselling duties such as filling out sales and market information reports or using a sales automation computer program. Chapter 18—Social Media and Marketing TRUE/FALSE 1. For most people, social media is a marketing experience. Answer: False Rationale: For most people, social media is meant to be a social experience, not a marketing experience. 2. A new alt-country band, Pike 27, wants to use a social media Web site to generate some buzz about their first album. Because of its safe, uncluttered, ad-free setting, MySpace would be a good site for the band to choose. Answer: False Rationale: What initially attracted users to Myspace (the freedom to code your own page, play with widgets and music, freely explore other people’s pages) became too much and drove users away—often to Facebook. 3. Social commerce combines social media with the basics of e-commerce. Answer: True Rationale: Social commerce leverages social media platforms to facilitate online buying and selling, integrating social interaction with e-commerce functionalities. 4. In 2000, Jones Soda used its Web site to ask its mostly teenage customers to suggest new flavors, names, and labels and let other customers vote on which flavors should be introduced into stores. Jones Soda was practicing co-creation. Answer: False Rationale: Using consumers to develop and market product is called crowdsourcing. 5. Erica was so impressed by the customer service she received from Mindware Toys that she tweeted all of her followers about her experience. Erica’s tweets are a good example of owned media. Answer: False Rationale: Erica’s tweets are an example of earned media, not owned media. Earned media includes viral videos, retweets, comments on blogs, and other forms of customer feedback resulting from a social media presence. 6. Marketers need to understand that, in cyberspace, paid media is essentially dead; consequently, they should no longer expend any resources into paid media. Answer: False Rationale: Paid media is not dead but should serve as a catalyst to drive customer engagement. 7. Customers are on social media, and they expect their favorite brands will be there as well. Answer: True Rationale: Customers use social media to engage with brands and expect their favorite companies to have a presence there for interaction and support. 8. The best way a company can increase sales using social media is to improve customer service. Answer: False Rationale: The clearest path to increasing the bottom line using social media is to get customers talking about products and services, which usually translates into sales. 9. While hundreds of metrics have been developed to measure social media’s value, these metrics are meaningless unless they are tied to key performance indicators. Answer: True Rationale: Metrics need to be aligned with key performance indicators (KPIs) to effectively measure social media’s impact on business goals. 10. Monette has posted over 100 book reviews on Amazon.com. She also frequently reviews products and services on opinions and posts her impressions of local restaurants on Yelp. As a social media user, Monette would best be characterized as a joiner. Answer: False Rationale: Joiners are those who maintain a social networking profile and visit other sites. Monette would best be characterized as a critic because she posts comments, ratings, and reviews of products and services on blogs and forums. 11. Research indicates that the number of people who contribute content on social media sites is actually decreasing. Answer: True Rationale: Studies have shown a decline in active content contributors on social media, with more users consuming rather than creating content. 12. A publicly accessible Web page that serves as an interactive journal is called a tweet. Answer: False Rationale: A publicly accessible Web page that serves as an interactive journal is called a blog. A tweet is the name given to a post on Twitter. 13. Anil hopes to use social media to find some new customers for his bicycle shop. Writing a corporate blog that discusses the shop’s inner workings would be a good tool for this purpose. Answer: True Rationale: A corporate blog can attract new customers by providing insights into the business, sharing expertise, and engaging with the community. 14. Mary is a stay-at-home mom who loves to cook. She shares recipes and cooking tips on her popular blog, What’s Cooking with Mary. Because of her blog’s popularity, many food and cookware companies send her products to try, hoping for a good review. Sometimes Mary even receives money in exchange for a review. Under Federal Trade Commission regulations, Mary is obligated to disclose any financial relationship she may have with these companies. Answer: True Rationale: FTC regulations require bloggers to disclose any financial relationships or received products to ensure transparency and trust with their audience. 15. A large part of Twitter’s success results from the one-dimensional nature of the platform—it is limited to text messages only, which makes it simpler for people to use than platforms that allow the sharing of photos or external links. Answer: False Rationale: Twitter is actually a very versatile platform, and tweets can be amended with photos, videos, and external links. 16. Facebook can best be characterized as a media sharing site. Answer: False Rationale: Facebook is best characterized as a social networking site. 17. Estefan works as a production editor for a major textbook publisher. He hopes to build a list of freelancers whom he can call on to edit and proofread materials for his company. LinkedIn would be a good place for Estefan to find such individuals. Answer: True Rationale: LinkedIn is a professional networking site where Estefan can connect with and find qualified freelancers for editing and proofreading. 18. Teenage boys make up the vast majority of YouTube users. Answer: False Rationale: YouTube attracts a diverse base of users: Age and gender demographics are remarkably balanced. 19. Alison visits Reddit and Digg several times each day, where she posts links to material she finds interesting. She also votes either up or down links that others have posted based on whether she liked or disliked them. Alison clearly enjoys using social news sites. Answer: True Rationale: Alison's active participation in posting and voting on links indicates her enjoyment of social news sites like Reddit and Digg. 20. A review site allows consumers to post, read, rate, and comment on opinions regarding a wide variety of goods and services. Answer: True Rationale: Review sites are designed for consumers to share their opinions and rate products and services, facilitating informed purchasing decisions. 21. Marketers have largely been frustrated in their attempts to engage with consumers through virtual worlds and online gaming because consumers of these environments are usually so intent on playing the games that they notice little else. Answer: False Rationale: The field of virtual worlds is a growing consideration for social media marketing. Several businesses have developed advantageous presences in virtual world environments. 22. Because of the widespread adoption of Android-based phones and other Smartphones, millions of applications have been developed for the mobile market. Answer: True Rationale: The popularity of smartphones has led to the development of millions of mobile applications catering to various needs and interests. 23. This morning, Katrina received the following text message on her cell phone: Don’t forget your haircut appointment today at 10 am with Jane at Hair Now. See you soon. This is an example of a mobile ad. Answer: False Rationale: Hair Now is using mobile marketing, but this specific example would best be characterized as an SMS (short message service), not a mobile ad. 24. Marketers have so far found few ways to integrate apps into their marketing strategies. Answer: False Rationale: An app can generate customer engagement when it is well branded and integrated into a company’s overall marketing strategy. 25. Sebastian operates a local coffee shop. He publishes a blog about the shop but updates it only occasionally. He usually posts entries on coffee-related trivia. Every now and then, he posts an entry promoting a new product. Given Sebastian’s relatively weak online presence, a marketing-oriented widget would likely be just the thing to generate some buzz about his coffee shop. Answer: False Rationale: A marketing-oriented widget can be an effective tool for organizations that regularly publish compelling online content, such as news, daily specials, or coupons—but Sebastian’s blog would not seem to be one of them. MULTIPLE CHOICE 1. Social media: A. cannot be compared to traditional advertising in any meaningful way. B. is thought of by most people as any tool or service that assists conversation. C. has less sophisticated ways of measuring how marketers meet and interact with consumers than traditional advertising. D. offer more one-to-one ways to meet consumers than traditional marketing media. E. has not yet had a major impact on the way marketing works. Answer: D Rationale: In many ways, social media can be defined relative to traditional advertising. For example, while traditional marketing media offers a mass media method of interacting with consumers, social media offers more one-to-one ways to meet consumers. 2. All of the following are examples of social media tools and platforms EXCEPT: A. a media sharing Web site. B. an AM/FM transistor radio. C. an iPhone. D. a blog. E. Twitter. Answer: B Rationale: Social media includes tools and platforms such as social networks, blogs, microblogs, and media sharing sites. 3. Marla is a 55-year-old woman who wants to use a social network site to stay in touch with her son, who is away at college. She also hopes to make contact with some old high school friends. Which of the following is Marla most likely to use for this purpose? A. Last.fm B. YouTube C. Facebook D. Bebo E. Flickr Answer: C Rationale: The largest area of growth in new profiles on Facebook is the baby boomer segment, who use it as a way to connect with old friends and keep up with family. Other social networks, like Last.fm and Bebo, offer alternative networks to other demographics. YouTube and Flickr are media sharing sites, not social network sites. 4. Noah is a budding photographer who is proud of his work. He wants to share his photos with others online. Which of the following is Noah MOST likely to use to share his photos? A. Twitter B. Google C. Bebo D. Flickr E. Groupon Answer: D Rationale: Flickr is a popular photo sharing site, where millions of people upload new photos daily. 5. Groupon.com offers discounts on products and services in specific cities. The concept of Groupon is that the discount for the product or service can be claimed only if a certain number of people purchase it. Groupon would BEST be described as a: A. media sharing site B. microblog C. social networking site D. virtual world E. social commerce site Answer: E Rationale: Social commerce is a subset of e-commerce that involves the interaction and user contribution aspects of social online media to assist the online buying and selling of products and services. 6. More and more companies are increasing their interactive marketing budgets. Most of this investment will be made in: A. search marketing. B. display advertising. C. mobile marketing. D. social media. E. e-mail marketing. Answer: A Rationale: Most spending in this area will go to search marketing (almost doubling by 2014), but substantial investments will also be made in mobile marketing and social media. 7. Because of the influence of social marketing, the interaction between producer and consumer has become LESS focused on: A. engaging. B. listening. C. understanding. D. entertaining. E. influencing. Answer: D Rationale: Because of social media, the interaction between producer and consumer has become less about entertaining and more about listening, influencing, and engaging. 8. Using consumers to develop and market product is called: A. crowdfunding. B. co-creation. C. brainstorming. D. closed innovation. E. crowdsourcing. Answer: E Rationale: This is the definition of crowdsourcing, which describes how the input of many people can be leveraged to make decisions that used to be based on the input of only a few people. 9. Which of the following is an example of owned media? A. The tweet that Winnie—who works out regularly at FitBody Bootcamp—sent to her followers about how much weight she’s lost since she’s been working out B. An e-mail sent by Garrett to the manager of FitBody Bootcamp, asking about what types of workout programs the gym offers C. The Facebook page created by FitBody Bootcamp to let members know about special events at the gym D. The display advertisement that FitBody Bootcamp purchased on the local newspaper’s Web site E. The personal blog “Feeling the Burn,” maintained by Andy, who is one of the instructors at FitBody Bootcamp Answer: C Rationale: Owned media is online content that an organization itself creates and controls. 10. Content that a company purchases to be placed online is called: A. owned media. B. paid media. C. controlled media. D. earned media. E. broadcast media. Answer: B Rationale: This is the definition of paid media, which is similar to marketing efforts that use traditional media, such as newspaper, magazine, and television advertisements. 11. One reason why negative customer comments and complaints are important to an organization is that they can: A. quickly be hidden from competitors. B. be used in future advertising campaigns. C. help the organization identify “problem” customers, who are then typically removed from mailing lists. D. bring to light unknown brand flaws. E. generate sympathy for the organization when consumers realize that “no one is perfect”. Answer: D Rationale: Negative comments and complaints are of particular importance to an organization because they can illuminate unknown brand flaws and because they are the comments that tend to go viral. 12. Angela and Dominic own Cincy by the Slice pizzeria. They are interested in keeping track of the number of mentions their store receives on sites like Twitter and Facebook, but they are unsure of which steps to take next. Which of the following resource would best meet their needs? A. A listening platform with key word report capabilities B. An insight platform that offers online focus group solutions C. Web analytics software D. An advanced customer database with predictive application E. A social customer relationship management system Answer: A Rationale: Such a platform would allow the organization to track discussions, understand sentiment, and identify influencers to improve overall marketing strategy. See Exhibit 18.1 13. After establishing a listening platform, an organization should: A. develop a list of objectives for its social media team to accomplish B. attempt to develop a unique social media platform to generate buzz about the brand C. identify potential consumers D. decide which tools and platforms to use as part of its social media strategy E. implement measurement tools to determine the benefits of social media Answer: A Rationale: These objectives must be developed with a clear understanding of how social media changes the communication dynamic with and for customers. 14. Social media strategist Jeremiah Owyang has identified eight stages of effective listening. Which of the following scenarios is an example of “Stage 8” listening? A. An online clothing retailer asks customers to complete a satisfaction survey, which it uses to improve customer service B. A local restaurant searches Twitter to find out what people are saying about today’s specials C. A software company allows customer service representatives to make real-time responses to customer queries D. A soft drink manufacturer uses social media notification tools to learn what the competition might be up to E. An online bookseller examines customers’ product reviews, comments, and past purchases to suggest additional titles customers might enjoy Answer: E Rationale: Stage 8 listening involves being proactive and anticipating customer demands; organizations at this level examine previous patterns of data and social behavior to anticipate customer needs. See Exhibit 18.1. 15. All of the following are practical ideas that marketing managers should consider when setting social media objectives EXCEPT: A. promoting products and services. B. listening and learning. C. building relationships and awareness. D. creating static messages to reach mass audiences. E. reaching out to unhappy customers and engaging them directly. Answer: D Rationale: Attempting to reach a mass audience with a static message will never be as successful as influencing people through conversation. 16. Crock of Love prepares and delivers from-scratch meals directly to their customers’ homes. Crock of Love has its own Web site, Facebook page, and Twitter account, but the company’s owner, Nikki, frequently makes helpful comments on food-related blogs and participates in online discussions about recipes and desserts. In so doing, Nikki is MOST LIKELY trying to: A. control what is being said online about Crock of Love. B. improve Crock of Love’s reputation. C. measure customer satisfaction with Crock of Love. D. improve customer service. E. monitor Crock of Love’s competitors. Answer: B Rationale: Organizations can position themselves as helpful and benevolent—and thus improve their reputations—by participating in other forums and discussions. 17. Which of the following statements about the use of social media in marketing is TRUE? A. All marketers understand that social media is mostly about return on investment (ROI). B. Marketers have plenty of hard evidence as to the effectiveness of the measurements and metrics that track social media. C. Given the relative ease and efficiency with which organizations can use social media, a positive return on investment is likely for many—if not most—organizations. D. Fewer than 30 percent of marketers plan on increasing their investments in social media. E. All of the above are true. Answer: C Rationale: Most marketers understand that social media is a worthwhile investment. 18. Without context, measurements are _____: A. Not very costly. B. invaluable. C. more accurate. D. hard to capture. E. meaningless. Answer: E Rationale: Without context, measurements are meaningless. 19. Which type of social media user posts comments, ratings, and reviews of products and services on blogs and forums? A. Critic B. Conversationalist C. Boaster D. Collector E. Assigner Answer: A Rationale: This is the definition of a critic. 20. Bethany follows several blogs every day and checks into YouTube frequently to watch the latest videos. She doesn’t have her own Web site, and she almost never makes any comments on the sites she visits; she just enjoys reading and watching. Bethany would best be characterized as which type of social media user? A. Creator B. Spectator C. Collector D. Joiner E. Inactive Answer: B Rationale: Spectators read blogs, listen to podcasts, watch videos, and generally consume media. 21. “Conversationalists” are social media users who: A. post comments, ratings, and reviews of products and services on blogs and forums. B. read blogs, listen to podcasts, and watch videos. C. visit many social media sites every day. D. post status updates on social networking sites and microblogging services such as Twitter. E. use RSS feeds to collect information and vote for Web sites online. Answer: D Rationale: The relatively new category of “conversationalists” represents 31 percent of social media users. 22. A publicly accessible Web page that functions as an interactive journal is called a(n): A. blog. B. profile. C. tweet. D. electronic diary. E. status page. Answer: A Rationale: This is the definition of a blog, on which readers can post comments on the author’s entries. 23. _____ disseminate marketing-controlled information and are effective platforms for developing thought leadership and fostering better relationships with stakeholders. A. press releases B. corporate blogs C. virtual worlds D. radio advertising E. media sharing sites Answer: B Rationale: Corporate blogs disseminate marketing-controlled information and are effective platforms for developing thought leadership, fostering better relationships with stakeholders, maximizing search engine optimization, attracting new customers, endearing the organization with anecdotes and stories about brands, and providing an active forum for testing new ideas. 24. Corina is a big fan of the British television program Doctor Who. She writes about her love of all things Whovian on her ______, called Wibbly Wobbly Ramblings, where readers can post comments about what she has written. A. social news site B. electronic diary C. toolbox D. blog E. virtual world Answer: D Rationale: Corina has a blog about Doctor Who. 25. The most popular microblog is: A. Twitter. B. YouTube. C. Facebook. D. Tumblr. E. Poink. Answer: A Rationale: Twitter is the most popular microblogging platform; it requires that posts be no more than 140 characters long. 26. Patrick goes to the movies three or four times each week. He operates a blog where he writes long, detailed reviews of the latest releases. Patrick’s blog would best be described as a: A. media sharing blog. B. Microblog. C. noncorporate blog. D. collaborative blog. E. corporate blog. Answer: C Rationale: Noncorporate blogs are independent and not associated with the marketing efforts of any particular company or brand. The information on noncorporate blogs is not controlled by marketers. 27. Blogs with strict post length limits are called: A. password-protected blogs. B. beta blogs. C. Miniblogs. D. private blogs. E. Microblogs. Answer: E Rationale: This is the definition of microblogs. 28. Marcie and Alyssa are going to the football game this afternoon. Alyssa notices that Marcie is sitting at her computer, and she calls out to ask if Marcie is ready to go. “Just a second,” replies Marcie. “I just want to tweet to everyone where we’ll be.” Because Alyssa knows a lot about social media, she knows that Marcie is: A. sending e-mail messages to their friends. B. posting a message on Facebook. C. leaving voice mail messages on their friends’ cell phones. D. using MapQuest to find out how to get to the football stadium. E. posting a message on Twitter. Answer: E Rationale: Twitter posts are commonly called tweets. 29. A company would probably NOT want to use Twitter to: A. provide a customer with complex, hard-to-understand instructions on how to use a product. B. direct customers to its corporate blog. C. promote a brand-new product. D. share links to the official company Web site. E. reply to customer queries. Answer: A Rationale: Twitter’s 140-character limit would likely make it difficult for a customer to follow difficult instructions. The company could, however, use Twitter to direct the customer to a Web page where the instructions could be found. 30. A Web site that allows individuals to connect with friends, peers, and business associates is called a: A. virtual world. B. social networking site. C. media sharing site. D. social news site. E. microblog. Answer: B Rationale: This defines a social networking site, which people use to make connections around shared interests, shared environments, or personal relationships. 31. Which of the following is NOT a social networking site? A. Match.com B. Last.fm C. LinkedIn D. Facebook E. All of the above are social networking sites. Answer: E Rationale: All of these are social networking sites. 32. Brands, organizations, and nonprofits that use Facebook: A. have thus far been unsuccessful in their attempts to use the platform to create viral marketing campaigns. B. are public and are thus subject to search engine indexing. C. must never use the Facebook Wall to communicate with consumers. D. need to use all of Facebook’s privacy options to ensure that only the right people have access to company information. E. create profiles, as opposed to pages. Answer: B Rationale: As opposed to individual profiles, corporate Facebook pages are public and subject to search engine indexing. 33. From a marketing standpoint, the most useful Facebook feature is probably _____, which allows a brand to communicate directly with fans via status updates. A. the Profile page B. the search engine C. the Fans list D. the Wall E. the Real or Fake application Answer: D Rationale: The Wall allows marketers to build databases of interested stakeholders. 34. Of the following, who is MOST likely to be a LinkedIn user? A. Erin, a 13-year-old junior high student and music lover B. Mitch, a 24-year-old songwriter who hopes to make it big in Nashville C. Sharika, a 35-year-old attorney just opening her own practice D. Jose, a 50-year-old Los Angeles Dodgers fan looking for conversation with other baseball lovers E. Amanda, a 70-year-old retiree and avid cook Answer: C Rationale: LinkedIn is used primarily by professionals who wish to build their personal brands online and businesses who are recruiting employees and freelancers. 35. TwitPic, Flickr, and Photobucket are all examples of: A. media sharing sites. B. corporate blogs. C. virtual worlds. D. microblogs. E. social networking sites. Answer: A Rationale: Media sharing sites allow users to upload and distribute multimedia content like videos and photos. 36. A movie studio wants to give fans a sneak peek of the trailer for an upcoming film. Which of the following would be the best platform for the studio to use? A. Picasa B. Photobucket C. YouTube D. Imgur E. TwitPic Answer: C Rationale: YouTube is the highest-trafficked video-based Web site. The other answer choices are photo sharing sites. 37. Videos made by teens that focus on fashion are called: A. flames. B. hauls. C. caches. D. feed dumps. E. heaves. Answer: B Rationale: Some teen clothing brands build followings on YouTube by posting hauls. 38. Which of the following is the best description of a podcast? A. Podcasts are like library card catalogs that users can search electronically to find information they need quickly and efficiently. B. Podcasts are like radio shows that are distributed through various means and not linked to a scheduled time slot. C. Podcasts are like newspapers that people can upload to their Smartphones and read anywhere. D. Podcasts are like calendars that people can use to keep track of the important events in their lives. E. Podcasts are like books that users can read directly on their PC screens. Answer: B Rationale: Podcasts are digital audio or video files that are distributed serially for other people to listen to or watch. 39. A _____ allows users to decide which content is promoted on a given Web site by voting that content up or down. A. media sharing site B. social networking site C. review site D. virtual world E. social news site Answer: E Rationale: This is the definition of a social news site, where users post news stories and multimedia for the community to vote on. 40. Social bookmarking sites such as Delicious and StumbleUpon differ from social news sites such as Reddit and Digg in that: A. the objective of their users is to collect, save, and share interesting and valuable links. B. they allow users to edit the stories they find for accuracy. C. users can change their profiles and update their status whenever they wish. D. they treat location-based micro-networking as a game. E. posts are limited to no more than 140 characters in length. Answer: A Rationale: On these sites, users categorize links with short, descriptive tags; search the sites’ databases of links by specific tags; or add their own tags to others’ links. 41. Location-based social networking sites: A. are a hybrid of media sharing and social networking sites. B. distribute digital audio and video files serially for other people to listen to or watch. C. are essentially short messaging systems designed primarily for internal communication. D. combine the fun of social networking with the utility of location-based GPS technology. E. allow users to archive and share photos and interesting external links. Answer: D Rationale: Location-based social networking sites combine the fun of social networking and location-based GPS technology. 42. Molly is a frequent Foursquare user; she enjoys earning badges and special statuses based on the number of: A. reviews she has written. B. visits she makes to particular locations. C. times she “Likes” a certain link or post. D. followers she has. E. times her posts are retweeted. Answer: B Rationale: Foursquare, one of the most popular location sites, treats location-based micro-networking as a game: Users earn badges and special statuses based on their number of visits to particular locations. 43. Which type of social media site allows users to post, read, rate, and comment on opinions about many different kinds of products and services? A. A virtual world B. A review site C. A corporate blog D. A media sharing site E. A microblog Answer: B Rationale: This defines a review site. Research indicates that people trust the consumer opinions they read on Internet review sites much more than traditional advertising. 44. Customers and business owners can do all of the following on the Yelp Web site EXCEPT: A. business owners can set their pages to block negative reviews. B. customers can write about bad experiences they’ve had at local businesses. C. business owners can edit their organizations’ pages. D. customers can read about what others think about particular businesses. E. business owners can respond directly to customers who have written reviews. Answer: A Rationale: Yelp allows customers to write and read reviews of local businesses. Owners can edit their organizations’ pages and respond directly to customers, but they cannot block negative reviews. 45. By far, the largest social network for gaming is: A. Twitter. B. LinkedIn. C. MySpace. D. Facebook. E. YouTube. Answer: D Rationale: Facebook is by far the largest social network for gaming, and nearly 25 percent of people play games within social networking sites like Facebook and MySpace. 46. Which of the following games is probably the MOST appropriate for marketing and advertising? A. Dungeon Runners B. World of Warcraft C. Texas Hold’em Poker D. Legends of Future Past E. The Chronicles of Spell born Answer: C Rationale: Social and real-world–like titles (as opposed to fantasy games) are the most appropriate for marketing and advertising. 47. Danny is a 19-year-old college student and avid gamer. Based on his demographics, which of the following games is Danny MOST likely to play regularly? A. Café World B. Mafia Wars C. Bejeweled Blitz D. Farmville E. World of Warcraft Answer: E Rationale: Massive multiplayer online games (MMOGs) such as World of Warcraft target 18- to 34-year-old males. The typical player of social games such as Farmville or Bejeweled Blitz is a 43-year-old woman with a full-time job. 48. Which of the following statements about online gaming is CORRECT? A. Games such as Angry Birds and CityVille have proven unattractive to marketers because it takes so long to play them. B. Almost 90% of people play games within social networking sites or on mobile devices. C. The typical online gamer is a 43-year-old woman with a full time job and college education. D. Social gaming is one of the few areas of social media that is not growing. E. The demographic of gamers who favor mobile devices tends to skew older than those who play on traditional PCs. Answer: C Rationale: This correctly describes the typical online gamer. 49. SMS is different from MMS in that: A. SMS allows the attachment of multimedia to text messages. B. SMS is designed specifically for viewing and navigation on mobile devices. C. MMS allows the attachment of multimedia to text messages. D. MMS is designed specifically for viewing and navigation on mobile devices. E. SMS and MMS are actually identical marketing tools. Answer: C Rationale: MMS is similar to SMS but allows the attachment of images, videos, ringtones, and other multimedia to text messages. 50. Which mobile marketing tool allows 160-character text messages to be sent to and from cell phones? A. Mobile ads B. SMS C. Web widgets D. MMS E. MOBI Answer: B Rationale: SMS (short message service) is typically integrated with other tools. 51. Much of the growth in social media sites can be attributed to: A. good old-fashioned word-of-mouth promotion. B. new platforms like iPads and Smartphones. C. media hype. D. the proliferation of new Web sites like Flickr and Reddit. E. new technology uses for social media. Answer: B Rationale: Much of the excitement in social media has centered on Web sites and new technology uses, but much of social media’s growth lies in new platforms. The major implication of new platforms mean consumers can access popular social media sites from almost anywhere. 52. _____ percent of the world’s mobile phones can receive text messages. A. 25 B. 40 C. 55 D. 75 E. 90 Answer: D Rationale: Worldwide, there are almost 5 billion mobile phones in use, more than 25 percent of which are smartphones and 75 percent of which can receive text messages. 53. Modern mobile advertising as a viable marketing strategy has become possible primarily because of the development of: A. the microchip. B. the PC. C. Smartphone technology. D. tablet laptops. E. GPS technology. Answer: C Rationale: With a Smartphone, individuals can read a blog, post to Facebook, write an e-mail, play a multiplayer game, use GPS technology—among many other tasks. Smartphone technology has opened the door to modern mobile advertising as a viable marketing strategy. 54. Johanna is a loyal customer of The Beanery, a regional coffee shop chain. Whenever Johanna is found to be within a geofenced area owned by The Beanery, she receives a text message on her BlackBerry offering her money off a coffee drink at a nearby store. The Beanery appears to be using: A. stealth marketing. B. viral marketing. C. Micro-networking. D. Bluetooth marketing. E. Widgets. Answer: D Rationale: With Bluetooth marketing, a signal is sent to Bluetooth-enabled devices, allowing marketers to send targeted messages to users based on their geographic locations. 55. One difference between apps and widgets is that: A. widgets are often cheaper to develop than apps. B. apps cannot be used in a mobile-ready format. C. widgets cannot be run entirely within online platforms. D. apps tend to broaden an organization’s listening system, while widgets narrow it. E. apps offer new content only, while widgets offer new and already-existing content. Answer: A Rationale: Widgets are software applications that run entirely within existing online platforms. They are often cheaper than apps to develop. 56. One of the most recent developments on the social media scene is that: A. it will soon be possible to repost another user’s message on Twitter. B. organizations will be able to host virtual networking events and “meet and greets” in the virtual world. C. searches on Bing will feature a Facebook module that shows you what your friends have liked as it relates to the search. D. marketers will be able to send short, 160-character text messages to your cell phone. E. YouTube users will not only be able to watch videos, they will also be able to upload their own videos to the site. Answer: C Rationale: The partnership linking bringing “like” data from Facebook to Bing searches is expected to be one of the next big things in social media. The other answer choices have been part of the social media scene for quite a while now. See Exhibit 18.3. Hallmark Cards Hallmark Cards is just one of thousands of companies that have a strong online presence. Hallmark’s Twitter account and Facebook page allow the company to talk directly to customers and also lets the customers talk to each other. Hallmark Social Calendar is a Facebook app that helps users stay on top of special events like birthdays and anniversaries; customers can use the app to send electronic cards to their friends’ PCs and cell phones. In addition, Hallmark operates several blogs, where it shares anecdotes about the company and often solicits opinions on various aspects of greeting card design and on ideas for gifts and their pricing. The company encourages Facebook fans to talk about their lifestyles and even upload photos of themselves so that it can better understand its market. The conversations consumers have with each other often result in interesting insights, such as gift ideas for specific occasions and the prices they are willing to pay for different gifts. 57. Refer to Hallmark Cards. When Hallmark asks it customers to weigh in on product design and pricing, it is practicing: A. virtual volunteering. B. mass customization. C. scripting. D. crowdsourcing. E. social commerce. Answer: D Rationale: Using consumers to develop and market product is called crowdsourcing. 58. Refer to Hallmark Cards. Hallmark’s Facebook page is best characterized as: A. owned media. B. secured media. C. shared media. D. paid media. E. earned media. Answer: A Rationale: Owned media is online content that an organization creates and controls. 59. Refer to Hallmark Cards. Kimberlee is a big fan of Hallmark products. She often uploads photos of her Hallmark ornament collection to the company’s Facebook page, she reads and comments on Hallmark blog posts, and she loves chatting with and tweeting other Hallmark lovers. What type of social media user is Kimberlee? A. A critic B. A joiner C. An inactive D. A noob E. A collector Answer: B Rationale: Joiners maintain a social networking profile and visit other sites. 60. Refer to Hallmark Cards. The blog operated by Hallmark Cards is an example of a: A. corporate blog. B. media sharing site. C. Microblog. D. noncorporate blog. E. location-based social networking site. Answer: A Rationale: Corporate blogs are sponsored by a company or one of its brands and are maintained by one or more of the company’s employees. 61. Refer to Hallmark Cards. Which of the following is an example of Hallmark’s use of mobile marketing? A. Asking customers’ opinions about products B. Encouraging customers to talk about their lifestyles C. Establishing a Twitter account D. Enabling customers to send electronic cards to friends’ cell phones E. Sharing company anecdotes on its blog Answer: D Rationale: Mobile marketing refers to marketing on or with a mobile device, such as a mobile phone. Allowing customers to send cards to their friends’ cell phones is one way Hallmark is using mobile marketing. Using Social Media Effectively Many marketers define their social media objectives around five core concepts: (1) listening––understanding what customers are saying and who those customers are, (2) talking––promoting products/services through conversations with customers, (3) energizing––finding enthusiastic customers and using them to sell to others, (4) supporting––enabling customers to help one another, and (5) embracing––using customers as a source of product innovation. Marketers should also keep in mind that, more and more, customers expect companies to use social media. According to a recent survey, more than half of all consumers want ongoing conversations with companies and brands—and of consumers who use social media themselves, a whopping 85 percent believe that companies should have an active social media presence. Marketers who embrace such consumers can harness the power of social media and ultimately improve profits and brand loyalty. 62. Refer to Using Social Media Effectively. Consumers’ opinions about the value of social marketing reflect: A. the decreasing influence of marketing as an important factor in business. B. the fact that paid media is no longer relevant in the marketing world. C. the widespread shift in marketing from one-to-many communication to many-to-many communication. D. marketers’ continuing reluctance to wholeheartedly dive into social media. E. customers can be easily manipulated by the onslaught of information available in the social media universe. Answer: C Rationale: Social media allows marketers to have conversations with consumers, which more and more consumers are demanding. 63. Refer to Using Social Media Effectively. Of the social media objectives often defined by marketers, the foundational objective is undoubtedly: A. energizing. B. talking. C. embracing. D. supporting. E. listening. Answer: E Rationale: The first action a marketing team should take when initiating a social media campaign is just to listen. Only after establishing an effective listening platform should an organization move on to developing a list of objectives for its social media team to accomplish. 64. Refer to Using Social Media Effectively. Which of the following resources would be MOST likely to help an organization achieve the goal of energizing customers? A. Buzzillions, a product review site that works with retailers to identify customers who have purchased products and asks them to write reviews B. Google Analytics, a free service that shows businesses how visitors found their site and how they interact with it C. Site Volume, a service that measures the frequency of search terms across Digg, MySpace, and YouTube D. Converseon, a service that helps marketers find out what people are saying about their product/brand on newsgroups, blogs, podcasts, and social media sites E. InnoCentive, a service that enables its clients to source ideas from their customers Answer: A Rationale: The clearest path to increasing sales using social media is to get customers talking about products and services. Ratings and reviews are a good way to achieve this; they are probably the easiest way to energize customers to sell to others. 65. Refer to Using Social Media Effectively. One way an organization might effectively use noncorporate blogs to help achieve its social media goals is by: A. providing bloggers with products and/or money in exchange for a review. B. flooding the comments section of such blogs with positive comments about the brand or company. C. purchasing the blogs from the owners and converting them into corporate blogs. D. endearing the organization with anecdotes and stories about the brand. E. designing their own corporate blogs to mimic the look of their noncorporate rivals. Answer: A Rationale: Because noncorporate blogs contain information not controlled by marketers, they are often perceived to be more authentic than corporate blogs, which can be highly beneficial to an organization. ESSAY 1. What is social media? Answer: Social media is any tool or service that uses the Internet to facilitate conversations. Social media have changed the way that marketers can communicate with their brands—from mass messages to intimate conversations. As marketing moves into social media, marketers must remember that for most people, social media are meant to be a social experience, not a marketing experience. 2. Social media offers more one-to-one ways for marketers to meet customers. Use this fact to identify five implications for marketers. Answer: 1. Marketers must realize that the often do not control the content. Having a great ad campaign is not enough—the product or service must be great, too. 2. The ability to share experiences amplifies the impact of word of mouth to whole new levels that can eventually affect the bottom line. Many companies are developing mascots (like Progressive’s Flo) to drive their marketing message on social media. 3. Social media allows marketers to listen. Domino’s, Dell, and Gatorade exemplify this. 4. Social media has more sophisticated methods of measuring how marketers meet and interact with customers than traditional advertising. Technology changes daily, offering consumers new ways to experience social media platforms. 5. Social media allows marketers to have much more meaningful and direct conversations with customers. It offers a form of relationship building that will ultimately bring the customer and brand closer. 3. How has social media changed the way marketers and consumers communicate with one another? How has it changed who is in control of the communication? Answer: Social media has facilitated the shift from one-to-many communication to many-to-many communication. Instead of simply creating a brand advertisement with no way to receive consumer feedback, social media allows marketers to have conversations with consumers, forge deeper relationships, and build brand loyalty. Social media also allows consumers to connect with each other, share opinions, and collaborate on new ideas according to their interests. With social media, the audience is often in control of the message, the medium, the response, or all three. Some companies have had trouble adjusting to this new distribution of control, but the focus of social marketing is on the audience and the brand must adapt to succeed. The interaction between producer and consumer becomes less about entertaining and more about listening, influencing, and engaging. 4. Distinguish between owned media and earned media and give several examples of each. Answer: OWNED MEDIA is online content that an organization creates and controls; examples include blogs, Web sites, Facebook pages, and other social media presences. EARNED MEDIA is a public relations term connoting free media such as mainstream media coverage; examples include viral videos, retweets, comments on blogs, and other forms of customer feedback resulting from a social media presence. 5. To leverage owned media, earned media, and paid media, marketers must follow three key guidelines. What are they? Answer: First, marketers must maximize owned media by reaching out beyond their existing Web sites to create portfolios of digital touch points. This is especially true for brands with tight budgets, as the organization may not be able to afford much paid media. Second, marketers must recognize that aptitude at public and media relations no longer translates into earned media. Instead, marketers must learn how to listen and respond to stakeholders. This will stimulate word of mouth. Finally, marketers must understand that paid media are not dead but should serve as a catalyst to drive customer engagement. 6. Identify and discuss five ideas that marketing managers should consider when setting social media objectives. Answer: LISTEN AND LEARN—Monitor what is being said about the brand and competitors, and glean insights about audiences. Use online tools and do research to implement the best social media practices. BUILD RELATIONSHIPS AND AWARENESS—Open dialogs with stakeholders by giving them compelling content across a variety of media. Engage in conversations and answer customers’ questions candidly. This will both increase Web traffic and boost your search engine ranking. PROMOTE PRODUCTS AND SERVICES—The clearest path to increasing the bottom line using social media is to get customers talking about products and services, which ultimately translates into sales. MANAGE YOUR REPUTATION—Develop and improve the brand’s reputation by responding to comments and criticisms that appear on blogs and forums. Organizations can also position themselves as helpful and benevolent by participating in other forums and discussions. IMPROVE CUSTOMER SERVICE—Customer comments about products and services will not always be positive. Use social media to find displeased customers and engage them directly to solve their service issues. 7. Name and briefly describe the six categories of social media users identified by Charlene Li and Josh Bernoff of Forrester Research. Answer: CREATORS—Those who produce and share online content such as blogs, Web sites, articles, and videos. CRITICS—Those who post comments, ratings, and reviews of products and services on blogs and forums. COLLECTORS—Those who use RSS feeds to collect information and vote for Web sites online. JOINERS—Those who maintain a social networking profile and visit other sites. SPECTATORS—Those who read blogs, listen to podcasts, watch videos, and generally consume media. INACTIVES—Those who do none of the above things. 8. Identify the two broad categories of blogs. Which is generally perceived to be more “authentic”? Why? Answer: The two broad categories of blogs are corporate blogs and noncorporate blogs such as personal blogs. Corporate blogs are sponsored by a company or one of its brands and are maintained by one or more of the company’s employees. Noncorporate blogs are independent and not associated with the marketing efforts of any particular company or brand. Because these blogs contain information not controlled by marketers, they are perceived to be more authentic than corporate blogs. 9. What is a blog? Why are blogs considered a key component in any interactive marketing strategy? Answer: A blog is a publicly accessible Web page that functions as an interactive journal, whereby readers can post comments on the author’s entries. Some experts believe that every company should have a blog that speaks to current and potential customers, not as consumers but as people. Blogs allow marketers to create content in the form of posts, which ideally build trust and a sense of authenticity in customers. Once posts are made, readers can provide feedback through comments. Because it opens a dialogue and gives customers a voice, the comments section of a blog post is one of the most important avenues of conversation between brands and consumers. 10. How might a marketer use Twitter to promote a brand? How might a celebrity use Twitter to promote him- or herself? What other types of applications can Twitter can be used for? Answer: Twitter is effective for disseminating breaking news, promoting longer blog posts and campaigns, sharing links, announcing events, and promoting sales. By following, retweeting, responding to potential customers’ tweets, and tweeting content that inspires customers to engage the brand, corporate Twitter users can lay a foundation for meaningful two-way conversation quickly and effectively. Celebrities also flock to Twitter to interact with fans, discuss tour dates, and efficiently promote themselves directly to fans. Twitter can be used to build communities; aid in customer service; gain prospects; increase awareness; and, in the case of nonprofits, raise funds. 11. What is a podcast? How are they used? Answer: A podcast is a digital audio or video file that is distributed serially for other people to listen to or watch. Podcasts can be streamed online, played on a computer, uploaded to a portable media player, or downloaded onto a Smartphone. Podcasts are like radio shows that are distributed through various means and not linked to a scheduled time slot. They have amassed a steadily growing number of loyal devotees. 12. Explain some ways marketers can effectively use location-based social networking sites. Answer: Location-based social networking sites such as Foursquare can be especially useful marketing tools for local businesses, particularly when combined with sales promotions such as coupons, special offers, contests, and events. Businesses can increase customers’ loyalty by rewarding them for repeat visits. Customers’ tips and reviews can help to build a sense of community around a brand or business. In addition, location-based social networking sites can introduce a business to nearby potential first-time customers who may not have otherwise known of its existence. Businesses might also create custom, real-time promotions to provide incentives for customers to check in and return for future visits. 13. Compare the demographics of a typical game player within a social networking site such as Facebook with those of a typical player of massive multiplayer online games such as World of Warcraft. Answer: The typical game player within social networking sites is a 43-year-old woman with a full-time job and college education; the typical massive multiplayer online game player is an 18- to 34-year-old male. 14. Identify five reasons for the recent popularity of mobile marketing. Answer: First, an effort to standardize mobile platform has resulted in a low barrier to entry. Second, especially given younger audiences, there are more consumers than ever acclimating to once-worrisome privacy and pricing policies. Third, because most consumers carry their mobile devices with them at all times, mobile marketing is uniquely effective at garnering consumer attention in real time. Fourth, mobile marketing can be easily and effectively measured. And finally, mobile marketing’s response rate is higher than that of traditional media types such as print and broadcast advertisement. 15. Briefly explain what apps are and describe how a business can use them from a marketing perspective. Answer: Millions of applications—or apps—have been developed for the mobile market. Dozens of new and unique apps that harness mobile technology are added to mobile marketplaces every day. While many apps perform platform-specific tasks, others convert existing content into a mobile-ready format. Whether offering new or existing content, when an app is well branded and integrated into a company’s overall marketing strategy, it can create buzz and generate customer engagement. Chapter 19—Pricing Concepts TRUE/FALSE 1. Price is defined as the value of a good or service as measured by a certain dollar amount. Answer: False Rationale: Price is not necessarily measured in terms of money. In bartering, other items of value may be exchanged. 2. Profit is the price charged to customers multiplied by the number of units sold. Answer: False Rationale: This is revenue. Profit is revenue minus expenses. 3. Today’s firms must develop specific, measurable, and attainable pricing objectives if they hope to survive in highly competitive markets. Answer: True Rationale: Clear and achievable pricing objectives help firms remain competitive and ensure financial stability in the market. 4. The only way to maximize profits is to reduce costs by operating more efficiently. Answer: False Rationale: Profit maximization can also be achieved by expanding revenue by increasing customer satisfaction. Companies can also attempt to reduce costs and expand revenue at the same time. 5. Target return on investment is the most common profit objective used by firms. Answer: True Rationale: Many firms use target return on investment (ROI) as a key profit objective to ensure their investments generate desirable financial returns. 6. Market share is a company’s product sales as a percentage of its total sales for that industry. Answer: True Rationale: Market share measures a company's product sales relative to the total sales within its industry, indicating its competitive position. 7. Maximization of cash should be a long-term objective. Answer: False Rationale: Maximization of cash should never be a long-run objective because cash maximization may mean little or no profitability. Without profits, a company cannot survive. 8. In most communities, the price of gas is more or less the same at all area service stations. This is an example of sales-oriented pricing. Answer: False Rationale: It is an example of status quo pricing. Status quo pricing seeks to maintain existing prices or to meet the competition’s prices. 9. When pricing goals are mainly sales oriented, cost considerations usually dominate. Answer: False Rationale: When pricing goals are mainly sales oriented, demand considerations usually dominate. 10. Profit maximization is the price at which supply and demand are equal, and there is no inclination for prices to rise or fall. Answer: False Rationale: Price equilibrium is the price at which supply and demand are equal, and there is no inclination for prices to rise or fall. 11. If demand for milk is inelastic, consumers will not change their purchasing habits greatly when the price of milk changes. Answer: True Rationale: Inelastic demand means that price changes have little effect on the quantity demanded by consumers. 12. Research indicates that when a country’s inflation rate is high, demand becomes more elastic. Answer: False Rationale: If E is greater than 1, demand is elastic. 13. Unitary elasticity means that an increase in sales exactly offsets a decrease in prices, so total revenue remains the same. Answer: True Rationale: Unitary elasticity implies that the percentage change in quantity demanded equals the percentage change in price, keeping total revenue constant. 14. When many substitute products are available, demand is inelastic. Answer: False Rationale: Demand is elastic when there are many substitute products available. 15. Yield management systems employ techniques such as discounting early purchases, limiting early sales at these discounted prices, and overbooking capacity. Answer: True Rationale: Yield management involves maximizing revenue from perishable resources (such as hotel rooms or airline seats) through dynamic pricing strategies. Techniques include offering discounts for early bookings to encourage advance purchases, limiting the availability of discounted prices to preserve higher margins, and overbooking to mitigate revenue losses from no-shows or cancellations. 16. Yield management systems can only be used by service industries. Answer: False Rationale: As the popularity of yield management systems increases, their use is spreading beyond service industries. 17. Costs should not be part of price determination, especially not as a floor for long run pricing. Answer: False Rationale: Costs should generally be part of any price determination, if only as a floor below which a good or service must not be priced in the long run. 18. Variable costs vary with changes in the level of output, whereas marginal costs do not vary as output changes. Answer: False Rationale: While variable costs do vary with changes in the level of output, marginal costs are the changes in total costs associated with a one-unit change in output. 19. The owner of Buffalo Mountain Coffee Shop pays the same amount in rent each month no matter how many customers she serves. The shop owner’s rent is an example of a marginal cost. Answer: False Rationale: Rent would be an example of a fixed cost. 20. Markup pricing, adding an amount to cost to cover expenses and profit, is the most popular method used by wholesalers and retailers to establish a selling price. Answer: True Rationale: Markup pricing is widely used because it simplifies pricing decisions by adding a predetermined percentage or fixed amount to the cost of a product. This ensures that costs, including expenses and desired profit margins, are covered in the selling price. It's particularly common among wholesalers and retailers because it provides a straightforward approach to setting prices that can adapt to changes in costs and market conditions. 21. A firm has maximized its profits when its marginal revenue exceeds its marginal cost. Answer: False Rationale: A firm has maximized its profits when its marginal revenue equals its marginal cost. 22. Break-even analysis determines what sales volume must be reached before total revenue equals total costs. Answer: True Rationale: Break-even analysis calculates the sales volume at which total revenue equals total costs, helping businesses understand the point at which they cover all expenses and begin to make a profit. 23. As products enter the growth stage of the product life cycle, prices generally begin to stabilize. Answer: True Rationale: During the growth stage of the product life cycle, demand for the product typically increases as it gains market acceptance and consumer awareness. As competition intensifies, firms often focus on expanding market share rather than engaging in aggressive price competition. This can lead to prices stabilizing or even increasing slightly due to perceived value or differentiation strategies. 24. The manufacturers that remain in the market toward the end of the maturity stage typically offer similar prices. Answer: True Rationale: In the maturity stage of the product life cycle, the market becomes saturated, and competition among firms offering similar products intensifies. As a result, manufacturers often adopt competitive pricing strategies to maintain or gain market share. This leads to price stabilization among competitors as they try to retain their customer base and profitability. 25. Prices always steadily decline for a product in the decline stage of the product life cycle. Answer: True Rationale: When only one firm is left in the market, prices begin to stabilize, but they may eventually rise dramatically if the product survives and moves into the specialty goods category, as horse-drawn carriages and vinyl records have. 26. Adequate distribution for a new product is often obtained by reducing the size of the profit margin for its resellers. Answer: False Rationale: Adequate distribution for a new product is often obtained by offering a larger-than-usual profit margin to its distributors. 27. One strategy to get adequate distribution for a new product is to offer dealers a large trade allowance to help offset the costs of promotion. Answer: True Rationale: Offering trade allowances (such as promotional discounts or incentives) to dealers can motivate them to promote and distribute a new product effectively. Distributors and retailers may face costs associated with advertising, stocking, and selling new products. Trade allowances help mitigate these costs, making it more attractive for dealers to invest in the promotion and distribution of the new product. 28. There are three general types of shopping bots: broad-based, niche-oriented, and enterprise-level. Answer: False Rationale: There are two general types of shopping bots. The first is the broad-based type that searches a wide range of product categories. The second is the niche-oriented type that searches for prices for only one type of product. 29. Even though businesses are spending billions on Internet auctions, consumer auctions are likely to be the dominant form in the future. Answer: False Rationale: The opposite appears to be true. 30. Price should not be used as a promotional tool. Answer: False Rationale: Price is often used as a promotional tool to increase consumer interest. 31. High purchase prices may create feelings of pleasure and excitement in consumers. Answer: True Rationale: In consumer behavior, perceptions of price often correlate with perceptions of quality, status, or exclusivity. Higher purchase prices can create a sense of prestige or excitement for consumers who perceive the product as valuable or desirable. This psychological effect is often leveraged in marketing strategies for luxury goods or premium products to enhance their perceived value and attractiveness to consumers. 32. Research has shown that products perceived to be of high quality tend to benefit less from price promotions than products perceived to be of lower quality. Answer: False Rationale: They tend to benefit more from price promotions that products perceived to be of lower quality. MULTIPLE CHOICE 1. Price is best described as: A. that which is given up in exchange to acquire a good or service B. money exchanged for a good or service C. the psychological results of purchasing D. the cost in dollars for a good or service as set by the producer E. the value of a barter good in an exchange Answer: A Rationale: Price is that which is given up in exchange to acquire a good or service. 2. At Walmart, Randi saw a bag of daffodil flower bulbs and a box of plant fertilizer. The items, which were sold together, retailed at $28.50 but were marked down to $19.99. The $19.99 is the: A. revenue. B. price. C. profit. D. liquidity value. E. amortized value. Answer: B Rationale: Price is that which is given in exchange to acquire a product. 3. Which of the following statements is NOT true about price? A. Price can relate to anything with perceived value, not just money. B. Price is that which is given up in an exchange to acquire a product. C. Price means the same thing to the consumer and the seller. D. The price paid is based on the satisfaction consumers expect to receive from a product. E. Customers are interested in obtaining a perceived reasonable price. Answer: C Rationale: Price means one thing to the consumer and something else to the seller. To the consumer, it is the cost of something. To the seller, price is revenue, the primary source of profits. 4. When goods and services are exchanged, the trade is called: A. exchange. B. substitution. C. barter. D. swap. E. bargaining. Answer: C Rationale: This is the definition of barter. 5. Revenue: A. equals quantity sold times profit margin B. equals price minus costs C. equals return on investment D. is synonymous with profit E. equals price of goods times quantity sold Answer: E Rationale: Revenue is the price charged to customers multiplied by the number of units sold. 6. _____ pay for every activity of the company. A. Revenues B. Investments C. Retained earnings D. Profits E. Prices Answer: A Rationale: Revenue is what pays for every activity of the company: production, finance, sales, distribution, and so on. 7. Money that is left over after paying for company activities is called: A. return on investment. B. a contribution margin. C. profit. D. net worth. E. a current asset. Answer: C Rationale: Profit is revenue minus expenses. 8. At Walmart, Randi saw a bag of daffodil flower bulbs and a box of plant fertilizer. The items, which were sold together, retailed at $28.50 but were marked down to $19.99. The retailer sold one at the $28.50 price and five at the $19.99 price. The retailer’s revenue is: A. $8.51 B. $19.99 C. $28.50 D. $128.45 E. $171.00 Answer: D Rationale: Revenue is the price charged to customers multiplied by the number of units sold. $28.50 + ($19.99 × 5) = $128.45. 9. Which of the following is not a real trend that has affected the consumer market? A. The increased availability of bargain-priced private and generic brands has put downward pressure on overall prices. B. Buyers evaluate the price of new products against the value of existing products. C. Many firms are trying to maintain or regain their market share by raising prices. D. The Internet has made comparison shopping easier. E. The United States was in a recession from late 2007 until 2009. Answer: C Rationale: Many firms are trying to maintain or regain their market share by cutting prices. 10. For convenience, pricing objectives can be divided into three categories. They are: A. refundable, competitive, and attainable B. perceived, actual, and unique-situational C. differentiated, niche, and undifferentiated D. profit oriented, sales oriented, and status quo E. monopolistic, fixed, and variable Answer: D Rationale: Profit-oriented objectives include profit maximization, satisfactory profits, and target return on investment. Sales-oriented pricing objectives are based either on market share or on dollar or unit sales. Status quo pricing seeks to maintain existing prices or to meet the competition’s prices. 11. An organization is using _____ when it sets its prices so that total revenue is as large as possible relative to total costs. A. profit maximization B. market share pricing C. demand-oriented pricing D. sales maximization E. status quo pricing Answer: A Rationale: Profit maximization is a type of profit-oriented pricing objective and means setting prices so that total revenue is as large as possible relative to total costs. 12. When Apple, Inc. originally introduced its iPhone, it was priced at what many believed to be about as high as the market would allow. Within weeks, Apple lowered the price of the iPhone. It appears that Apple entered the market with a _____ approach to pricing the iPhone. A. market share pricing B. profit maximization C. demand-oriented D. sales maximization E. status quo pricing Answer: B Rationale: Profit maximization means setting prices so that total revenue is as large as possible relative to total costs. 13. When Insight Research Associates quotes a marketing research project, management will first estimate the cost to conduct the research and produce and deliver the final client report. The next step in determining the price is to add 30 percent to that cost estimate. This becomes the price estimate given to the potential research client. This suggests that Insight Research Associates uses a(n) _____ pricing objective. A. profit-oriented B. market share maximization C. status quo D. sales maximization E. supply–demand equalization Answer: A Rationale: Target return on investment is one of the most common types of profit-oriented pricing objectives. 14. Thompson Pool and Patio is known for quality pool installations, excellent customer service, and reasonable prices. If you want to have a Thompson pool, you will have to wait about six months due to demand for their product. While Thompson could probably price its product higher, given the demand, they don’t. Instead, the company sets its price so that it will earn a reasonable level of profits. Thompson seems to base its pricing policy on: A. profit maximization. B. earning satisfactory profits. C. creating retained earnings. D. making the most money as possible. E. decreasing consumer demand. Answer: B Rationale: The objective of satisfactory profits is characterized by seeking a level of profits that is satisfactory to management and owner(s). 15. _____ is equal to net profit after taxes divided by total assets. A. Return on investment B. Economic order quantity C. Target-on-sales D. Retained earnings E. Efficiency maximization Answer: A Rationale: This is the definition of return on investment (ROI). 16. Pierre’s Ice Cream Company produces ultra-rich ice cream, which it sells in the Cleveland, Ohio, area. Last year, it managed to exceed its target return on investment (ROI) for the current fiscal year. The following results were found on its financial statements: What was the actual ROI for Peirre’s Ice Cream Company? A. 6.67 percent B. 10 percent C. 22 percent D. 28 percent E. 100 percent Answer: B Rationale: ROI is net profits after taxes divided by total assets: $50,000 ÷ 500,000 = 10 percent. 17. Britney is fifteen years old and wants to open her own business selling cupcakes to local coffee shops and restaurants. She is having a tough time deciding whether to base her pricing objectives on market share, dollar sales, or unit sales. Regardless of which she chooses, her pricing objective can be categorized as: A. status quo. B. profit oriented. C. need oriented. D. cost oriented. E. sales oriented. Answer: E Rationale: Sales-oriented pricing objectives are based on either market share or dollar or unit sales. 18. A company using market share pricing has a _____ pricing objective. A. profit-oriented B. sales-oriented C. demand-oriented D. supply-oriented E. status quo Answer: B Rationale: Sales-oriented pricing objectives are based either on market share or on dollar or unit sales. 19. _____ is a company’s product sales as a percentage of total sales for that industry. A. Return on investment B. Profit share C. Revenue share D. Market share E. Contribution Answer: D Rationale: This is the definition of market share, and sales can be reported in dollars or in units of product. 20. At a price of $1,192,057, the Bugatti Veyron may be the most expensive street-legal car on the market today. Obviously, Bugatti is NOT using a(n) _____ pricing objective in setting the price for this car. A. inelastic or supply-oriented B. market share or sales maximization C. profit maximization or target return on investment D. status quo or satisfactory profits E. demand-oriented or supply-oriented Answer: B Rationale: A lower price allows a company to maximize sales and build market share, but Bugatti's high price is geared towards the other options. 21. At the end of the summer, the Bloomin’ Garden Center reduced the price on all of its plants, fertilizer, and potting soil by 50 percent in order to liquidate this inventory. What type of pricing strategy is being used in this example? A. Supply oriented B. Sales maximization C. Target return on investment D. Satisfactory profit E. Profit maximization Answer: B Rationale: Sales maximization ignores profit and competition for the purpose of raising cash. 22. Dixie Furniture Company has recently moved to a new, larger location. At this new location, it has been unable to attract sufficient customers. Its owner did not have the cash to pay the current loan installment due on the building and inventory, so he decided to reduce all merchandise prices by at least 50 percent for a weekend sale to earn enough to make his loan payment. His pricing objective can be classified as: A. market share maximization. B. satisfactory profits. C. asset maximization. D. sales maximization. E. target ROI. Answer: D Rationale: The strategy described will maximize sales dollars but will not maximize or improve any of the other objectives in the long term. 23. As a short-term pricing objective, _____ can be effectively used on a temporary basis to sell off excessive inventory. A. profit maximization B. profit-oriented pricing C. status quo pricing D. sales maximization E. market share pricing Answer: D Rationale: Sales maximization pricing is a short-term price reduction to increase sales. 24. If a company’s pricing objective is to meet the competition or to maintain existing prices, it is using _____ pricing. A. head-on B. target return on investment C. status quo D. market share E. demand-oriented Answer: C Rationale: This defines status quo pricing. 25. When the local Shell station raises or lowers its prices on its gasoline, the Marathon station across the street makes the same changes in its pricing. This is an example of _____ pricing. A. status quo B. target return C. market share D. predatory E. cost-plus Answer: A Rationale: Status quo pricing is best described as meeting the competition. 26. Which of the following statements describes an advantage of status quo pricing? A. Status quo pricing is derived from actual costs of manufacturing. B. Status quo pricing maintains the organization’s differential advantage. C. Status quo pricing is active, not reactive. D. Status quo pricing causes price wars. E. Status quo pricing requires little planning. Answer: E Rationale: Status quo pricing requires little planning because it involves just copying the competitions’ pricing policies. 27. Although many factors can influence price, the primary determinants are: A. costs of manufacturing and distribution B. the demand for the good and the cost to the seller C. demand by the consumer and perceived quality D. distribution and promotion strategies E. stage of the product life cycle and costs to the consumer Answer: B Rationale: The price that managers set for each product depends mostly on two factors: the demand for the good or service and the cost to the seller for that good or service. 28. The quantity of a product that will be sold in the market at various prices for a specified period is called: A. market share. B. demand. C. supply. D. value. E. revenue. Answer: B Rationale: This is the definition of demand. 29. The price of the good or service is a key decision for a marketer because it most significantly and directly affects the product’s: A. distribution. B. costs. C. demand. D. promotion. E. quality. Answer: C Rationale: The quantity of a product that people will buy depends on its price. 30. Most demand curves slope: A. horizontally B. upward and to the right C. downward and to the left D. vertically E. downward and to the right Answer: E Rationale: For most products when prices increase, demand will decrease. 31. Peggy’s Twist Shack sells soft-serve ice cream. Peggy graphed the demand per week for vanilla ice cream cones. The graph indicates a demand schedule that slopes downward and to the right. This graph indicates that the quantity of vanilla ice cream cones demanded increases as: A. cost increases. B. supply decreases. C. price increases. D. price decreases. E. supply increases. Answer: D Rationale: The lower the price, the more goods or services will be demanded. 32. The _____ is the quantity of a product that will be sold in the market at various prices for a specified period, and _____ is the quantity of a product that will be offered to the market by suppliers at various prices for a specified period. A. demand; inventory B. demand; supply C. supply; demand D. inventory; demand E. inventory; supply Answer: B Rationale: These are the definitions of demand and supply, respectively. 33. _____ is the quantity of a product that will be offered to the market at various prices for a specified period. A. Distribution B. Supply C. Price D. Equilibrium E. Elasticity Answer: B Rationale: This is the definition of supply. 34. When the price of a product is set at a level where demand and supply are the same, price _____ has been achieved. A. equilibrium B. stability C. leverage D. symmetry E. status quo Answer: A Rationale: Price equilibrium is the price at which demand and supply are equal. 35. At a price of $6,000, only 191 of the Moulton 60 model bicycle are being made. If Moulton sells each one of the bicycles at that price, then a state of _____ has been achieved. A. symmetry B. marketing balance C. unitary economics D. commerce stability E. price equilibrium Answer: E Rationale: Price equilibrium is achieved at the price at which supply is equal to demand. 36. Bottles of Pure Hawaiian Air contain air that smells like the floral bouquet that greets tourists as they get off the plane in Hawaii. When a tourist shop began selling Pure Hawaiian Air, it charged $5 per bottle and could not keep up with the demand. It has since raised the price to $7. Now the shop is still selling all the bottles of Pure Hawaiian Air it carries, but the owner is not forced to reorder on a daily basis. The $7 price is probably a(n): A. supply schedule. B. symmetrical price. C. price equilibrium. D. inventory equalizer. E. inelastic price. Answer: C Rationale: When demand and supply are approximately equal, price equilibrium is reached. 37. Consumers’ responsiveness or sensitivity to changes in price is known as: A. break-even. B. Equilibrium. C. unitary revenue. D. asymmetrical demand. E. elasticity of demand. Answer: E Rationale: This is the definition of elasticity of demand. 38. When consumers are sensitive to price changes, _____ occurs. A. inelastic demand B. elastic supply C. elastic demand D. inelastic supply E. unitary elasticity Answer: C Rationale: This is the definition of elastic demand. 39. While the sales of the Apple iPhone have been great from the beginning, when Apple released its iPhone 4S and cut the price of the iPhone 4 from $399 to $199, sales exploded with one million iPhone 4s sold the first weekend. Demand for the iPhone appears to be: A. unitary. B. predictable. C. synergistic. D. inelastic. E. elastic. Answer: E Rationale: Elastic demand occurs when consumers by more or less of a product when the price changes. 40. What happens when demand is elastic? A. As price goes up, revenue goes down. B. As price goes down, revenue goes down. C. As price goes up, revenue goes up. D. As price goes up, revenue does not change. E. As price goes down, revenue does not change. Answer: A Rationale: If demand is elastic, an increase in price will decrease demand by a larger amount, reducing total revenue 41. If price _____ and revenue _____, demand is elastic. A. goes up; goes down B. goes down; goes down C. goes down; goes up D. down; stays the same E. goes up; stays the same Answer: C Rationale: If price goes down and revenue goes up, demand is elastic. 42. _____ occurs when an increase in sales exactly offsets a decrease in price so that total revenue remains exactly the same. A. Inelastic demand B. Functional elasticity of demand C. Unitary elasticity D. Highly elastic demand E. Fixed elasticity Answer: C Rationale: Unitary elasticity is a situation in which total revenue remains the same when prices change. 43. When price decreases and total revenue falls, demand is: A. elastic. B. inelastic. C. absolute. D. unitary. E. stable. Answer: B Rationale: This is characteristic of inelastic demand, which means that an increase or decrease in price will not significantly affect the demand for the product. 44. If price goes up or down and revenue stays the same: A. elasticity is universal. B. elasticity is quantum. C. elasticity is solitary. D. elasticity is unitary. E. None of the above. Answer: D Rationale: If price goes up or down and revenue stays the same, elasticity is unitary. 45. When the NES Group lowered the price of its professional-grade meat slicers from $2,300 to $1,600, demand doubled from four units sold per month to eight units per month. However, total revenue dropped. This is an example of: A. substitute goods B. unitary elasticity C. elastic demand D. consumer shortage E. inelastic demand Answer: E Rationale: Inelastic demand is characterized by price and revenue both falling. 46. When Nesco brand food hydrators sold for $59.99, Nesco sold 90 dehydrators. When the company dropped the price of its dehydrators to $44.95, it sold 145 dehydrators. Demand for the food dehydrators appears to be: A. elastic. B. inelastic. C. unitary. D. symmetrical. E. asymmetrical. Answer: A Rationale: The first price is $59.99 with total revenue of $5,399.10; the second price is $44.95 with total revenue of $6,517.75. Therefore, price dropped, and total revenue went up. 47. Demand for which of the following products or services is most likely inelastic? A. Fishing boats B. Wheat bread C. Pedicures D. Filet mignon steaks E. Digital cameras Answer: B Rationale: If there is a crop shortage, prices escalate, but consumers still maintain the same level of demand because food—particularly bread—is a necessity. 48. All of the following factors directly affect the elasticity of demand EXCEPT: A. a product’s other uses. B. inputs needed to manufacture the product. C. availability of substitute goods. D. price relative to a consumer’s purchasing power. E. product durability. Answer: B Rationale: Inputs at time of manufacture only indirectly affect the demand, if at all. 49. Which of the following would imply elastic demand? A. Price is low relative to purchasing power B. Nondurable product C. Low inflation rate D. Many substitute products E. All of these choices Answer: D Rationale: When there are many substitute products, the consumer can easily switch from one product to another, making demand elastic. The other situations make demand inelastic. 50. The greater the number of different uses for a product, the more _____ demand tends to be. A. elastic B. inelastic C. unitary D. volatile E. stable Answer: A Rationale: If a product has only one use, the quantity purchased probably will not vary as price varies. 51. What does “YMS” stands for? A. Yorkshire manufacturing sector. B. Yield management systems. C. Yes-man syndrome. D. Yardstick measurement scale. E. Year-end marketing services. Answer: B Rationale: The initialism “YMS” stands for yield management systems. 52. Yield management systems were first developed by which industry? A. The manufacturing industry. B. The airline industry. C. The retail industry. D. The healthcare industry. E. The automobile industry. Answer: B Rationale: Yield management systems were first developed by the airline industry. 53. Yield management systems are used to: A. determine the availability of product substitutes in complex industries that are experiencing rapid change B. profitably fill unused capacity C. predict necessary service levels to achieve revenue goals D. determine whether it is financially more feasible to buy a new product or repair a broken one E. create elastic demand for low-involvement products Answer: B Rationale: Yield management systems use complex mathematical software to profitably fill unused capacity by discounting early purchases, limiting early sales at these discounted prices, and overbooking capacity. 54. _____ use complex mathematical software to profitably fill unused capacity. A. Yield management systems B. Capacity correlation systems C. Service forecasting tools D. Service management systems E. Capacity management software Answer: A Rationale: Yield management systems use complex mathematical software to profitably fill unused capacity by discounting early purchases, limiting early sales at these discounted prices, and overbooking capacity. 55. Allstate has more than 1,500 price levels determined by complex algorithms that analyze 16 credit report variables, including late payments and card balances. Allstate is using a _____ to set prices. A. yield management system B. capacity correlation system C. service forecasting tools D. service management system E. capacity maintenance tool Answer: A Rationale: A yield management system is complex mathematical software used to profitably fill unused capacity. 56. Which of the following statements about yield management systems (YMS) is true? A. The first use of YMS was in the U.S. car industry as it looked for ways to compete with imports. B. YMS eliminate the problem of simultaneous production and consumption from services. C. YMS cannot be used by any other businesses but services. D. YMS are complex pricing systems used to set equilibrium pricing points. E. YMS are mathematically complex systems to make use of underutilized capacity and reduce the cost of perishability. Answer: E Rationale: YMS was first used in the airline industry, but it is now used by automobile manufacturers to make use of underutilized capacity. 57. Chad has calculated the sales volume at which his lemonade stand’s costs equal revenue. Over dinner, he announced to his family that he only needed to sell 50 glasses of lemonade at $5 per glass to cover all his costs (such as lumber and nails for the stand, lemons, and sugar). Which important factor has Chad excluded from his analysis? A. Fixed and variable cost determination B. Consumer demand C. Target return pricing D. Break-even analysis E. Market share Answer: B Rationale: Chad’s analysis only includes company costs and does not consider consumer demand. 58. Total variable costs divided by quantity of output equals: A. average total cost B. mean intermittent cost C. fixed cost D. marginal cost E. average variable cost Answer: E Rationale: This is the definition of average variable cost. 59. The two types of costs a marketer needs to consider when setting prices are: A. primary and secondary. B. variable and fixed. C. marginal and absolute. D. short term and long term. E. elastic and inelastic. Answer: B Rationale: A variable cost is a cost that varies with changes in the level of output (such as cost of materials), whereas a fixed cost does not change as output is increased or decreased (such as rent). 60. A cost that changes with the level of output is called a(n) _____ cost. A. liquidity B. variable C. fixed D. asset E. elastic Answer: B Rationale: An example of a variable cost is the cost of materials. 61. Which of the following is most likely to be a variable cost for an Internet retailer that sells spices, herbs, and seasonings to consumers? A. Annual lease on mixer used to blend seasonings B. Executive salaries C. Rent for building where spices and herbs are repackaged for consumers D. Workers’ insurance E. Postage for shipping spices and herbs Answer: E Rationale: Postage is the only item that varies depending upon the amount of units sold. 62. For a nail salon, the costs associated with the purchase of nail polish and other products like nail polish remover, sterilized equipment, laundry service for the towels, and the beverages given to customers, are all examples of _____ costs. A. marginal B. variable C. fixed D. promotional E. liquidity Answer: B Rationale: A cost that changes with the level of output is called a variable cost. 63. _____ costs do not change as output is increased or decreased. A. Asset B. Variable C. Fixed D. Symmetrical E. Status quo Answer: C Rationale: This is the definition of fixed costs. 64. Central Bark is a dog resort where pets are pampered. Which of the following is the BEST example of one of its fixed costs? A. Payment on the building used by Central Bark B. Dog biscuits C. Dog collars and leashes D. Bubble bath E. Advertisements in local magazines Answer: A Rationale: The payment on the building remains the same, no matter how many dogs are visit the resort. 65. Mitch owns an accounting agency. The monthly payment on the land he purchased for his business, the mortgage on his small office building, and his business license are all examples of _____ costs. A. marginal B. variable C. fixed D. promotional E. demand Answer: C Rationale: Fixed costs do not change as output changes. 66. _____ cost is the change in total costs associated with a one-unit change in output. A. Variable B. Intermittent C. Elastic D. Marginal E. Flex Answer: D Rationale: This is the definition of marginal cost. 67. Monthly output at Leisure-Time, Inc. changed from 12 to 13 prefabricated gazebos, and the total costs changed from $9,000 to $10,500. What is the marginal cost for this company? A. $1,500 B. $2,000 C. $1,200 D. $10,000 E. $12,000 Answer: A Rationale: Marginal cost is the change in total costs associated with a one-unit change in output. 68. When a seller determines the selling price by adding to cost an amount for profit and expenses not previously accounted for, the seller is using _____ pricing. A. profit maximization B. demand-oriented C. break-even D. target return E. markup Answer: E Rationale: Markup pricing does not directly analyze the costs of production; rather, is uses the cost of buying the product from the producer, plus amounts for profit and for expenses otherwise not accounted for. 69. The most popular method used by wholesalers and retailers in establishing a sales price is _____ pricing. A. markup B. status quo C. formula D. marginal revenue E. break-even Answer: A Rationale: Markup pricing does not directly analyze the costs of production; rather, is uses the cost of buying the product from the producer, plus amounts for profit and for expenses otherwise not accounted for. 70. Cowboy Malone’s Electric City pays a wholesaler $700 for a television and sells it to a customer for $1,500. The markup on the television is: A. $240 B. $160 C. $700 D. $800 E. $1,500 Answer: D Rationale: Markup is selling price minus cost: $1,500 – $700 = $800. 71. An office supply store can buy a desk for $300. If the store owner sells the desk for $450, what is the markup based on cost? A. 15 percent B. 20 percent C. 25 percent D. 33 percent E. 50 percent Answer: E Rationale: Price – Cost = Markup $450 – $300 = $150 $150 ÷ $300 = 50% markup 72. An office supply store can buy a desk for $300. If the store owner sells the desk for $450, what is the markup based on the selling price? A. 15 percent B. 20 percent C. 25 percent D. 33 percent E. 50 percent Answer: D Rationale: Price – Cost = Markup $450 – $300 = $150 $150 ÷ $450 = 33% markup 73. The difference between the retailer’s cost and the selling price is the: A. gross margin. B. markup percentage. C. profit. D. keystone. E. breakeven profit. Answer: A Rationale: Gross margin is the amount added to cost to determine price. 74. The owner of specialty kitchen retail store wants to determine what price she should put on a set of mixing bowls. They cost her $7. She desires a markup of 30 percent based on selling price. Which of the following is closest to the price she should charge her customers? A. $19 B. $12 C. $15 D. $10 E. $18 Answer: D Rationale: When desired markup is based on selling price, then selling price can be calculated as follows: Retail price = Cost ÷ (1 – Desired return on sales). 75. _____ is the practice of marking up prices by 100 percent (or doubling the cost to set the selling price). A. Margin pricing B. Key stoning C. Mark-on adding D. Formula doubling E. Symmetrical pricing Answer: B Rationale: This is the definition of key stoning. 76. Key stoning is: A. the practice of marking up prices by 100 percent. B. a method used for determining the point of elasticity. C. a plan for reducing marginal costs. D. the practice of maintaining variable costs at one-half of total fixed costs. E. a method of changing consumers’ perceptions about price. Answer: A Rationale: Key stoning simply doubles the cost. 77. The Nest is a retail store owned and operated by an interior designer. The markup on all items in the store is 100 percent over cost (or double the cost). In this case, we would say the designer uses: A. key stoning. B. target ROI pricing. C. break-even pricing. D. marginalizing. E. double sourcing. Answer: A Rationale: Key stoning is the practice of marking up prices by 100 percent (or doubling the cost to set the selling price). 78. Profit maximization occurs when: A. total costs equals average fixed revenue. B. average variable costs are larger than average total costs. C. total costs equal total variable costs. D. marginal variable costs equal average revenues. E. marginal revenue equals marginal cost. Answer: E Rationale: As long as the revenue of the last unit produced and sold is greater than the cost of the last unit produced and sold, the firm should continue manufacturing and selling the product, but maximum profit occurs when marginal revenue equals marginal cost. 79. _____ is the extra revenue associated with selling an additional unit of output. A. Average revenue B. Marginal revenue C. Marginal cost D. Net profit E. Average variable cost Answer: B Rationale: Marginal revenue is also defined as the change in total revenue with a one-unit change in output. 80. As long as the revenue of the last unit produced and sold is greater than the cost of the last unit produced and sold, a firm should: A. continue manufacturing. B. not use formula pricing. C. continue using price equilibrium. D. consider using sales maximization pricing. E. reach its break-even point very shortly. Answer: A Rationale: Diminishing returns have not set in, so the firm should continue manufacturing. 81. The point at which marginal cost and marginal revenue are equal always results in: A. maximization of elasticity. B. maximization of revenue. C. maximization of costs. D. maximization of profits. E. break-even equilibrium. Answer: D Rationale: Until the point where MR = MC, each unit of sales has contributed to additional profit; therefore, profit, not revenue or costs, has been maximized at MR = MC. 82. _____ determine what sales volume must be reached before the company’s total revenue equals total costs and no profits are earned. A. Marginal revenue estimates B. Price equilibrium analyses C. Break-even analyses D. Average total cost (ATC) figures E. Marginal costs of goods sold Answer: C Rationale: At the break-even point, costs are equal to revenue, and profit is zero. 83. The typical break-even model assumes a given fixed cost and a: A. variable per unit cost. B. constant inventory turnover. C. markup cost attained through key stoning. D. constant production schedule. E. constant average variable cost. Answer: E Rationale: Break-even quantity = Total fixed costs ÷ (Price – Average variable cost). 84. Fixed cost contribution equals: A. price times the average fixed cost. B. price plus the average variable cost. C. average variable cost plus average fixed cost. D. break-even quantity times price. E. price minus the average variable cost. Answer: E Rationale: Fixed cost contribution is what is left over after variable costs are covered, so it is equal to price minus the average variable cost. 85. Your Memory Lane creates custom art prints that use graphs and icons in a street scene to commemorate special occasions. Suppose that Your Memory Lane has priced its product at $350 per print. Further, it has determined that the company’s fixed cost is $12,500, with average variable costs per print of $250. What is the fixed cost contribution per print? A. $225 B. $100 C. $605 D. $2 E. $1 Answer: B Rationale: Fixed cost contribution is the price minus the average variable cost: $350 – $250 = $100. 86. Your Memory Lane produces custom-made art prints that include graphics and icons to celebrate life’s special moments. For example, on his wedding anniversary, David had an art print produced that celebrated highlights of his ten years with his wife, Kathy. Suppose that Your Memory Lane sells the custom artwork for $500. It estimates its average variable costs to be $200 per unit produced. It figures its fixed costs to be $900,000 per year. How many prints does it have to sell to break even? A. 2,000 prints B. 1,200 prints C. 3,000 prints D. 2,500 prints E. 6,000 prints Answer: C Rationale: Break-even quantity equals the total fixed costs ($900,000) divided by the fixed cost contribution per unit ($500 – $200 = $300). 87. Pet’s Eye View Digital Camera is a small, durable digital camera that pet owners can clip to their pets’ collars. A programmable timer takes pictures every few minutes, recording a photographic diary of the pet’s day. The camera sells for $25. The average variable cost for each camera is $10 and the total annual fixed costs for plant operation are $45,000. What is the break-even point in units? A. 1,800 B. 2,500 C. 3,000 D. 4,500 E. 5,000 Answer: C Rationale: Break-even quantity equals the total fixed costs ($45,000) divided by the fixed cost contribution per unit ($25 – $10 = $15). 88. Regency, Inc. makes disposable cap and gown sets for graduations. Each cap and gown set sells for $15. The average variable cost for manufacturing ten cap and gown sets is $100. Total fixed costs for the year equal $65,000. Calculate the break-even point in units. A. 650 B. 765 C. 1,300 D. 4,334 E. 13,000 Answer: E Rationale: Break-even point = $65,000 ÷ [15 – (100 ÷ 10)] = 13,000 89. Ceylon Express sells bottled pasteurized tea to retailers. It has the following revenues and costs: What is the annual break-even point in units for the company? A. 50,000 B. 250,000 C. 100,000 D. 166,667 E. 500,000 Answer: B Rationale: Break-even quantity is the total fixed costs ($50,000.00) divided by fixed cost contribution per unit ($0.20). 90. Which of the following statements describes a limitation associated with break-even analysis? A. It is sometimes difficult to ascertain whether a cost is fixed or variable. B. It requires the calculation of marginal revenue. C. It strictly considers demand. D. It assumes variable cost per item, which is difficult to calculate. E. It can only be expressed as a break-even point in dollar amounts. Answer: A Rationale: Not all costs are easily categorized because a cost may be fixed when viewed in the short term but variable when considered over a longer period of time. 91. Which of the following statements about pricing strategies throughout the product life cycle is NOT true? A. During product decline, prices may also decline until there is only one competitor left in the market. B. Price increases during the maturity stage are cost initiated instead of demand initiated. C. The maturity stage often brings about price decreases. D. Prices stabilize when the product enters the growth stage. E. With inelastic demand, price will be set low during the introduction stage. Answer: E Rationale: With inelastic demand, prices are set high at introduction. 92. Prices generally begin to stabilize: A. as a product enters the growth stage. B. when a product if first introduced onto the market. C. as a result of key stoning. D. when a product enters the decline stage of the life cycle. E. in hotly competitive markets. Answer: A Rationale: As the product enters the growth stage, prices generally begin to stabilize. 93. When Apple, Inc. developed and introduced the iPad, it was unique as it essentially combined a touch-based portable computer, a wireless marketplace, and an eBook reader. As such, in the short run, it seemed that demand for the product would be inelastic, with no real existing competition. The recommend pricing strategy in such a situation would be: A. low initial price, rising slightly when entering the growth stage. B. high initial price, falling slightly when entering the growth stage. C. high price, continuing through growth and maturity. D. low price, continuing through growth and maturity. E. low price initially, rising constantly through growth and into maturity. Answer: B Rationale: A high initial price is used when a new product faces little competition, needs to recoup research and development costs, and has inelastic demand. Prices will fall slightly when entering the growth stage. 94. As a product enters the growth stage, prices generally begin to stabilize. One reason for this is that: A. the product has begun to appeal to a broader market. B. most competitors have been eliminated from the market. C. manufacturers hope to recover their development costs quickly. D. the available supply decreases. E. All of the above are correct. Answer: A Rationale: Other reasons include competitors entering the market and economies of scale. 95. Kroger supermarkets will place well-known brands on the shelves at high prices while offering their own Kroger brand at lower prices. This practice is an example of: A. illegal pricing. B. selling against the brand. C. price pressurization. D. brand cutting. E. private label cannibalization. Answer: B Rationale: Selling against the brand with private labels causes sales of the higher-priced brands to decline. 96. Manufacturers can do all of the following to regain some control over the price their products are sold for at the retail level EXCEPT: A. require resellers to maintain prices in line with competitors’ prices. B. developing brand loyalty in consumers by delivering quality and value. C. avoiding doing business with price-cutting discounters. D. franchising. E. using an exclusive distribution system. Answer: A Rationale: Manufacturers can also package merchandise with the selling price marked on it or place goods on consignment. 97. Shopping bots: A. encourage a more creative use of advertising. B. link manufacturers, suppliers, and customers. C. create opportunities for prestige pricing. D. search the Web for the best price. E. create inelastic demand. Answer: D Rationale: Bot is short for robot, and shopping bots theoretically give pricing power to the consumer. 98. Which of the following statements about the Internet is true? A. The Internet has shifted all shopping power to consumers. B. Consumer reviews tend to be equal in quality. C. Business-to-business auctions on the Internet are likely to be more important than consumer auctions in the future. D. Fraud is not a problem on the Internet. E. Extranets are programs that search the Internet for the best price for a particular product. Answer: C Rationale: The Internet has shifted some, but not all, shopping power to consumers. Consumer reviews vary in quality. Fraud is a huge problem. 99. During the hot summer months, or the week before a new class starts if there is still space available, the Nick Price golf school in Orlando, Florida, offers a 25 percent reduction to get golfers during the off-season or those making a last-minute decision. This is an example of pricing strategy used as a(n): A. distribution tool. B. price enhancer. C. product strategy. D. direct sales tool. E. promotion strategy. Answer: E Rationale: Price is often used as a promotional tool to increase consumer interest. 100. Many consumers, especially when faced with an uncertain purchase decision, think that a high price: A. is a signal of quality. B. is an indication that consumers are being ripped off. C. will always lead to major price discounts to wholesalers and retailers that distribute it. D. is a sign of the company’s overall market share. E. indicates that the brand was slipping into the decline stage of the product life cycle but has had a sudden resurgence of growth. Answer: A Rationale: Numerous studies have shown that consumers equate high price with good quality. 101. David likes New Balance running shoes. However, when he stopped by the Foot Locker to buy a new pair of running shoes, he noticed that Nike had a new pair of running shoes that cost $350. To David, the higher price of the Nike shoe indicated that it would be a better pair of running shoes. This is an example of: A. premium pricing. B. price lining. C. prestige pricing. D. exclusive pricing. E. selective pricing. Answer: C Rationale: Prestige pricing is charging a high price to help promote a high-quality image. 102. When the Apple iPhone 3G was introduced, users could buy the “I Am Rich” app for a price of $1,000. The app did nothing but display a red gem on the iPhone’s screen. The description of the app stated that this red icon would remind you (and others you show it to) “that you were rich enough to afford this.” Six of the applications were sold before Apple, Inc. removed the app from iTunes. At the $1,000 price, the author of the app was using _____ pricing as part of his marketing approach. A. snob appeal B. prestige C. exclusive D. selective E. unique Answer: B Rationale: Prestige pricing strategy sets high prices to connote high product quality and exclusiveness. 103. Marketing managers who attempt to raise the quality image of their product by selling it at high prices are following a(n) _____ strategy. A. profit maximization B. market share C. maintained markup pricing D. prestige pricing E. investment asset Answer: D Rationale: Prestige pricing strategy sets high prices to connote high product quality. 104. Laurie knows little about cooking and does not want to spend the time to learn how to make a quiche. However, she has been asked to bring a quiche to an office retirement party. Not wanting to make a poor choice, she is likely to: A. intuitively make the right choice. B. avoid making a decision by not attending the party. C. buy the most expensive pre-made quiche (perhaps paying too much), guessing that the price is related to quality. D. research the product and buy the least expensive frozen quiche she can find. E. buy the least expensive frozen quiche because most consumers feel that price is not directly related to quality. Answer: C Rationale: Most consumers equate price and quality. 105. When Jerry took delivery of his brand new (and very expensive) Jaguar automobile, he was filled with pleasure and excitement. This is an example of the _____ effect associated with the price-quality relationship. A. durability B. allocative C. prestige D. hedonistic E. performance Answer: D Rationale: Hedonistic consumption refers to pursuing emotional responses associated with using a product, such as pleasure, excitement, arousal, and fun. 106. The dimensions of quality that are important to consumers include: A. versatility. B. serviceability. C. performance. D. ease of use. E. all of these choices. Answer: E Rationale: Durability and prestige are other dimensions. Tesla Motors “Going green” doesn’t have to be boring. The Tesla Roadster Sport is an electric car that goes from 0 to 60 in four seconds and drives more like a race car than an environmentally friendly ride. But that level of performance will set you back $128,500. As of 2009, Silicon Valley–based Tesla Motors, Inc. was the only company offering highway-compatible electric cars. Most Roadster Sport buyers are car enthusiasts and are buying them for the “fun toy” aspect of having an electric car rather than for environmental reasons. 107. Refer to Tesla Motors. The price of the Roadster was set so that the marginal revenue equals marginal cost. This represents a _____ approach. A. profit maximization B. market share pricing C. demand-oriented pricing D. sales maximization E. status quo pricing Answer: A Rationale: Profit maximization means setting the price so that marginal revenue equals marginal cost. 108. Refer to Tesla Motors. If Tesla had assets of $5 million and net profits after taxes of $550,000, what is Tesla’s return on investment? A. 1 percent B. 9 percent C. 11 percent D. $14,135 E. $4,450,000 Answer: C Rationale: Return on investment = Net profit after taxes ÷ Total assets OR Return on investment = $550,000 ÷ $5,000,000 109. Refer to Tesla Motors. What is the fixed cost contribution for the Roadster given average variable costs of $50,500? A. $50,500 B. $78,000 C. $128,500 D. $179,000 E. $500,000 Answer: B Rationale: Fixed cost contribution is the price minus the average variable cost: $128,500 – $50,500 = $78,000. 110. Refer to Tesla Motors. If total fixed costs are $23,400,000 and the average variable cost is $50,500, how many Roadsters must Tesla sell to break even? A. 130 B. 182 C. 250 D. 300 E. 463 Answer: D Rationale: Breakeven quantity = Total fixed costs ÷ (Price – Average variable costs) OR Break-even quantity = 23,400,000 ÷ (128,500 – 50,500) OR Break-even quantity = Total fixed costs ÷ Fixed cost contribution (23,400,000 ÷ 78,000) 111. Refer to Tesla Motors. Say Tesla set the price of the Roadster high because the company wanted to promote a high-quality image. What type of pricing does this represent? A. Prestige pricing B. Elite pricing C. Penetration pricing D. Quality pricing E. Complete pricing Answer: A Rationale: Prestige pricing is charging a high price to help promote a high-quality image. Specialty Cakes Imagine that you’re planning an after-symphony fund-raising party, and you need a life-size grand piano cake. Or, you are a developer proposing a new shopping center to a group of investors, and you want to serve a cake shaped like an architectural rendition of the center. Is this impossible? No, you just need to contact Cecilia Villaveces Cakes. She actually built a life-size grand piano for a gala in Macon, Georgia. You can expect to pay anywhere from $75 to $10,000 for one of Cecilia’s artistic creations, depending on complexity of design and size. She uses only the best ingredients, and no two cakes are ever quite alike. 112. Refer to Specialty Cakes. Although many factors determine the prices charged by Cecilia Villaveces Cakes, the two primary determinants are: A. costs of manufacturing and distribution costs. B. stage of the product life cycle and costs to the consumers. C. the demand for the good and the cost to the seller. D. demand by the consumer and perceived quality. E. distribution and promotion strategies used by the cake maker. Answer: C Rationale: If there was reduced demand, then Cecilia would have to lower prices. Also, she must charge for the time, energy, and resources that go into each cake. 113. Refer to Specialty Cakes. Many party planners in the Southeast will only use Cecilia Villaveces Cakes at their parties––no matter what the price is. They know that Cecilia’s cakes can make a party a success. Moreover, the cakes are what people remember most about the parties. From this description, you should assume Cecilia Villaveces Cakes have a(n): A. elastic demand. B. unitary elasticity. C. inelastic supply. D. inelastic demand. E. elastic supply. Answer: D Rationale: Price increases do not decrease demand for cakes. 114. Refer to Specialty Cakes. Which of the following is the BEST example of a fixed cost for Cecilia Villaveces Cakes? A. Eggs, butter, sugar B. Delivery costs C. Part-time employees D. Electricity consumption E. Food preparation licenses Answer: E Rationale: Delivery costs, electricity consumption, and use of part-time employees will vary according to the job. 115. Refer to Specialty Cakes. Which of the following is the BEST example of a variable cost for Cecilia Villaveces Cakes? A. Life insurance on Cecilia B. Flour and sugar C. Ovens used for cooking cakes D. Business licenses E. Interest payments to the bank Answer: B Rationale: Only the consumption of flour and sugar would vary from cake to cake. 116. Refer to Specialty Cakes. To set the price of her cakes, Cecilia simply doubles her costs, which often include several hours of labor and expensive raw materials. This method of price setting is called: A. mark-on pricing. B. premium pricing. C. key stoning. D. add-on pricing. E. superimposed pricing. Answer: C Rationale: The practice of marking prices up 100 percent is called key stoning. 117. Refer to Specialty Cakes. There are many occasions for which people may need to buy a cake, but most people do not have the time or interest to learn about cakes and their bakers. These people who do not know about the quality of the Cecilia Villaveces Cakes might choose them because they: A. equate price and quality. B. know cakes are in the mature stage of their product life cycle. C. realize that this is a monopolistic industry. D. believe that there is not a relationship between price and quality. E. desire value-added services. Answer: A Rationale: Most consumers equate price and quality. American Girl Doll The American Girl catalog began as a concept to introduce today’s girls to those who lived in the past. Each historically accurate doll is carefully crafted and dressed and has books to describe her life. For example, Kristen is an 1854 pioneer girl who is growing up in Minnesota. Her story begins with her long sea voyage from Sweden. The basic doll, dressed in a calico dress and striped apron plus the hardcover story of how she got to Minnesota, costs $90.00. Six more hardback books of Kristen’s life are available for $74.95. Kristen’s nightgown costs $20.00, and a matching one for the doll owner is an additional $38.00. Buy both together and the price is only $50.00. A hand-painted wooden bed and trunk for Kristen are available for $213.00. Shipping costs vary with the price of the merchandise ordered. 118. Refer to American Girl Doll. What is the revenue to American Girl if it sells 20 basic Kristen dolls? A. $90.00 B. $100.95 C. $427.95 D. $1,800.00 E. $3,600.00 Answer: D Rationale: Revenue equals price times number of items sold. 119. Refer to American Girl Doll. American Girl is the primary seller of historically accurate dolls with accompanying books in a market where there is very little competition. It has no cash flow problems and is not interested in maximizing its sales. From this information, you should know American Girl has _____ pricing objectives. A. status quo B. psychological C. profit-oriented D. sales-oriented E. supply-derived Answer: C Rationale: Because it has no competitors to speak of, status quo objectives are unlikely. It is not interested in maximizing its sales, so sales-oriented objectives are also not likely. 120. Refer to American Girl Doll. The popularity of the American Girl dolls is so great that an increase in the price of the basic Kristen doll and books by 5 percent will not significantly affect the demand for the product. The means that the demand for the American Girl doll is: A. elastic. B. derived. C. a multiplier. D. inelastic. E. symmetrical. Answer: D Rationale: Inelastic demand means that an increase or decrease in price will not significantly affect demand for the product. 121. Refer to American Girl Doll. Based on the information in the narrative, which of the following factors is most likely to affect the elasticity of demand for the doll? A. The absence of substitutes B. The existence of complementary products C. The price relative to purchasing power D. Product durability E. A variety of alternative uses for the product Answer: A Rationale: There is nothing like the American Girl doll in terms of historical accuracy. 122. Refer to American Girl Doll. In terms of the costs of producing the doll and its accessories, the salary of the graphic designer who does the layout for the American Girl catalog is a(n): A. markup cost. B. variable cost. C. fixed cost. D. derived cost. E. elastic cost. Answer: C Rationale: Salaries are considered fixed costs. 123. Refer to American Girl Doll. In terms of producing the doll and its accessories, the calico fabric used to make Kristen’s dress is an example of a(n): A. markup cost. B. variable cost. C. fixed cost. D. derived cost. E. elastic cost. Answer: B Rationale: How much fabric used depends on how many dolls made. 124. Refer to American Girl Doll. You can buy a doll at Walmart for $5.99. The high price of the American Girl doll is used to promote a high-quality image. American Girl uses a _____ pricing strategy. A. markup B. demand-based C. prestige D. penetration E. supply-derived Answer: C Rationale: Prestige pricing is charging a high price to help promote a high-quality image. Smelly Fruit At first glance, there is little to like about the durian. The durian is a fruit popular in Thailand that is spiky outside and stinky on the inside. When confronted with the durian for the first time, the Wall Street Journal recently reported that westerners often describe its distinctive sulphurous smell with words like stinky socks and manure. However, the durian is so popular in Thailand that one variety of the durian fruit, called the Kan Yao, has been selling for as high as $200. Even at that price, supply of the Kan Yao cannot keep up with demand. There are approximately 30 varieties of the durian grown in Thailand, with the most plentiful selling in the $15 range. The yellow flesh of the durian, the part you eat, has very powerful smell but possesses a sweet, nutty taste that Thais cannot seem to get enough of at any price. 125. Refer to Smelly Fruit. If it cost a Thai farmer $100 to produce and $25 to market the Kan Yao durian that she sells for $200 at the marketplace, her revenue, for each durian sold, would be: A. $125 B. $200 C. $25 D. $100 E. $325 Answer: B Rationale: Revenue is the price charged to customers multiplied by the number of units sold (in this case, one). 126. Refer to Smelly Fruit. Suppose that a Thai farmer sells ten Kan Yao durians in the marketplace at the going rate of $200 each. If it cost a Thai farmer $125 to produce and market the Kan Yao durian that she has sold, the difference between these two numbers ($75), times the number sold (ten), represents the farmer’s: A. ROI B. revenue C. profit D. returns E. COGS Answer: C Rationale: What’s left over after covering all costs would be the farmer’s profit. 127. Refer to Smelly Fruit. Suppose that you have decided to buy land in Thailand and become a durian producer. You see that the customary price for a Kan Yao is $200, so that is the price you decide to charge for your durian crop. This suggests you are using a _____ approach to setting your price. A. profit maximization B. market share C. return on investment (ROI) D. sales maximization E. status quo Answer: E Rationale: Status quo pricing seeks to maintain existing prices or to meet competition’s price. 128. Refer to Smelly Fruit. At $200 per Kan Yao, demand for the fruit appears to be higher than supply. Suppose that at a price of $225, the amount demanded exactly meets the amount farmers are willing and able to supply. In this case, the $225 price would be considered the _____ price. A. price equilibrium B. sales maximization C. profit maximization D. ROI maximization E. yield management Answer: A Rationale: When demand and supply are equal, you have met the price equilibrium price. 129. Refer to Smelly Fruit. Over the past two years, the price for Kan Yao durian fruit has increased by 50 percent. If the amount sold has remained almost constant, we would say that demand is: A. elastic B. inelastic C. unitary D. highly elastic E. moderately elastic Answer: B Rationale: Inelastic demand means that when prices go up it does not significantly affect sales. 130. Refer to Smelly Fruit. The durian fruit is a unique product. To many people in Thailand, no other fruit tastes or smells anything like the durian. Its uniqueness, in the eyes of the customer, would tend to have which of the following? A. It would have little effect. B. It would tend to make the durian more price elastic. C. It would tend to make the durian more price inelastic. D. It would tend to raise the price in comparison to purchasing power. E. It would tend to impact the durian’s stage in the PLC. Answer: C Rationale: The lack of availability of substitutes leads to inelastic demand. ESSAY 1. Define price and discuss the two roles price plays in the evaluation of product alternatives. Answer: Price is that which is given up in an exchange to acquire a good or service. Price plays two roles in the evaluation of product alternatives: as a measure of sacrifice and as an information cue. Since price is “that which is given up,” it means what is sacrificed to get a good or service. In the United States, that usually means money, but it can mean other things as well. It may also be time lost while waiting to acquire the good or service. The information effect of price implies that many consumers use price as an indication of quality. That is, higher quality equals higher price. The information effect of price may also extend to favorable price perceptions by others because higher prices can convey the prominence and status of the purchaser to other people. 2. One of the most stressful and pressure-filled tasks of the marketing manager is attempting to set the right price. Specify three aspects of the current pricing environment in consumer markets that have contributed to the difficulty in setting correct prices. Answer: Five factors in the consumer products market are listed in the text: (1) confronting a flood of new products, potential buyers carefully evaluate the price of each one against the value of existing products; (2) the increased availability of bargain-priced private and generic brands has put downward pressure on overall prices; (3) many firms are trying to maintain or regain their market share by cutting prices; (4) the Internet has made comparison shopping easier; and (5) the United States was in a recession from late 2007 until late 2009. 3. List the three categories of pricing objectives and then two specific strategies in each category that a marketer could implement to achieve those objectives. Answer: PROFIT-ORIENTED PRICING OBJECTIVES include: • profit maximization • satisfactory profits • target return on investment SALES-ORIENTED PRICING OBJECTIVES include: • market share • sales maximization (dollar or unit sales) STATUS QUO PRICING OBJECTIVES include: • maintaining the existing price • meeting the competition 4. Last quarter, Abingdon Company sold 1,000 decorative decals for $1 each; Cedar Decaliania sold 200 decorative decals at $4 each; Creative Decals sold 500 decals at $2 each; and Donnelly, Inc. sold 300 decals for $4 each. Assuming that the four companies are the only firms competing in the decorative decal market, calculate unit and dollar market share for each company for last quarter. For each company, which market share figure might be used in an advertisement for that company? Answer: The following calculation table shows the resultant unit and dollar shares. Market share should be expressed in percentage points. Firms often state market share in terms that are most flattering to the company. Abingdon would express unit shares in an advertisement, while Cedar Decaliania and Donnelly would use dollar shares. Creative Decals’ market share is 25 percent for either calculation. See Exhibit 19.1. 5. List the two primary determinants of price. What other factors can affect price setting? Answer: The price established depends primarily on (1) DEMAND for the good or service and (2) the COST to the seller for that good or service. Other factors that would influence price include distribution strategies, promotion strategies, perceived quality, demands of large customers, the Internet, and the stage of the product life cycle. 6. The daily demand for bottled water is 35 bottles when the price is set at $1. However, if the price is raised to $5, the demand is only five bottles. The bottled water producer is willing to supply 40 bottles if the price is set at $5 per bottle but will only supply 10 bottles if the price is set at $2. Draw the supply and demand curves for the water bottles on the graph below. Label each curve and each axis. At what level does equilibrium occur? What are the areas of surplus and shortage? Answer: The vertical axis should be labeled as price, and the horizontal axis should be labeled as quantity. The demand curve should show a negative slope, crossing the positively sloped supply curve at the equilibrium point of $3 and 20 units. Surplus occurs in the area between the curves and above the equilibrium point, while shortage occurs below the equilibrium point. 7. Define elasticity of demand and compare and contrast the three types of demand: elastic, inelastic, and unitary. What would the demand curve for elastic and inelastic demand look like when graphed? Answer: Elasticity of demand refers to consumers’ responsiveness or sensitivity to changes in price. ELASTIC DEMAND occurs when consumers buy more or less of a product when the price changes. The demand curve is almost horizontal or exactly horizontal if demand is perfectly elastic. INELASTIC DEMAND means that an increase or a decrease in price will not significantly affect demand for the product. The demand curve is almost vertical or exactly vertical if demand is perfectly inelastic. UNITARY ELASTICITY exists when the increase in sales exactly offsets a decrease in prices, so total revenue remains the same. 8. List five factors that affect elasticity of demand and briefly describe how each affects demand. Answer: AVAILABILITY OF SUBSTITUTES. When many product substitute products are available, the consumer can easily switch from one product to another. This makes demand more elastic. PRICE RELATIVE TO PURCHASING POWER. If a price is so low that it is an inconsequential part of an individual’s budget, demand will be inelastic. PRODUCT DURABILITY. Consumers often have the option of repairing durable products rather than replacing them, thus prolonging their useful life. In other words, people are sensitive to the price increase, and demand is elastic. A PRODUCT’S OTHER USES. The greater the number of different uses for a product, the more elastic demand tends to be. As price varies for a product with a wide variety of applications, substitutability becomes an issue. RATE OF INFLATION. Recent research has found that when a country’s inflation rate (the rate at which the price level is rising) is high, demand becomes more elastic. In other words, rising price levels make consumers more price sensitive. 9. Explain yield management systems (YMS) and discuss the types of industry where they are most appropriate. Answer: Yield management systems (YMS) are a technique for adjusting prices that use complex mathematical software to profitably fill unused capacity by discounting early purchases, limiting early sales at these discounted prices, and overbooking capacity. YMS are used to raise prices to maximize revenues. They were first used in the service industries—specifically the airline industry—but have been recently discovered by manufacturers and others as a way to make more efficient use of resources. 10. What are the problems associated with the use of a cost-based pricing strategy? What contribution does cost make to the setting of prices? Answer: Setting prices based solely on costs ignores demand and other important factors such as marketing mix components or consumer needs and wants. Prices determined strictly on the basis of cost may be too high for the target market, thereby reducing or eliminating sales. Cost-based prices may also be too low, causing the firm to earn a lower return than it should. Costs play an important role in price setting, however. Costs serve as floor below which a good or service must not be priced in the long run. 11. What is the difference between fixed and variable costs? Give examples of each type of cost. Answer: FIXED COSTS are those expenses of the firm that are stable and do not change with the level of output. Examples include rent and executive salaries. VARIABLE COSTS are those expenses of the firm that vary directly with the level of output. Examples of variable costs associated with output include cost of materials, direct labor, and packaging. 12. Calculate answers for the following scenarios if retailer markups are based on their selling price: A. A retailer sells a set of measuring cups for $2.50 after adding $0.50 to the original cost. What is the markup percentage? B. The cost of a food blender for the retailer is $40 and the retailer applies a markup of $60. What is the retail markup percentage? C. A retailer marks up all products by 20 percent. If a set of glasses costs the retailer $10, what will be the final selling price? D. A retailer marks up all products by 75 percent. If the selling price of a set of plastic bowls is $4, what was the cost to the retailer? Answer: The dollar markup is calculated as selling price minus cost, and percentage markup can be calculated by dividing dollar markup by selling price. 13. What is marginal revenue? Based on the provided schedule from the Chesapeake Bay Swing Company, at which quantity should Chesapeake Bay stop producing additional swings? Answer: Marginal revenue is the extra revenue associated with selling an extra unit of output. As long as the revenue of the last unit produced and sold is greater than the cost of the last unit produced and sold, the firm should continue manufacturing. The student should find the point in the schedule where marginal revenue is equal to marginal cost, which is at five units. The firm would not stop producing at four units, even though the addition of the fifth unit did not add any profits. This is because the firm could not determine an increase or decrease in profits after the fourth unit until an additional unit had been produced. 14. What is a break-even point? The Catera Company makes and sells cotton candy machines. What is the break-even volume for Catera machines in units? Answer: A break-even point is that level of units sold at a certain price at which no profit or loss is incurred. Break-even analysis determines what sales volume must be reached for a product before the company breaks even (total costs are equal to total revenue).Using the break-even formula indicates that Catera must sell 800 cotton candy machines to break even. Fixed cost contribution = Selling price – Average variable cost Fixed cost contribution = $600 – $350 = $250 Break-even quantity = Total fixed costs ÷ Fixed cost contribution Break-even quantity = $200,000 ÷ $250 = 800 units 15. Name two advantages and two disadvantages associated with the use of break-even analysis. Answer: Advantages of using break-even analysis include that it (1) provides a quick estimate of how much the firm must sell to break even, (2) provides information about how much profit can be earned if a higher sales volume is obtained, and (3) reduces the dependence on marginal cost and revenue data, which are frequently unavailable. Disadvantages of using break-even analysis include that (1) it is difficult to know whether a cost is fixed or variable and (2) failure to account for the concept of demand. For example, a firm may not be able to sell the break-even number of units because demand may be limited to fewer units. 16. As a product moves through its life cycle, the demand for the product and the competitive conditions tend to change. For each stage in the product life cycle, discuss pricing strategies appropriate for that stage. Answer: INTRODUCTION. Management usually sets prices high during the introductory stage. One reason is that it hopes to recover its development costs quickly. In addition, demand originates in the core of the market (the customers whose needs ideally match the product’s attributes) and thus is relatively inelastic. GROWTH. As the product enters the growth stage, prices generally begin to stabilize for several reasons. First, competitors have entered the market, increasing the available supply. Second, the product has begun to appeal to a broader market, often lower-income groups. Finally, economies of scale are lowering costs, and the savings can be passed on to the consumer in the form of lower prices. MATURITY. Maturity usually brings further price decreases as competition increases and inefficient, high-cost firms are eliminated. The manufacturers that remain in the market toward the end of the maturity stage typically offer similar prices. At this stage, price increases are usually cost initiated, not demand initiated. DECLINE. The final stage of the life cycle may see further price decreases as the few remaining competitors try to salvage the last vestiges of demand. When only one firm is left in the market, prices begin to stabilize. In fact, prices may eventually rise dramatically if the product survives and moves into the specialty goods category. 17. Explain how the Internet and extranets affect price. In particular, how do shopping bots and business-to-business auction Web sites affect pricing? Answer: The Internet and extranets are linking people, machines, and companies around the globe—and connecting sellers and buyers as never before. These links are enabling buyers to compare products and prices, putting them in a better bargaining position. A shopping bot is a program that searches the Web for the best price for a particular item that you wish to purchase. Shopping bots theoretically give pricing power to the consumer. The business-to-business auction world is shifting from haggling over prices to niggling over parameters of the deal. Warranties, delivery dates, transportation methods, customer support, financing options, and quality have all become bargaining chips. 18. Discuss how consumers use the price–quality relationship to evaluate goods and explain how marketers can take advantage of this consumer response. Answer: Consumers tend to rely on price as an indicator of product quality; that is, a higher price indicates higher quality in the form of better materials, more careful workmanship, or higher service levels. Conversely, lower price indicates lower quality, as illustrated by the adage, “you get what you pay for.” Marketers can take advantage of the price–quality phenomenon by increasing the price of the product to enhance the image of their product. This is known as a prestige pricing strategy. Chapter 20—Setting the Right Price TRUE/FALSE 1. The first step in setting the right price for a new product is to estimate demand, costs, and profits. Answer: False Rationale: The first step in setting the right price is to establish pricing goals. 2. All pricing objectives have trade-offs that managers must weigh. Answer: True Rationale: Different pricing objectives, such as profit maximization, sales growth, or market share expansion, involve trade-offs in terms of profitability, market positioning, and customer perception. 3. After estimating how much market share and profit can be earned at each possible price point, managers should establish price goals and determine corresponding costs for each price. Answer: False Rationale: After establishing pricing goals, managers should estimate total revenue at a variety of prices. Next, they should determine corresponding costs for each price. They are then ready to estimate how much profit, if any, and how much market share can be earned at each possible price. 4. Most companies do a very good job of doing research to create a price strategy. Answer: False Rationale: In fact, only about 15 percent of companies surveyed conduct serious pricing research to support the development of an effective pricing strategy. 5. Penetration pricing means charging a relatively low price for a product as a way to reach the mass market. Answer: True Rationale: Penetration pricing aims to attract customers quickly by setting a lower price than competitors, often to gain market share rapidly. 6. It makes the most sense to use price skimming when an above-average price makes the market hesitant to buy the product. Answer: False Rationale: It makes the most sense to use price skimming when the market is willing to buy the product even though it carries an above-average price. 7. Procter & Gamble entered the electric toothbrush market with the Crest Spinbrush at a price considerably lower than those of lesser-known competitors. It used penetration to gain market share. Answer: True Rationale: This strategy describes penetration pricing, where a company sets a low initial price to penetrate the market quickly and gain market share. 8. Sometimes multinational firms will follow a penetration strategy in developed countries and a skimming strategy in developing countries. Answer: False Rationale: The opposite is true. 9. There are two limousine services that drive customers from communities in North Georgia to the Atlanta airport. Whenever one reduces its fare, its competitor reduces its fares by the same amount. This is an example of status quo pricing. Answer: True Rationale: Status quo pricing involves matching competitors' price changes without initiating pricing innovations or changes independently. 10. In the United States, price fixing is only illegal in some instances. Answer: False Rationale: Price fixing is illegal under the Sherman Act and the Federal Trade Commission Act. Price fixing is one area where the law is quite clear, and the Justice Department’s enforcement is vigorous. 11. Price discrimination can sometimes be justified if certain criteria are not met. Answer: True Rationale: Price discrimination can be legally justified if it is based on differences in production costs, market segmentation, or the nature of the product or service being offered. 12. A winery that makes a huge profit on merlot wines may lower its price on pinot noir wines to cause damage to wineries that only produce pinot noir. This is an example of predatory pricing. Answer: True Rationale: Predatory pricing involves intentionally lowering prices to harm competitors and gain market share, which can be illegal under antitrust laws. 13. Manufacturers know the general price level they can expect when establishing a product price. This general price is called the base price. Answer: True Rationale: The base price refers to the general expected price level that manufacturers consider when setting initial prices for their products. 14. An awning manufacturer that is allowed to deduct 3 percent from its total bill if it pays by a specific date is receiving a promotional discount. Answer: False Rationale: This is describing a cash discount. 15. Functional discounts are typically calculated as the wholesale price times the accumulated margin percentages. Answer: False Rationale: Functional discounts are typically a percentage discount from the base price. 16. Sears and John Deere run major sales on their lines of riding lawn mowers every fall. This is an example of a seasonal discount. Answer: True Rationale: Seasonal discounts involve temporary price reductions offered during specific times of the year to stimulate sales, such as during seasonal peaks like fall. 17. Rebates involve a cash refund for the purchase of a product during a specific period. Answer: True Rationale: Rebates offer consumers a partial refund on a purchased product after they provide proof of purchase within a specified period. 18. A markdown allowance is a payment to a dealer for promoting the manufacturer’s products. Answer: False Rationale: This is the definition of a promotional (or trade) allowance. 19. The basic assumption with price skimming is that the firm is customer driven, seeking to understand the attributes customers want in goods and services they buy and the value of that bundle of attributes to customers. Answer: False Rationale: This is the basic assumption for value-based pricing. 20. Evergreen Lighting, a manufacturer of decorative, energy-efficient lighting products, requires its buyers to pay for the cost of transportation from the manufacturing site to their place of businesses. Evergreen Lighting uses FOB origin pricing. Answer: True Rationale: FOB (Free on Board) origin pricing means that the buyer assumes responsibility for transportation costs from the seller's location, typically used to indicate when ownership of goods transfers from seller to buyer. 21. Uniform delivered pricing is illegal because it discriminates against buyers who are located close to the point of shipping because they pay the same amount as buyers located far from the point of shipping pay. Answer: False Rationale: This geographic pricing practice is not illegal. 22. Flexible pricing enables a seller to close a sale with price-conscious consumers. Answer: True Rationale: Flexible pricing allows sellers to adjust prices based on customer segments, market conditions, or negotiation, which can help attract and close sales with price-sensitive consumers. 23. A catalog retailer offers three styles of khaki pants at three price levels. The special pricing tactic used by the catalog retailer is best described as variable psychological pricing. Answer: False Rationale: The catalog retailer is using price lining as a pricing tactic. 24. Loss-leader pricing is a price tactic that tries to get consumers into a store through false or misleading price advertising. Answer: False Rationale: Leader pricing, also called loss-leader pricing, is a price tactic in which a product is sold near or even below cost in the hope that shoppers will buy other items once they are in the store. 25. One example of price bundling occurs when Nintendo sells the Wii Fit balance board with the Wii game system at a lower price than the total price of each if bought separately. Answer: True Rationale: Price bundling involves offering two or more products or services together at a lower combined price than if purchased separately, encouraging customers to buy both items. 26. Kiddieland Amusement Park charges customers an admission fee of $10. Customers must pay 50 cents for each ride they want to ride while inside the park. This is an example of two-part pricing. Answer: True Rationale: Two-part pricing involves charging customers a fee for the basic service (admission fee in this case) and then charging an additional fee for each unit of service used (rides in this example). 27. Rag fibers for paper and cotton seeds for cottonseed oil are two by-products of the cotton textile industry. Because these products are produced together, they are complementary products. Answer: False Rationale: Complementary products are those that are consumed together, whereby an increase in the sale of one causes an increase in demand for the other. 28. More and more businesses are adopting consumer penalties—extra fees paid by consumers for violating the terms of a purchase agreement. Answer: True Rationale: Consumer penalties are additional charges imposed on consumers for failing to meet the terms or conditions of a purchase agreement, becoming increasingly common in various industries. 29. Costs that are shared in the manufacturing and marketing of several products in a product line are called joint costs. Answer: True Rationale: Joint costs are costs incurred in the production and marketing of multiple products within a product line, which are shared among those products based on a common production process or marketing effort. 30. Escalator pricing and price shading are two examples of cost-oriented pricing tactics. Answer: False Rationale: Escalator pricing is an example of a cost-oriented pricing tactic; price shading is an example of a demand-oriented pricing tactic. 31. Many businesses find recessions to be an excellent time to build market share through the use of price shading. Answer: False Rationale: Recessions are an excellent time to build market share through value-based pricing, bundling, and unbundling. MULTIPLE CHOICE 1. The marketing manager of Icruise.com (a travel Web site targeted to consumers who want a luxury vacation) finds that the firm can gain market share and become the industry leader if it slashes prices by 50 percent during the month of December. However, the VP of finance is committed to reporting a 25 percent return on investment at all times. This conflict illustrates: A. a need to eliminate low-profit products. B. a lack of corporate concentration on the marketing concept. C. how pricing operates in a mature marketplace. D. the need for trade-offs in pricing objectives. E. how target markets can be ignored. Answer: D Rationale: Different individuals in an organization may have pricing objectives that are not mutually compatible and will involve trade-offs. 2. After establishing pricing goals, managers should estimate total revenue at a variety of prices. Next, they should _____. Only after performing this task are they are ready to estimate how much profit and how much market share can be earned at each possible price. A. choose the ROI target B. determine corresponding costs for each price C. estimate industry supply D. implement pricing segmentation E. establish geographic pricing heuristics Answer: B Rationale: Managers will need to know costs to determine profit. 3. Which of the following is a pricing policy whereby a firm charges a high introductory price, often coupled with heavy promotion? A. Penetration pricing B. Price skimming C. Price capping D. Profit pricing E. Price maximization Answer: B Rationale: This is the definition of price skimming, which is sometimes called a “market-plus” approach to pricing because is denotes a high price relative to the prices of competing products. 4. A 16-ounce bottle of Prairie Herb vinegar sells for $4.95, and a 16-ounce bottle of Heinz vinegar costs $1.05. Prairie Herb vinegar is new to the market, perceived to be of higher quality, and provides a unique flavor to foods even though it is used in the same way as Heinz vinegar. Prairie Herb vinegar is most likely using a _____ policy. A. penetration pricing B. status quo pricing C. price skimming D. bundling cost pricing E. geodemographic pricing Answer: C Rationale: Price skimming is common for products in the introductory stage of their product life cycle. 5. In which of the following countries is Procter & Gamble MOST likely to sell razor blades using a penetration pricing strategy? A. the United States B. France C. Bangladesh D. Japan E. Canada Answer: C Rationale: Sometimes multinational firms will follow a skimming strategy in developed countries and a penetration strategy in developing countries. 6. A shortage of blood for transfusions for injured animals has resulted in the introduction of a synthesized product called Oxyglobin, which can be used effectively as a blood replacement. The manufacturer of the product has put a high price on the product in order to recoup its research and development costs. The manufacturer of Oxyglobin is using a _____ policy. A. price banding B. penetration pricing C. price lining D. bundling costs E. price skimming Answer: E Rationale: Price skimming is a pricing policy whereby a firm charges a high introductory price. 7. The price skimming strategy is sometimes called a “market-plus” approach to pricing because it denotes a high price relative to the prices of competing products. This strategy works best when: A. competition is abundant. B. revenues are equal to expenses. C. supply is greater than demand. D. production capacity is large and flexible. E. The product is perceived as having unique advantages. Answer: E Rationale: Companies often use this strategy for new products when the product is perceived by the target market as having unique advantages. 8. When the Mosquito Magnet was introduced, it was designed to rid the immediate area of mosquitoes and other annoying insects. The technology for the Mosquito Magnet had taken years to develop. It is a patented grill-like apparatus that emits carbon dioxide to attract bugs to a fan that draws them into the device where they die. What type of pricing policy would you recommend the company use to introduce this product to the market? A. Status quo pricing B. Penetration pricing C. Price skimming D. Flexible pricing E. Leader pricing Answer: C Rationale: The price skimming strategy will recoup the research and development costs quickly. Also, patents will limit or prohibit direct competition. 9. For which of the following situations would a price skimming strategy be most appropriate? A. The addition of a new comic book series with a gay protagonist B. The introduction of a new brand of bottled water C. The elimination of demand for low-wattage light bulbs D. The introduction of a unique, roomy automobile model that has extremely low energy and fuel costs E. The introduction of a Barbie Olympic champion doll by Mattel and the International Olympic Committee Answer: D Rationale: The automobile will justify a price skimming strategy because the manufacturer will need to recoup research and development costs, and it will take several years for the competition to catch up. 10. The DCS Stainless Steel Gas Grill for outside cooking costs $3,995. The market for a grill that could easily replace a kitchen range is limited even though a lot of people have seen articles about this grill in cooking magazines and in the cooking section of newspapers. There is no potential competitor for this grill. The _____ strategy is probably best. A. price skimming B. penetration pricing C. status quo D. cost bundling E. price lining Answer: A Rationale: Like products in the introductory stage of the product life cycle, this grill has no competition. 11. When a firm introduces a new product at a relatively low price because it hopes to reach the mass market, it is following a _____ strategy. The low price is designed to capture a large share of a substantial market and produce lower production costs. A. penetration pricing B. price-insensitive demand C. price skimming D. price elasticity E. cost bundling Answer: A Rationale: Penetration pricing means charging a relatively low price for a product in order to reach the mass market. 12. Marketers must take care when using _____ since a lower price often signals to consumers that product quality is also low. A. price skimming B. status quo pricing C. penetration pricing D. unbundling E. cost sharing Answer: C Rationale: Penetration pricing uses low prices to gain market share. 13. The market for turkey products is large. If a major producer of turkeys were to introduce a boneless fresh turkey wrapped around savory dressing, most of the large market for this new product would be aware of its existence. The market is price sensitive, and there is some potential competition. The appropriate strategy would be: A. price skimming. B. penetration pricing. C. status quo. D. cost bundling. E. price lining. Answer: B Rationale: The market for the new product is price sensitive and has greater competitive pressure. 14. Jones Soda Company and Big Sky Brands have introduced Jones Soda Carbonated Candy, a candy that delivers a blast of the most popular Jones Soda flavors along with an oddly enjoyable tongue-tingling sensation. Which pricing strategy would be appropriate if the company wants to convince price-sensitive consumers to try it and not buy some other brand? A. Price lining B. Price fixing C. Status quo pricing D. Penetration pricing E. Price skimming Answer: D Rationale: Consumers are price sensitive in this market. 15. Pharmacies are a new addition to Sam’s Clubs. They could exert a greater influence on the marketplace for prescription drugs than their newness indicates. Sam’s has a stated philosophy of marking up merchandise a maximum of 14 percent. When that philosophy is applied to prescription drugs, especially generics, warehouse club prices can be dramatically lower than those of conventional drugstores, supermarkets, or discount store pharmacies. Sam’s is using a _____ strategy to convince consumers to use its pharmacies rather than its competitors. A. penetration pricing B. price-insensitive demand C. price skimming D. price elasticity E. cost bundling Answer: A Rationale: When a firm introduces a new product at a relatively low price because it hopes to reach the mass market, it is following a penetration pricing strategy. The low price is designed to capture a large share of a substantial market and produce lower production costs. 16. A penetration strategy tends to be effective in a price-sensitive market. Thus, one of the purposes of penetration pricing is to: A. recoup product development costs quickly. B. discourage competitors from entering the market. C. produce a large margin of profit per unit. D. develop exclusive distribution. E. attract the price-insensitive buyer who demands the latest in technology. Answer: B Rationale: A low price will mean a low profit margin and will only be attractive if a large volume of business can be seized. The first company on the market that uses penetration pricing has a great advantage. 17. A penetration pricing strategy tends to be most effective: A. when demand is relatively inelastic. B. under unitary conditions. C. in price-sensitive markets. D. when the company can only perform small production runs. E. if unit costs are high. Answer: C Rationale: Penetration pricing is the logical choice given an elastic demand curve. 18. Penetration pricing means charging a relatively low price for a product as a way to reach the mass market. The low price is designed to capture a large share of a substantial market. Thus, penetration pricing: A. tends to be more effective in a less price-sensitive market. B. tempts competitors to enter the market. C. provides a large profit per unit sold. D. recoups product development costs quickly. E. tends to lower production costs. Answer: E Rationale: The low price is designed to capture a large share of the market, resulting in lower production costs. Production costs are lowered because of economies of scale in production. 19. A firm charging a price identical to or very close to the competition’s price is using a _____ strategy. A. differentiation pricing B. penetration pricing C. preemptive pricing D. status quo pricing E. leader pricing Answer: D Rationale: Status quo pricing is also called meeting the competition or going rate pricing. 20. JCPenney sends representatives to shop at similar retailers to make sure it is charging comparable prices for its products. JCPenney probably uses a _____ strategy. A. leader pricing B. preemptive pricing C. status quo pricing D. flexible pricing E. functional pricing Answer: C Rationale: Status quo pricing means meeting the competition. 21. State laws that put a lower limit on wholesale and retail prices are called _____. In states that have these laws, selling below cost is illegal. A. unfair trade practice acts B. price floor laws C. protectionism acts D. transparency laws E. price edicts Answer: A Rationale: Unfair trade practice acts are laws that prohibit wholesalers and retailers from selling below cost. 22. States developed unfair trade practice acts to: A. enforce the Sherman Act that makes bait pricing illegal. B. prevent oligopoly leaders from getting together and fixing prices at the highest the market will bear. C. establish penalties for companies that break the Clayton Act by engaging in predatory pricing. D. make sure that all pricing policies are equitable. E. protect small local firms from giant companies that operate efficiently on razor-thin profit margins. Answer: E Rationale: Unfair practice acts protect small businesses. 23. In 2008, United Airlines and American Airlines disclosed settlements in a class-action lawsuit over allegations of airfreight price fixing. This means the companies: A. tried to charge fees for airfreight that were below costs. B. charged customers different amounts for the same shipments. C. agreed on the price they would charge customers for airfreight. D. used uniform geographic pricing. E. created an artificial demand for shipping. Answer: C Rationale: Price fixing is an agreement between two or more firms on the price they will charge for a product. 24. South Africa’s Competition Commission accused South African Airways of conspiring with its partner, Germany’s Lufthansa, to set prices on flights between Cape Town, Johannesburg, and Frankfurt. As a result, the two airlines were charged with: A. price discrimination. B. price fixing. C. bait pricing. D. unfair trade practices. E. channel control pricing tactics. Answer: B Rationale: Price fixing is an agreement between two or more firms on the price they will charge for a product. 25. Which of the following prohibits any firm from selling to two or more different buyers, within a reasonably short time, commodities (not services) of like grade and quality at different prices where the result would be to substantially lessen competition? A. Sherman Act B. Federal Trade Commission Act C. Food and Drug Administration Act D. Anti-Discrimination Act E. Robinson-Patman Act Answer: E Rationale: The Robinson-Patman Act also makes it illegal for a seller to offer two buyers different supplementary services and for buyers to use their purchasing power to force sellers into granting discriminatory prices or services. 26. Acme Lawnmowers sells its mowers to retailers at different prices, depending on whether they are independent stores or members of a national chain. It uses: A. unfair trade practices. B. price fixing. C. price discrimination. D. predatory pricing. E. bait pricing. Answer: C Rationale: Price discrimination occurs when sellers charge different customers different prices for the same products. 27. All of the following elements must be present for a pricing practice to be considered discriminatory under the Robinson-Patman Act EXCEPT: A. The seller must charge different prices to different customers for the same product. B. The seller must make two or more actual sales within a reasonably short time. C. The transaction must occur in interstate commerce. D. The products sold must not be commodities. E. There must be significant competitive injury. Answer: D Rationale: The products sold must be commodities or other tangible goods. 28. The practice of charging a very low price for a product with the intent of driving competitors out of business or out of a market is called: A. price discrimination. B. predatory pricing. C. price fixing. D. price manipulation. E. anti-competitive pricing. Answer: B Rationale: This is the definition of predatory pricing. Firms using such pricing charge very low prices, and once competitors have been driven out; the firm raises its prices. 29. When Microsoft introduced its Zune MP3 player, many people thought it would capture the MP3 player market by pricing its product so low that a smaller competitor, like the Apple iPod, would be unable to compete. If Microsoft had used this approach, it would have been would be guilty of: A. predatory pricing. B. unfair trade practices. C. channel manipulation pricing. D. price fixing. E. price discrimination. Answer: A Rationale: Predatory pricing is the practice of charging a very low price for a product with the intent of driving competitors out of business or out of a market. 30. After managers understand both the legal and the marketing consequences of price strategies, they should set a _____ price––the general level at which a company expects to sell a good or service. A. functional B. zone C. demand D. leader E. base Answer: E Rationale: One the base price is set then the marketer fine-tunes it using discounts, allowances, rebates, and value-based pricing. 31. Toyota periodically offers customers _____, allowing purchasers to borrow money to pay for new cars without incurring an interest charge. A. markdown money B. zero percent financing C. promotional allowances D. make-up allowances E. leader pricing reductions Answer: B Rationale: This tactic creates a huge increase in sales—but not without cost to the automakers. 32. All of the following are tactics for fine-tuning the base price EXCEPT: A. functional discounts. B. cash discounts. C. rebates. D. quality discounts. E. quantity discounts. Answer: D Rationale: Additional tactics include seasonal discounts, promotional allowances, zero-percent financing, and value-based pricing. 33. Last year, a single infield box ticket for an Atlanta Braves baseball game cost $40, but fans who bought a season pass for the same seat got a reduced price. This $40 price was a _____ price. A. base B. zone C. demand D. channel leader E. functional Answer: A Rationale: The base price is the general price level at which a company intends to sell its product. 34. When a buyer pays a lower price for buying multiple units or above a specified dollar amount for a single order, the buyer is receiving a _____ discount. A. promotional B. quantity C. frequent buyer D. functional E. cumulative Answer: B Rationale: A quantity discount is a price reduction offered to buyers buying in multiple units or above a specified dollar amount. 35. Redline Editorial Services sent a $1,000 invoice to a customer for copyediting a booklet. According to the terms of the invoice, the customer would receive a 3 percent discount on the invoice price if the invoice was paid within 15 days. This is an example of a: A. Quantity discount. B. Cash discount. C. Rebate. D. Functional discount. E. Promotional allowance. Answer: B Rationale: This is the definition of a cash discount. 36. When the salesperson from Affiliated Food, Inc., a grocery distributor, calls on a grocery store, she is authorized to offer a 15 percent discount from the list price in recognition of activities (such as unpacking items and stocking shelves) that retailers perform for the distributor. This 15 percent discount is a: A. quantity discount. B. promotional allowance. C. functional discount. D. seasonal discount. E. channel allowance. Answer: C Rationale: A functional, or trade, discount is a discount to wholesalers and retailers for performing channel functions. 37. Ace Hardware’s spring snow blower sale is an example of which of the following pricing tactics? A. Quantity discount B. Seasonal discount C. Temporal discount D. Promotional allowance E. Functional discount Answer: B Rationale: A seasonal discount is a price reduction for buying merchandise out of season. 38. A(n) _____ discount is a deduction from list price that applies to the buyer’s total purchases made during a specific period. A. cumulative quantity B. noncumulative quantitative C. functional D. cash E. integrated Answer: A Rationale: This is the definition of a cumulative quantity discount. 39. Which type of quantity discount is a deduction from the list price that applies to a single order? A. Base discount B. Cumulative discount C. Noncumulative discount D. Cash discount E. Functional discount Answer: C Rationale: A noncumulative discount applies to one order and is intended to encourage orders in large quantities. 40. Quantity discounts are most often used to: A. reward the buyer who pays in cash. B. encourage buying in multiple units. C. increase supply for a specific raw material. D. reward a channel intermediary for performing some service. E. shift the storage function backward to the supplier. Answer: B Rationale: Quantity discounts encourage wholesalers or retailers to buy in larger amounts. 41. An Internet picture frame manufacturer offers retailers reduced prices on any combination of size or style frames purchased. The discount is shown as they shop and adjusted as the quantity of frames purchased increases. What common form of purchase discount is the frame manufacturer using? A. Rebate B. Cash discount C. Quantity discount D. Promotional allowance E. Functional discount Answer: C Rationale: Quantity discounts offer lower prices for buying in multiple units or above a specified dollar amount. 42. A discount off the base price to customers who pay immediately, or within a specified time period, is called a: A. functional discount. B. quantity discount. C. base discount. D. cash discount. E. promotional allowance. Answer: D Rationale: A cash discount is a price reduction offered to a consumer, an industrial user, or a marketing intermediary in return for prompt payment of a bill. Prompt payment saves the seller carrying charges and billing expenses and allows the seller to avoid bad debt. 43. When a channel intermediary is compensated for the ordinary services and tasks performed within the channel of distribution, the compensation usually comes in the form of a discount from base price. This discount is called a: A. seasonal discount. B. promotional allowance. C. cumulative or noncumulative quantity discount. D. functional (or trade) discount. E. rebate or refund. Answer: D Rationale: A functional, or trade, discount is a discount to wholesalers and retailers for performing channel functions. 44. A _____ is a price reduction that shifts the storage function forward to the purchaser and enables manufacturers to maintain steady production year-round. A. functional discount B. base allowance C. promotional allowance D. quantity discount E. seasonal discount Answer: E Rationale: A seasonal discount is a price reduction for buying merchandise out of season. 45. Apple’s “Back to School” program offered students who purchased an iMac computer and an iPod Touch MP3 player a $250 refund. The $250 check was essentially a: A. rebate. B. reciprocal allowance. C. cash discount. D. functional discount. E. trade promotion. Answer: A Rationale: Rebates are cash refunds. 46. Hunter’s Alley is a chain of stores targeted to people who are proud of their National Rifle Association memberships. It has agreed to set up a special display of Swartklip ammunition near its rifle and shotgun aisles, and also to run an advertisement in newspapers in communities where its stores are located. Swartklip has agreed to supply the display material free and to pay for half the cost of the advertisement. This is an example of a: A. bundled pricing tactic. B. functional discount. C. promotional allowance. D. quantity discount. E. direct allowance. Answer: C Rationale: A promotional allowance may be used to offer free goods or displays to a retailer in return for promotion of a manufacturer’s products, or it may pay for some or all of the advertising costs. 47. _____ are cash refunds given for the purchase of a product during a specific period. A. Rebates B. Loss leaders C. Reciprocal allowances D. Demand discounts E. Promotional allowances Answer: A Rationale: This is the definition of a rebate. 48. _____ occurs when a firm is customer driven and seeks to understand the attributes customers want in the goods and services they buy and the value of those attributes to customers. Thus, the price of the product is set at a level that seems to the customer to be a good price compared with the prices of other options. A. Value-based pricing B. Noncumulative pricing C. CRM pricing D. Market concept pricing E. Price bundling Answer: A Rationale: Instead of figuring prices based on costs or competitors’ prices, value-based pricing starts with the customer, considers the competition, and then determines the appropriate price. 49. With value-based pricing: A. the firm is sales driven. B. the firm is both customer driven and competitor driven. C. increased profitability for wholesalers will increase the number of services they are willing to perform. D. consumers are more concerned about price than quality. E. additional long-term costs to manufacturers will increase. Answer: B Rationale: The basic assumption is that the firm is customer driven, seeking to understand its customers. Because it is unlikely to be operating as a monopoly, it must also pay attention to what its competitors are doing. 50. One pharmaceutical manufacturer did not price a new antiulcer drug by adding up the costs of developing and manufacturing the medication and tacking on the amount of profit it wanted to make. Instead, the company justified a higher price than it might otherwise have been able to get from medical insurers by using studies that showed the new drug could help patients avoid expensive surgery and save the insurance companies money. The pharmaceutical company used: A. value-based pricing. B. noncumulative pricing. C. CRM pricing. D. price bundling. E. market concept pricing. Answer: A Rationale: Value-based pricing occurs when a firm is customer driven and seeks to understand the attributes customers want in the goods and services they buy and the value of those attributes to customers. Thus, the price of the product is set at a level that seems to the customer to be a good price compared with the prices of other options. 51. Sometimes managers price their products too low, resulting in a loss of company profits. One reason this happens is that: A. managers attempt to buy market share through aggressive pricing, but the cuts are quickly met by competitors, which wipe out any gain in market share. B. consumers tend to equate lower prices with low-quality goods, and they are never able to regain that lost market share. C. price-skimming strategies only work in the short-term, and always eventually result in lower profits. D. most managers simply lack good business sense, especially in regard to finances. E. all of the above are reasons. Answer: A Rationale: Another reason is that managers have a natural tendency to want to make decisions that can be justified objectively, but companies often lack hard data to make good decisions. 52. A price tactic that requires the purchaser to absorb the freight costs from the shipping point is called _____. In this case, the farther buyers are from sellers, the more they pay, because transportation costs generally increase with the distance merchandise is shipped. A. basing-point pricing B. zone pricing C. uniform delivered pricing D. freight absorption pricing E. FOB origin pricing Answer: E Rationale: This is the definition of FOB origin pricing. 53. The term FOB is an acronym for: A. free on board. B. fee on buyer. C. first on board. D. freight on board. E. freight origin buyer. Answer: A Rationale: FOB means “free on board” and is called FOB origin pricing or FOB shipping point. It is a price tactic that requires the buyer to absorb the freight costs from the shipping point. 54. With _____, the seller pays the actual freight charges and bills every purchase with an identical, flat freight charge. A. uniform delivered pricing B. zone pricing C. FOB origin pricing D. freight absorption pricing E. basing-point pricing Answer: A Rationale: If the marketer wants total costs, including freight, to be equal for all purchasers of identical products, the firm will adopt uniform delivered pricing, or “postage stamp” pricing. 55. Uniform delivered pricing enables a firm to: A. charge each customer the actual cost of shipping its products. B. stir up price competition between buyers. C. offer every purchaser an identical, flat freight charge. D. discriminate in favor of buyers that are geographically closer to the seller. E. charge each customer its fair share of the cost of shipping. Answer: C Rationale: With uniform delivered pricing, all customers will pay the same price regardless of their location. 56. Uniform delivered pricing: A. creates no geographic price discrimination. B. is sometimes called “postage stamp” pricing. C. is prevalent in the steel, cement, corn oil, and lead industries. D. is common where freight costs are a large portion of total costs. E. is calculated from regional base points. Answer: B Rationale: With uniform delivered pricing, all customers pay the same amount for freight regardless of location. 57. L.L. Bean charges all customers the same flat freight rate. It uses: A. FOB origin pricing. B. zone pricing. C. freight absorption pricing. D. basing-point pricing. E. uniform delivered pricing. Answer: E Rationale: In uniform delivery pricing, all customers are charged the same flat freight rate. 58. All of the following are geographic pricing methods EXCEPT: A. latitude pricing. B. FOB origin pricing. C. zone pricing. D. freight absorption pricing. E. basing-point pricing. Answer: A Rationale: Another geographic pricing method is uniform delivered pricing. 59. Claxton Bakery recently began selling its fruitcakes online. If Claxton wants a simple pricing system that allows for different shipping charges depending on geographic segment or region, the company should use _____ pricing. A. two-part B. uniform delivered C. freight absorption D. flexible E. zone Answer: E Rationale: Zone pricing sets freight prices according to geographic areas. 60. An Alabama-based catalog retailer sells fireplace equipment such as screens and andirons. Its customers in New England are charged one shipping rate, and customers west of the Rocky Mountains are charged a different rate. Customers in the midwestern states are charged yet another rate. What kind of geographic pricing is the catalog retailer using? A. FOB origin pricing B. FOB factory C. Zone pricing D. Freight absorption pricing E. Uniform delivered pricing Answer: C Rationale: Zone pricing is a modification of uniform delivered pricing that divides the United States into segments or zones and a charges a flat rate to all customers in that zone. 61. If a company decides to divide its market area into segments or regions and charge a flat rate for freight to all customers in a given region, the company is using _____ pricing. A. zone B. uniform delivered C. freight absorption D. FOB origin E. basing-point Answer: A Rationale: Zone pricing divides the total market into segments or zones and charges a flat freight rate to all customers in a given zone. 62. Dancing Pigs Bar-B-Que Sauce is a product of the Bar-B-Q Shop located in Memphis, Tennessee. It’s also sold online for $88 a case, including shipping and handling. The Bar-B-Q shop covers the cost of shipping and uses _____ pricing policy. A. penetration B. skimming C. zone D. basing-point E. freight absorption Answer: E Rationale: In freight absorption pricing, the seller pays the actual freight charges and does not pass the charges along to the customer. 63. Trade-ins often go hand-in-hand with: A. price skimming. B. professional services pricing. C. flexible pricing. D. single-pricing. E. penetration pricing. Answer: C Rationale: Trade-ins and flexible pricing often go hand-in-hand. 64. If the seller pays all or part of the actual freight charges and does not pass them on to the buyer, the seller is using _____ pricing. A. freight absorption B. uniform delivered C. zone D. FOB origin E. basing-point Answer: A Rationale: This is the definition of freight absorption pricing, and the manager may use this tactic in intensely competitive areas or as a way to break into a new market. 65. If a manufacturer designates a shipping point from which to calculate all freight charges and charges customers the freight costs from that point (even if the goods were shipped from another location), the manufacturer is using _____ pricing. A. freight absorption B. uniform delivered C. zone D. FOB origin E. basing-point Answer: E Rationale: With a basing-point price, the seller designates a location as a basing point and charges all buyers the freight costs from that point. 66. A national manufacturer of car parts has six warehouses and has a pricing policy of charging freight from the closest warehouse to the customer, regardless of where parts are shipped from. For instance, if the customer is in Vancouver, British Columbia, the closest warehouse to the customer is in Seattle, Washington. If the ordered car part actually comes from the Alabama warehouse, the customer still pays freight from Seattle. The manufacturer uses _____ pricing. A. FOB origin B. uniform delivered C. zone D. basing-point E. freight absorption Answer: D Rationale: In basing-point pricing, customers pay freight from a set base point, regardless of the location from which the goods are shipped. 67. Which of the following is a price tactic that offers all goods and services at the same price (or perhaps two or three prices)? A. Primary pricing B. Uniform-price tactic C. Single-price tactic D. Constant-pricing E. EDLP Answer: C Rationale: Single-price selling removes price comparisons from the buyer’s decision-making process. 68. The 99-Center is a retail store where all of the merchandise is priced at 99 cents. This retailer uses: A. a single-price tactic. B. flexible pricing. C. price lining. D. price bundling. E. leader pricing. Answer: A Rationale: The single-price tactic offers all goods and services at the same price (or perhaps two or three prices). 69. Single-price selling: A. removes price comparisons from the buyer’s decision-making process. B. does not benefit the retailer. C. is most effective when used during an inflationary period. D. encourages clerical errors. E. is accurately described by none of these choices. Answer: A Rationale: A single-price tactic offers all goods and services at the same price (or perhaps two or three prices). 70. The Used Car Mall lets salespeople charge different customers different prices for essentially the same automobile depending on how good the customer is at negotiating price. It uses: A. two-part pricing. B. an illegal pricing policy. C. flexible pricing. D. bait and switch practices. E. price maintenance. Answer: C Rationale: Flexible pricing means selling essentially the same product to different customers for different prices. 71. The tactic that allows different customers to pay different prices for essentially the same merchandise bought in equal quantities is called _____. It is often found in the sale of shopping goods, specialty merchandise, and most industrial goods except supply items. A. zoning (or basing) pricing B. illegal price fixing C. price maintenance D. psychological (or odd–even) pricing E. flexible (or variable) pricing Answer: E Rationale: Car dealers, many appliance retailers, and manufacturers of industrial installations, accessories, and component parts commonly follow this practice. 72. All of the following are potential disadvantages of a flexible pricing policy EXCEPT: A. it causes inconsistent profit margins. B. it enables a seller to close a sale with a price-conscious customer. C. it causes ill will among customers if they discover that other customers are paying lower prices. D. it enables salespeople to automatically lower the price to make a sale. E. it can spark a price war with competitors. Answer: B Rationale: Flexible pricing may allow a salesperson to negotiate with a price-sensitive customer, which is an advantage. The other answer choices are disadvantages. 73. Suppose an advertising agency develops logos for its clients. It charges $10,000 per logo––whether the team that’s working on the logo takes 30 minutes or days to design the logo. Agency management explain that clients pay for the agency’s expertise and creativity, not the amount of time it literally takes to develop a logo. This pricing approach is known as: A. professional services pricing. B. potential (or base) pricing. C. price maintenance. D. psychological pricing. E. flexible (or variable) pricing. Answer: A Rationale: Professional services pricing is used by people with lengthy experience, training, and often certification by a licensing board. 74. Which of the following statements is NOT true with regard to trade-ins? A. Flexible pricing and trade-ins often go hand in hand. B. If a trade-in is involved, the consumer must negotiate two prices, one for the new product and one for the existing product. C. Research found that trade-in customers tend to care more about the trade-in value they receive than the price they pay for the new product. D. About 95 percent of all new car sales involve a trade-in. E. On average, customers who trade-in an automobile when purchasing a new one end up paying more than customers who simply buy a new car from a dealer. Answer: D Rationale: A little over one half (57 percent) of all new car sales involve a trade-in. 75. Lea Kirkham is a physician. She charges each patient the same price for a physical examination, whether the procedure takes 10 minutes or a full hour. Which pricing policy is Dr. Kirkham following? A. Professional services B. Potential (or base) C. Price maintenance D. Psychological E. Flexible (or variable) Answer: A Rationale: Professional services pricing is used by people with lengthy experience, training, and often certification by a licensing board. 76. Often a seller will establish a series of prices for a family of merchandise items. There may be several different models at specific price points but no prices in between. This policy is called: A. price lining. B. price bracketing. C. family pricing. D. variable pricing. E. price bundling. Answer: A Rationale: Price lining is the practice of offering a product line with several items at specific price points. 77. At the Greenville Florist, there are four different prices for funeral bouquets. The smallest bouquet sells for $30; there is also a $40 version and a $75 version. For those who want to express their grief through the purchase of a dramatic floral arrangement, the florist also offers a $150 version. The owner of the florist shop has chosen price lining because it will: A. enable the shop to carry a larger total inventory. B. maintain all of the product line at the same stage in the product life cycle. C. confuse customers and allow salespeople to sell more of the expensive models. D. reach several different target market segments. E. thwart competitors that are trying to sell similar products. Answer: D Rationale: Price lining allows a retailer to appeal to several different target markets. It is not an uncommon strategy, and competitors probably use it. It should not affect inventory overall and will not confuse customers. 78. Price lining presents certain drawbacks to sellers, especially if: A. costs are continually rising. B. competition suddenly increases. C. profit margins are lowered. D. demand is rising. E. costs are flat. Answer: A Rationale: Price lining can be risky for sellers if costs are continually rising. 79. Why is price lining a valuable tactic for marketing managers? A. Price lining results in a greater inventory carrying charge. B. It reduces confusion for its customers. C. A company that uses price lining has more price markdowns and greater markup. D. The price lining strategy allows the company to gain brand loyalty from its targeted segments. E. Price lining tends to confuse customers and requires them to listen closely to the salesperson’s pitch. Answer: B Rationale: It reduces confusion for both the salesperson and the consumer. 80. The owner of a neighborhood hardware store has decided to sell a set of three padlocks for $5. He hopes the below-cost price for the locks will attract current and new customers who will also buy regularly priced items. The owner is encouraging store patronage through: A. deceptive pricing. B. incentive pricing. C. pricing lining. D. cumulative pricing. E. leader pricing. Answer: E Rationale: Leader pricing involves selling a product near or even below cost to attract business. 81. Leader pricing is used to: A. attract customers to a store so they can be persuaded to buy a more expensive product instead. B. bundle products together for sale. C. attract customers to the store so they will buy other products in addition to the leader product. D. price products at odd-numbered amounts to stimulate demand. E. maintain a status quo pricing strategy. Answer: C Rationale: Leader pricing involves selling a product near or even below cost in the hope that shoppers will buy other items once they are in the store. 82. Every week, Keller’s Grocery runs a weekly ad in the newspaper touting its sale prices on a number of products. For example, this week the store is selling cherries for $1.50/pound and boneless chicken breasts for 99 cents/pound. Keller’s sells these products at a below-market price to lure customers into the store in hope that while they are in the store to buy chicken and cherries, they will also buy other grocery items that have a much higher markup. The store is using: A. price lowballing. B. price maintenance. C. price lining. D. leader pricing. E. functional pricing. Answer: D Rationale: Leader pricing involves selling a product near or even below cost in the hope that shoppers will buy other items once they are in the store. 83. _____ tries to get customers into the store with misleading advertising and then uses high-pressure selling to persuade the consumer to buy something else more expensive. A. Functional pricing B. Bait pricing C. Sales-oriented pricing D. Production-oriented pricing E. Decoy pricing Answer: B Rationale: The Federal Trade Commission considers bait pricing a deceptive act and has banned its use in interstate commerce. Most states also ban bait pricing, but sometimes enforcement is lax. 84. Cashtown Used Cars aired a radio spot announcing, “Today only, previously owned cars are only $200!” Meghan just wanted some kind of in-town transportation. When she went to Cashtown, the salesperson said, “We have only one $200 car left, and it’s not the kind of car I’d want my wife to drive. However, we do have some great deals on newer models.” Meghan went home with an $8,000 used car. Cashtown is probably practicing: A. decoy pricing. B. deal pricing. C. functional pricing. D. bait pricing. E. price pressuring. Answer: D Rationale: Bait pricing tries to get customers into the store with misleading advertising and then uses high-pressure selling to persuade the consumer to buy something else more expensive. 85. Which type of pricing means pricing at odd-numbered prices to connote bargains and pricing at even-numbered prices to imply quality? A. Bait pricing B. Price bundling C. 1–2 pricing D. Odd–even pricing E. Two-part pricing Answer: D Rationale: This describes odd–even pricing (or psychological pricing). 86. If a marketer decides to price goods at odd-numbered dollar amounts to denote bargains and at even-numbered amounts to denote quality, he or she is using: A. two-part pricing. B. price lining. C. price bracketing. D. decoy pricing. E. psychological pricing. Answer: E Rationale: Odd–even pricing is also called psychological pricing. 87. Leader pricing is also called: A. psychological pricing. B. follower pricing. C. frontrunner pricing. D. price bundling. E. loss-leader pricing. Answer: E Rationale: Leader pricing is also called loss-leader pricing. 88. Marketing two or more products in a single package for a special price is known as: A. price bundling. B. two-part pricing. C. psychological pricing. D. price lining. E. family pricing. Answer: A Rationale: This is the definition of price bundling. 89. AMC Theaters offers customers a package that includes two movie tickets, two small drinks, and one small popcorn all priced together at $29.99. this pricing technique is called: A. price lining. B. two-part pricing. C. horizontal pricing. D. price bundling. E. bait pricing. Answer: D Rationale: Marketing two or more products in a single package for a special price is called price bundling. 90. The Comcast Triple Play package includes cable television, Internet, and telephone service for a price significantly lower than the cost of the three services priced separately. This is an example of: A. multiple unit pricing. B. professional services pricing. C. price lining. D. price bundling. E. two-part pricing. Answer: D Rationale: Marketing two or more products in a single package for a special price is called price bundling. 91. In a catalog targeted to people who like to bake, customers can buy a single yeast bread mix designed specifically to be baked in bread machines for $3.95 each or 12 different mixes for $37.50. This is an example of: A. price bundling. B. CRM pricing. C. psychological pricing. D. penetration pricing. E. status quo pricing. Answer: A Rationale: The 12 items sold together are priced less expensively than if purchased separately. 92. Reducing the services that come with the basic product is called: A. Demarketing. B. Contraction. C. two-part pricing. D. retroactive pricing. E. Unbundling. Answer: E Rationale: Unbundling is done instead of raising the price. 93. Tickets to a combined amusement park and water slide were $49 for the day. Then the company gave customers the option to purchase tickets for either the amusement park or the water slide for $18. To help keep costs in line, the park management also began charging its customers a small parking fee. Initially, the cost of parking was figured into the $49 price. The amusement park is using: A. price lining. B. potential (or base) pricing. C. Unbundling. D. professional services pricing. E. price maintenance. Answer: C Rationale: Reducing the bundle of services that comes with the basic product is called unbundling. 94. Louisiana State University (LSU) football season ticket holders have to pay a fee (also known as a “donation”) to the LSU Foundation every January, which is not deemed to be part of the ticket price. They pay for their tickets later in the year for the next football season. The Foundation fee must be paid in order to retain the rights to purchase a season ticket. LSU is using: A. multiple unit pricing. B. professional services pricing. C. price lining. D. price bundling. E. two-part pricing. Answer: E Rationale: Two-part pricing involves two separate charges to consume a single good or service. 95. Consumers sometimes prefer two-part pricing because: A. prices are often perceived as quality indicators. B. consumers like to be in control of costs. C. consumers are uncertain about the number and types of activities they might use at places like an amusement park. D. consumers prefer a limited number of choices. E. prices have little or no psychological influence on most consumers. Answer: C Rationale: This is one of the reasons consumers sometimes prefer two-part pricing. 96. _____ are extra fees paid by consumers for violating the terms of purchase agreements. A. Decoy fees B. Misuse discounts C. Punitive fees D. Consumer penalties E. Financial judgments Answer: D Rationale: This defines consumer penalties. Businesses impose consumer penalties for two reasons: They will allegedly (1) suffer an irrevocable revenue loss and/or (2) incur significant additional transaction costs should customers be unable or unwilling to complete their purchase obligations. 97. Consumers are required to pay consumer fees because businesses allegedly: A. will sell more if the consumer is unaware of the actual costs. B. suffer an irrevocable revenue loss. C. are required by the federal government to charge this fee. D. avoid additional transactional costs when purchase agreements are violated. E. incur greater fixed costs as a result of the purchase agreement violation. Answer: B Rationale: There is no such federal law that requires charging of a fee. Businesses allege they incur more transactional costs as a result of purchase agreements violations. Fixed costs do not change. 98. Costs that are shared in the manufacturing and marketing of several products in a product line are called: A. joint costs. B. integrated costs. C. fixed costs. D. variable costs. E. combined costs. Answer: A Rationale: This is the definition of joint costs, which are a unique problem in product pricing. 99. Post makes several varieties of cereals. In promoting this product line, Post offers a 50-cents-off coupon that can be used to purchase any of its cereals. Therefore, Post must consider _____ when pricing its cereals. A. joint costs B. differential costs C. bundling costs D. potential (or basing) costs E. factorial costs Answer: A Rationale: Joint costs are costs that are shared in the manufacturing and marketing of several products in a product line. 100. Alissa Dunn is the owner and operator of Dunn’s Best Jams, which she sells at craft festivals. She only makes and sells three types of jams––pecan pie jam, chocolate pie jam, and lemon tart jam. The costs of leasing her professional kitchen for manufacturing, travel to craft shows, insurance, and so on are allocated on an equal basis to the three types of jam sold. In other words, these costs are: A. derived costs. B. elastic costs. C. joint costs. D. revenue impediments. E. synergistic costs. Answer: C Rationale: Joint costs are costs that are shared in the manufacturing and marketing of several products in a product line. 101. Nestlé Purina sells chicken feed and Wheat Chex, but the sale of one of these products has no known impact on demand for the other. In this case, the two products have a(n) _____ relationship. A. inverse B. neutral C. complementary D. substitute E. negative Answer: B Rationale: Demand for one of the products is unrelated to demand for the other. 102. Best Buy charges customers a 15 percent restocking fee on some returned items. A restocking fee is for putting a returned item back into inventory. This is an example of a: A. bait-and-switch. B. trade-in. C. consumer penalty. D. product absorption strategy. E. price bundle. Answer: C Rationale: Consumer penalties are extra fees paid by consumers for violating the terms of a purchase agreement. 103. If items are _____, an increase in the sale of one good causes an increase in the sale of the other (and vice versa). A. inverse B. neutral C. complementary D. substitute E. negative Answer: C Rationale: This defines a complementary relationship. 104. Kule, Inc. produces three different lines of car racks for transporting large, bulky items. Total company net annual profit = $40,000 Included in the cost of goods sold is $12,000 of annual rent (a fixed cost) that is distributed equally among the three product lines. As a consultant to Kule, will you recommend that it drop the luggage rack line? A. No, dropping the line will actually decrease overall net profits. B. Yes, dropping the line will increase company net profits. C. No, dropping the line will result in increased fixed costs. D. Yes, dropping the line will reduce joint costs. E. Yes, dropping the line will reduce cost of goods sold and increase revenues. Answer: B Rationale: Current net profit is $40,000. Dropping the luggage rack line will increase profit by $10,000 initially, but the fixed rent costs ($4,000) that are being covered by that line will have to be distributed to the other two lines. That means the cost of goods sold for each remaining line will increase by $2,000 (or $4,000 ÷ 2). Cost of goods sold for the Bicycle line will increase to 112,000, resulting in profit of $28,000; cost of goods sold for the Ski line will increase to 142,000, resulting in a profit of $18,000. Total profit will be $46,000, or $6,000 more than if the company kept the Luggage line. Therefore, Kule should drop the line. 105. When using _____, price is not set on the product until the item is either finished or delivered. A. price shading B. escalator pricing C. delayed-quotation pricing D. bid pricing E. two-part pricing Answer: C Rationale: This tactic is used for industrial installations and many accessory items. 106. Delayed-quotation pricing: A. requires the seller to place a later date on the product invoice to help accounts receivable in recording transactions. B. allows the final selling price to reflect cost increases incurred between the time the order is placed and the final delivery takes place, often over a period of years. C. prevents the competitor from submitting an earlier bid. D. requires a seller to submit a bid after the closing date. E. is also known as price-shading bidding. Answer: B Rationale: Delayed-quotation pricing delays the setting of the final price. 107. A(n) _____ allows for price increases based on the cost-of-living index or some other formula. A. consumer penalty B. price shade C. price allowance D. escalator price clause E. elasticity quotient Answer: D Rationale: Escalator pricing is similar to delayed-quotation pricing in that the final selling price reflects cost increases incurred between the time an order is placed and the time delivery is made, but increases based on a formula set at the beginning of the job. 108. Escalator pricing is: A. a demand-oriented pricing tactic. B. similar to delayed-quotation pricing. C. similar to price shading. D. a form of market penetration pricing. E. also called “postage stamp” pricing. Answer: B Rationale: Escalator pricing allows for price increases and delays the setting of the final price. 109. Business-to-business salespeople often use _____ to increase demand for one or more products in a line. It is a discounting practice that is often done routinely without much forethought. A. decremental pricing B. price lining C. devaluation D. price shading E. consumer discounts Answer: D Rationale: Price shading is the use of discounts by salespeople to increase demand for one or more products in a line. 110. What can a marketing manager do to make demand for his or her product more inelastic? A. Eliminate brand equity B. Eliminate any unique products from the product line C. Cultivate selected demand D. Avoid making any product changes E. Implement escalator pricing Answer: C Rationale: The marketing manager would need to use some demand-oriented tactic. Escalator pricing is a cost-oriented tactic. 111. All of the following are strategies to make the demand for a good or service more inelastic EXCEPT: A. reducing consumer awareness. B. cultivating selected demand. C. changing the package design. D. creating a unique offering. E. heightening buyer dependence. Answer: A Rationale: All of the other choices are strategies to make the demand for a good or service more inelastic and will create buyer dependency. 112. For sports marketers, an inelastic demand curve means that they have greater flexibility in making pricing decisions. What can a sports marketer do to make demand for his or her product more inelastic? A. Have a winning team that people want to see play. B. Sell the rights to buy a season pass. C. Eliminate all quantity discounts. D. Use discriminatory pricing. E. Any of these strategies will help render demand more inelastic. Answer: A Rationale: There are several strategies to make demand more inelastic, and choice A is an illustration of the “create unique offerings” tactic. 113. Which of the following pricing methods can be used to build market share during a recession? A. Price lining B. Resale price maintenance C. Psychological pricing D. Bundling E. Variable pricing Answer: D Rationale: The other commonly used technique is value-based pricing. 114. During a recent worldwide recession when wine usage was declining, Nickel & Nickel launched a new brand of wine, which it sold at $125 a bottle. The wine is allowed to age three times as long as lower-priced wines, and the grapes used in the wine’s production are hand-picked. Wine lovers appreciate how both production techniques improve wine quality. Nickel & Nickel used _____ to build market share. A. value-based pricing B. unbundling C. price lining D. status quo pricing E. leader pricing Answer: A Rationale: Value-based pricing indicates the consumers are getting value for their money. 115. Two effective pricing tactics that can be used to hold or build market share during a recession are: A. flexible pricing and price shading. B. price shading and price lining. C. unbundling and price shading. D. value-based pricing and bundling. E. price lining and escalator pricing. Answer: D Rationale: Two effective pricing tactics to hold or build market share during a recession are value-based pricing and bundling. 116. During a recent recession, many manufacturers determined that their suppliers were an excellent source of cost savings. Specific cost reduction strategies manufacturers have used with their suppliers include: A. offering help in boosting productivity of suppliers. B. renegotiating contracts. C. setting annual across-the-board cost reduction targets for suppliers. D. improving economies of scale by cutting number of suppliers. E. all of the choices. Answer: E Rationale: Tough tactics like these help keep companies afloat during economic downturns. Art Supplies It’s September and Sophia wants to buy some arts and crafts supplies for an after-school program she is developing for her daughter’s elementary school. In her Sunday newspaper was a flyer from Michaels, an arts and crafts retailer. As she looked through the newspaper insert, she noticed that if she purchased three or more bottles of Alene’s Tacky Glue, the regular price of $1.50 each was reduced to $1.15 each. She also saw that the store priced its plastic storage boxes at $1.99, $3.99, and $5.99. She thought they would be useful for storing each child’s projects. On the front page of the flyer was an ad for Funky Girls Gel Pens, something she knew her daughter would love to use. The price at Michael’s was $6.99 lower than the price she had found at the other stores that carried the pens. She thought that some of the older girls might like to start a scrapbook and was pleased to find that Michaels had a scrapbook starter kit, which includes scissors, book, pages, and stickers for only $15. The items could be purchased separately for $19.99. The flyer also announced that all flag-related items leftover from its Fourth of July sale were reduced by 40 percent. 117. Refer to Art Supplies. Which type of discount is being used to price the tacky glue? A. Noncumulative quantity discount B. Promotional allowance C. Seasonal discount D. Cash discount E. Cumulative quantity discount Answer: A Rationale: A noncumulative quantity discount is a reduction from list price that applies to a single order rather than to the total volume placed during a specific period. 118. Refer to Art Supplies. Which of the following merchandise is offered in the flyer with a seasonal discount? A. Tacky glue B. Plastic storage boxes C. Flag-related items D. Funky Girls Gel Pens E. Scrapbook starter kit Answer: C Rationale: Seasonal discounts are pricing reductions for buying merchandise out of season. 119. Refer to Art Supplies. What pricing practice was used to price the plastic storage boxes? A. Seasonal pricing B. Price shading C. Price lining D. Inelastic pricing E. Cumulative pricing Answer: C Rationale: Price lining is the practice of offering a product line with several items at specific price points. 120. Refer to Art Supplies. Michaels was trying to get consumers into the store with the Funky Girls Gel Pens promotion in hope that they will purchase other, higher-margin items. This is an example of: A. seasonal pricing. B. psychological pricing. C. price lining. D. price bundling. E. leader pricing. Answer: E Rationale: Leader pricing is an attempt by the marketing manager to attract customers by selling a product near or even below cost in the hope that shoppers will buy other items once they are in the store. 121. Refer to Art Supplies. What pricing practice was used with the scrapbook starter kit? A. Seasonal pricing B. Psychological pricing C. Price lining D. Price bundling E. Leader pricing Answer: D Rationale: Price bundling is marketing two or more products in a single package for a special price. Apple iPhone Apple, Inc.’s iPhone first went on sale on June 29, 2007. Apple’s loyal and enthusiastic customer base is known for rushing to purchase its new products, and the iPhone enjoyed a tremendous amount of buzz before its introduction. As expected, the iPhone entered the market at what many believed to be a high price ($599). However, within weeks, the price was reduced to $399. By the end of 2007, over eight million iPhones had sold in the U.S. marketplace. By most, if not all measures, the original iPhone was a huge success for Apple and its then-exclusive U.S. carrier AT&T. On July 11, 2008, Apple, Inc. released the iPhone 3G, which it advertised as being twice as fast as the original iPhone for half the cost. However, in order to obtain an iPhone at the new price of $199, buyers had to agree to a two-year service contract with AT&T. This approach succeeded, and over a million iPhone 3Gs were sold during the introductory weekend. In 2011, the iPhone 4S—the fifth generation iPhone—led cellular phone sales with more than 25 million units sold. Several Android-based phones manufactured by Samsung were not far behind, however. 122. Refer to Apple iPhone. When Apple introduced the iPhone at a high price, it was probably using a _____ strategy to maximize profits. A. price bracketing B. penetration pricing C. price lining D. price-fixing E. price skimming Answer: E Rationale: Price skimming is a pricing policy whereby a firm charges a high introductory price, especially for a heavily promoted product. 123. Refer to Apple iPhone. When the iPhone 3G was released at half the cost of the current iPhone, it appeared that Apple’s strategic focus had shifted from maximizing profits to gaining market share. Its lowered price was consistent with the _____ approach. A. price bracketing B. penetration pricing C. price lining D. price-fixing E. price skimming Answer: B Rationale: Penetration pricing uses a relatively low price to build market share. 124. Refer to Apple iPhone. Samsung recently introduced the Galaxy S III cellular phone, apparently to compete directly with the iPhone 4S. If Samsung checked the price of the iPhone at the Apple Store and leading cellular carrier locations and then set the price of the Galaxy S III to match the iPhone’s price, it would be using a _____ pricing approach. A. bracketing B. penetration C. status quo D. retain maintenance E. skimming Answer: C Rationale: Status quo pricing is basically meeting the price of competition. 125. Refer to Apple iPhone. Best Buy also carries the iPhone. If Best Buy, Apple, and leading cellular carriers meet to agree on a price for the iPhone, they are likely guilty of _____. A. price fixing B. retail price maintenance C. price discrimination D. penetration pricing E. price skimming Answer: A Rationale: Price fixing is an agreement between two or more firms on the price they will charge for a product. 126. Refer to Apple iPhone. Apple has several options available for competing with Samsung and its Galaxy S III phone. If Apple chooses to compete by pricing its product at a low price to drive Samsung out of the market, this would be considered: A. price fixing B. retail price maintenance C. price discrimination D. predatory pricing E. fair competition Answer: D Rationale: Predatory pricing means charging a very low price for a product with the intent of driving competitors out of business or out of a market 127. Refer to Apple iPhone. HSBC Group is the world’s largest banking group. When the iPhone 3G was released, HSBC considered switching from BlackBerry handsets to iPhone 3Gs. This would mean ordering 200,000 iPhones, so HSBC would probably receive special pricing incentives, including a: A. functional discount. B. cash discount. C. seasonal discount. D. rebate. E. quantity discount. Answer: E Rationale: A quantity discount applies when customers buy in multiple units or spend above a specified dollar amount. 128. Refer to Apple iPhone. For one fee, the basic AT&T cellular package includes 450 minutes of cellular calls, with free nights and weekend minutes, unlimited data, visual voice mail, 200 text messages, rollover minutes, and unlimited mobile-to-mobile service within the AT&T network. AT&T is using price: A. bundling B. skimming C. baiting D. leading E. lining Answer: A Rationale: Price bundling means marketing two or more products in a single package for a special price. 129. Refer to Apple iPhone. Consumers who agree to the two-year AT&T service contract are required to pay a cancellation fee if they leave AT&T prior to the end of the two-year period. This fee is a consumer: A. bundling. B. penalty. C. lining. D. stimulus. E. markdown. Answer: B Rationale: Consumer penalties are extra fees paid by consumers for violating the terms of a purchase agreement. ESSAY 1. List in order the four steps used to set the right price for a product. Answer: 1. Establish pricing goals. 2. Estimate demand, costs, and profits. 3. Choose a price strategy to help determine a base price. 4. Fine-tune the base price with pricing tactics. 2. What activities occur once the marketing manager has established pricing goals? Why are these activities important? Answer: The marketing manager must first estimate quantity demand levels and elasticity of demand, which allows for an estimate of revenues at a variety of price levels. Next, corresponding costs should be determined for each price. Then the manager can estimate the amount of profit and market share that can be earned at each possible price. Alternative pricing policies can be examined in terms of revenues, costs, and profits. This information becomes the core of the price policy by determining which price can best meet the firm’s pricing goals. 3. Name and describe the three basic strategies for setting a price on a new good or service. Under what conditions is each of the three basic pricing methods successful? Answer: PRICE SKIMMING. This method is sometimes called a “market-plus” approach to pricing because it denotes a high price relative to the prices of competing products. Often companies will use skimming and then lower prices over time. This strategy is successful when (1) the market is willing to buy the product even though it carries an above-average price, (2) when a product is well protected legally, (3) when it represents a technological breakthrough, and (4) when it has in some other way blocked the entry of competitors. As long as demand is greater than supply, skimming is an attainable strategy. PENETRATION PRICING. This method is at the opposite end of the spectrum from skimming. With this method, a firm charges a relatively low price, hoping to reach the mass market in the early stages of the product life cycle. The low price allows the product to penetrate a large portion of the market, resulting in large market share and lower production costs. This strategy is successful when (1) the market is price sensitive, (2) economies of mass production are feasible, and (3) the firm has substantial resources to sustain the short-run losses necessary to obtain penetration. STATUS QUO PRICING. With this method, price is set identical or close to that of the competition. This strategy may be used more often by small firms for survival or ease of use but ignores demand and cost. This strategy can be successful when (1) the firm is comparatively small and (2) the firm needs a safe route to long-term survival. 4. Name one advantage and one disadvantage associated with using each of the three basic pricing methods. Answer: PRICE SKIMMING advantages include (1) quick recovery of product development or educational costs, (2) pricing flexibility that allows subsequent lowering of price, and (3) the ability to market prestige products successfully. Disadvantages include encouragement of competitive entry into the market. PENETRATION PRICING advantages include (1) a tendency to discourage competitive entry, (2) large market share due to high volume sold, and (3) lower production costs resulting from economies of scale. Disadvantages include (1) lower profits per unit, (2) higher volume required to reach the break-even point, (3) slow recovery of development costs, and (4) inability to later raise prices. STATUS QUO POLICIES have the advantage of simplicity. Their disadvantage is that the strategy may ignore demand or cost or both. 5. Describe which pricing method (skimming, penetration, or status quo) would be most appropriate for each of the following products: (1) a new kind of automatic vacuum cleaner; (2) brightly colored wooden blocks to be used as a child’s toy; (3) a new, low-cost, no-calorie fat substitute; (4) a home computer; and (5) a designer perfume. Briefly justify your answers. Answer: VACUUM CLEANER. Skimming could be used because there are innovators and early adopters who would like to be “first” to own the product. It is likely that competition could also follow fairly quickly, further justifying a skimming policy. CHILD’S BLOCKS. Status quo pricing could be used because this type of toy is a mature product with many substitutes. Penetration pricing could be argued if one assumes the producer found manufacturing cost advantages. FAT SUBSTITUTE. Penetration pricing would quickly gain a large market share and is appropriate for a low-cost item. Skimming could be argued if one assumes greater demand than supply for the product. HOME COMPUTER. Penetration pricing is appropriate because of the large amount of competition in this particular market. PERFUME. Skimming would be appropriate because a lower introductory price might reduce the high-prestige perception of the product. 6. Some pricing decisions are subject to government regulation. Name and define three pricing practices that are illegal. Answer: UNFAIR TRADE PRACTICES occur when firms sell below costs. Many state unfair trade practice acts put a lower limit on wholesale and retail prices; wholesalers and retailers must take a minimum percentage markup. PRICE FIXING is an agreement between two or more firms on the price they will charge for a product or service. The Sherman Act and the Federal Trade Commission Act govern price fixing practices. PRICE DISCRIMINATION occurs when a firm sells to two or more different buyers, within a reasonably short time, commodities (not services) of like grade and quality at different prices where the result would be to substantially lessen competition. Price discrimination can also occur if the seller discriminates between buyers in terms of supplementary services provided, or if the buyers use their power to force sellers into discriminatory practices. The Robinson-Patman Act of 1936 prohibits these forms of price discrimination. PREDATORY PRICING is the practice of charging a very low price for a product with the intent of driving competitors out of business or out of the market. This practice is illegal under the Sherman Act and the Federal Trade Commission Act. 7. A base price may be lowered by a discount. Discounts take a variety of forms and have several different objectives. Name and define three types of discounts (do not include allowances or rebates). State the main objective of each type of discount you identify. Answer: QUANTITY DISCOUNTS are offered to buyers who purchase multiple units or above a specified dollar amount. The objectives of the quantity discount include selling large volumes (through noncumulative quantity discounts) and encouraging customer loyalty (through cumulative quantity discounts). CASH DISCOUNTS are price reductions offered to consumers, industrial users, or marketing intermediaries who pay promptly. One objective is to save the seller carrying charges and billing expenses. Another objective is to avoid bad debt. FUNCTIONAL (or TRADE) DISCOUNTS are compensation to wholesalers and retailers for performing channel functions. The objective is to compensate the channel member for services rendered or to encourage additional functions to be performed by the trade. SEASONAL DISCOUNTS are price reductions for buying merchandise out of season. The objectives of seasonal discounts include shifting the storage function forward to the purchaser and enabling a steady manufacturing schedule. 8. Distinguish between a cumulative and a noncumulative discount. Answer: A cumulative quantity discount is a deduction from list price that applies to the buyer’s total purchases made during a specific period; it is intended to encourage customer loyalty. In contrast, a noncumulative quantity discount is a deduction from list price that applies to a single order rather than to the total volume of orders placed during a certain period. It is intended to encourage orders in large quantities. 9. What is a promotional allowance? What is the difference between a promotional allowance and a functional discount? Give two specific examples of promotional allowances. Answer: A promotional allowance is a payment to a dealer for promoting the manufacturer’s products. A promotional allowance is similar to a functional discount as a pricing tool but also serves as a promotional device. Like functional discounts (and other forms of discount), promotional allowances must be made available to all purchasers on essentially the same terms. Examples of promotional allowances include cooperative advertising (in which the manufacturer pays for a portion of retailer-based advertising) or display assistance (in which the manufacturer pays for a special display or provides free goods for the display). 10. What is value-based pricing? What is the basic assumption marketers must make about their markets before implementing a value-based pricing strategy? Answer: Value-based pricing, also called value pricing, is a pricing strategy that has grown out of the quality movement. Instead of figuring prices based on costs or competitors’ prices, it starts with the customer, considers the competition, and then determines the appropriate price. The basic assumption is that the firm is customer driven, seeking to understand the attributes customers want in the goods and services they buy and the value of that bundle of attributes to customers. 11. Discuss the two reasons why managers sometimes price their products too low, thereby reducing company profits. Answer: First, managers attempt to buy market share through aggressive pricing. Usually, these price cuts are quickly met by competitors. Thus, any gain in market share is short-lived, and overall industry profits end up falling. Second, managers have a natural tendency to make decisions that can be justified objectively. Managers, however, often lack the hard data needed to make an accurate assessment of the market and what pricing strategy should be used. Managers tend to make pricing decisions based on easily gathered, short-term focused information, such as costs, sales, market share, and competitors’ prices rather than on long-term profitability. 12. Geographically dispersed sellers often result in significant freight costs. Name and describe the five types of geographic pricing tactics that can be selected by a marketing manager to moderate the impact of freight costs on its more dispersed customers. For each tactic defined, specify the circumstances that would prompt the selection of that geographic pricing tactic. Answer: FOB ORIGIN PRICING. This price tactic requires the buyer to absorb the freight costs from the shipping point. A manager would choose to use FOB origin pricing if he or she is not concerned about total costs varying among the firms’ clients or if freight charges are not a significant pricing variable. UNIFORM DELIVERED PRICING. With this price tactic, the seller pays the actual freight charges and bills every purchaser an identical, flat freight charge. This equalizes the total cost of the product for all buyers, regardless of location. A manager would select this policy if the firm is trying to maintain a nationally advertised price or when transportation charges are a minor part of total costs. ZONE PRICING. This price tactic is a modification of uniform delivered pricing in which the geographic selling area is divided into segments or zones. A flat freight rate is charged to all customers in a given zone, but different rates will apply to each zone. A marketing manager would use this strategy to equalize total costs among buyers within large geographic areas. FREIGHT ABSORPTION PRICING. With this price tactic, the seller pays all or part of the actual freight charges and does not pass these charges along to the buyer. A manager would choose this tactic if competition is extremely intense or if the firm is trying to break into new market areas. BASING-POINT PRICING. This method requires the seller to designate a location as a basing point and charges all buyers the freight cost from that point (regardless of the point from which the goods are actually shipped). This tactic has waned in popularity due to several adverse court rulings. 13. What type of geographic pricing policy would a marketing manager most appropriately choose for the following products: (1) nationally advertised bubble gum, (2) rebuilt engines for jet airplanes, and (3) bulk amounts of a rare spice harvested from a single mountain in Canada and used in high-priced restaurants. Justify your answers, and specify any assumptions you used to arrive at your answer. Answer: Geographic pricing policies should be compatible with the total price structure of the firm, so assumptions about the company’s pricing objectives and other pricing policies will affect responses. Additionally, assumptions about competitors’ practices and customs in the industry are important. BUBBLE GUM. In this case, identical retail prices would be sought, so the bubble gum should be delivered at the same price throughout the country. This would result in a pricing policy of uniform delivered pricing or possibly freight absorption pricing if competition is extremely intense. ENGINES. Because engines are large and heavy, transportation costs are an important component of pricing. In this case, FOB origin pricing could be used to put the burden of transportation on the purchasers. This pricing policy could be assumed if there were few competitors in the jet engine business. Otherwise, basing-point pricing would be the most appropriate pricing policy, although this type of pricing is being used less frequently due to adverse court rulings. SPICE. Zone pricing would be appropriate in this case. Because the spices are sold in bulk, it should be assumed that transportation costs are not insignificant. Uniform delivered pricing is not appropriate because there is no heavy competition. FOB origin pricing could be used as well because other pricing aspects of the spice might outweigh the transportation costs. 14. Marketing managers can use a wide variety of special pricing tactics beyond discounts and allowances to fine-tune prices. Name and define five of the other pricing tactics that are wholly legal. For each tactic, give an example of a specific company, industry, or product that would use the tactic. Answer: SINGLE-PRICE TACTIC means all goods and services are offered at the same price (or perhaps two or three prices). Examples of retailers include One Price Clothing Stores, MATTER, Your $10 Store, and Fashions $9.99. FLEXIBLE PRICING OR VARIABLE PRICING means different customers pay different prices for essentially the same merchandise purchased in equal quantities. Car dealers and many appliance retailers commonly use this method. TRADE-INS and flexible pricing go hand-in-hand. Car dealers are especially associated with trade-ins, but they also occur for other products such as musical instruments, video games, sporting goods, jewelry, and some appliances. PROFESSIONAL SERVICES PRICING is used by people with lengthy experience, training, and often certification by a licensing board. This pricing refers to the charging of an hourly rate or a fee based on some problem solution or performance. Lawyers, physicians, and family counselors are some examples. PRICE LINING is the practice of offering a product line with several items at specific price points. Text examples include cell phone carriers and Apple’s iTunes. LEADER PRICING is a method used to attract customers to a store by offering a product near or even below cost in the hope that shoppers will buy other merchandise once they are in the store. Supermarkets, social coupon sites, and health clubs use leader pricing. ODD–EVEN PRICING OR PSYCHOLOGICAL PRICING uses a price ending in an odd number to connote a bargain and a price ending in an even number to connote quality. Discount stores tend to use odd pricing, and specialty boutiques commonly use even pricing. PRICE BUNDLING is marketing two or more products in a single package for a special price. Microsoft offers “suites” of software that bundle items such as spreadsheets, word processing, graphics, e-mail, and so forth. Others include the telecommunications industry and ski resorts. TWO-PART PRICING means establishing two separate charges to consume a single good or service. Examples would include health and tennis clubs that charge both a membership fee and a fee each time the facilities are used. PAY WHAT YOU WANT PRICING involves consumers deciding what they want to pay for a product or service. Restaurants and music acts have used this concept. 15. What are consumer penalties? What two reasons do businesses give for requiring consumers to pay them? Answer: Consumer penalties are extra fees paid by consumers for violating the terms of purchase agreements. Businesses impose consumer penalties for two reasons: They will allegedly (1) suffer an irreversible revenue loss and/or (2) incur significant additional transaction costs should customers be unwilling or unable to comply with the purchase agreement. 16. What is product line pricing? What three relationships among products in the line must managers be aware of before setting prices? For each relationship, give an example of a product that fits the situation. Answer: Product line pricing is setting prices for the entire line of products rather than for a single component of the line. In product line pricing, the marketing manager attempts to achieve maximum profits or other goals for the entire line. Before determining price, the manager must determine the type of relationship that exists among the various products in the line. Three types of relationships exist: complementary, substitute, and neutral. COMPLEMENTARY PRODUCTS. In this case, an increase in sales for one item in the line will increase demand for a complementary product in the line. Examples of complementary products include shampoo and conditioner or skis and ski poles. PRODUCT SUBSTITUTES. Items in a line may also act as substitutes for one another. If a buyer purchases one item in the line, he or she will be less likely to purchase a second, substitutable item in the line. Examples of substitutable items would be liquid and powder Tide laundry detergent or paste and liquid Turtle Wax car polish. NEUTRAL RELATIONSHIP. In this case, the demand for one product is not related to the demand for any other product. Examples include Nestlé Purina’s sale of chicken feed and Wheat Chex or Gillette’s sale of disposable razors and disposable writing pens. 17. Nellie Tompkins is the owner and operator of Hot Mamma Salsa, which she sells at craft festivals. She only makes and sells three types of salsa––peach, pear, and pineapple. The joint costs of leasing her professional kitchen for manufacturing, travel to craft shows, insurance, and so on are allocated on an equal basis to the three types of salsas sold. Last year’s sales figures and allocated joint costs follow. Should Hot Mamma Salsa stop selling its pear salsa? Why or why not? Answer: Hot Mamma Salsa should continue to produce and sell all three types of salsas. An investigation of overall figures shows that a $6,000 profit was earned on the three items in the line: The pear salsa should not be dropped just because it is currently showing a loss; the joint costs would have to be allocated to the remaining two lines: Equal allocation of joint costs may not be the right way to distribute the costs. Other allocation bases that may be used include weighting, market value, or quantity sold. Other allocation methods would change the figures for each type of salsa, but not overall figures. 18. When the economy is characterized by high inflation, special pricing tactics are often necessary. One popular cost-oriented tactic is culling low-profit margin products from the product line. Why might this tactic backfire? What two other cost-oriented tactics can be used to guard against inflation? Describe these tactics. Answer: Culling low-profit margin products from a product line may backfire because of (1) the high volume and thus high profitability of a low profit margin item; (2) a loss of economies of scale as certain products are eliminated, which lowers the margins on other items; or (3) a lowering of the price–quality image of the entire line. Instead of culling these products, two other cost-oriented tactics may be used: delayed-quotation pricing and escalator pricing. DELAYED-QUOTATION PRICING. With this tactic, price is not set on the product until the item is either finished or delivered. This is a popular tactic for builders of nuclear power plants, ships, and airports. ESCALATOR PRICING. With this tactic, the final selling price will reflect cost increases incurred between the time when the order is placed and delivery is made. This tactic is used for complex products of long duration, with new customers, or with inelastic demand products. 19. How do value-based pricing, unbundling, and bundling help marketers hold on to market share during a recession? Answer: VALUE-BASED PRICING stresses to customers that they are getting a good value for their money. Products that typically use prestige pricing can introduce products at a lower price, thereby earning lower profit margins, which are typically offset by increased sales volume. UNBUNDLING allows a marketer to start charging separately for some products that were sold together prior to the economic recession. By selling items separately, the markets have a lower base price, which often appeals to consumers. BUNDLING can stimulate demand. When features are added to a product, consumers may perceive the offering as having greater value, which appeals to some consumers. Test Bank for MKTG Charles W. Lamb, Jr. Hair, Joseph F., Carl McDaniel 9781285091860

Document Details

Related Documents

person
Olivia Johnson View profile
Close

Send listing report

highlight_off

You already reported this listing

The report is private and won't be shared with the owner

rotate_right
Close
rotate_right
Close

Send Message

image
Close

My favorites

image
Close

Application Form

image
Notifications visibility rotate_right Clear all Close close
image
image
arrow_left
arrow_right